Sei sulla pagina 1di 78

III. TERMINATION OF EMPLOYMENT he never showed up.

Respondent was later caught by surprise when


A. EMPLOYER’S RIGHT TO DISCIPLINE petitioner filed the instant case for recovery of separation pay.
Respondent claimed that he never terminated the services of petitioner
59 Negros Slashers, Inc., et al. v. Alvin L. Teng and that during their mandatory conference, he even told the latter that
(BASKETBALL players are contractual employees) he could go back to work anytime but petitioner clearly manifested that
he was no longer interested in returning to work and instead asked for
Facts: separation pay.
Respondent Alvin Teng is a professional basketball player who
started his career as such in the Philippine Basketball Association and ISSUE:
then later on played in the Metropolitan Basketball Association (MBA). is Villaruel entitled to separation pay?
Some time in one of his games, particularly Game Number 4 of
the MBA Championship Round for the year 2000 season, Teng had a HELD:
below-par playing performance. Because of this, the coaching staff YES, but only financial assistance as a measure of social
decided to pull him out of the game. Teng then sat on the bench, untied justice Article 284 of the Labor Code reads:
his shoelaces and donned his practice jersey. On the following game, An employer may terminate the services of an employee
Game Number 5 of the Championship Round, Teng called-in sick and did who has been found to be suffering from any disease and whose
not play. continued employment is prohibited by law or is prejudicial to his
On March 16, 2001, because of what happened, the health as well as to the health of his co-employees: Provided, That
management of Negros Slashers came up with a decision, and through he is paid separation pay equivalent to at least 1 month salary or to
its General Manager, petitioner Rodolfo Alvarez, wrote Teng informing ½ month salary for every year of service whichever is greater, a
him of his termination from the team. fraction of at least six months being considered as 1 whole year.
A plain reading of the abovequoted provision clearly presupposes that it
Issue: is the employer who terminates the services of the employee found to be
Whether or not Teng’s dismissal from the Negros Slashers suffering from any disease and whose continued employment is
Team was unjustified and too harsh considering his misconduct. prohibited by law or is prejudicial to his health as well as to the health of
his co-employees. It does not contemplate a situation where it is the
Ruling: employee who severs his or her employment ties.
YES. As ruled in Sagales v. Rustan’s Commercial Corporation, The Court agrees with the CA in its observation of the following
while the employer has the inherent right to discipline, including that circumstances as proof that respondent did not terminate Villaruel’s
of dismissing its employees, this prerogative is subject to the employment: first, the only cause of action in petitioner’s original
regulation by the State in the exercise of its police power. complaint is that he was “offered a very low separation pay”; second,
In this regard, it is a hornbook doctrine that infractions there was no allegation of illegal dismissal, both in petitioner’s original
committed by an employee should merit only the corresponding penalty and amended complaints and position paper; and, third, there was no
demanded by the circumstance. The penalty must be commensurate with prayer for reinstatement. This is tantamount to resignation.
the act, conduct or omission imputed to the employee and must be Resignation is defined as the voluntary act of an employee
imposed in connection with the disciplinary authority of the employer. who finds himself in a situation where he believes that personal reasons
In the case at bar, the penalty handed out by the petitioners cannot be sacrificed in favor of the exigency of the service and he has no
was the ultimate penalty of dismissal. There was no warning or other choice but to disassociate himself from his employment
admonition for respondent’s violation of team rules, only outright However, there is no provision in the Labor Code which grants
termination of his services for an act which could have been punished separation pay to voluntarily resigning employees. In fact, the rule is that
appropriately with a severe reprimand or suspension. an employee who voluntarily resigns from employment is not entitled to
separation pay, except when it is stipulated in the employment contract
B. DEFINITIONS; ACTUAL & CONSTRUCTIVE DISMISSAL or CBA, or it is sanctioned by established employer practice or policy.
Since petitioner was not terminated from his employment and,
60 VILLARUEL VS. YEO HAN GUAN instead, is deemed to have resigned therefrom, he is not entitled to
separation pay under the provisions of the Labor Code.
FACTS: **
Villaruel filed with the NLRC NCR-Quezon City a Complaint for It may not be amiss to point out at this juncture that aside from
payment of separation pay against Yuhans Enterprises. Article 284 of the Labor Code, the award of separation pay is also
Petitioner alleged that in June 1963, he was employed as a authorized in the situations dealt with in Article 283 of the same Code
machine operator by Ribonette Manufacturing Company, an enterprise and under Section 4 (b), Rule I, Book VI of the IRR of the said Code
engaged in the business of manufacturing and selling PVC pipes and is where there is illegal dismissal and reinstatement is no longer feasible.
owned and managed by herein respondent Yeo Han Guan.Petitioner By way of exception, this Court has allowed grants of separation pay to
further alleged that in October 1998, he got sick and was confined in a stand as “a measure of social justice” where the employee is validly
hospital; In December 1998, he reported for work but was no longer dismissed for causes other than serious misconduct or those reflecting
permitted to go back because of his illness; he asked that respondent on his moral character.
allow him to continue working but be assigned a lighter kind of work but This Court, in a number of cases, has granted financial
his request was denied; instead, he was offered a sum of P15,000.00 as assistance to separated employees as a measure of social and
his separation pay; however, the said amount corresponds only to the compassionate justice and as an equitable concession. Taking into
period between 1993 and 1999; petitioner prayed that he be granted consideration the factual circumstances obtaining in the present case,
separation pay computed from his first day of employment in June 1963, the Court finds that petitioner is entitled to this kind of assistance. The
but respondent refused. Court notes that there is no evidence on record to show that petitioner
On the other hand, respondent averred that petitioner was has any derogatory record during his long years of service with
hired as machine operator from March 1993 until he stopped working respondent and that his employment was severed not by reason of any
sometime in February 1999 on the ground that he was suffering from infraction on his part but because of his failing physical condition. Based
illness; after his recovery, petitioner was directed to report for work, but
on the foregoing, the Court finds that the award of financial assistance is Respondent was hired by petitioner as security guard. Almost 4
deemed equitable under the circumstances. years after, he was relieved from service and was not given any
assignment thereafter. He filed a complaint for constructive dismissal and
nonpayment of 13th month pay. Petitioner presented a different version.
61 The University of Immaculate Conception vs National Labor and It alleged that respondent was not constructively or illegally dismissed,
Relations Commission but had voluntarily resigned.
The LA declared respondent to have been constructively
FACTS: dismissed. On appeal, the NLRC modified the LA decision. It declared
Teodora Axalan is a regular faculty member in the University of that respondent was neither constructively terminated nor did he
the Immaculate Conception holding the position of Associate Professor II. voluntarily resign. As such, respondent remained an employee of
Aside from being a regular faculty member, Axalan is the elected petitioner. The NLRC thus ordered respondent to immediately report to
President of the Employees' Union from 18 November to 22 November petitioner and assume his duty. The CA set aside the resolutions of the
2002. Axalan attended a seminar in Quezon City on website NLRC and reinstated that of the LA. The CA sustained respondent’s
development. Axalan then received a memorandum from Dean Maria claim of constructive dismissal and pointed out that respondent remained
Rosa Celestial asking her to explain in writing why she should not be on floating status for more than six (6) months, and petitioner offered no
dismissed for having been absent without official leave. Axalan claimed credible explanation why it failed to provide a new assignment to
that she held online classes while attending the seminar. She explained respondent
that she was under the impression that faculty members would not be
marked absent even if they were not physically present in the classroom ISSUE:
as long as they conducted online classes. From 28 January to 3 Whether or not the CA erred in sustaining respondent’s claim
February 2003, Axalan attended a second seminar in Baguio City on of constructive dismissal.
advanced paralegal training on which dates Axalan was absent. An Ad
Hoc Grievance Committee was created, and upon its recommendation HELD:
Axalan was suspended for a year for her AWOLcharges. On 1 December The petition lacks merit.
2003, Axalan filed a complaint against the University for illegal LABOR LAW: Floating status
suspension and constructive dismissal in the Labor Arbiter. The In cases involving security guards, a relief and transfer order in
University moved to dismiss on the ground that the Labor Arbiter had no itself does not sever employment relationship between a security guard
jurisdiction over the subject matter of the complaint. The university and his agency. An employee has the right to security of tenure, but this
maintained that jurisdiction lay in the voluntary arbitrator. does not give him a vested right to his position as would deprive the
company of its prerogative to change his assignment or transfer him
UNIVERSITY: The university maintains that Axalans suspension does where his service, as security guard, will be most beneficial to the client.
not constitute constructive dismissal and that the Labor Arbiters decision (ARTICLE 286.) Jurisprudence is trite with pronouncements
treating it as such is an attempt to make it appear that the voluntary that the temporary inactivity or “floating status” of security guards should
arbitrator has no jurisdiction. The university points out that for continue only for six months. Otherwise, the security agency concerned
constructive dismissal to exist, there must be severance of employment could be liable for constructive dismissal. In this case, respondent
by the employee because of unbearable act of discrimination, remained on “floating status” for more than six months. He was relieved
insensibility, or disdain on the part of the employer leaving the employee on January 30, 2006, and was not given a new assignment at the time he
with no choice but to forego continued employment. The university claims filed the complaint on August 2, 2006.
that on the contrary, Axalan eagerly reported for work as soon as the Petition is DENIED.
one-year suspension was over.
AXALAN: Finding of LA as constructively dismissed 62.1 William Barroga vs Data Center College et al
(ADDITIONAL ALLOWANCE)
ISSUE:
Whether Axalan was constructively dismissed? FACTS:
In November 1991, William Barroga was hired as an instructor
RULING: by Data Center College in its Laoag City, Ilocos Norte campus. In June
No. Constructive dismissal occurs when there is cessation of 1992, Barroga was re-assigned to Vigan, Ilocos Sur. Part of the deal for
work because continued employment is rendered impossible, his re-assignment was that Barroga will receive a monthly allowance of
unreasonable, or unlikely as when there is a demotion in rank or P1,200.00 for board and lodging while performing his job in Vigan.
diminution in pay or when a clear discrimination, insensibility, or disdain However, Data Center made it clear in writing that Barroga is only
by an employer becomes unbearable to the employee leaving the latter entitled to the additional allowance while assigned in Vigan and such
with no other option but to quit.29 allowance may be changed or forfeited if he will be re-assigned
somewhere. In 1994, he was recalled to Laoag. Later, Barroga was also
In this case however, there was no cessation of employment relations assigned as the temporary Head of Education; he was also given a
between the parties. It is unrefuted that Axalan promptly resumed scholarship grant to support his post-graduate studies. In 2003, Barroga
teaching at the university right after the expiration of the suspension was advised that he will be transferred to Bangued, Abra. Barroga
period. In other words, Axalan never quit. Hence, Axalan cannot claim refused because his father was sick and second, he found out that there
that she was left with no choice but to quit, a crucial element in a finding will be no additional allowance this time and that he will be working there
of constructive dismissal. Thus, Axalan cannot be deemed to have been as an instructor and not as a Head of Education. In the same year, he
constructively dismissed. filed a labor case against Data College for constructive dismissal.
Barroga alleged that the real purpose of his transfer is to demote him to
the rank of an instructor from being the Head for Education performing
62 NATIONWIDE SECURITY AND ALLIED SERVICES, INC.,
administrative functions and that his re-assignment will entail an indirect
Petitioner, v. RONALD P. VALDERAMA, Respondent.
reduction of his salary or diminution of pay considering that no additional
(Security Guard)
allowance will be given to cover for board and lodging expenses. He
claims that such additional allowance was given in the past and therefore
FACTS:
cannot be discontinued and withdrawn without violating the prohibition redundancy or lack of a posting commensurate to her position at the
against non-diminution of benefits. Project. Leynes was offered by NHPI the sum of P28,188.16
representing her unpaid wages, proportionate 13th month pay, tax refund
ISSUE: and service incentive leave pay (SILP).
Whether or not the absence of additional allowance in The LA found that NHPI act of putting Leynes on floating status
Barroga’s supposed re-assignment constitutes a diminution of benefits. was equivalent to termination from employment without just cause and
compliance with the twin requirements of notice and hearing.
HELD: On appeal, the NLRC reversed the LA decision. Leynes
No. It is true that as a general rule, benefits and perks enjoyed elevated the case to the CA on a Rule 65 petition for certiorari and the
by employees cannot be reduced and discontinued or diminished. But CA reversed the NLRC decision.
this rule is only applicable to grants or benefits which are founded on an ISSUE:
express policy or has ripened into a practice over a long period which is Whether or not the CA erred in finding that Leynes was
consistent and deliberate. In the case at bar, Barroga’s additional constructively dismissed when she was placed on floating status prior to
allowance while in Vigan is not permanent. In fact, Data College made her termination from employment on the ground of redundancy?
clear that such allowance is only applicable while Barroga is in Vigan and
such allowance is no longer applicable if he is going to be assigned HELD:
somewhere. Further, Data College showed that it is experiencing Although the CA correctly found that the record is bereft of any
financial difficulties hence the need to withdraw the scholarship showing that Leynes was unacceptable to BGCC, the evidence the
previously granted to Barroga. On the issue of his removal as Head for parties adduced a quo clearly indicates that petitioners were not in bad
Education, the same is valid. Barroga was merely assigned in a faith when they placed the former under floating status. Disgruntled by
temporary capacity, such designation is terminable at the pleasure of NHPI countermanding of her decision to bar Engr. Cantuba from the
Data College which made such appointment. Project, Leynes twice signified her intention to resign from her position. In
her application letter for an immediate emergency leave, Leynes also
63 NIPPON HOUSING PHIL. INC., and/or TADASHI OTA, HOROSHI distinctly expressed her dissatisfaction over NHPI resolution of her
TAKADA, YUSUHIRO KAWATA, MR. NOBOYUSHI and JOEL dispute with Engr. Cantuba and announced her plan of coordinating with
REYES,Petitioners, v. MAIAH ANGELA LEYNES, Respondent. her lawyer regarding her resignation letter.
(OFF-SETTING) In view of the sensitive nature of Leynes position and the
critical stage of the Project business development, NHPI was constrained
FACTS: to relay the situation to BGCC which, in turn, requested the immediate
Nippon Housing Philippines, Inc. (NPHI) hired respondent adoption of remedial measures from Takada, including the appointment
Maiah Angela Leyneson 26 March 2001 for the position of Property of a new Property Manager for the Project. Upon BGCC
Manager. recommendation, NHPI consequently hired Engr. Jose on 13 February
On 6 February 2002, Leynes had a misunderstanding with 2002 as Leynes replacement. Far from being the indication of bad faith
Engr. Honesto Cantuba, the Building Engineer assigned at Bay Gardens the CA construed the same to be, these factual antecedents suggest that
Condominium Project (the Project), regarding the extension of the latter NHPI immediate hiring of Engr. Jose as the new Property Manager for
working hours. Aside from instructing the security guards to bar Engr. the Project was brought about by Leynes own rash announcement of her
Cantuba from entry into the Project and to tell him to report to the NHPI intention to resign from her position. Although she subsequently changed
main office in Makati, Leynes also sent a letter dated 8 February 2002 by her mind and sent Reyes a letter by telefax on 13 February 2002
telefax to Joel Reyes, NHPI HR Head, apprising the latter of Cantuba announcing the reconsideration of her planned resignation and her
supposed insubordination and disrespectful conduct. With Engr. Cantuba intention to return to work on 15 February 2002, Leynes evidently had
submission of a reply in turn accusing Leynes of pride, conceit and poor only herself to blame for precipitately setting in motion the events which
managerial skills, Hiroshi Takada, NHPI VP, went on to issue the 12 led to NHPI hiring of her own replacement.
February 2002 memorandum, attributing the incident to "simple personal The record, moreover, shows that NHPI simply placed her on
differences" and directing Leynes to allow Engr. Cantuba to report back floating status "until such time that another project could be secured" for
for work. her. Traditionally invoked by security agencies when guards are
Disappointed with the foregoing management decision, Leynes temporarily sidelined from duty while waiting to be transferred or
submitted to a letter asking for an emergency leave of absence for the assigned to a new post or client, Article 286 of the Labor Code has been
supposed purpose of coordinating with her lawyer regarding her applied to other industries when, as a consequence of the bona fide
resignation letter. While NHPI offered the Property Manager position to suspension of the operation of a business or undertaking, an employer is
Engr. Carlos Jose on 13 February 2002 as a consequence Leynes constrained to put employees on floating status for a period not
signification of her intention to resign, it also appears that Leynes sent exceeding six months.
another letter to Reyes by telefax on the same day, expressing her Considering that even labor laws discourage intrusion in the
intention to return to work on 15 February 2002 and to call off her employer's judgment concerning the conduct of their business, courts
planned resignation upon the advice of her lawyer. On 22 February 2002, often decline to interfere in their legitimate business decisions, absent
Leynes was further served with a letter and memorandum relieving her showing of illegality, bad faith or arbitrariness. Indeed, the right of
from her position and directing her to report to NHPI main office while employees to security of tenure does not give them vested rights to their
she was on floating status. positions to the extent of depriving management of its prerogative to
Aggrieved, Leynes lost no time in filing against NHPI and its change their assignments or to transfer them. The record shows that
above-named officers a complaint for illegal dismissal, unpaid salaries, Leynes filed the complaint for actual illegal dismissal from which the case
benefits, damages and attorney fees before the NLRC. NHPI and its originated on 22 February 2002 or immediately upon being placed on
officers asserted that the management exercise of the prerogative to put floating status as a consequence of NHPI hiring of a new Property
an employee on floating status for a period not exceeding six months Manager for the Project. The rule is settled, however, that "off-
was justified in view of her threatened resignation from her position and detailing" is not equivalent to dismissal, so long as such status
BGCC request for her replacement. During the pendency of the case, does not continue beyond a reasonable time and that it is only
however, Reyes eventually served the DOLE and Leynes with a notice when such a "floating status" lasts for more than six months that
terminating her services effective 22 August 2002, on the ground of the employee may be considered to have been constructively
dismissed. A complaint for illegal dismissal filed prior to the lapse of said The BSSI denied Bello’s claim of constructive dismissal,
six-month and/or the actual dismissal of the employee is generally arguing that no promotion took place; Bello’s designation as assistant
considered as prematurely filed. detachment commander or detachment commander was not an
Viewed in the light of the foregoing factual antecedents, the employment position but a duty-related assignment; Bello abandoned his
Court finds that the CA reversibly erred in holding petitioners liable for job when he went on an indefinite leave of absence and did not report for
constructively dismissing Leynes from her employment. There is said to work.[7]
be constructive dismissal when an act of clear discrimination, insensitivity
or disdain on the part of the employer has become so unbearable as to The Labor Arbiter’s Ruling
leave an employee with no choice but to forego continued employment. In his December 29, 2005 decision,[8] Labor Arbiter Cresencio
Constructive dismissal exists where there is cessation of work because G. Ramos, Jr. found that Bello was illegally dismissed, noting that the
continued employment is rendered impossible, unreasonable or unlikely, BSSI failed to adduce evidence that Bello abandoned his employment.
as an offer involving a demotion in rank and a diminution in pay. Stated Thus, he ordered Bello’s reinstatement and awarded him backwages
otherwise, it is a dismissal in disguise or an act amounting to dismissal amounting to P391,474.25.
but made to appear as if it were not. After the NLRC dismissed the BSSI’s belated appeal and subsequent
With no other client aside from BGCC for the building motion for reconsideration,[9] the latter filed a petition for certiorari with
management side of its business, the Court finds that NHPI was acting the CA. The CA granted the petition,[10] thus reinstating BSSI’s appeal
well within its prerogatives when it eventually terminated Leynes services with the NLRC.
on the ground of redundancy. One of the recognized authorized causes In its March 26, 2008 resolution, the NLRC affirmed the labor
for the termination of employment, redundancy exists when the service arbiter’s decision, finding that Bello had been constructively dismissed
capability of the workforce is in excess of what is reasonably needed to when he was demoted to the rank-and-file position of traffic marshal after
meet the demands of the business enterprise. A redundant position is occupying the supervisory position of assistant detachment commander
one rendered superfluous by any number of factors, such as over hiring and detachment commander.[11] The denial of BSSI’s subsequent
of workers, decreased volume of business, dropping of a particular motion for reconsideration led it back to the CA on a petition for certiorari
product line previously manufactured by the company or phasing out of under Rule 65 of the Rules of Court.[12]
service activity priorly undertaken by the business. It has been held that
the exercise of business judgment to characterize an employee service The CA Ruling
as no longer necessary or sustainable is not subject to discretionary The CA nullified the NLRC resolutions, finding the records
review where, as here, it is exercised there is no showing of violation of bereft of evidence substantiating the labor arbiter’s and the NLRC’s
the law or arbitrariness or malice on the part of the employer. conclusions that Bello had been constructively dismissed.[13] It noted
Having been validly terminated on the ground of redundancy, that Bello offered no evidence to prove that there was a series of
Leynes is entitled to separation pay equivalent to one-month salary for promotions that would justify his claim of subsequent demotion. The CA
every year of service but not to the backwages adjudicated in her favor denied the BSSI’s motion for reconsideration,[14] paving the way for the
by the Labor Arbiter. present petition.
GRANTED
The Petition
Bello insists that he was constructively dismissed when he was
64 Francis Bello vs Bonifacio Security Services demoted to a mere traffic marshal after having been promoted to the
(TRANSFERRING AND REASSIGNING NOT CONSTRUCTIVE positions of supervisor, assistant detachment commander, and
DISMISSAL) detachment commander.

The Factual Background The Case for the BSSI


Respondent Bonifacio Security Services, Inc. (BSSI) is a The BSSI prays for the petition’s outright dismissal due to a
domestic private corporation engaged in the business of providing defective verification, arguing that the special power of attorney (SPA) of
security services. In July 2001, the BSSI hired Bello as a roving traffic Bello’s attorney-in-fact, Geraldine Bello-Ona, was limited to representing
marshal to manage traffic and to conduct security and safety-related him in the NLRC case only and not to the present petition; and that Bello-
operations in the Bonifacio Global City (BGC). In August 2001, Bello was Ona has no personal knowledge of the allegations in the petition. On the
posted at the Negros Navigation Company in Pier 2, North Harbor, to merits of the case, the BSSI contends that the CA correctly ruled that
supervise sectoral operations. In November 2001, he was assigned at there was no evidence to substantiate the NLRC’s finding of constructive
BGC as assistant detachment commander. After a week, he was dismissal.
transferred to Pacific Plaza Towers as assistant detachment commander
and later as detachment commander. In June 2002, he was assigned at The Issues
Pier 2, North Harbor as assistant detachment commander, but later The core issues boil down to: whether the petition should be
reassigned to BGC. In August 2002, the BSSI hired a new operations dismissed outright for defective verification; and whether the CA erred in
manager, resulting in the reorganization of posts. In October 2002, Bello annulling the NLRC’s resolutions.
was assigned as roving traffic marshal at the BGC. On October 25, 2002,
he filed an indefinite leave of absence when his new assignment took The Court’s Ruling
effect. The petition lacks merit.
On November 5, 2002, Bello filed a complaint against the BSSI Verification of a pleading is a formal, not jurisdictional,
and its General Manager, respondent Samuel Tomas, with the National requirement intended to secure the assurance that the matters alleged in
Labor Relations Commission (NLRC),[5] claiming that he had been a pleading are true and correct.[15] Thus, the court may simply order the
constructively dismissed when he was demoted from a detachment correction of unverified pleadings or act on them and waive strict
commander to a mere traffic marshal. He alleged that he received a compliance with the rules.[16] It is deemed substantially complied with
series of promotions from 2001 to 2002, from traffic marshal to when one who has ample knowledge to swear to the truth of the
supervisor, to assistant detachment commander, and to detachment allegations in the complaint or petition signs the verification, and when
commander.[6] matters alleged in the petition have been made in good faith or are true
and correct.[17]
In this case, we find that the petition’s verification distribution centers under the principle that 'things that are alike should
substantially complied with the requirements of the rules. The SPA be treated alike' since they also hold the position of 'distribution
authorized Bello-Ona to represent Bello in the case entitled “Francis personnel.'"
Bello v. Bonifacio Security Services, Inc. and/or Samuel Tomas, (CA)
Case No. 047829-06; NLRC-N[CR] Case No. 00-11-09529-2002”[18] –
the case from which the present petition originated. As the daughter of ISSUE:
Bello, Bello-Ona is deemed to have sufficient knowledge to swear to the Is constructive dismissal applicable to the respondents?
truth of the allegations in the petition, which are matters of record in the
tribunals and the appellate court below. HELD:
On the merits of the case, we find no reason to disturb the CA The concept of constructive dismissal is inapplicable to
conclusion that there was no constructive dismissal. Case law defines respondents. Constructive dismissal is a derivative of dismissal without
constructive dismissal as a cessation of work because continued cause; an involuntary resignation, nay, a dismissal in disguise. It occurs
employment has been rendered impossible, unreasonable, or unlikely, as when there is cessation of worker caused continued employment is
when there is a demotion in rank or diminution in pay, or both, or when a rendered impossible, unreasonable, or unlikely as when there is a
clear discrimination, insensibility, or disdain by an employer becomes demotion in rank or diminution in pay or when a clear discrimination,
unbearable to the employee.[19] insensibility, or disdain by an employer becomes unbearable to the
We note that, other than his bare and self-serving employee leaving the latter with other option but to quit. In turn, dismissal
allegations, Bello has not offered any evidence that he was promoted in a without cause is prohibited because of the Constitutional security of
span of four months since his employment as traffic marshal in July 2001 tenure of workers.
to a detachment commander in November 2001. During his six-month Simply put, security of tenure from which springs the concept of
probationary period of employment,[20] it is highly improbable that Bello constructive dismissal is not an absolute right. It cannot be pleaded to
would be promoted after just a month of employment, from a traffic avoid the transfer or assignment of employees according to the
marshal in July 2001 to supervisor in August 2001, and three months requirements of the employer's business. Such transfer or assignment
later to assistant detachment commander and to detachment commander becomes objectionable only when it is not for "reasonable returns on
in November 2001. At most, the BSSI merely changed his assignment or investments," and for "expansion and growth" which are constitutionally
transferred him to the post where his service would be most beneficial to recognized employer's rights, but is sought merely as a convenient cover
its clients. The management's prerogative of transferring and for oppression.
reassigning employees from one area of operation to another in Records will reveal that [respondents] Domingo, Remigio,
order to meet the requirements of the business is generally not Norico, Marcelo and Ozaraga as accountants or employees
constitutive of constructive dismissal.[21] We see this to be the case performing accounting functions were affected by the Shared
in the present dispute so that the consequent reassignment of Bello to a Services Policy of the Company. Thus, after the provincial depots
traffic marshal post was well within the scope of the BSSI’s management were closed down, they were reassigned to [Unilab’s] Finance
prerogative. Division to service the accounting requirement of the Unilab group
WHEREFORE, we hereby DENY the petition and of companies. Thereafter, [respondents] Norico, Marcelo and
AFFIRM the assailed CA decision and resolution in CA-G.R. SP. No. Ozaraga voluntarily resigned while respondentns Domingo and
105402. Costs against the petitioner. Remigio remained with [Unilab].
SO ORDERED. Most recently, The University of the Immaculate Concepcion v.
National Labor Relations Commission26 iterated that a crucial element
in a finding of constructive dismissal is a cessation of employment
65 United Laboratories, Inc., Petitioner, v. Jaime Domingo relations between the parties.
Substituted by his spouse Carmencita Punzalan Domingo, et al. Certainly, the Court cannot accept the proposition that when an
(Retirement vs. Redundancy; cessation of work = CD) employee opposes his employer’s decision to transfer him to another
work place, there being no bad faith or underhanded motives on the part
FACTS: of either party, it is the employee’s wishes that should be made to prevail.
Sometime in 2001, under a Physical Distribution Master Plan On the basis of the qualifications, training and performance of the
(PDMP), Unilab consolidated its finished goods inventories and logistics employee, the prerogative to determine the place or station where he or
activities (warehousing, order processing and shipping) into one she is best qualified to serve the interests of the company belongs to the
distribution center located in Metro Manila. As a result, Unilab closed employer
down its sixteen (16) provincial depots. The job functions of the Respondents are laboring under a cloud of confusion.
employees working thereat were declared redundant and their positions Retirement and redundancy
were abolished. Unilab gave the redundant employees a separation
package of two and a half (2 1/2) months' pay for every year of service. ART. 283. Closure of establishment and reduction of personnel. – The
In the succeeding year, on 7 January 2002, respondents wrote employer may also terminate the employment of any employee due to
Unilab requesting for their separation or retirement from service under a the installation of labor saving devices, redundancy, retrenchment to
separation package similar or equivalent to that of the redundant prevent losses or the closing or cessation of the operation of the
establishment or undertaking unless the closing is for the purpose of
employees in the provincial depots. Respondents referred to this
circumventing the provisions of this Title, by serving a written notice on
separation package as the Bagong Sibol Program the workers and the Department of Labor and Employment at least one
On 9 April 2002, respondents' counsel, on their behalf, wrote (1) month before the intended date thereof. In case of termination due to
Unilab reiterating respondents' previous request to be separated from the installation of labor saving devices or redundancy, the worker
service under Unilab's purported Bagong Sibol Program. Particularly, affected thereby shall be entitled to a separation pay equivalent to at
respondents were keen on retiring and receiving 2 1/2 months' pay for least his one month pay or to at least one month pay for every year of
every year of service, and all the other benefits which Unilab had service, whichever is higher. In case of retrenchment to prevent losses
and in cases of closures or cessation of operations of establishment or
extended to the redundant employees in the provincial depots. The undertaking not due to serious business losses or financial reverses, the
message and sentiment were that "they should likewise be retired under separation pay shall be equivalent to one (1) month pay or to at least
the same redundancy plan or retirement scheme [because] their one-half (1/2) month pay for every year of service, whichever is higher. A
positions are similarly situated [to] the 'retired employees' of [Unilab's] fraction of at least six (6) months shall be considered one (1) whole year.
ART. 287. Retirement. – Any employee retirement may be retired upon conduct and action are for valid and legitimate grounds such as
reaching the retirement age established in the collective bargaining genuine business necessity and that the transfer is not
agreement or other applicable employment contract. unreasonable, inconvenient or prejudicial to the employee. If the
employer cannot overcome this burden of proof, the employees
In case of retirement, the employees shall be entitled to receive such transfer shall be tantamount to unlawful constructive dismissal.
retirement benefits as he may have earned under existing laws and any
Record shows that HCPTI miserably failed to discharge the
collective bargaining, and other agreement: Provided, however, the
employee’s retirement benefits under any collective bargaining and other foregoing on us. While there was a lack of showing that the transfer or
agreement shall not be less than those provided herein. reassignment entailed a diminution of salary and benefits, one fact that
must not be lost sight of was that Morales was already occupying the
In the absence of retirement plan or agreement providing for retirement position of Division Manager at HCPTIs Accounting Department as a
benefits of employee upon reaching the age of sixty (60) years or more, consequence of his promotion to said position on 22 October 2002.
but not beyond sixty-five (65) years which is hereby declared the Concurrently appointed as member of HCPTIs Management
compulsory retirement age, who has served at least five (5) years in the Committee (MANCOM) on 2 December 2002, Morales was subsequently
said establishment, may retire and shall be entitled to retirement pay
reassigned by HCPTI "from managerial accounting to Operations Cost
equivalent to at least one-half (1/2) month salary for every year of service
, a fraction of at least six (6) months being considered as one whole year. Accounting" on 27 March 2003, without any mention of the position to
which he was actually being transferred. That the reassignment was a
Unless the parties provide for broader inclusions, the term one-half (1/2) demotion is, however, evident from Morales new duties which, far from
month salary shall mean fifteen (15) days plus one-twelfth of the 13th being managerial in nature, were very simply and vaguely described as
month pay and the cash equivalent of not more than five (5) days of inclusive of "monitoring and evaluating all consumables requests, gears
service incentive leaves.1âwphi1 and equipments related to HCPTIs operations" as well as "close
interaction with its sub-contractor Bulk Fleet Marine Corporation."
66 JONATHAN V. MORALES, Petitioner, v. HARBOUR CENTRE Admittedly, the right of employees to security of tenure does
PORT TERMINAL, INC. Respondent. not give them vested rights to their positions to the extent of depriving
management of its prerogative to change their assignments or to transfer
FACTS: them. By management prerogative is meant the right of an employer to
Regularized on 17 November 2000, Morales was promoted to regulate all aspects of employment, such as the freedom to prescribe
Division Manager of the Accounting Department, for which he was work assignments, working methods, processes to be followed,
compensated a monthly salary of P33,700.00, plus allowances starting 1 regulation regarding transfer of employees, supervision of their work, lay-
July 2002. Subsequent to HCPTIs transfer to its new offices at Vitas, off and discipline, and dismissal and recall of workers.
Tondo, Manila on 2 January 2003, Morales received an inter-office Although jurisprudence recognizes said management
memorandum dated 27 March 2003, reassigning him to Operations Cost prerogative, it has been ruled that the exercise thereof, while ordinarily
Accounting, tasked with the duty of "monitoring and evaluating all not interfered with, is not absolute and is subject to limitations imposed
consumables requests, gears and equipment" related to the corporations by law, collective bargaining agreement, and general principles of fair
operations and of interacting with its sub-contractor, Bulk Fleet Marine play and justice. Thus, an employer may transfer or assign employees
Corporation. from one office or area of operation to another, provided there is no
Morales wrote Singson, protesting that his reassignment was a demotion in rank or diminution of salary, benefits, and other privileges,
clear demotion since the position to which he was transferred was not and the action is not motivated by discrimination, made in bad faith, or
even included in HCPTIs plantilla. Singson, the Administration Manager, effected as a form of punishment or demotion without sufficient cause.
answered by stating that the transfer was a management prerogative. Indeed, having the right should not be confused with the manner in which
For the whole of the ensuing month Morales was absent from that right is exercised.
work and/or tardy. Singson issued to Morales a 29 April 2003 inter-office GRANTED
memorandum denominated as a First Warning. In view of the absences
Morales continued to incur, HCPTI issued a Second Warning. C. GROUNDS FOR TERMINATION; SUBSTANTIVE & PROCEDURAL
In the meantime, Morales filed a complaint dated 25 April 2003
against HCPTI, Filart and Singson, for constructive dismissal, moral and 66.1 ROBINSONS GALLERIA/ROBINSONS SUPERMARKET
exemplary damages as well as attorneys fees. CORPORATION and/or JESS MANUEL, petitioners, vs. IRENE R.
LA dismissed the complaint for lack of merit. It ruled that RANCHEZ, respondents
Morales reassignment was a valid exercise of HCPTIs management (SUBSTANTIVE & PROCEDURAL PROCESS IS MANDATORY;
prerogative which cannot be construed as constructive dismissal absent PROBATIONAL EMPLOYEE)
showing that the same was done in bad faith and resulted in the
diminution of his salary and benefits. The NLRC however, reversed the FACTS:
decision. Its subsequent denial of HCPTIs motion for reconsideration Respondent Ranchez was a probationary employee for 5
prompted the latter to file a petition for certiorari before the CA. The CA months. She was hired as a cashier by Robinsons sometime within that
reversed the findings of the NLRC. Hence, this petition. period. Two weeks after she was hired, she reported the loss of cash
which she had placed in the company locker. She offered to pay for the
ISSUE: lost amount (P20,299.00) but the Operations Manager of Robinsons had
Whether or not petitioner was constructively dismissed her strip-searched then reported her to the police even though they found
nothing on her person. An information for Qualified Theft was filed with
HELD: the Quezon City Regional Trial Court. She was detained for 2 weeks for
Yes. CA Decision reversed and set aside failure to immediately post bail. Weeks later, respondent Ranchez filed a
Constructive dismissal exists where there is cessation of work complaint for illegal dismissal and damages. A year later, Robinsons sent
because "continued employment is rendered impossible, unreasonable to respondent by mail a notice of termination and/or notice of expiration
or unlikely, as an offer involving a demotion in rank or a diminution in pay of probationary employment.
and other benefits. The Labor Arbiter dismissed the complaint for illegal dismissal,
In cases of a transfer of an employee, the rule is settled alleging that at the time of filing respondent Ranchez had not yet been
that the employer is charged with the burden of proving that its terminated. She was merely investigated. However, the NLRC reversed
this ruling, stating that Ranchez was illegally dismissed and that
Robinson's should reinstate her. It held that Ranchez was deprived of D. TERMINATION BY EMPLOYER; JUST CAUSE FOR TERMINATION
due process when she was strip-searched and sent to jail for two weeks 1. SERIOUS MISCONDUCT
because such amounted to constructive dismissal, making it impossible
for the respondent to continue under the employment. Even though she 67 PASTOR DIONISIO V. AUSTRIA vs. HON. NATIONAL LABOR
was merely a probationary employee, the lapse of the probationary RELATIONS COMMISSION
contract did not amount to a valid dismissal because there was already (Church – employer; Minister – employee)
an unwarranted constructive dismissal beforehand. (BREACH OF TRUST; MISCONDUCT)
The NLRC denied Robinson's motion for reconsideration. The
CA affirmed the decision of the NLRC. FACTS:
Petitioner had worked with the private respondent Seventh Day
ISSUE: Adventists (SDA) for 28 years before he was terminated. Prior to said
Whether respondent was illegally terminated from employment termination, petitioner was asked to admit accountability for the church
by petitioners. offerings collected by his wife in the amount of P15,078.10.
Petitioner refused since it was private respondents Pastor Buhat and
HELD: Eufronio Ibesate who authorized his wife to collect. Thereafter petitioner
The petition is unmeritorious. requested Pastor Buhat to convene the Executive Committee to settle
LABOR LAW: Probationary employees; termination of the dispute between him and Pastor Rodrigo, but the latter denied the
employment same, and heated arguments between the two ensued until petitioner
There is probationary employment when the employee upon banged the attaché case of Pastor Buhat on the table, scattered the
his engagement is made to undergo a trial period during which the books and threw the phone. Later, an Executive Committee meeting was
employer determines his fitness to qualify for regular employment based held where the non-remittance of church collections and the events that
on reasonable standards made known to him at the time of engagement. transpired were discussed. Subsequently, petitioner received a letter of
A probationary employee, like a regular employee, enjoys dismissal citing therein grounds for the termination of his services:
security of tenure. However, in cases of probationary employment, aside misappropriation of denominational funds, willful breach of trust,
from just or authorized causes of termination, an additional ground is serious misconduct, gross and habitual neglect of duties, and
provided under Article 281 of the Labor Code,i.e., the probationary commission of an offense against the person of employers duly
employee may also be terminated for failure to qualify as a regular authorized representative.
employee in accordance with reasonable standards made known by the Petitioner then filed a complaint for illegal dismissal and a decision was
employer to the employee at the time of the engagement. Thus, the rendered in his favor.
services of an employee who has been engaged on probationary basis
may be terminated for any of the following: ISSUE:
(1) a just or Whether or not Austria’s termination was valid? NO!
(2) an authorized cause; and
(3) when he fails to qualify as a regular employee in HELD:
accordance with reasonable standards prescribed by the employer.
Article 277(b) of the Labor Code mandates that the employer BREACH OF TRUST - no basis for the alleged loss of confidence and
shall furnish the worker, whose employment is sought to be terminated, a breach of trust. Settled is the rule that under Article 282 (c) of the Labor
(1) written notice containing a statement of the causes of Code, the breach of trust must be willful. A breach is willful if it is done
termination, and (2) shall afford the latter ample opportunity to be intentionally, knowingly and purposely, without justifiable excuse, as
heard and to defend himself with the assistance of a representative distinguished from an act done carelessly, thoughtlessly, heedlessly or
if he so desires, in accordance with company rules and regulations inadvertently.[38] It must rest on substantial grounds and not on the
pursuant to the guidelines set by the Department of Labor and employers arbitrariness, whims, caprices or suspicion; otherwise, the
Employment. employee would eternally remain at the mercy of the employer. The
In the instant case, based on the facts on record, petitioners stenographic notes on the testimony of Naomi Geniebla, the Negros
failed to accord respondent substantive and procedural due process. The Mission Church Auditor and a witness for private respondents, show that
haphazard manner in the investigation of the missing cash, which was Pastor Austria was able to remit all his collections to the treasurer of the
left to the determination of the police authorities and the Prosecutor's Negros Mission.
Office, left respondent with no choice but to cry foul. Administrative
investigation was not conducted by petitioner Supermarket. On the same SERIOUS MISCONDUCT AND COMMISSION OF AN OFFENSE
day that the missing money was reported by respondent to her AGAINST THE PERSON - Misconduct has been defined as improper or
immediate superior, the company already pre-judged her guilt without wrong conduct. It is the transgression of some established and definite
proper investigation, and instantly reported her to the police as the rule of action, a forbidden act, a dereliction of duty, willful in character,
suspected thief, which resulted in her languishing in jail for two weeks. and implies wrongful intent and not mere error in judgment. For
The due process requirements under the Labor Code are misconduct to be considered serious it must be of such grave and
mandatory and may not be replaced with police investigation or court aggravated character and not merely trivial or unimportant. Based on this
proceedings. An illegally or constructively dismissed employee, standard, we believe that the act of petitioner in banging the attached
respondent is entitled to: (1) either reinstatement, if viable, or separation case on the table, throwing the telephone and scattering the books in the
pay, if reinstatement is no longer viable; and (2) backwages. These two office of Pastor Buhat, although improper, cannot be considered as grave
reliefs are separate and distinct from each other and are awarded enough to be considered as serious misconduct. After all, though
conjunctively. petitioner committed damage to property, records show that he did not
In this case, since respondent was a probationary employee at physically assault Pastor Buhat or any other pastor present during the
the time she was constructively dismissed by petitioners, she is entitled incident. Hence, there is no basis for the allegation that petitioners act
to separation pay and backwages. Reinstatement of respondent is no constituted serious misconduct or that the same was an offense against
longer viable considering the circumstances. the person of the employers duly authorized representative. As such, the
DENIED cited actuation of petitioner does not justify the ultimate penalty of
dismissal from employment. While the Constitution does not condone tampered school records, violated school policies and committed
wrongdoing by the employee, it nevertheless urges a moderation of the misconduct. Respondent was investigated for her act of increasing the
sanctions that may be applied to him in light of the many disadvantages grades of her students while she was on maternity leave. The
that weigh heavily on hi m like an albatross on his neck. Where a penalty investigating panel reached the conclusion that respondent altered her
less punitive would suffice, whatever missteps may have been committed student grades while she was on leave, which is, according to them, a
by the worker ought not be visited with a consequence so severe such as case of education malpractice or grave misconduct and grossly
dismissal from employment. For the foregoing reasons, we believe that prejudicial to the good name of the petitioner school. On these bases, the
the minor infraction committed by petitioner does not merit the ultimate members of the investigating committee ruled to terminate respondent
penalty of dismissal. services. Respondent filed a Complaint for illegal dismissal.

GROSS AND HABITUAL NEGLECT OF DUTIES - Suffice it to say that ISSUE:


all private respondents had were allegations but not proof. Aside from WON respondents act of giving failing students higher grades
merely citing the said ground, private respondents failed to prove than what they actually earned is tantamount to serious misconduct
culpability on the part of petitioner which justified her dismissal.

HELD:
68 PHILIPPINE LONG DISTANCE TELEPHONE COMPANY vs. NLRC NO. Respondent had committed a misconduct. However, such
and MARILYN ABUCAY misconduct is not serious enough to warrant her dismissal from
(NO SEPARATION PAY FOR SERIOUS MISCONDUCT) employment under paragraph (a) of Article 282 of the Labor Code.
Misconduct is defined as improper or wrong conduct. It is the
FACTS: transgression of some established and definite rule of action, a forbidden
Marilyn Abucay, a traffic operator of the Philippine Long act, a dereliction of duty, willful in character and implies wrongful intent
Distance Telephone Company, was accused by two complainants of and not mere error of judgment. The misconduct to be serious within
having demanded and received from them the total amount of P3,800.00 the meaning of the act must be of such a grave and aggravated
in consideration of her promise to facilitate approval of their applications character and not merely trivial or unimportant. Such misconduct,
for telephone installation. Investigated and heard, she was found guilty however serious, must nevertheless be in connection with the work of the
as charged and accordingly separated from the service. She went to the employee to constitute just cause from his separation. In order to
Ministry of Labor and Employment claiming she had been illegally constitute serious misconduct which will warrant the dismissal of an
removed. The case was dismissed but she was granted with a separation employee under paragraph (a) of Article 282 of the Labor Code, it is not
pay. sufficient that the act or conduct complained of has violated some
established rules or policies. It is equally important and required that the
ISSUE: act or conduct must have been performed with wrongful intent. There
WON Abuca is entitled to separation pay. is no evidence to show that there was ulterior motive on the part of the
respondent when she decided to pass her students. Also, it was not
HELD: shown that respondent received immoral consideration when she did the
NO. The rule embodied in the Labor Code is that a person same. Respondent has maintained her stand that her decision to pass
dismissed for cause as defined therein is not entitled to separation pay. the concerned students was done out of humanitarian consideration. It
Separation pay shall be allowed as a measure of social justice only in is settled that a misconduct, which is not serious or grave, cannot be a
those instances where the employee is validly dismissed for causes other valid basis for dismissing an employee.
than serious misconduct or those reflecting on his moral character.
Where the reason for the valid dismissal is, for example, habitual
intoxication or an offense involving moral turpitude, like theft or illicit 70 EDEN LLAMAS vs OCEAN GATEWAY MARITIME AND
sexual relations with a fellow worker, the employer may not be required MANAGEMENT, INC.
to give the dismissed employee separation pay, or financial assistance, (GROSS MISCONDUCT; REQUISITES)
or whatever other name it is called, on the ground of social justice. In the
case at bar, the grant of separation pay in the case at bar is unjustified. FACTS:
Abuca has been dismissed for dishonesty and as she herself has Ocean Gateway Maritime and Management, Inc. hired Eden
impliedly admitted. The fact that she has worked with the PLDT for more Llamas as an accounting manager. Mary Anne T. Macaraig, respondents
than a decade, if it is to be considered at all, should be taken against her Chief Executive Officer, called Llamas’s attention to her failure, despite
as it reflects a regrettable lack of loyalty that she should have repeated demands, to accomplish the long overdue monthly and annual
strengthened instead of betraying during all of her 10 years of service company financial reports and to remit the companys contributions to the
with the company. If regarded as a justification for moderating the penalty Social Security System (SSS) and PhilHealth for November and
of dismissal, it will actually become a prize for disloyalty, perverting the December 2001. Subsequently or on February 20, 2002, Mary Anne
meaning of social justice and undermining the efforts of labor to cleanse again instructed petitioner to remit on that day or until the following day
its ranks of all undesirables. the companys contributions to the SSS and PhilHealth for January 2002.
By petitioners claim, she failed to comply with the instruction as money
for the purpose was not, as of February 20, 2002, credited to the
69 NLRC, ST. JUDE CATHOLIC SCHOOL, et.al., vs MA. company’s account at the bank. The following day, petitioner did not
BERNADETTE S. SALGARINO report for work as she was allegedly suffering from hypertension, hence,
(ALTERATION OF GRADE WITHOUT WRONGFUL INTENT NOT A she was again unable to remit the contributions. On February 26, 2002
SERIOUS MISCONDUCT) Mary Anne sent a memorandum to petitioner charging her with gross and
habitual neglect of duty and/or misconduct or willful disobedience and
FACTS: insubordination, detailing therein the bases of the charges, and requiring
Maria Bernadette A. Salgarino was employed by St. Jude her to submit a written explanation why she should not be penalized or
Catholic School as Mathematics teacher. She was directed to submit dismissed from employment. Complying with the show cause order,
herself to a panel of investigators and explain why she had allegedly petitioner claimed that the delay was due to the fact that she was
overloaded with work and undermanned. On account of the delay in the (2) WON petitioners are illegally dismissed?
remittance of those contributions, respondent was penalized in the
amount of P18,580.41 which it charged to petitioner via salary Held:
deductions. Later, the company sent Llamas notice of termination from (1) In order to resolve the issue of whether P&G is the
employment anchored on gross and habitual neglect of duty and/or employer of petitioners, it is necessary to first determine whether Promm-
serious misconduct or willful disobedience/insubordination Gem and SAPS are labor-only contractors or legitimate job contractors.
Clearly, the law and its implementing rules allow contracting
HELD: arrangements for the performance of specific jobs, works or services.
On petitioner’s declaration that I believe that I did something However, in order for such outsourcing to be valid, it must be made to
good for our office when our declaration of gross income submitted to an independent contractor because the current labor rules expressly
City Hall for the renewal of our municipal license was lower than our prohibit labor-only contracting.
actual gross income for which the office had paid a lower amount, the To emphasize, there is labor-only contracting when the
Court finds the same as betraying a streak of dishonesty in her. It contractor or sub-contractor merely recruits, supplies or places workers
partakes of serious misconduct. to perform a job, work or service for a principal and any of the following
xxx elements are present:
Misconduct has been defined as improper or wrong conduct. It 1. i) The contractor or subcontractor does not have substantial capital
is the transgression of some established and definite rule of action, or investment which relates to the job, work or service to be
a forbidden act, a dereliction of duty, willful in character, and performed and the employees recruited, supplied or placed by such
implies wrongful intent and not mere error of judgment. The contractor or subcontractor are performing activities which are
misconduct to be serious must be of such grave and aggravated directly related to the main business of the principal; or
character and not merely trivial and unimportant. Such misconduct, 1. ii) The contractor does not exercise the right to control over the
however serious, must nevertheless be in connection with the performance of the work of the contractual
employees work to constitute just cause for his separation. Thus,
for misconduct or improper behavior to be a just cause for Under the circumstances, Promm-Gem cannot be considered as a
dismissal, (a) it must be serious; (b) must relate to the performance labor-only contractor. We find that it is a legitimate independent
of the employees duties; and (c) must show that the employee has contractor.
become unfit to continue working for the employer. Indeed, an Considering that SAPS has no substantial capital or investment
employer may not be compelled to continue to employ such person and the workers it recruited are performing activities which are directly
whose continuance in the service would be patently inimical to his related to the principal business of P&G, we find that the former is
employers interest. For her act of understating the companys profits or engaged in “labor-only contracting”.
financial position was willful and not a mere error of judgment, committed
as it was in order to save costs, which to her warped mind, was Where labor-only contracting exists, the Labor Code itself
supposed to benefit respondent. It was not merely a violation of company establishes an employer-employee relationship between the employer
policy, but of the law itself, and put respondent at risk of being made and the employees of the labor-only contractor. The statute establishes
legally liable. Verily, it warrants her dismissal from employment as this relationship for a comprehensive purpose: to prevent a
respondents Accounting Manager, for as correctly ruled by the appellate circumvention of labor laws. The contractor is considered merely an
court, an employer cannot be compelled to retain in its employ someone agent of the principal employer and the latter is responsible to the
whose services is inimical to its interests. employees of the labor-only contractor as if such employees had been
directly employed by the principal employer.

71 Aliviado vs. Procter and Gamble (2) Yes. Misconduct has been defined as improper or wrong
(ERROR IN JUDGEMENT NOT A SERIOUS MISCONDUCT) conduct; the transgression of some established and definite rule of action, a
forbidden act, a dereliction of duty, unlawful in character implying wrongful intent
Facts: and not mere error of judgment. The misconduct to be serious must be of such
80 Petitioners worked as merchandisers of P&G. They all grave and aggravated character and not merely trivial and unimportant.[46] To be a
individually signed employment contracts with either Promm-Gem or just cause for dismissal, such misconduct (a) must be serious; (b) must relate to
SAPS. They were assigned at different outlets, supermarkets and stores the performance of the employees duties; and (c) must show that the employee
where they handled all the products of P&G. They received their wages has become unfit to continue working for the employer.[47] In the instant case,
from Promm-Gem or SAPS. petitioners-employees of Promm-Gem may have committed an error of judgment
SAPS and Promm-Gem imposed disciplinary measures on in claiming to be employees of P&G, but it cannot be said that they were
erring merchandisers for reasons such as habitual absenteeism, motivated by any wrongful intent in doing so. As such, we find them guilty of only
dishonesty or changing day-off without prior notice. simple misconduct for assailing the integrity of Promm-Gem as a legitimate and
To enhance consumer awareness and acceptance of the independent promotion firm. A misconduct which is not serious or grave, as that
products, P&G entered into contracts with Promm-Gem and SAPS for the existing in the instant case, cannot be a valid basis for dismissing an employee.
promotion and merchandising of its products.
In December 1991, petitioners filed a complaint against P&G NOTE:
for regularization, service incentive leave pay and other benefits with 1. Respondent filed MR, which was denied.
damages. 2. In its resolution, the Court upheld its decision declaring SAPS has
Due to the express admission of the petitioners, that they no substantial capital, therefore, labor-only contractor.
consider themselves as an employee of P&G and assailing the integrity
of the Company as legitimate and independent promotion firm, Promm-
Gem terminated their services for disloyalty assailing: serious misconduct 72 NORMAN YABUT, Petitioner, v. MANILA ELECTRIC COMPANY
and breach of trust. AND MANUEL M. LOPEZ, Respondents.

Issue: FACTS:
(1) WON P&G is the employer of petitioners?
This case stems from a complaint for illegal dismissal and Article 282 (a) provides that an employer may terminate an
monetary claims filed by herein petitioner Norman Yabut (Yabut) against employment because of an employee's serious misconduct, a cause that
respondents Manila Electric Company (Meralco) and Meralco officer was present in this case in view of the petitioner's violation of his
Manuel M. Lopez (Lopez). employer's code of conduct. Misconduct is defined as the transgression
The petitioner had worked with Meralco from February 1989 of some established and definite rule of action, a forbidden act, a
until his dismissal from employment on February 5, 2004. Meralco's dereliction of duty, willful in character, and implies wrongful intent and not
Inspection Office issued a memorandum informing it of an illegal service mere error in judgment. For serious misconduct to justify dismissal, the
connection at the petitioner's residence. Given this report, Meralco's following requisites must be present:
Head of Investigation-Litigation Office issued to the petitioner a notice of (a) it must be serious;
investigation. (b) it must relate to the performance of the employee's
Meralcos Litigation Investigation Office summarized the results duties; and
of Meralco's findings in a memorandum which indicated that Yabuts (c) it must show that the employee has become unfit to
electric service was disconnected for account delinquency. continue working for the employer.
Notwithstanding the disconnection and the fact that Meralcos service had The dismissal is also justified as the act imputed upon the
not been reconnected, Yabut's meter registered electric consumption. In petitioner qualifies as fraud or willful breach by the employee of the trust
view of these findings, respondent Meralco, issued a notice of dismissal reposed in him by his employer or duly authorized representative under
addressed to the petitioner. The notice cites violation of Section 7, Article 282 (c) of the Labor Code. While the petitioner contests this
paragraph 3 of Meralco's Company Code on Employee Discipline and ground by denying that his position is one of trust and confidence, it is
Article 282 (a), (c), (d) and (e) of the Labor Code of the Philippines as undisputed that at the time of his dismissal, he was holding a supervisory
bases for the dismissal. position after he rose from the ranks since commencement of his
Aggrieved by the decision of the management, Yabut filed with employment with Meralco. As a supervisor with duty and power that
the National Labor Relations Commission (NLRC) a complaint for illegal included testing of service meters and investigation of violations of
dismissal and money claims against Meralco and Lopez. contract of customers, his position can be treated as one of trust and
Labor Arbiter Antonio R. Macam rendered his Decision, confidence, requiring a high degree of honesty as compared with
declaring the petitioner illegally dismissed from the service and hence, ordinary rank-and-file employees.
entitled to reinstatement plus backwages and attorney's fees. We emphasize that dismissal of a dishonest employee is to the
NLRC rendered its Resolution dismissing the herein best interest not only of the management but also of labor. As a measure
respondents' appeal for lack of merit. of self-protection against acts inimical to its interest, a company has the
CA rendered the now assailed Decision reversing the rulings of right to dismiss its erring employees. An employer cannot be compelled
the NLRC. In finding the petitioner's dismissal lawful, the appellate court to continue employing an employee guilty of acts inimical to the
attributed unto Yabut authorship of the meter tampering and illegal use of employer's interest, justifying loss of confidence in him.
electricity acts which it regarded as serious misconduct. DENIED

ISSUE: 2. WILLFUL DISOBEDIENCE


Whether or not petitioners dismissal is illegal?
73 GOLD CITY INTEGRATED PORT SERVICES, INC. (INPORT) vs.
HELD: NLRC (WILLFUL DISOBEDIENCE; REQUISITES;
Court of Appeals decision is sustained. PROPORTIONALITY & CIRCUMSTANCES DETERMINE PENALTY)
LABOR LAW
Article 279 of the Labor Code of the Philippines provides that FACTS:
(i)n cases of regular employment, the employer shall not terminate the Jose Bacalso was employed as an admeasurer by the Gold
services of an employee except for a just cause or when authorized by City Integrated Port Services, Inc. He was suspected by management of
this Title. x x x The just causes are enumerated in Article 282, which under measuring cargo. One time, the cargo control officer ordered two
provides: (2) other admeasurers to re-measure three (3) pallets of bananas which
Article 282.Termination by employer. - An employer may had already been measured by private respondent. The re-measurement
terminate an employment for any of the following causes: revealed that respondent had under-measured the bananas. Bacalso felt
(a) Serious misconduct or willful disobedience by the insulted by the re-measurement and so the next day he confronted Nigel
employee of the lawful orders of his employer or representative in Mabalacad, one who had re-checked his work. Bacalso quarreled with
connection with his work; Mabalacad in the presence of their immediate superior Gunacao.
(b) Gross and habitual neglect by the employee of his Guanaco directed Bacalso to stop provoking Mabalacad and told both
duties; that being in his office, they should behave properly but a fistfight erupted
(c) Fraud or willful breach by the employee of the trust then and there between him and Mabalacad. Bacalso was then charged
reposed in him by his employer or duly authorized representative; with assaulting a co-employee and falsifying reports and records of the
(d) Commission of a crime or offense by the employee company relative to the performance of his duties, and was preventively
against the person of his employer or any immediate member of his suspended pending investigation of his case by the union-management
family or his duly authorized representative; and grievance committee. Bacalso later received a notice of termination of
(e) Other causes analogous to the foregoing. services upon the grounds of assaulting a co-employee and of
Significantly, tampering with electric meters or metering insubordination for failure to heed Guangco's order to stop
installations of the Company or the installation of any device, with the provoking Mabalacad constituted insubordination or disrespect
purpose of defrauding the Company is classified as an act of dishonesty towards a superior officer punishable by dismissal under the
from Meralco employees, expressly prohibited under company rules. It is Schedule of disciplinary sanctions and norms of conduct, incorporated in
reasonable that its commission is classified as a severe act of the existing Collective Bargaining Agreement ("CBA") with the union.
dishonesty, punishable by dismissal even on its first commission, given
the nature and gravity of the offense and the fact that it is a grave wrong ISSUE:
directed against their employer. Whether private respondent was dismissed for a just cause.
10:00 p.m., Filoteo was informed that there would be no "butchering" of
HELD: tuna that night. Filoteo then sought permission to go home, which was
NO. Wilful disobedience of the employer's lawful orders, as a granted. Filoteo then hurriedly got his things and dashed off to the exit
just cause for the dismissal of an employee, envisages the concurrence gate to catch the service jeep provided by Permex. The next day, Filoteo
of at least two (2) requisites: (1) the employee's assailed conduct must reported for work as usual. He then remembered that he had to make a
have been wilful or intentional, the wilfulness being characterized by a re-entry in his daily time record for the previous day. He proceeded to the
"wrongful and perverse attitude"; and (2) the order violated must have Office of the Personnel Manager to retime his DTR entry. Later, he
been reasonable, lawful, made known to the employee and must pertain received a memorandum from the Assistant Personnel Officer asking him
to the duties which he had been engaged to discharge. Both requisites to explain, in writing, the entry he made in his DTR. Filoteo complied and
are present in the instant case. By private respondent Bacalso's own submitted his written explanation that same evening. Filoteo was
admission, he felt insulted by the re-measurement of the cargo he suspended indefinitely. His explanation was found unsatisfactory.
had already measured. He was apparently much offended by the Thereafter, he was dismissed from employment for allegedly violating
implication he perceived that management was uncertain either about his Article 2 of the company rules and regulations. The offense charged was
honesty or his competence or possibly both. He determined to lose his entering in his DTR that he had worked from 8:45 p.m. of July 31, 1994
temper, became very angry and picked a fight with one of the co-workers to 7:00 a.m. of August 1,1994, when in fact he had worked only up to
who had been instructed by their common superior to carry out the re- 10:00 p.m.
measurement of private respondent's pallets of bananas. In the process,
Bacalso completely disregarded the courtesy and respect due from a ISSUE:
subordinate to his superior. Indeed, he may have been, consciously or Whether or not private respondent was illegally terminated from
otherwise, precisely sending a signal to his superior officer in whose his employment.
presence he provoked and then engaged in physical violence with his co-
worker. Prior to the fistfight, Guangco had warned Bacalso to desist from HELD:
further provoking his co-worker with insulting language. This warning YES. To constitute a valid dismissal from employment, two
constituted an order from private respondent's immediate superior not requisites must concur: (a) the dismissal must be for any of the causes
to breach the peace and order of the Surveyors'(Admeasurers') Division; provided for in Article 282 of the Labor Code; and (b) the employee must
Guangco was obviously attempting to maintain basic employee discipline be afforded an opportunity to be heard and defend himself. This means
in the workplace. It does not follow, however, that private respondent that an employer can terminate the services of an employee for just and
Bacalso's services were lawfully terminated either under Article 282 (a) of valid causes, which must be supported by clear and convincing evidence.
the Labor Code or under the CBA Schedule of penalties. We believe It also means that, procedurally, the employee must be given notice, with
that not every case of insubordination or wilful disobedience by an adequate opportunity to be heard, before he is notified of his actual
employee of a lawful work connected order of the employer or its dismissal for cause. In the present case, the NLRC found that the two-
representative is reasonably penalized with dismissal. For one thing, fold requirements for a valid dismissal were not satisfied by the
Article 282 (a) refers to "serious misconduct or wilful disobedience". petitioners. First, petitioner's charge of serious misconduct of falsification
There must be reasonable proportionality between, on the one hand, the or deliberate misrepresentation was not supported by the evidence on
wilful disobedience by the employee and, on the other hand, the penalty the record. Such dismissal was too harsh a penalty for an unintentional
imposed therefor. Examination of the circumstances surrounding private infraction, not to mention that it was his first offense committed without
respondent's assault upon his co-employee shows that no serious or malice, and committed also by others who were not equally penalized. It
substantial danger had been posed by that fistfight to the well -being of is clear that the alleged false entry in private respondent's DTR was
his other co-employees or of the general public doing business with actually the result of having logged his scheduled time-out in advance on
petitioner employer; and neither did such behavior threaten substantial July 31, 1994. But it appears that when he timed in, he had no idea that
prejudice for the business of his employer. The fistfight occurred inside his work schedule (night shift) would be cancelled. When it was
the offices of the Surveyors' Division, more particularly, Mr. Guangco's confirmed at 10:00 p.m. that there was no "butchering" of tuna to be
office, away from the view of petitioner's customers or of the general done, those who reported for work were allowed to go home, including
public. Considering that private respondent Bacalso's unruly temper did private respondent. In fact, Filoteo even obtained permission to leave
not become an effective threat to his co-workers or the safety of the from the Assistant Production Manager. Considering the factory practice
customers dealing with his employer, or to the goodwill of his employer, which management tolerated, we are persuaded that Filoteo, in his rush
and considering further that he had been quite candid in admitting that he to catch the service vehicle, merely forgot to correct his initial time-out
had been at fault as soon as the investigation began in the company entry. Nothing is shown to prove he deliberately falsified his daily time
level, we agree with the NLRC that termination of his services was a record to deceive the company. The NLRC found that even
disproportionately heavy penalty. We believe that suspension without p management's own evidence reflected that a certain Felix Pelayo, a co-
ay for three (3) months would be an adequate penalty for the assault on worker of private respondent, was also allowed to go home that night and
a co-worker and act of insubordination that private respondent Bacalso like private respondent logged in advance 7:00 a.m. as his time-out. This
actually committed. supports Filoteo's claim that it was common practice among night-shift
workers to log in their usual time-out in advance in the daily time record.

74 PERMEX INC. vs NLRC (JUST & VALID CAUSE; PRIOR NOTICE)


75 ROLANDO APARENTE, SR. vs. NLRC and COCA-COLA
FACTS: BOTTLERS PHILIPPINES, INC.
Permex initially hired Emmanuel Filoteo on October 1, 1990, as (WILLFUL DISOBEDIENCE OF COMPANY’S RULES AND
a mechanic. Eventually, Filoteo was promoted to water treatment REGULATIONS – DRIVING WITHOUT LICENSE)
operator, a position he held until his termination on August 29, 1994. As
water treatment operator, Filoteo did not have a fixed working schedule. FACTS:
His hours of work were dependent upon the company's shifting Rolando Aparante, Sr. was first employed by private
production schedules. One evening he inquire if "butchering" of fish respondent Coca -Cola Bottlers Phils., Inc. (CCBPI), General Santos City
would be done so they could start operating the boiler. They were Plant as assistant mechanic in April 1970. He rose through the ranks to
advised to wait from 9:30 p.m. to 10:00 p.m. for confirmation. At or about eventually hold the position of advertising foreman until his termination
on May 12, 1988 for alleged violation of company rules and regulations. officer. In his written reply Nuez mentioned a personal appointment in
One day, he met an accident wherein he sideswiped a kid diving private justification for his refusal to render "overtime" service and that "ferrying
respondent’s advertising truck. He reported the incident to private employees . . . was not a kind of emergency that . . . warrants (the)
respondent then submitted himself to the police authorities at Polomolok, charge of disobedience." Later, the company terminated Nuez for
South Cotabato for investigation where it was discovered that petitioner insubordination.
had no driver’s license at the time of the accident. In view thereof, an
insurer of private respondent’s vehicles, did not reimburse the latter for ISSUE:
the expenses it incurred in connection with the kid hospitalization. Private WON Nuez was illegally terminated.
respondent conducted an investigation of the incident where petitioner
was given the opportunity to explain his side and to defend himself but HELD:
eventually private respondent dismissed petitioner from employment for NO. It is the discretion of the employer to regulate all aspects
having violated the company rules and regulations particularly Sec. 12 of of employment as well as the corresponding obligation of the workers to
Rule 005-85 for blatant disregard of established control procedures obey company rules and regulations. Deliberate disregard or
resulting in company damages. disobedience of the rules cannot be countenanced and any justification
for the violation is deemed inconsequential. In fact, this is one ground the
ISSUE: Labor Code provides for termination of employment since an employer
Whether or not the infraction committed by petitioner warrants cannot be compelled to continue retaining a worker found guilty of
the penalty of dismissal despite the fact that it was his first offense during maliciously committing acts detrimental to its interests. A contrary rule
his eighteen (18) long years of satisfactory and unblemished service. would render a mockery of the regulations the employees are required to
observe. The existence of an emergency situation is irrelevant to the
HELD: charge of willful disobedience; an opposite principle would allow a worker
YES. Article 282 (a) of the Labor Code of the Philippines to shield himself under his self-designed concept of "nonemergency
sanctions termination by the employer of the employees services for situation" to deliberately defy the directive of the employer. Neither is the
serious misconduct or willful disobedience by the employee of the lawful resulting damage vital. The heart of the charge is the crooked and
orders of his employer or representative in connection with his work. In anarchic attitude of the employee towards his employer. Damage
the instant case, petitioner Aparente was terminated from service after aggravates the charge but its absence does not mitigate nor negate the
having been found guilty of driving without a valid driver’s license, which employee's liability. The fact that a replacement driver was able to
is a clear violation of the company’s rules and regulations. In order that perform the task could neither alter the gravity of the charge, this
an employer may dismiss an employee on the ground of wilful responsibility being personal to the perpetrator. The length of service
disobedience, there must be concurrence of at least two requisites: (1) rendered by the employee is also inconsequential for it does not lessen a
The employees assailed conduct must have been willful or intentional, bit the rebellious temper of the employee object of the charge. We thus
the willfulness being characterized by a wrongful and perverse attitude; find no grave abuse of discretion in the finding of the NLRC that there is
and (2) the order violated must have been reasonable, lawful, made a just ground for the termination of petitioner from the services.
known to the employee and must pertain to the duties which he had been
engaged to discharge. We have found these requisites to be present in
the case at bar. The extant evidence on record clearly reveals the willful 77 LORES REALTY ENTERPRISES, INC., LORENZO Y. SUMULONG
act of petitioner Aparente in driving without a valid driver’s license, a fact III, Petitioners, v. VIRGINIA E. PACIA, Respondent.
that he even tried to conceal during the investigation conducted by (NON PREPARATION OF CHECKS DESPITE ORDERED FOR THE
private respondent. Such misconduct should not be rewarded with re- PROTECTION OF THE COMPANY AGAINST BOUNCING CHECK
employment and backwages, for to do so would wreak havoc on the LAW IS NOT WILLFUL DISOBEDIENCE)
disciplinary rules that employees are required to observe. The law
warrants the dismissal of an employee without making any distinction FACTS:
between a first offender and a habitual delinquent where the totality of Respondent Virginia E. Pacia (Pacia) was hired by Lores
the evidence was sufficient to warrant his dismissal. In protecting the Realty Enterprises, Inc.(LREI). LREI's acting general manager, petitioner
rights of the laborer, the law authorizes neither oppression nor self- Sumulong, on two occasions, directed Pacia to prepare checks as partial
destruction of the employer. payment for LREI's outstanding obligation to the Bank of the Philippine
Islands. Pacia was slow in obeying the order. When asked to explain for
76 FEDERICO NUEZ vs. NLRC, PHILIPPINE OVERSEAS her refusal to immediately follow the directive, Pacia reasoned out that
TELECOMMUNICATIONS CORPORATION the funds in LREI's account were not sufficient to cover the amounts to
(PHILCOMSAT), ET.AL. be indicated in the checks and that she only wanted to protect LREI from
(WILLFUL OBEDIENCE FOR NOT DRIVING DUE TO PERSONAL liability under the Bouncing Checks Law. Pacia received a notice of
APPOINTMENT) termination stating that she was being dismissed because of her willful
disobedience and their loss of trust and confidence in her .Pacia then
FACTS: filed a Complaint for illegal dismissal. The Labor Arbiter (LA) rendered a
Nuez was a driver of PHILCOMSAT assigned to its station in decision finding that the dismissal of Pacia was for a just and valid cause.
Baras, Antipolo, Rizal, from seven-thirty in the morning to three-thirty in On appeal, the NLRC reversed the LA's Decision. The CA affirmed the
the afternoon. At one-thirty that afternoon, Engr. Jeremias Sevilla, the decision of the NLRC. It held that LREI and Sumulong failed to establish
officer in charge and the highest ranking official of the station, asked with substantial evidence that the dismissal of Pacia was for a just cause.
Nuez to drive the employees to the Makati head office to collect their It found that Pacias initial reluctance to obey the orders of her superiors
profit shares. Nuez declined saying that he had an important personal was for a good reason.
appointment right after office hours. At two-thirty that same afternoon, he
also declined a similar order given on the phone by his vehicle ISSUE:
supervisor, Pedro Sibal, reasoning that "Ayaw kong magmaneho dahil Whether or not Pacia was illegally dismissed? Yes!
may bibilhin ako sa Lagundi. Kung gusto mo yong 'loyalist' ang
magmaneho." Nuez was then required to explain why he should not be HELD:
administratively dealt with for disobeying an order of their most senior Court of Appeals decision is affirmed.
LABOR LAW penalized for his previous infractions, this does not and should not
The offense of willful disobedience requires the concurrence of mean that his employment record would be wiped clean of his infractions.
two (2) requisites: After all, the record of an employee is a relevant consideration in
(1) the employee's assailed conduct must have been willful, determining the penalty that should be meted out since an employee's
that is characterized by a wrongful and perverse attitude; and past misconduct and present behavior must be taken together in
(2) the order violated must have been reasonable, lawful, made determining the proper imposable penalty.
known to the employee and must pertain to the duties which he had been But, the employer, is not exempt from observing due process
engaged to discharge. for every infraction. The Supreme Court found the memorandum asking
There is nothing unlawful in the directive of Sumulong to for a written explanation within 24 hours
prepare checks in payment of LREI's obligations. The availability or to be unreasonable.
unavailability of sufficient funds to cover the check is immaterial in the Also, there is no indication that Graphics, Inc. issued a second
physical preparation of the checks. Pacias initial reluctance to prepare notice, informing the petitioner of his dismissal. The respondents admit
the checks, however, which was seemingly an act of disrespect and that Graphics, Inc. decided to terminate the petitioner’s employment after
defiance, was for honest and well intentioned reasons. Protecting LREI he ceased reporting for work from the time he received the memorandum
and Sumulong from liability under the Bouncing Checks Law was requiring him to explain and subsequent to his failure to submit a written
foremost in her mind. It was not wrongful or willful. Neither can it be explanation. However, there is nothing on record showing that Graphics,
considered an obstinate defiance of company authority. The Court takes Inc. placed its decision to dismiss in writing and that a copy thereof was
into consideration that Pacia, despite her initial reluctance, eventually did sent to the petitioner. Dispositive: The petition is DENIED. The Decision
prepare the checks on the same day she was tasked to do it. of the Court of Appeals in CA-G.R. SP No. 106928 is AFFIRMED with
MODIFICATION in that respondent New Age Graphics, Inc. is hereby
78 Realda vs New Age Graphics Inc ordered to pay petitioner Billy M. Realda nominal damages in the amount
(JUST CAUSE WITH FAILURE TO OBSERVE DUE PROCESS = of Thirty Thousand Pesos (P30,000.00) because such dismissal was for
NOMINAL DAMAGES) a just cause but there is a lack of due process.

Facts: 79 Kakampi and Its Members Panuelos vs. Kingspoint Express &
Petitioner Billy Realda was the former machine operator of Logistics
respondent New Age Graphics Inc. (DRIVER’S DISOBEDIENCE TO MANDATORY DRUG TESTING
The company dismissed him on the ground of repeated WITHOUT EXPLAINING THEMSELVES EVEN AFFORDED TO, IS
violations of company’s rules and regulations, namely: insubordination, WILLFUL; FAILURE TO OBSERVE DUE PROCESS = NOMINAL
deliberate slowdown of work, habitual tardiness, absence without official DAMAGES)
leave and inefficiency.
Furthermore, private respondent’s refusal to render overtime Facts:
work when required upon him, contributed to losses incurred by the Petitioners were former drivers of the respondent Kingspoint
petitioner. Express, a sole proprietorship under the name of Co which is engaged in
Nonetheless, while the CA recognized the existence of just the business of transporting goods. They were dismissed from service on
causes for petitioner’s dismissal, it found that the petitioner is entitled to January 20, 2006 on the grounds of serious misconduct, dishonesty, loss
nominal damages due to Graphics, Inc.’s failure to observe the of trust and confidence and commission of acts inimical to the interest of
procedural requirements of due process. Kingspoint Express.
Kingspoint Express issued separate notices to explain to the
Issue: individual petitioners on January 16, 2006 the charges of dishonesty,
Whether or not the petitioner exhibited willful disobedience to a serious misconduct and loss of confidence by filing with the NLRC false,
reasonable order from his employer thus making his dismissal valid malicious and fabricated cases against the company, and their allegedly
unwarranted refusal to undergo drug testing. They were required to
Held: submit their answer to the charges within forty-eight (48) hours from
Yes, the dismissal is valid but there is a lack of due process. receipt of the notices with a warning that failure to do so would mean
waiver of their answer. They were also placed under preventive
Ratio: suspension in the meantime.
In the present case, the company’s business is a printing press Petitioners failed to submit their written explanation within the
whose production schedule is sometimes flexible and varying. It is only stated period. Subsequently, Kingspoint Express issued to them separate
reasonable that workers are sometimes asked to render overtime work in yet uniformly worded notices on January 20, 2006, informing them of
order to meet production deadlines. their dismissal for the abovementioned charges based on the following
The petitioner’s arbitrary defiance to Graphics, Inc.’s order for acts: fabrication of baseless money claims against the company,
him to render overtime work constitutes willful disobedience. misleading fellow co-workers to sign the malicious complaint for money
Security of tenure is guaranteed by the Constitution but it is not claims against the company, refusal to undergo the company's general
an absolute rule and cannot be used as a legal shield by an employee drug test, and extorting money from co-workers to fund activities that
who has exhibited habitual tardiness and absenteeism, and willful they were never fully informed of. Also, petitioner Dacara was dismissed
disobedience. for consummating his sexual relations with Co’s helper inside her
In Merin v. National Labor Relations Commission, this Court residence and thus impregnating the help.
expounded on the principle of totality of infractions as follows: A complaint for illegal dismissal was subsequently filed,
The totality of infractions or the number of violations committed alleging that the charges against them were fabricated and that their
during the period of employment shall be considered in determining the dismissal was prompted by Kingspoint Express' aversion to their union
penalty to be imposed upon an erring employee. The offenses committed activities. The Labor Arbiter ruled in favor of the petitioners as the
by petitioner should not be taken singly and separately. Fitness for charges are purportedly mere unsubstantiated allegations. This was
continued employment cannot be compartmentalized into tight little affirmed by the NLRC on appeal but the latter reversed itself on a
cubicles of aspects of character, conduct and ability separate and subsequent MR filed by Kingspoint. The CA initially reversed the NLRC’s
independent of each other. While it may be true that petitioner was
ruling but on an MR, they too reversed their earlier ruling and favored 1993. There appeared to be a discrepancy in one of the deposits she
Kingspoint. Thus, this petition for certiorari before the SC. made where the amount indicated in the deposit slip and the money
actually received by the bank did not tally. A sum of P50, 000.00 was
Issue: missing and such loss was blamed exclusively on her by the private
WON the dismissal was valid. respondents, after considering the separate reports of the National
Bureau of Investigation and the Diaz Murillo Dalupan Auditing Firm. Prior
Ruling: to her dismissal, petitioner was suspended for a total of 90 days. Two
Yes, the dismissal was valid. It is fundamental that in order to months after, she then filed a complaint for illegal dismissal.
validly dismiss an employee, the employer is required to observe both
substantive and procedural due process — the termination of ISSUE:
employment must be based on a just or authorized cause and the WON petitioner was validly dismissed for gross negligence and
dismissal must be effected after due notice and hearing. for loss of trust and confidence
As to the substantive requirements of due process, the
employees' refusal to submit themselves to drug test is a just cause HELD:
for their dismissal. YES. Bearing in mind that the position of cashier is a highly
An employer may terminate an employment on the ground of sensitive position, requiring as it does the attributes of absolute trust and
serious misconduct or willful disobedience by the employee of the lawful honesty because of the temptations attendant to the daily handling of
orders of his employer or representative in connection with his work. money, petitioner's acts could not help but sow mistrust and loss of
Willful disobedience requires the concurrence of two elements: confidence on the part of respondent employer. The Court agrees with
(1) the employee's assailed conduct must have been willful, the Commission that the resulting breach of trust constitutes a valid
that is, characterized by a wrongful and perverse attitude; and, (2) the cause for the dismissal of petitioner.
order violated must have been reasonable, lawful, made known to the
employee, and must pertain to the duties which he had been engaged to 81 CHALLENGE SOCKS CORPORATION vs. COURT OF APPEALS
discharge. Both elements are present in this case. (HABITUAL ABSENTISM W/O LEAVE CONSTITUTE GROSS
As to the first element, the dismissed employees did not deny NEGLIGENCE)
their refusal to undergo drug testing nor did they explain their refusal.
The utter lack of reason or justification for their insubordination indicates FACTS:
that it was prompted by mere obstinacy, hence, willful and warranting of Elvie Buguat was hired by petitioner Challenge Socks
dismissal. As to the second element, the subject order is relevant in the Corporation as knitting operator. In the course of her employment, she
performance of their functions as drivers of Kingspoint Express. As the incurred absences and tardiness without prior approval and had been
NLRC correctly pointed out, drivers are indispensable to Kingspoint neglectful of her duties. On May 25, 1998, she failed to check the socks
Express' primary business of rendering door-to-door delivery services. It she was working on causing excess use of yarn and damage to the
is common knowledge that the use of dangerous drugs has adverse socks design. She was suspended for five days and warned that a
effects on driving abilities that may render the dismissed employees repetition of the same act would mean dismissal from the service.
incapable of performing their duties to Kingspoint Express and acting On February 2, 1999, she committed the same infraction and was given
against its interests, in addition to the threat they pose to the public. a warning. Despite the previous warnings, Buguat continued to be
The existence of a single just cause is enough to order their habitually absent and inattentive to her task. On March 1, 1999, she
dismissal and it is now inconsequential if the other charges against them again failed to properly count the bundle of socks assigned to her. Thus,
do not merit their dismissal from service. Nonetheless, while Kingspoint on March 2, 1999, petitioner terminated her services on grounds of
Express had reason to sever their employment relations, this Court finds habitual absenteeism without prior leave, tardiness and neglect of work.
its supposed observance of the requirements of procedural due process ISSUE:
pretentious. While Kingspoint Express required the dismissed employees WON Buguat was validly terminated.
to explain their refusal to submit to a drug test, the two (2) days afforded
to them to do so cannot qualify as "reasonable opportunity", which the HELD:
Court construed in King of Kings Transport, Inc. v. Mamacas a period of YES. One of the just causes for terminating an employment
at least five (5) calendar days from receipt of the notice. under Article 282 of the Labor Code is gross and habitual neglect by the
Thus, even if Kingspoint Express' defective attempt to comply employee of her duties. This cause includes gross inefficiency,
with procedural due process does not negate the existence of a just negligence and carelessness. Such just causes is derived from the right
cause for their dismissal, Kingspoint Express is still liable to indemnify of the employer to select and engage his employees. In the instant case,
the dismissed employees, with the exception of Panuelos, Dizon and there is no doubt that Buguat was habitually absent, tardy and neglectful
Dimabayao, who did not appeal the dismissal of their complaints, with of her duties. Habitual neglect implies repeated failure to perform ones
nominal damages in the amount of P30,000.00. duties for a period of time. Buguats repeated acts of absences without
leave and her frequent tardiness reflect her indifferent attitude to and lack
3. GROSS AND HABITUAL NEGLECT OF DUTIES of motivation in her work. Her repeated and habitual infractions,
committed despite several warnings, constitute gross misconduct.
80 EVELYN J. GARCIA vs. NLRC, HOLY TRINITY ACADEMY Habitual absenteeism without leave constitute gross negligence
(DISCREPANCY OF AMOUNT TENDERED) and is sufficient to justify termination of an employee. We find the
penalty of dismissal from the service reasonable and appropriate to
FACTS: Buguats infraction. Her repeated negligence is not tolerable; neither
Petitioner served as school cashier for private respondent Holy should it merit the penalty of suspension only. The record of an employee
Trinity Academy (the school) from June 1974 until her dismissal on is a relevant consideration in determining the penalty that should be
October 5, 1993 for alleged loss of confidence, gross negligence of duty, meted out. Buguat committed several infractions in the past and despite
gross inefficiency and dishonesty. As school cashier, she was the the warnings and suspension, she continued to display a neglectful
custodian of all school funds, including tuition fees, the petty cash and attitude towards her work. An employees’ past misconduct and p resent
canteen cash receipts. In her position paper, she alleged that her behavior must be taken together in determining the proper imposable
termination was brought about by an incident which occurred on June 15, penalty. The totality of infractions or the number of violations committed
during the period of employment shall be considered in determining the the patient and her niece did not press charges against her by reason of
penalty to be imposed upon an errin g employee. The offenses the subject incident.
committed by him should not be taken singly and separately but in their Neglect of duty, to be a ground for dismissal, must be both
totality. Fitness for continued employment cannot be compartmentalized gross and habitual. Gross negligence connotes want of care in the
into tight little cubicles of aspects of character, conduct, and ability performance of one's duties. Habitual neglect implies repeated failure to
separate and in dependent of each other. It is the totality, not the perform one's duties for a period of time, depending upon the
compartmentalization, of such company infractions that Buguat had circumstances. A single or isolated act of negligence does not constitute
consistently committed which justified her dismissal. a just cause for the dismissal of the employee.
Negligence is defined as the failure to exercise the standard of
82 HOSPITAL MANAGEMENT SERVICES, INC.MEDICAL CENTER care that a reasonably prudent person would have exercised in a similar
MANILA, petitioner, vs. HOSPITAL MANAGEMENT SERVICES, situation. The Court emphasizes that the nature of the business of a
INC.MEDICAL CENTER MANILA EMPLOYEES ASSOCIATION-AFW hospital requires a higher degree of caution and exacting standard of
and EDNA R. DE CASTRO, respondents. diligence in patient management and health care as what is involved are
(NEGLIGENCE; REQUISITES – GROSS & HABITUAL) lives of patients who seek urgent medical assistance. An act or omission
that falls short of the required degree of care and diligence amounts to
FACTS: serious misconduct which constitutes a sufficient ground for dismissal.
Respondent De Castro was a staff nurse at Medical Center However, in some cases, the Court had ruled that sanctioning
Manila. Calixijan, HRD Officer of Medical Center Manila issued a notice an erring employee with suspension would suffice as the extreme penalty
of termination upon respondent De Castro for alleged (1) negligence to of dismissal would be too harsh. Considering that this was the first
follow company policy on what to do with patient Rufina Causaren who offense of respondent De Castro in her nine (9) years of employment
fell from a hospital bed; (2) failure to record and refer the incident to the with petitioner hospital as a staff nurse without any previous derogatory
physician-on-duty allowing a significant lapse of time before reporting the record and, further, as her lapse was not characterized by any wrongful
incident; (3) deliberately instructing the staff to follow her version of the motive or deceitful conduct, the Court deems it appropriate that, instead
incident in order to cover up the lapse; and (4) negligence and of the harsh penalty of dismissal, she would be suspended for a period of
carelessness in carrying out her duty as staff nurse-on-duty when the six (6) months without pay, inclusive of the suspension for a period of 14
incident happened. days which she had earlier served. Thereafter, petitioner hospital should
Respondent De Castro, with the assistance of Medical Center reinstate respondent Edna R. De Castro to her former position without
Manila Employees Association-AFW, filed a Complaint for illegal loss of seniority rights, full backwages, inclusive of allowances and other
dismissal against petitioners with prayer for reinstatement and payment benefits, or their monetary equivalent, computed from the expiration of
of full backwages without loss of seniority rights, P20,000.00 moral her suspension of six (6) months up to the time of actual reinstatement.
damages, P10,000.00 exemplary damages, and 10% of the total
monetary award as attorney's fees. 83 NISSAN MOTORS PHILS., INC. vs. VICTORINO ANGELO
The Labor Arbiter ruled in favor of respondent De Castro, G.R. No. 164181, 14 September 2011
stating that although De Castro committed the act complained of, being (FAILURE TO TURNOVER FUNCTIONS TO SOMEONE CAPABLE OF
her first offense, the penalty to be meted should not be dismissal from PERFORMING THE VITAL TASKS EVEN BECAUSE OF A VALID
service, but merely 7 to 14 days suspension as the same was classified CAUSE CONSTITUTES GROSS NEGLECT)
as a less serious offense under the Employee's Handbook.
The NLRC reversed the decision of the Labor Arbiter, stating FACTS:
that respondent De Castro lacked diligence and prudence in carrying out Angelo was employed by Nissan as one of its payroll staff. His
her duty when, instead of personally checking on the condition of patient sick leave and vacation leave resulted in the non-
Causaren after she fell from the bed, she merely sent ward-clerk orientee preparation of the payroll for that particular period.
Guillergan to do the same in her behalf and for influencing her staff to He then received a Memorandum informing him that the compa
conceal the incident. ny is considering his dismissal from employment on the grounds of seriou
The CA reversed the NLRC's ruling and reinstated the Labor s misconduct, willful disobedience and gross neglect of duties. He was th
Arbiter's ruling. en placed on preventive suspension effective immediately. Unsatisfied wi
th his answer, they then issued a Notice of Termination.
ISSUE:
Whether the CA erred in affirming the illegal dismissal of ISSUE:
respondent De Castro? NO! Whether or not petitioner was validly dismissed? YES!

HELD: RULING:
Article 282 (b) of the Labor Code provides that an employer SERIOUS MISCONDUCT - This Court found evidence to support the
may terminate an employment for gross and habitual neglect by the allegation of serious misconduct or insubordination. Petitioner claims that
employee of his duties. The CA ruled that per the Employees Handbook the language used by respondent in his Letter-Explanation is akin to a
of petitioner hospital, respondent De Castros infraction is classified as a manifest refusal to cooperate with company officers, and resorted to
less serious offense for "commission of negligent acts during working conduct which smacks of outright disrespect and willful defiance of
time" as set forth in subparagraph 11, paragraph 3 (B) of Chapter XI[10] authority or insubordination.
thereof. Petitioners anchor respondent De Castros termination of
employment on the ground of serious misconduct for failure to personally WILLFUL DISOBEDIENCE - This allegation of willful disobedience can
attend to patient Causaren who fell from the bed as she was trying to still be adduced and proven from the same Letter-Explanation cited
reach for the bedpan. Based on her evaluation of the situation, earlier. One of the fundamental duties of an employee is to obey all
respondent De Castro saw no necessity to record in the chart of patient reasonable rules, orders and instructions of the employer. Disobedience,
Causaren the fact that she fell from the bed as the patient did not suffer to be a just cause for termination, must be willful or intentional, willfulness
any injury and her vital signs were normal. She surmised that the incident being characterized by a wrongful and perverse mental attitude rendering
was not of a magnitude that would require medical intervention as even the employees act inconsistent with proper subordination. A willful or
intentional disobedience of such rule, order or instruction justifies
dismissal only where such rule, order or instruction is (1) reasonable and Fraud or willful breach by the employee of the trust reposed in him by his
lawful, (2) sufficiently known to the employee, and (3) connected with the employer or duly authorized representative; (d) Commission of a crime or
duties which the employee has been engaged to discharge. offense by the employee against the person of his employer or any
immediate family member of his family or his duly authorized
HABITUAL NEGLIGENCE - the NLRC correctly ruled that the latter's representative; and (e) Other causes analogous to the foregoing. Further,
failure to turn over his functions to someone capable of performing the due process requires that employers follow the procedure set by the
vital tasks which he could not effectively perform or undertake because of Labor Code. Under Art. 277, workers may be dismissed only for a just
his heart ailment or condition constitutes gross neglect. cause and enjoy the right of due process which includes notice and the
ample opportunity to be heard and to defend his or her side.
LETTER: In this case, Padao was dismissed by PNB for gross and
“Again, it's not negligence on my part and I'm not alone to be blamed. It's habitual neglect of duties under Article 282 (b) of the Labor Code. Gross
negligence on your part [Perla Go] and A.A. Del Rosario kasi, noong negligence connotes want of care in the performance of ones duties,
pang April 1999 ay alam ninyo na hindi ako ang dapat may while habitual neglect implies repeated failure to perform ones duties for
responsibilidad ng payroll kundi ang Section Head eh bakit hindi ninyo a period of time, depending on the circumstances. Padao was accused of
pinahawak sa Section Head noon pa. Pati kaming dalawa sa payroll, having presented a fraudulently positive evaluation of the business, credit
kasama ko si Thelma.Tinanggal nyo si Thelma. Hindi nyo ba naisip na standing/rating and financial capability 13 loan applicants.
kailangan dalawa ang tao sa payroll para pag absent ang isa ay may The role that a credit investigator plays in the conduct of a
gagawa. Dapat noon nyo pa naisip iyan. Ang tagal kong gumawa ng banks business cannot be overestimated. The amount of loans to be
trabahong hindi ko naman dapat ginagawa.” extended by a bank depends upon the report of the credit investigator on
the collateral being offered. If a loan is not fairly secured, the bank is at
84 PHILIPPINE NATIONAL BANK, Petitioner, v. DAN PADAO, the mercy of the borrower who may just opt to have the collateral
Respondent. foreclosed. If the scheme is repeated a hundredfold, it may lead to the
(13 FRADULENT POSITIVE EVALUATION OF BUSINESSES BY A collapse of the bank.
CREDIT INVESTIGATOR CONSTITUTES GROSS AND HABITUAL Padao's repeated failure to discharge his duties as a credit
NEGLECT OF DUTIES) investigator of the bank amounted to gross and habitual neglect of duties
under Article 282 (b) of the Labor Code. He not only failed to perform
FACTS: what he was employed to do, but also did so repetitively and habitually,
On August 21, 1981, Padao was hired by PNB as a clerk at its causing millions of pesos in damage to PNB. Thus, PNB acted within the
Dipolog City Branch. He was later designated as a credit investigator in bounds of the law by meting out the penalty of dismissal, which it
an acting capacity on November 9, 1993. He was ultimately promoted to deemed appropriate given the circumstances.
the position of Loan and Credit Officer IV. The CA was correct in stating that when the violation of
In 1994, PNB became embroiled in a scandal involving "behest company policy or breach of company rules and regulations is tolerated
loans" as anomalous loans were being granted by its officers. In line with by management, it cannot serve as a basis for termination. Such ruling,
this, Padao was administratively charged with Dishonesty, Grave however, does not apply here. The principle only applies when the
Misconduct, Gross Neglect of Duty, Conduct Prejudicial to the Best breach or violation is one which neither amounts to nor involves fraud or
Interest of the Service, and violation of R.A. No. 3019 (Anti-Graft and illegal activities. In such a case, one cannot evade liability or culpability
Corrupt Practices Act). The case against Padao was grounded on his based on obedience to the corporate chain of command.
having allegedly presented a deceptively positive status of the business, Padao, in affixing his signature on the fraudulent reports,
credit standing/rating and financial capability of 13 loan applicants. After attested to the falsehoods contained therein. Moreover, by doing so, he
due investigation, PNB found Padao guilty of gross and habitual neglect repeatedly failed to perform his duties as a credit investigator.
of duty and ordered him dismissed from the bank. Padao appealed to the
banks Board of Directors. Velasco, Padaos colleague, was also held 85 MANSION PRINTING CENTER and CLEMENT CHENG,
guilty of the offenses charged, and was similarly meted the penalty of Petitioners, v. DIOSDADO BITARA, JR., Respondent.
dismissal. Her motion for reconsideration, however, was later granted by (GROSS AND HABITUAL ABSENTISM = GROSS AND HABITUAL
the bank, and she was reinstated. NEGLECT OF DUTIES)

ISSUES FACTS:
I. Whether the position of a credit investigator is one imbued Petitioners engaged the services of respondent as a helper
with the trust and confidence of the employer (kargador). Respondent was later promoted as the company’s sole driver
II. Whether the act of falsifying the credit and appraisal reports tasked to pick-up raw materials for the printing business, collect account
and that of affixing ones signature in a false report by another is one and receivables and deliver the products to the clients within the delivery
the same degree of misconduct which warrants the same penalty schedules.
Petitioners aver that the timely delivery of the products to the
HELD: clients is one of the foremost considerations material to the operation of
While it is an employer’s basic right to freely select or the business. It being so, they closely monitored the attendance of
discharge its employees, if only as a measure of self-protection respondent. They noted his habitual tardiness and absenteeism.
against acts inimical to its interest, the law sets the valid grounds Thus, petitioners issued a Memorandum requiring respondent
for termination as well as the proper procedure to be followed when to submit a written explanation why no administrative sanction should be
terminating the services of an employee. imposed on him for his habitual tardiness.
Thus, in cases of regular employment, the employer is Despite respondents undertaking to report on time, however,
prohibited from terminating the services of an employee except for a just he continued to disregard attendance policies.
or authorized cause. Such just causes for which an employer may Consequently, Davis Cheng, General Manager of the company
terminate an employee are enumerated in Article 282 of the Labor Code: and son of petitioner Cheng, issued another Memorandum (Notice to
(a) serious misconduct or willful disobedience by the employee of the Explain) requiring respondent to explain why his services should not be
lawful orders of his employer or representative in connection with his terminated. He personally handed the Notice to Explain to respondent
work; (b) Gross and habitual neglect by the employee of his duties; (c) but the latter, after reading the directive, refused to acknowledge receipt
thereof. He did not submit any explanation and, thereafter, never We cannot simply tolerate injustice to employers if only to
reported for work. protect the welfare of undeserving employees. As aptly put by then
Davis Cheng personally served another Memorandum (Notice Associate Justice Leonardo A. Quisumbing:
of Termination) upon him informing him that the company found him Needless to say, so irresponsible an employee like petitioner
grossly negligent of his duties, for which reason, his services were does not deserve a place in the workplace, and it is within the
terminated. management’s prerogative xxx to terminate his employment. Even as the
On even date, respondent met with the management law is solicitous of the welfare of employees, it must also protect the
requesting for reconsideration of his termination from the service. rights of an employer to exercise what are clearly management
However, after hearing his position, the management decided to prerogatives. As long as the company’s exercise of those rights and
implement the Memorandum. Nevertheless, the management, out of prerogative is in good faith to advance its interest and not for the purpose
generosity, offered respondent financial assistance in the amount of of defeating or circumventing the rights of employees under the laws or
P6,110.00 equivalent to his one month salary. Respondent demanded valid agreements, such exercise will be upheld.
that he be given the amount equivalent to two (2) months salary but the Procedural due process entails compliance with the two-notice
management declined as it believed it would, in effect, reward rule in dismissing an employee, to wit: (1) the employer must inform the
respondent for being negligent of his duties. employee of the specific acts or omissions for which his dismissal is
Respondent filed a complaint for illegal dismissal against the sought; and (2) after the employee has been given the opportunity to be
petitioners before the Labor Arbiter. heard, the employer must inform him of the decision to terminate his
Labor Arbiter dismissed the complaint for lack of merit. employment.
On appeal to the National Labor Relations Commission, the
findings of the Labor Arbiter was AFFIRMED en toto. 4. FRAUD
Before the Court of Appeals, respondent sought the annulment
of the Commissions Resolution on the ground that they were rendered 86 SAN MIGUEL CORPORATION vs. NLRC
with grave abuse of discretion and/or without or in excess of jurisdiction.
The Court of Appeals found for the respondent and reversed FACTS:
the findings of the Commission. The complainants were former security guards of the petitioner
which dismissed them for falsification of their time cards. They made
ISSUE: Whether or not respondent is illegally dismissed? NO! false entries in their time cards showing that they reported for work on
February 19 and 20, 1983 when the truth was that they went on a
HELD: hunting tap to San Juan, Batangas, with their chief Major Martin
In order to validly dismiss an employee, the employer is Asaytuno, then head of the Administrative Services Department of the
required to observe both substantive and procedural aspects the Security Directorate of the petitioner. Besides the falsification of the
termination of employment must be based on a just or authorized cause entries for February 19 and 20, 1983 in their time cards, complainant Mi
of dismissal and the dismissal must be effected after due notice and solas was caught red handed by Security Guard Romeo Martin at 7:45
hearing. A.M. on March 2, 1983 punching in not only his own time card but also
The imputed absence and tardiness of the complainant are the time cards of Delen and Querubin. Seeing Misolas in a tight fix,
documented. He faltered on his attendance 38 times of the 66 working Querubin rushed to the bundy clock and punched in a time card (which
days. His last absences on 11, 13, 14, 15 and 16 March 2000 were turned out to be the card of one Rodrigo de Castro) to save Misolas and
undertaken without even notice/permission from management. These to make it appear to Martin that he (Querubin), punched in his own time
attendance delinquencies may be characterized as habitual and are card. Hence, he was dismissed.
sufficient justifications to terminate the complainant’s employment.
On this score Valiao v. Court of Appealsis instructive: ISSUE: WON complainants were validly terminated.
xxx It bears stressing that petitioners absences and tardiness
were not isolated incidents but manifested a pattern of habituality. xxx HELD:
The totality of infractions or the number of violations committed during YES. The falsification and fraud which the private respondents
the period of employment shall be considered in determining the penalty committed against their employer were inexcusable. Major Asaytuno's
to be imposed upon an erring employee. The offenses committed by him initials on the false entries in their time cards did not purge the
should not be taken singly and separately but in their totality. Fitness for documents of their falsity. Their acts constituted dishonesty and serious
continued employment cannot be compartmentalized into tight little misconduct, lawful grounds for their dismissal under Art. 282, sub-pars.
cubicles of aspects of character, conduct, and ability separate and (a) and (c), of the Labor Code, which provides: ART. 282. Termination by
independent of each other. employer. An employer may terminate an employment for any of the
In Valiao,we defined gross negligence as want of care in the following just causes: (a) Serious misconduct or willful disobedience
performance of ones duties and habitual neglect as repeated failure to by the employee of the lawful orders of his employer or representative in
perform ones duties for a period of time, depending upon the connection with his work. xxx xxx xxx (c) Fraud or willful breach by the
circumstances.51 These are not overly technical terms, which, in the first employee of the trust reposed in him by his employer or duly authorized
place, are expressly sanctioned by the Labor Code of the Philippines, to representative.
wit:
ART. 282.Termination by employer.- An employer may 5. WILLFUL BREACH OF TRUST & LOSS OF CONFIDENCE
terminate an employment for any of the following causes:
(a) xxx 87 NATIONAL SUGAR REFINERIES CORPORATION (NASUREFCO)
(b)Gross and habitual neglect by the employee of his duties; vs. NLRC and SUSAN PABIONA
Xxx (LOSS OF CONFIDENCE)
Clearly, even in the absence of a written company rule defining
gross and habitual neglect of duties, respondents omissions qualify as FACTS:
such warranting his dismissal from the service. Pabiona was appointed as Sugar Accountant-Bookkeeper. She
was tasked to maintain records of all transactions pertaining to the Raw
and Refined Sugar Exchange Program, validate Raw Sugar Quedans
submitted by Exchange participants prior to issuance of the Refined that the Company was terminating his employment for loss of confidence
Sugar Delivery Orders and prepare and issue Refined Sugar Delivery and breach of trust, effective as of the time he was placed under
Orders only after validation procedures have been properly complied preventive suspension.
with. When the books of NASUREFCO were audited in 1990 anomalous
and irregular transactions were uncovered in the Raw Sugar Movement ISSUE:
Report. NASUREFCO found Pabionas written explanation flawed, WON Falguera’s termination was valid.
unsatisfactory. Hence, NASUREFCO charged Pabiona with several
violations of accounting policies. A formal investigation was conducted HELD:
thereafter, NASUREFCO terminated the services of Pabiona for willful YES. It is not disputed that the petitioner is a rank-and-file
violation of company policies, gross and habitual neglect of duties, and employee. Ordinarily, a rank and-file employee is not reposed with a high
willful breach of trust. Complainant herself admits that she may be degree of trust and confidence expected of a supervisory or managerial
negligent yet it was not gross and habitual; that her acts in violating employee. It must, however, be noted that the petitioner served as a
company policies as basis for her dismissal may be viewed by warehouseman and was in charge of the custody, safekeeping, and
respondent as breach of trust, yet the same is not wilful. release of the Company's materials. The nature of his work and the
scope and special character of his duties, therefore, involved utmost trust
ISSUE: and confidence. Among the just causes or valid grounds for termination
WON the dismissal of Pabiona was for a just and valid cause of employment by the employer is "fraud or willful breach by the
employee of the trust reposed in him by his employer or duly authorized
HELD: representative." Ordinary breach will not suffice; it must be willful
YES. The preparation and validation of documents for and without justifiable excuse; there must be basis therefor, and it
purposes of withdrawing refined sugar from NASUREFCO's warehouse must be supported by substantial evidence and not merely by the
involve trust and confidence. It is only through the issuance by Pabiona whims or caprice of the employer. In the instant case, we find no difficulty
of a Refined Sugar Delivery Order that the planters could avail of the in agreeing with the public respondents that the petitioner committed
refined sugar of NASUREFCO. The rule is settled that if the employee willful breach of the trust and confidence reposed in him by the
is guilty of breach of trust or that his employer has justifiable Company.
reason to distrust him, the labor tribunal cannot justly deny the
freedom and authority to dismiss his employee. The basic premise
for dismissal on the ground of loss of confidence is that the employee 89 JAMES BEN L. JERUSALEM, Petitioner, v. KEPPEL MONTE
concerned holds a position of trust and confidence. It is the breach of this BANK, HOE ENG HOCK, SUNNY YAP and JOSEFINA PICART,
trust that results in the employer’s loss of confidence in the employee. Respondents.
Under Art. 282 of the Labor Code, as amended, loss of confidence would (BREACH OF TRUST; REQUISITES)
be the result of fraud or willful breach by the employee of the trust
reposed in him by his employer or duly authorized representative, a just FACTS:
cause for termination. It cannot be gainsaid that the breach of trust must James Ben L. Jerusalem (James) was employed by Keppel
be related to the performance of the employee's functions. The Monte Bank (Keppel) as Assistant Vice-President and was later was
infractions committed by Pabiona were directly within the purview of her assigned as Head of the newly created VISA Credit Card Department.
job description. It was only through her active participation and The bank subsequently re-organized the said Department and reduced it
involvement in the illicit infringement of the companys accounting to a mere unit. Afterwards, James, carrying the same rank, was
procedures that some clients of NASUREFCO were able to withdraw reassigned as Head of the Marketing and Operations of the Jewelry
refined sugar in larger quantities to the prejudice of the latter. Department.
James received from Jorge Javier (Jorge) a sealed envelope
said to be containing VISA Card application forms. Jorge is a Keppel
88 ALEX A. FALGUERA vs. NLRC, PHILIPPINE REFINING CO. (PRC) Visa Card Holder since December 1998. James immediately handed
or UNILEVER-PRC, and JESUS over the envelope with accomplished application forms to the VISA
JAVELONA Credit Card Unit. All in all, the VISA credit card applications referred by
(WAREHOUSEMAN; SUBSTANTIAL EVIDENCE) Jorge which James forwarded to the VISA Credit Card Unit numbered
67, all of which were subsequently approved. As it turned out, all the
FACTS: accounts under these approved applications became past due.
Falguera was an employee of Philippine Refining Co., Inc., now James received a Notice to Explain why no disciplinary action
known as Unilever Philippines (PRC), Inc. One day, the assistant should be taken against him for referring/endorsing fictitious VISA card
soapery engineer of the Company observed an unusual increase in the applicants. He explained that he had no participation in the processing of
reported requisitions by the soapery department of Parker packing the VISA card applications since he was no longer connected with the
materials for the month of June. Upon his examination of the green VISA Credit Card Unit at the time of such transactions. He explained that
copies of the MRs of the soapery department, he discovered that he can only endorse the applications referred by Jorge to the VISA Credit
P27,025.00 worth of Parker packing materials chargeable to his Card Unit because he was already transferred to Jewelry Department, as
department could not be accounted for and were not reflected in the said Head. However, he was issued a Notice of Termination informing him
copies. He therefore sought the original white copies of the MRs from the that he was found guilty of breach of trust and confidence for knowingly
accounting department. A meticulous scrutiny disclosed that while the and maliciously referring, endorsing and vouching for VISA card
original MRs contained entries of the packing items worth P27,050.00, applicants who later turned out to be impostors resulting in financial loss
they, however, showed alterations, super impositions, and erasures. to Keppel. Thus, the filing of a complaint foe illegal dismissal.
Pending investigation, the petitioner was placed under preventive The LA found Keppel guilty of illegal dismissal and ordered the
suspension pending the investigation of the anomaly. Upon investigation, payment of backwages from the time of his illegal termination and in lieu
it found that eight original MRs were tampered by two engineering of reinstatement, a separation pay should be given. The NLRC affirmed
storemen, in particular, Falguera and Felipe Viado. The latter admitted the LA’s decision. However, the CA reversed and set aside the said
having tampered one MR and offered to testify against the petitioner. decision and dismissed the complaint.
Thereafter, Falguera received a letter signed by Javelona informing him
lawyer (Atty. Edmundo V. Buensuceso) and a military man (one Col.
ISSUE: Flordeliza). After the meeting, the bank found no reason to reconsider
Whether Keppel legally terminated James’s employment on the and reiterated its decision to dismiss Lopez.[8]
ground of willful breach of trust and confidence? NO! Lopez filed a complaint for illegal dismissal and money claims
against the bank, Bosano and Tong.
HELD:
Article 282(c) of the Labor Code prescribes two separate and The Compulsory Arbitration Proceedings
distinct grounds for termination of employment, namely: (1) fraud; or (2) Lopez alleged before the labor arbiter that he issued the POs
willful breach by the employee of the trust reposed in him by his as part of his strategy to enhance the bank's business, in line with his
employer or duly authorized representative. In order to constitute a just duty as branch manager to promote the growth of the bank. He claimed
cause for dismissal, the act complained of must be ‘work-related’ such that the bank honored the first PO for P1.8M from which the bank derived
as would show the employee concerned to be unfit to continue working an income of P142,000.00. He added that the second PO did not
for the employer. materialize because Mr. James Puyat Concepcion, a Hertz incorporator
There are 2 requisites for dismissal on the ground of loss of and director who opened the Hertz account, stopped depositing with the
trust and confidence, these are: 1) employee concerned must be holding bank because of the negative credit rating he received from the bank's
a position of trust and confidence; and 2) there must be an act that would credit committee. Allegedly, the committee discovered that James Puyat
justify the loss of trust and confidence. In the case at hand, the first Concepcion had several pending court cases.
requisite is present for James held a position of trust and confidence as For its part, the bank denied approving the first PO, arguing
Assistant Vice-President of the Jewelry Department. As for the second that Lopez did not have the authority to issue the POs for the Hertz
requisite, the court ruled that Keppel’s evidence against James fails to account as there was a standing advice that no Hertz loan application
clearly and convincingly establish a willful breach of trust. was to be approved. It stressed that Lopez committed a serious violation
From the findings of both the Labor Arbiter and the NLRC it is of company rules when he issued the POs.
clear that James did nothing wrong when he handed over to Marciana, In a decision dated April 28, 2004,[9] Labor Arbiter Cesar D.
the unit head, the envelope containing the applications of persons under Sideño ruled that Lopez was illegally dismissed. Accordingly, the labor
the referred accounts of Jorge who were later found to be fictitious. As arbiter ordered Lopez's immediate reinstatement, and awarded him
the records now stand, James was no longer connected with the VISA backwages of P392,000.00, moral and exemplary damages of P8M, and
Credit Card Unit when the 67 applications for VISA card were approved. P550,000.00 -- the purchase price of a Toyota Revo which Lopez
At such time, he was already the Head of the Marketing and Operations allegedly brought over from his stint with Global Bank (now Metrobank).
of the Jewelry Department. His act therefore of forwarding the already The labor arbiter found that contrary to the bank's claim, the evidence
accomplished applications to the VISA Credit Card Unit is proper as he is showed that Lopez had been issuing POs which the bank had paid,
not in any position to act on them. The processing and verification of the including the first of the two POs that led to his dismissal.[10]
identities of the applicants would have been done by the proper On appeal by the bank, the National Labor Relations
department, which is the VISA Credit Card Unit. Therefore, it is Commission (NLRC) rendered a decision on October 11,
incumbent upon Marciana as Unit Head to have performed her duties. As 2005[11] reversing the labor arbiter's ruling. It dismissed the complaint for
correctly observed by the Labor Arbiter, Keppel had gone too far in lack of merit. The NLRC found merit in the bank's submission that by
blaming James for the shortcomings and imprudence of Marciana. The issuing the questioned POs without authority and against the bank's
invocation of Keppel of the loss of trust and confidence as ground for express orders, Lopez thereby committed a willful disobedience against
James’s termination has therefore no basis at all. Thus, the Court his superiors -- a sufficient basis for the bank to lose its trust and
declared that such dismissal based on the ground of loss of trust and confidence in him as branch manager. It thus found that Lopez had been
confidence was illegal. dismissed for cause after the observance of due process. Lopez moved
for reconsideration, but the NLRC denied the motion in its resolution of
90 Elmer Lopez vs Keppel Bank Philippines, Inc. January 25, 2006.[12] Lopez sought relief from the CA through a petition
(BRANCH MANAGER OF BANK ISSUING CREDIT DESPITE for certiorari, charging the NLRC with grave abuse of discretion for
EXPRESSLY DISALLOWED) setting aside the labor arbiter's decision.

The Antecedents The CA Decision


The facts, as set out in the assailed CA decision, are On December 19, 2006, the CA rendered its now assailed
summarized below. decision,[13]denying the petition and affirming the October 11, 2005
Petitioner Elmer Lopez was the Branch Manager of the decision of the NLRC. It fully agreed with the NLRC finding that Lopez
respondent Keppel Bank Philippines, Inc. (bank) in Iloilo City. Allegedly, had not been illegally dismissed.
through his efforts, Hertz Exclusive Cars, Inc. (Hertz) became a client of Lopez moved for, but failed to obtain, a reconsideration of the
the bank. CA decision. The CA denied the motion on February 7, 2007.[14]
By notice dated August 12, 2003,[4] the bank asked Lopez to
explain in writing why he should not be disciplined for issuing, without The Case for Lopez
authority, two purchase orders (POs) for the Hertz account amounting to Through the present petition,[15] the reply to the bank's
a total of P6,493,000.00, representing the purchase price of 13 Suzuki comment dated February 11, 2008,[16] and the memorandum dated
Bravo and two Nissan Exalta vehicles. September 22, 2008,[17] Lopez entreats the Court to nullify the CA
Lopez submitted his written explanation on the same day,[5] but decision, contending that the CA erred in: (1) not ruling that the bank's
the bank refused to give it credit. Through respondents Manuel Bosano appeal with the NLRC should have been dismissed on the ground of non-
III (Vice-President and Head of Retail Banking Division/Consumer perfection; and (2) affirming the decision of the NLRC that he was
Banking Division) and Stefan Tong Wai Mun (Vice- dismissed for a just cause (loss of trust and confidence) and that he was
President/Comptroller), the bank terminated Lopez's employment afforded due process.
effective immediately.[6] Lopez argues, with respect to the first assignment of error, that
Lopez asked the bank for reconsideration.[7] In response, the the bank failed to comply with Sections 4 and 6, Rule VI, of the 2002
bank, through the respondent officers, met with Lopez at its headquarters Rules of Procedure of the NLRC.[18] He points out that the bank did not
in Cubao, Quezon City on September 25, 2003. Lopez came with his file a notice of appeal together with its memorandum of appeal, which in
turn was not supported by a certificate of non-forum shopping; and bank to put on hold the loan application of Hertz until the adverse finding
neither did the bank furnish him, as appellee, a certified copy of the could be cleared. It insists that Lopez willfully and knowingly disobeyed
appeal bond. this order.
On the substantive aspect of the case, Lopez posits that the Further, the bank questions Lopez's submission, through a
bank failed to justify his dismissal on the ground of loss of trust and supplemental addendum to his position paper, of evidence that it
confidence. He insists that, as branch manager, he had the authority to honored and paid POs issued by Lopez in the past. It maintains that it
issue POs as in fact he issued several of them in the past, which POs was not furnished a copy of this submission; hence, it was unable to
were honored and paid by the bank. The labor arbiter properly relied on controvert this evidence.
the past transactions in his decision. These included, he reiterates, the On the procedural due process issue, the bank denies Lopez's
first PO for the Hertz account which was paid by the bank on July 18, allegation that he was not given the opportunity to defend himself. It
2003, a transaction where the bank even earned a substantial income points out that both the NLRC and the CA confirmed that Lopez was not
(P142,000.00). He maintains that the bank failed to substantiate its deprived the opportunity to be heard; the opportunity commenced with:
position that he was not authorized to issue the POs. He adds that the (1) the notice for him to explain his side regarding his unauthorized
bank's claim that his issuance of the POs exposed the bank to financial issuance of POs; (2) the notice of his termination from employment; and
loss is a lame excuse to justify the termination of his employment. (3) the hearing called in response to his motion for reconsideration where
Lopez argues that his dismissal was a mere afterthought on the he was assisted by his lawyer and his soldier friend.
part of the bank management, particularly Bosano, to cover up its
embarrassment when he (Lopez) made inquiries and discovered that The Court's Ruling
Hertz's James Puyat Concepcion had no pending court cases and was
therefore credit worthy. He adds that assuming that he did not have the The procedural issue
authority to issue POs, still, he cannot be held guilty of willful Lopez faults the CA for not ruling that the bank's appeal to the
disobedience; even if he had been guilty, dismissal was a very harsh NLRC should have been dismissed for non-perfection. He argues that no
penalty. notice of appeal accompanied the memorandum of appeal; neither was
Finally, Lopez submits that the bank failed to accord him due there a certificate of non-forum shopping nor any copy furnished to him of
process because the bank did not give him the opportunity to prepare for the certified true copy of the appeal bond.
his defense. He points out that his written explanation (dated August 12, The procedural question is a non-issue. Lopez did not raise it
2003)[19] preceded the bank's letter (of the same date)[20] that required before the CA; in fact, he challenged the NLRC decision of October 11,
him to explain why he issued the POs in question. Lopez contends in this 2005[23] on its merits and not on its form. We, therefore, see no need to
regard that on August 12, 2003, he went to Bosano's office in Quezon further discuss this argument.
City all the way from Iloilo City and there, he was cornered by Bosano
who verbally instructed him to immediately write down his explanation The merits of the case
even before he was served with the bank's August 12, 2003 letter. He On the substantive aspect of the case, we note that Lopez was
maintains that Bosano's preemptive move deprived him of the dismissed from the service by reason of loss of trust and confidence, a
opportunity to secure the services of a counsel. just cause for an employee's dismissal under the law.[24] Lopez insists
While Lopez believes his dismissal to be illegal, he does not though that the act which triggered the dismissal action does not justify
seek reinstatement due to the antagonism that has developed between his separation from the service.
him, and the bank and its officers, due to the present case. He only asks
for separation pay of one month pay for every year of service, full Is Lopez liable for loss of trust and confidence for issuing the two
backwages, allowances and other benefits. Additionally, he prays for disputed POs?
moral and exemplary damages, as well as attorney's fees, to The right of an employer to freely select or discharge his
compensate him for a dismissal that was attended by bad faith and employee is a recognized prerogative of management; an employer
effected in a wanton, oppressive and malevolent manner. cannot be compelled to continue employing one who has been guilty of
acts inimical to its interests. When this happens, the employer can
The Case for the Bank and its Officers, dismiss the employee for loss of confidence.[25]
Through its comment to the petition[21] and At the same time, loss of confidence as a just cause of
memorandum,[22] the bank submits that the CA committed no reversible dismissal was never intended to provide employers with a blank check
error in denying Lopez's petition for certiorari, and in affirming the ruling for terminating employment. Loss of confidence should ideally apply only
of the NLRC that Lopez was dismissed for a just cause and after due (1) to cases involving employees occupying positions of trust and
process. confidence, or (2) to situations where the employee is routinely charged
The bank is puzzled why Lopez is standing firm on his position with the care and custody of the employer's money or property. To the
that he did nothing wrong when he issued the questioned POs despite first class belong managerial employees, i.e., those vested with the
the express directive not to proceed with the Hertz loan application powers and prerogatives to lay down management polices and/or to hire,
unless its adverse credit investigation report is explained to the bank's transfer, suspend, lay-off, recall, discharge, assign or discipline
credit committee. It posits that no bank would gamble to maintain as employees, or effectively recommend such managerial actions. To the
branch manager a person who dares to supplant a major decision of the second class belong cashiers, auditors, property custodians, or those
bank's top leadership with his personal decision. It argues that in this who, in the normal and routine exercise of their functions, regularly
situation, the law (Labor Code) provides protection to the employer handle significant amounts of money or property.[26]
through its management prerogative rights and the right to dismiss As branch manager, Lopez clearly occupies a "position of
employees on just and valid grounds. trust." His hold on his position and his stay in the service depend on the
The bank refutes Lopez's contention that there was no willful employer's trust and confidence in him and on his managerial
disobedience that warranted his dismissal. It points out that there was an services.[27] According to the bank, Lopez betrayed this trust and
order for him not to proceed with the Hertz loan application. The order confidence when he issued the subject POs without authority and despite
was very reasonable as it is the standard policy of every bank to conduct the express directive to put the client's application on hold. In response,
an investigation on the credit worthiness of any loan applicant. Since it Lopez insists that he had sufficient authority to act as he did, as this
appeared from the investigation of its credit committee that James Puyat authority is inherent in his position as bank manager. He points to his
Concepcion of Hertz had various court cases, it was only proper for the record in the past when he issued POs which were honored and paid by
the bank and which constituted the arbiter's "overwhelming heard; when he moved for reconsideration of the bank's decision to
evidence"[28] in support of the finding that "complainant's dismissal from terminate his employment, it scheduled a hearing where he appeared
work was without just cause, hence, illegal."[29] together with his lawyer and a military man. This was an opportunity to
We disagree with Lopez's contention. Despite evidence of his be heard that the law recognizes.
past exercise of authority (as found by the labor arbiter), we cannot In fine, we find no merit in the petition.
disregard evidence showing that in August 2003, the bank specifically WHEREFORE, premises considered, we hereby DENY the
instructed Lopez not to proceed with the Hertz loan application because petition for lack of merit. The assailed decision and resolution of the
of the negative credit rating issued by the bank's credit committee. We Court of Appeals are AFFIRMED. Costs against petitioner Elmer Lopez.
find it undisputed that Lopez processed the loan despite the adverse SO ORDERED.
credit rating. In fact, he admitted that he overlooked the "control aspects"
of the transaction as far as the bank was concerned because of his
eagerness to get a bigger share of the market.[30] 91 Jumuad vs. Hi-Flyer Food, G.R. No. 187887, September 7, 2011
Lopez's good intentions, assuming them to be true, are beside (RESTAURANT; RESPONDEAT SUPERIOR)
the point for, ultimately, what comes out is his defiance of a direct order
of the bank on a matter of business judgment. He went over the heads of Facts:
the bank officers, including the credit committee, when, based on Petitioner Pamela Florentina P. Jumuad began her
inquiries he made on his own regarding the credit worthiness of James employment with respondent Hi-Flyer Food, Inc. as management trainee.
Puyat Concepcion, he simply proceeded to act on the basis of his own Based on her performance through the years, Jumuad received several
judgment. Evident in his written explanation[31] was his failure to inform promotions until she became the area manager for the entire Visayas-
the credit committee of his own efforts to check on the committee's Mindanao 1 region. Sometime on October 2004, Hi-Flyer conducted a
adverse findings against Hertz and his independent action based solely food safety, service and sanitation audit and revealed several sanitation
on his own authority. violations, such as the presence of rodents and the use of a defective
As a bank official, the petitioner must have been aware that it is chiller for the storage of food. When asked to explain, Jumuad first
basic in every sound management that people under one's supervision pointed out that she had already taken steps to prevent the further
and direction are bound to follow instructions or to inform their superior of infestation of the branch. As to why the branch became infested with
what is going on in their respective areas of concern, especially rodents, Jumuad faulted management's decision to terminate the
regarding matters of vital interest to the enterprise. Under these facts, we services of the branch's pest control program and to rely solely on the
find it undisputed that Lopez disobeyed the bank's directive to put the pest control program of the mall.
Hertz loan application on hold, and did not wait until its negative credit Hi-Flyer audited the account of one of its branches and found
rating was cleared before proceeding to act. That he might have been out irregularities of cash shortages. Another sanitation audit was made
proven right is immaterial. Neither does the submission that the bank and signs of rodent gnawing/infestation were found. This time, Jumuad
honored and paid the first PO and even realized a profit from the explained to management that she had been busy conducting
transaction, mitigate the gravity of Lopez's defiance of the directive of management team meetings and that, at the date the audit was
higher authority on a business judgment. What appears clear is that the conducted, she had no scheduled visit. Hi-Flyer sent Jumuad an
bank cannot in the future trust the petitioner as a manager who would Irregularities Report and Notice of Charges. Jumuad submitted her
follow directives from higher authorities on business policy and written explanation. Hi-Flyer held an administrative hearing where
directions. The bank can be placed at risk if this kind of managerial Jumuad appeared with counsel. Apparently not satisfied with her
attitude will be repeated, especially if it becomes an accepted rule among explanations, Hi-Flyer served her a Notice of Dismissal. This prompted
lower managers. Jumuad to file a complaint against Hi-Flyer for illegaldismissal.
In Nokom v. NLRC,[32] we reiterated the guidelines for the
application of loss of confidence as follows: (1) loss of confidence, should Issue:
not be simulated; (2) it should not be used as a subterfuge for causes Whether Jumuad was illegally dismissed
which are improper, illegal or unjustified; (3) it may not be arbitrarily
asserted in the face of overwhelming evidence to the contrary; and (4) it HELD:
must be genuine, not a mere afterthought to justify an earlier action taken The Court is convinced that Jumuad cannot be dismissed on
in bad faith. the ground of gross and habitual neglect of duty. The Court notes the
Under the circumstances of this case, we are convinced that apparent neglect of Jumuad of her duty in ensuring that her subordinates
the bank was justified in terminating Lopez's employment by reason of were properly monitored and that she had dutifully done all that was
loss of trust and confidence. He admitted issuing the two POs, claiming expected of her to ensure the safety of the consuming public. The nature
merely that he had the requisite authority. He could not present any proof of the anomalies uncovered were each of a different nature, the Court
in this regard, however, except to say that it was part of his inherent duty finds that her acts or lack of action in the performance of her duties is not
as bank manager. He also claimed that the bank acquiesced to the born of habit.
issuance of the POs as it paid the first PO and the POs he issued in the Despite saying this, it cannot be denied that Jumuad willfully
past. This submission flies in the face of the bank's directive for him not breached her duties as to be unworthy of the trust and confidence of Hi-
to proceed unless matters are cleared with the bank's credit committee. Flyer.
The bank had a genuine concern over the issue as it found through its Based on established facts, the mere existence of the grounds
credit committee that Hertz was a credit risk. Whether the credit for the loss of trust and confidence justifies petitioner's dismissal. In the
committee was correct or not is immaterial as the bank's direct order left present case, the reports of Hi-Flyer show that there were anomalies
Lopez without any authority to clear the loan application on his own. After committed in the branches managed by Jumuad. On the principle of
this defiance, we cannot blame the bank for losing its confidence in respondeat superior or command responsibility alone, Jumuad may be
Lopez and in separating him from the service. held liable for negligence in the performance of her managerial duties.
She may not have been directly involved in causing the cash shortages
The due process issue but her involvement in not performing her duty monitoring and supporting
As the NLRC and the CA did, we find Lopez to have been the day to day operations of the branches and ensure that all the facilities
afforded due process when he was dismissed. He was given the required and equipment at the restaurant were properly maintained and serviced,
notices. More importantly, he was actually given the opportunity to be could have truly prevented the whole debacle from ever occurring.
Moreover, it is observed that rather than taking proactive steps to prevent work for the employer.[50] In the instant case, the petitioners-employees
the anomalies at her branches, Jumuad merely effected remedial of Promm-Gem have not been shown to be occupying positions of
measures. In the restaurant business where the health and well-being of responsibility or of trust and confidence. Neither is there any evidence to
the consuming public is at stake, this does not suffice. Thus, there is show that they are unfit to continue to work as merchandisers for Promm-
reasonable basis for Hi-Flyer to withdraw its trust in her and dismissing Gem.
her from its service.
All told, we find no valid cause for the dismissal of petitioners-employees
of Promm-Gem.
92 JOEB M. ALIVIADO, et al., Petitioners, v. PROCTER & GAMBLE
PHILS., INC., and PROMM-GEM INC., Respondents.
(EMPLOYEES MUST HOLD A POSITION/DESIGNATION OF TRUST)
93 Grand Asian Shipping Lines v. Galvez
(BREACH OF TRUST; MANAGERIAL VS. RANK AND FILE)
FACTS:
Petitioners worked as merchandisers of P&G from various
Facts:
dates, allegedly starting as early as 1982 or as late as June 1991, to
 Petitioner Grand Asian Shipping Lines, Inc (GASLI) is a
either May 5, 1992orMarch 11, 1993.
domestic corporation engaged in transporting liquified petroleum gas
They all individually signed employment contracts with either
(LPG) from Petron’s refinery in Bataan to Pasig and Cavite.
Promm-Gem or SAPS for periods of more or less five months at a
time.They were assigned at different outlets, supermarkets and stores  Respondents are crewmembers of one of GASLI’s vessels,
where they handled all the products of P&G.They received their wages M/T Dorothy Uno.
from Promm-Gem or SAPS.  On January 2000, Richard Abis (vessel’s oiler) reported to
SAPS and Promm-Gem imposed disciplinary measures on GASLI an alleged illegal activity being committed by respondent who
erring merchandisers for reasons such as habitual absenteeism, would misdeclare the consume fuel in the Engineer’s Voyage Reports
dishonesty or changing day-off without prior notice. and the save fuel oil were sold to other vessel out at sea (at nighttime).
P&G is principally engaged in the manufacture and production Profits would be divided amongst themselves.
of different consumer and health products, which it sells on a wholesale  After investigation, from the period of June 30, 1999 to Feb
basis to various supermarkets and distributors. To enhance consumer 15, 2000 the fuel it consumption was overrate by 6,954.3 liters amounting
awareness and acceptance of the products, P&G entered into contracts to 74,737.86.
with Promm-Gem and SAPS for the promotion and merchandising of its  Acting upon the anomaly, GASLI placed respondents under
products. preventive suspension and after conducting administrative hearings
In December 1991, petitioners filed a complaint against P&G decided to terminate them for breach of trust, commission of crime
for regularization, service incentive leave pay and other benefits with against employer.
damages.The complaint was later amendedto include the matter of their  Respondents filed with the NLRC separate complaint for
subsequent dismissal. illegal suspension and dismissal, underpayment/nonpayment of
On November 29, 1996, the Labor Arbiter dismissed the salaries/wages, overtime pay, premium pay for holiday and rest day,
complaint for lack of merit and ruled that there was no employer- service incentive pay, tax refunds and indemnities for damages and
employee relationship between petitioners and P&G.He found that the attorney’s fees against petitioner.
selection and engagement of the petitioners, the payment of their wages,  On August 30, 2001, the Labor Arbiter rendered decision
the power of dismissal and control with respect to the means and finding the dismissal of 21 complainants to be illegal.
methods by which their work was accomplished, were all done and  Petitioner then filed a Notice of Appeal with Motion to
exercised by Promm-Gem/SAPS.He further found that Promm-Gem and Reduce Bond before the NLRC citing economic depression, legality of
SAPS were legitimate independent job contractors. On appeal, the NLRC termination, and compliance with labor standards. NLRC denied
dismissed the same. Petitioners filed a motion for reconsideration but the petitioner’s motion to reduce bond and directed an additional bond.
motion was denied in theNovember 19, 1998Resolution.  Despite petitioner’s failure the pay the bond, NLRC found the
Petitioners likewise failed to have a favrable decision in the CA appeal meritorious and ruled for petitioners. Stating that the dismissal
hence, this petition. was valid with the exception of Sales.
 NLRC struck down the monetary awards given by the Labor
ISSUE: Arbiter as they were based on computations made by respondents.
Whether or not Petitioners are validly dismissed for wilful  On appeal to the CA, the court ruled in favor of respondent
breach of trust? stating that the NLRC’s decision had jurisdictional error since petitioner
did not comply with the additional bond.
HELD:
NO! Loss of trust and confidence, as a ground for dismissal, must be LABOR LAW
based on the willful breach of the trust reposed in the employee by his Issue:
employer. Ordinary breach will not suffice. A breach of trust is willful if it I. WON the CA erred in holding that respondents were illegally
is done intentionally, knowingly and purposely, without justifiable excuse, dismissed on the ground of breach of trust?
as distinguished from an act done carelessly, thoughtlessly, heedlessly
or inadvertently.[49] Held/Ratio:
Respondent’s termination due to loss of trust and confidence
Loss of trust and confidence, as a cause for termination of employment,
should have a distinction between managerial and rank and file
is premised on the fact that the employee concerned holds a position of
employees. Rank-and-file employees require proof of involvement while
responsibility or of trust and confidence. As such, he must be invested
managerial employees mere existence of a basis for belief is sufficient.
with confidence on delicate matters, such as custody, handling or care
Given that Galvez and Gruta have managerial positions there is some
and protection of the property and assets of the employer. And, in order
basis for the loss of employer’s confidence—regarding the overstatement
to constitute a just cause for dismissal, the act complained of must be
of fuel consumption without any evidence to the contrary. While the
work-related and must show that the employee is unfit to continue to
others, who are ordinary rank and file employees, were not proven to
have any involvement in the loss of the vessel’s fuel. Rendering their Subsequently petitioner filed a case for illegal dismissal against
dismissals illegal. The employer bears the burden of proof in illegal respondent and two days later respondent filed a criminal case for
dismissal cases thus the employer must first establish by qualified theft against petitioner. The petitioner insisted on her innocence,
substantial evidence the fact of dismissal. reiterating that on the time the alleged crime took place she, together
In the case before us, Galvez, as the ship captain, is with her two salesgirls, had first counted the cash before placing it in a
considered a managerial employee since his duties involve the plastic bag that she deposited inside the drawer of the cabinet with the
governance, care and management of the vessel.56 Gruta, as chief knowledge of the other salesgirls. One of the salesgirls however averred
engineer, is also a managerial employee for he is tasked to take that she had left the petitioner alone because the latter had still to change
complete charge of the technical operations of the vessel.57 As captain her clothes; and that that was the first time that the petitioner had ever
and as chief engineer, Galvez and Gruta perform functions vested with asked to be left behind, for they had previously left the kiosk together.
authority to execute management policies and thereby hold positions of Respondent Vina declared that the petitioner did not call the office of
responsibility over the activities in the vessel. Indeed, their position Minex for the pick-up of the P39,194.50 cash sales on that faithful day in
requires the full trust and confidence of their employer for they are violation of the standard operating procedure (SOP) requiring cash
entrusted with the custody, handling and care of company property and proceeds exceeding P10,000.00 to be reported for pick-up if the amount
exercise authority over it. could not be deposited in the bank. After the preliminary investigation,
Thus, we find that there is some basis for the loss of the fiscal rendered a resolution finding probable cause for qualified theft
confidence reposed on Galvez and Gruta. The certification issued by De and recommending the filing of an information against the petitioner.
la Rama stated that there is an overstatement of fuel Thus, she was charged with qualified theft before the Regional Trial
consumption. Notably, while respondents made self–serving allegations Court. The petitioner argued that there was no evidence at all upon which
that the computation made therein is erroneous, they never questioned Minex could validly dismiss her considering that she had not yet been
the competence of De la Rama to make such certification. Neither did found guilty beyond reasonable doubt of the crime of qualified theft.
they question the authenticity and validity of the certification. Thus, the
fact that there was an overstatement of fuel consumption and that there Issues:
was loss of a considerable amount of diesel fuel oil remained Whether or not there was valid ground to terminate the
unrefuted. Their failure to account for this loss of company property petitioner.
betrays the trust reposed and expected of them. They had violated
petitioners’ trust and for which their dismissal is justified on the ground of Ruling:
breach of confidence. The petitioner’s argument is not novel. It has been raised and
As for Arguelles, Batayola, Fresnillo, Noble, Dominico, rejected many times before on the basis that neither conviction beyond
Nilmao and Austral, proof of involvement in the loss of the vessel’s fuel reasonable doubt for a crime against the employer nor acquittal after
as well as their participation in the alleged theft is required for they are criminal prosecution was indispensable. Nor was a formal charge in court
ordinary rank and file employees. And as discussed above, no for the acts prejudicial to the interest of the employer a pre-requisite for a
substantial evidence exists in the records that would establish their valid dismissal. The criminal charges initiated by the company against
participation in the offense charged. This renders their dismissal illegal, private respondents and the finding after preliminary investigation of their
thus, entitling them to reinstatement plus full backwages, inclusive of prima facie guilt of the offense charged constitute substantial evidence
allowances and other benefits, computed from the time of their dismissal sufficient to warrant a finding by the Labor Tribunal of the existence of a
up to the time of actual reinstatement. just cause for their termination based on loss of trust and confidence.
The Labor Tribunal need not have gone further as to require
94 Concepcion vs. Minex Import Corp., G.R. No. 153569, January 24, private respondent’s conviction of the crime charged, or inferred
2012 innocence on their part from their release from detention, which was
(CONVICTION OR ACQUITAL IS NOT A NECESSARY ELEMENT FOR mainly due to their posting of bail.
DISMISSAL BASED ON BREACH OF TRUST) While there is a valid ground to terminate petitioner,
respondent however failed to comply with the requirements of due
Facts: process prior to the termination under the implementing rules and
Respondent is engaged in the retail of semi-precious stones, regulations of the Labor Code.
selling them in kiosks or stalls installed in various shopping centers. It In all cases of termination of employment, the following
employed the petitioner initially as a salesgirl then later on as supervisor. standards of due process shall be substantially observed. For termination
Working under her supervision were salesgirls Cristina Calung and Lida of employment based on just causes as defined in Article 282 of the
Baquilar. Labor Code:
One day the petitioner and her salesgirls had sales of crystal (i) A written notice served on the employee specifying the
items totaling P39,194.50. At the close of business that day, they ground or grounds for termination, and giving said employee reasonable
conducted a cashcount of their sales proceeds, including those from the opportunity within which to explain his side.
previous two days and determined their total for the three days to be (ii) A hearing or conference during which the employee
P50,912.00. The petitioner wrapped the amount in a plastic bag and concerned, with the assistance of counsel if he so desires is given
deposited it in the drawer of the locked wooden cabinet of the kiosk. opportunity to respond to the charge, present his evidence, or rebut the
The following day petitioner phoned respondent Vina Mariano evidence presented against him.
to report that the P50,912.00 was missing, explaining how she and her (iii) A written notice of termination served on the employee,
salesgirls had placed the wrapped amount at the bottom of the cabinet indicating that upon due consideration of all the circumstances, grounds
the night before, and how she had found upon reporting to work that have been established to justify his termination.
morning that the contents of the cabinet were in disarray and the money. In this case the respondents immediately had her arrested and
Later, while the petitioner was giving a detailed statement on investigated by the police authorities for qualified theft which constitutes
the theft to the security investigator of Harrison Plaza, Vina and Sylvia a denial of her right to due process of law, consisting in the opportunity to
Mariano, her superiors, arrived with a policeman who immediately placed be heard and to defend herself. In fact, their decision to dismiss her was
the petitioner under arrest and brought her to a police station where she already final even before the police authority commenced an
was investigated her and detained for a day. investigation of the theft, the finality being confirmed by no less than
Sylvia Mariano herself telling the petitioner during their phone
conversation following the latter’s release from police custody that she and not on the weakness of that adduced by the employee, in keeping
(Sylvia) “no longer wanted to see” her. with the principle that the scales of justice should be tilted in favor of the
The fact that the petitioner was the only person suspected of latter in case of doubt in the evidence presented by them. Often
being responsible for the theft aggravated the denial of due process. described as more than a mere scintilla, the quantum of proof is
substantial evidence which is understood as such relevant evidence as a
reasonable mind might accept as adequate to support a conclusion, even
95 Blue Sky Trading Co. vs. Blas, G.R. No. 190559, March 7, 2012 if other equally reasonable minds might conceivably opine otherwise.
(BURDEN OF PROOF; SUBSTANTIAL EVIDENCE;) Failure of the employer to discharge the foregoing onus would mean that
the dismissal is not justified and therefore illegal.
Facts: We find no error in the CA's findings that the petitioners had
Petitioner Blue Sky Trading Company, Inc. (Blue Sky) is a duly not adequately proven by substantial evidence that Arlene and Joseph
registered domestic corporation engaged in the importation and sale of indeed participated or cooperated in the commission of theft relative to
medical supplies and equipment. The respondents Arlene P. Blas the six missing intensifying screens so as to justify the latter's termination
(Arlene) and Joseph D. Silvano (Joseph) were regular employees of Blue from employment on the ground of loss of trust and confidence.
Sky and they respectively held the positions of stock clerk and We note that the parties disagree as to what tasks were
warehouse helper before they were dismissed from service on February actually and regularly performed by Arlene and Joseph. They are at odds
5, 2005. as to the issue of whether or not Arlene and Joseph had custody of the
An incident occurred where six pairs of intensifying screens missing screens. We observe though that neither of the parties presented
were missing. On February 3, 2005, Jean B. De La Paz (Jean), Human any documentary evidence, such as employment contracts, to establish
Resource Department Head issued notices to explain/preventive their claims relative to the actual nature of Arlene and Joseph's daily
suspension to Arlene, Joseph, delivery personnel Jayde Tano-an (Jayde) tasks.
and maintenance personnel/driver Wilfredo Fasonilao (Wilfredo). The The petitioners also argue that if Arlene and Joseph had not
notices informed them that they were being accused of gross dishonesty been grossly negligent in the performance of their duties, Blue Sky would
in connection with their alleged participation in and conspiracy with other not have incurred the loss. We observe though that in the notices sent to
employees in committing theft against company property, specifically Arlene and Joseph, first charging them with theft, and later, informing
relative to the loss of the six intensifying screens. them of their dismissal from service, gross negligence was not stated
On February 5, 2005, Jean issued to Arlene, Joseph, Jayde therein as a ground. Hence, Arlene and Joseph could not have defended
and Wilfredo notices of dismissal for cause stating therein that evidence themselves against the charge of gross negligence. They cannot be
that they had conspired with each other to commit theft against company dismissed on that ground lest due process be violated.
property was too glaring to ignore. Blue Sky had lost its trust and Other Matters: (For Discussion Purposes)
confidence on them and as an act of self-preservation, their termination Impropriety of the Preventive Suspension
from service was in order. The purpose of the suspension is to prevent an employee from
On February 8, 2005, Arlene, Joseph, Helario, Jayde and causing harm or injury to his colleagues and to the employer. The
Wilfredo filed with the National Labor Relations Commission (NLRC) a maximum period of suspension is 30 days, beyond which the employee
complaint for illegal dismissal and suspension, underpayment of overtime should either be reinstated or be paid wages and benefits due to him.
pay, and non-payment of emergency cost of living allowance (ECOLA), While we do not agree with Blue Sky's subsequent decision to terminate
with prayers for reinstatement and payment of full backwages. them from service, we find no impropriety in its act of imposing
Meanwhile, an entrapment operation was conducted by the preventive suspension upon the respondents since the period did not
police during which Jayde and Helario were caught allegedly attempting exceed the maximum imposed by law and there was a valid purpose for
to sell to an operative an ultrasound probe worth around P400,000.00 the same.
belonging to Blue Sky. Though eventually, Jayde and Helario executed In lieu of reinstatement, separation pay If reinstatement proves
affidavits of desistance stating that their dismissal was for cause. impracticable, and hardly in the best interest of the parties, perhaps due
The Labor Arbiter denied the claims of the respondents of to the lapse of time since the employee's
illegal suspension and dismissal since they failed in their duties to dismissal, or if the employee decides not to be reinstated, the latter
exercise utmost protection, care, or custody of respondent's property. should be awarded separation pay in lieu of reinstatement.
Hence, their dismissal from the service is warranted. In the case at bar, Arlene and Joseph were dismissed from
The first decision of the NLRC ruled that respondents were not service on February 5, 2005. We find that the lapse of more than seven
holding positions of trust and must therefore be reinstated and be paid years already renders their reinstatement impracticable. Further, from the
their backwages. Their second decision on the other hand reversed the stubborn stances of the parties, to wit, the petitioners' insistence that
previous one which in turn reinstated the Labor Arbiter’s dismissal of the dismissal was valid on one hand, and the respondents' express prayer
complaint saying that respondents were holding positions of trust and for the payment of separation pay on the other, we find that
that the loss of the company’s property are substantially proven. The CA reinstatement would no longer be in the best interest of the contending
on the other hand found merit on their claims, though found respondents parties.
to have positions of trust and confidence, petitioner in this case failed to Liability of Corporate Officers
sufficiently establish the charge against respondents which was the basis As a general rule, a corporate officer cannot be held liable for
for its loss of trust and confidence that warranted their dismissal. acts done in his official capacity because a corporation, by legal fiction,
has a personality separate and distinct from its officers, stockholders,
Issue: and members. In illegal dismissal cases, corporate officers may only be
Whether or not respondents Blas and Silvano committed a held solidarily liable with the corporation if the termination was done with
breach of trust malice or bad faith. We find that the aforementioned circumstance did not
obtain in the case of Jose (vice-president) and Linda (secretary) relative
Ruling: to Arlene and Joseph's dismissal from service.
The rule is long and well settled that, in illegal dismissal cases
like the one at bench, the burden of proof is upon the employer to show 96 Manila Electric Co. vs. Beltran, G.R. No. 173774, January 30, 2012
that the employee's termination from service is for a just and valid cause. (FAILURE TO REMIT IN PROPER TIME; BURDEN OF PROOF)
The employer's case succeeds or fails on the strength of its evidence
Arbiter that there were no serious grounds to warrant Beltran’s dismissal.
Facts: The CA held that the penalty of dismissal is harsh considering the
Beltran was employed by MERALCO and at the time material infraction committed and Beltran’s nine years of unblemished service
to this case, she was holding the position of Senior Branch Clerk at with MERALCO.
MERALCO’s Pasig branch. While rendering overtime work on September
28, 1996, a Saturday, Beltran accepted P15,164.48 from Collection Issue:
Route Supervisor Berlin Marcos (Marcos), which the latter received from Whether or not Beltran dismissal is valid finding that she is
customer Andy Chang (Chang). The cash payment was being made in guilty of withholding company funds.
lieu of a returned check earlier issued as payment for Chang’s electric
bill. Beltran received the payment and issued Auxiliary Receipt No. Ruling:
87964 which she dated September 30, 1996, a Monday, instead of Supreme Court support the CA’s finding that there are no
September 28, 1996. This was done to show that it was an sufficient grounds to warrant Beltran’s dismissal. For loss of trust and
accommodation, an accepted practice in the office. She thereafter placed confidence to be a valid ground for dismissal, it must be based on a
the money and the original auxiliary receipt and other documents willful breach of trust and founded on clearly established facts. A breach
pertinent to the returned check underneath her other files inside the is willful if it is done intentionally, knowingly and purposely, without
drawer of her table. justifiable excuse, as distinguished from an act done carelessly,
Beltran, however, was only able to remit Chang’s payment on thoughtlessly, heedlessly or inadvertently. In addition, loss of trust and
January 13, 1997. Thus, in a Memorandum dated January 16, 1997, she confidence must rest on substantial grounds and not on the employer’s
was placed under preventive suspension effective January 20, 1997 arbitrariness, whims, caprices or suspicion.
pending completion of an investigation. MERALCO considered as In the case at bench, Beltran attributed her delay in turning
misappropriation or withholding of company funds her failure to over Chang’s payment to her difficult family situation as she and her
immediately remit said payment in violation of its Code on Employee husband were having marital problems and her child was suffering from
Discipline. an illness. Admittedly, she was reminded of Chang’s payment by her
Garcia, the Administrative Supervisor of MERALCO’s Pasig supervisor on January 7, 1997 but denied having been ordered to remit
branch, on the other hand, testified that while doing an accounting of all the money on that day. She then reasoned that her continued delay was
outstanding returned checks sometime in December 1996, she noticed caused by an inevitable need to take a leave of absence for her to attend
that Chang’s returned check was missing. Upon further inquiry, she to the needs of her child who was suffering from asthma.
discovered that Chang had already redeemed the returned check after MERALCO cannot claim or conclude that Beltran misappropriated the
paying P15,164.48 to Beltran, who in turn issued an Auxiliary Receipt money based on mere suspicion. And even if Beltran delayed handing
dated September 30, 1996. It was also discovered that the payment has over the funds to the company, MERALCO still has the burden of proof to
not yet been remitted. This prompted her to inquire from Beltran on show clearly that such act of negligence is sufficient to justify termination
January 7, 1997 about the supposed payment and immediately ordered from employment. Beltran was remiss in her duties for her failure to
the remittance of the same. Beltran, however, failed to do so on that day immediately turn over Chang’s payment to the company. Such
and even on the next day when she reported for work. Beltran negligence, however, is not sufficient to warrant separation from
subsequently went on leave of absence on January 9 and 10, 1997. It employment. To justify removal from service, the negligence should be
was only on January 13, 1997 that the money with the pertinent gross and habitual. “Gross negligence x x x is the want of even slight
documents was handed over. In a memorandum dated February 25, care, acting or omitting to act in a situation where there is duty to act, not
1997, the investigator found Beltran guilty of misappropriating and inadvertently but willfully and intentionally, with a conscious indifference
withholding Chang’s payment of P15,164.48 and recommended her to consequences insofar as other persons may be affected.” Habitual
dismissal from service. neglect, on the other hand, connotes repeated failure to perform one’s
Beltran filed a complaint for illegal dismissal against duties for a period of time, depending upon the circumstances. No
MERALCO. She argued that she had no intention to withhold company concrete evidence was presented by MERALCO to show that Beltran’s
funds. Besides, it was not her customary duty to collect and remit delay in remitting the funds was done intentionally. Neither was it shown
payments from customers. She claimed good faith, believing that her that same is willful, unlawful and felonious contrary to MERALCO’s
acceptance of Chang’s payment is considered goodwill in favor of both finging as stated in the letter of termination it sent to Beltran. Surely,
MERALCO and its customer. If at all, her only violation was a simple Beltran’s single and isolated act of negligence cannot justify her
delay in remitting the payment, which caused no considerable harm to dismissal from service.
the company.
In a Decision of the Labor Arbiter regarded the penalty of
dismissal as not commensurate to the degree of infraction committed as 97
there was no adequate proof of misappropriation on the part of Beltran. If DOLORES T. ESGUERRA vs. VALLE VERDE COUNTRY CLUB, INC.
there was delay in Beltran’s remittance of Chang’s payment, it was (COST CONTROL SUPERVISOR – 2ND CLASS OF POSITIONS OF
unintentional and same cannot serve as sufficient basis to conclude that TRUST)
there was misappropriation of company funds. In fact, Beltran did not
even attempt to deny possession of, or refuse to hand in, the money. FACTS:
The Labor Arbiter thus gave compassionate consideration for Valle Verde hired Esguerra as Head Food Checker and was pr
the neglect to remit the money promptly, stating that it is excusable for omoted to Cost Control Supervisor. The Management found out that proc
Beltran to commit lapses in her work due to serious family difficulties. eeds had been remitted to the accounting department for an event were l
Upon appeal, the NLRC reversed the Labor Arbiter’s Decision acking. There were also unauthorized charges of food on one of the parti
and dismissed Beltran’s complaint against MERALCO in its Decision. It cipants. To resolve the issue, Valle Verde conducted an investigation; the
found that Beltran withheld company funds by failing to remit it for almost employees who were assigned in that event were summoned and made
four months. The NLRC thus ruled that MERALCO validly dismissed to explain, in writing, what had transpired. A memorandum was sent to E
Beltran from the service in the exercise of its inherent right to discipline sguerra requiring her to show cause as to why no disciplinary action shou
its employees. ld be taken against her for the nonremittance of the Ballroom’s sales. Esg
When Beltran brought the case to the CA the NLRC’s ruling uerra was placed under preventive suspension with pay, pending investig
was reversed. The CA instead agreed with the findings of the Labor ation. Unsatisfied with the explanation, Esguerra was terminated.
Petitioner said that she couldn’t be dismissed on the ground of l s principally charged with the function and responsibility to accept payme
oss of trust and confidence for she was only a regular employee and did nt of fees required for the installation of electric service and facilitate issu
not occupy a supervisory position vested with trust and confidence. Esgu ance of meter sockets.” The duties of his position require him to always a
erra also questions the manner of dismissal since the notice was insuffici ct with the highest degree of honesty, integrity and sincerity, as the comp
ent since it failed to contain any intention to terminate her. any puts it. In light of his fraudulent act, Meralco, an enterprise imbued wi
th public interest, cannot be compelled to continue Dejan’s employment,
ISSUE: as it would be inimical to its interest. Needless to say, “the law, in protecti
 Whether or not intention to terminate should be included in the notice ng the rights of the laborer, authorizes neither oppression nor selfdestruct
of informing of charges against an employee. ion of the employer.” For sure, Dejan was validly dismissed for serious mi
 Whether or not Cost Control Supervisor can be dismissed on the gro sconduct, and loss of trust and confidence.
und of loss of trust and confidence.

HELD: 99ROMEO E. PAULINO vs. NLRC and PHILIPPINE LONG DISTANCE


1.) No. The law does not require that an intention to terminate o TELEPHONE COMPANY, INCORPORATED
ne’s employment should be included in the first notice. It is enough that e (PROOF BEYOND REASONABLE DOUBT NOT REQUIRED;
mployees are properly apprised of the charges brought against them so t ACQUITAL IMMATERIAL)
hey can properly prepare their defenses; it is only during the second notic
e that the intention to terminate one’s employment should be explicitly sta FACTS:
ted. Members of the PNP, armed with a search warrant, searched t
The existence of an actual, formal “trialtype” hearing, although he house of Paulino, Cable Splicer III of PLDT, found items belonging to
preferred, is not absolutely necessary to satisfy the employee’s right to b PLDT which was in there for 1 month and 11 days
e heard. Esguerra was able to present her defenses; and only upon prop Petitioner did not present any documents or requisition slips th
er consideration of it did Valle Verde send the second memorandum term at would justify his possession of the materials. Consequently, PLDT cau
inating her employment. Since Valle Verde complied with the twonotice r sed the filing of an Information for qualified theft against him. PLDT then
equirement, no procedural defect exists in Esguerra’s termination. received a security report stating that petitioner had engaged in the illicit
2.) Esguerra held the position of Cost Control Supervisor and disposal of its plant materials, which were recovered during the search co
had the duty to remit to the accounting department the cash sales procee nducted at his residence. He was then terminated.Three years later, after
ds from every transaction she was assigned to. This is not a routine task the criminal case for qualified theft had been terminated for failure of the
that a regular employee may perform; it is related to the handling of busin prosecution to prove his guilt beyond reasonable doubt, petitioner filed a
ess expenditures or finances. For this reason, Esguerra occupies a positi Complaint for Illegal Dismissal which the Labor Arbiter .
on of trust and confidence a position enumerated in the second class of p
ositions of trust(first is for the managerial employees). Any breach of the t ISSUE:
rust imposed upon her can be a valid cause for dismissal. Whether or not an employee acquitted of the criminal charge ar
ising from the same incident which caused his dismissal, can be reinstate
d.
98
MANILA ELECTRIC COMPANY (MERALCO) vs. HERMINIGILDO H. D RULING:
EJAN No. The LA, the NLRC and the CA all acknowledged that, notwi
(BRANCH REPRESENTATIVE – HIGHEST DEGREE OF HONESTY) thstanding petitioner’s acquittal in the criminal case for qualified theft, res
pondent PLDT had adequately established the basis for the company’s lo
FACTS: ss of confidence as a just cause to terminate petitioner. Since proof bey
Dejan was then Meralco’s branch representative in its San Ped ond reasonable doubt of an employee’s misconduct is not required in di
ro, Laguna branch. His work consisted of accepting payments of the requ smissing an employee. Rather, as opposed to the “proof beyond reasona
ired fees from applicants for electric service installation and issuing the c ble doubt” standard of evidence required incriminal cases, labor suits req
orresponding meter sockets/bases after payment of a deposit, preceded uire only substantial evidence to prove the validity of the dismissal
by an inspection of the premises to be energized by a Meralco field perso However, assuming that he lawfully possessed the materials, P
nnel. LDT still had ample reason or basis to already distrust petitioner. For mor
One afternoon, 20 pieces of meter sockets take out from the br e than a month, he did not even inform PLDT of the whereabouts of the p
anch and was in possession of a Meralco field representative. Dejan was lant materials. Instead, he stocked these materials at his residence even i
asked to explain the incident. f they were needed in the daily operations of the company. In keeping wit
Dejan received a letter charging him with the unauthorized taki h the honesty and integrity demanded by his position, he should have tur
ng of 20 meter sockets, in violation of Section 7, paragraphs 4 and 11 of t ned over these materials to the plant’s warehouse.
he Company Code of Employee Discipline, in relation to Article 282 of th It would have been unfair for PLDT to keep petitioner in its emp
e Labor Code. In the Formal investigation, Dejan alleged that he released loy. Petitioner displayed actions that made him untrustworthy. Thus, as a
the items even without authorization as it had been the accepted practic measure of selfprotection, PLDT validly terminated his services for seriou
e in the office, provided the deposit fee had been paid. Unconvinced, his s misconduct and loss of confidence.
employment was terminated. NLRC and CA both ordered his reinstateme
nt. 100
VICENTE VILLANUEVA, JR. vs. NLRC THIRD DIVISION, MANILA EL
ISSUE: ECTRIC COMPANY
Whether or not an employee dismissed on the ground of loss of (ADDITIONAL GUIDELINES)
trust and confidence should be reinstated as judicially ordered.
FACTS:
RULING: There was a report regarding “unusual contract modifications” i
No. Meralco cannot be blamed for losing its trust and confidenc n the transactions handled by Villanueva. There are discrepancies not co
e in Dejan. He is no ordinary employee. As branch representative, “he wa vered by any receipt. Pursuant to the complaints, a field investigation wa
s conducted by the companydesignated investigator who was able to obt A. SEXUAL HARRASMENT
ain sworn statements from nine (9) out of twenty four (24) complaining cu
stomers that it was Villanueva whom they have transacted with. 101 CARLOS G. LIBRES vs. NLRC & NATIONAL STEEL
Meralco denied the request of Villanueva’s counsel to crossexa CORPORATION
mine the witnesses (complaining customers) who were not Meralco empl (SEXUAL HARRASMENT = LACK OF TRUST AND CONFIDENCE)
oyees. He was then served a notice of termination.
FACTS:
ISSUE: Whether Petitioner was validly dismissed in the ground of wilful Carlos G. Libres, an electrical engineer, was holding a
breach of trust? YES! managerial position with National Steel Corporation (NSC) as Assistant
Manager. One day, he received a Notice of Investigation from his
RULING: immediate superior, requesting him to submit a written explanation
As a safeguard against employers who indiscriminately use loss of trust relative to the charge of sexual harassment made by Susan D. Capiral,
and confidence to justify arbitrary dismissal of employees, the Court, in Hynsons secretary, allegedly committed by Libres, and subsequently to
addition to the above elements, came up with the following guidelines answer clarificatory questions on the matter. The notice also warned him
for the application of the doctrine: (1) loss of confidence should not be that failure to file his written explanation would be construed as a waiver
simulated; (2) it should not be used as a subterfuge for causes which are of his right to be heard. Libres submitted his written explanation denying
improper, illegal or unjustified; (3) it may not be arbitrarily asserted in the the accusation against him and offering to submit himself for clarificatory
face of overwhelming evidence to the contrary; and (4) it must be interrogation.
genuine, not a mere afterthought, to justify an earlier action taken in bad After deliberation, it was concluded that the charges against
faith.[21] petitioner constituted a violation of Item 2, Table V, of the Plants Rules
and Regulations. It opined that touching a female subordinates hand
In this case, the above requisites have been met. Meralcos loss of trust and shoulder, caressing her nape and telling other people that
and confidence arising out of Villanuevas act of misappropriation of Capiral was the one who hugged and kissed or that she responded to
company funds in the course of processing customer applications has the sexual advances are unauthorized acts that damaged her honor.
been proven by substantial evidence, thus, justified. Verily, the issuance Referring to the Manual of the Philippine Daily Inquirer in defining sexual
of additional receipts for excessive payments exacted from customers is harassment, the MEC finally concluded that petitioners acts clearly
a willful breach of the trust reposed in him by the company. constituted sexual harassment as charged and recommended petitioners
suspension for thirty (30) days without pay. Libres filed a complaint for
One. Villanueva worked for Meralco as a Branch Representative whose
illegal suspension and unjust discrimination against respondent NSC
tasks included the issuance of Contracts for Electric Service after receipt
of the amount due for service connection from customers. Obviously, he
ISSUE:
was entrusted not only with the responsibility of handling company funds
WON Libres was illegally suspended.
but also to cater to customers who intended to avail of Meralcos
services. This is nothing but an indication that trust and confidence were
HELD:
reposed in him by the company.
NO. His suspension was both fitting and appropriate since it
Two. Villanuevas acts of issuing contracts indicating therein an amount singularly addressed the issue of a managerial employee committing
less than the actual payment made by the customers and, thereafter, sexual harassment on a subordinate. It even invited the attention of the
issuing a receipt in an attempt to document the discrepancy are certainly Court to focus on sexual harassment as a just and valid cause for
work-related. This is, in fact, the core of his position as a Branch termination.
Representative. Whereas petitioner Libres was only meted a 30-day
suspension by the NLRC, Villarama, in the other case was penalized with
Three. Meralcos charge against Villanueva was adequately proven by termination. As Mr. Justice Puno elucidated, As a managerial employee,
substantial evidence. The records provide an extensive showing of petitioner is bound by more exacting work ethics. He failed to live up to
evidence against Villanueva. The affidavits of co-employees and, more his higher standard of responsibility when he succumbed to his moral
especially those of the customers themselves, bear weight in establishing perversity. And when such moral perversity is perpetrated against his
the specific acts constituting the charge against him. In fact, no subordinate, he provides a justifiable ground for his dismissal for lack of
inconsistencies among these statements were found. Villanueva likewise trust and confidence. It is the right, nay, the duty of every employer
failed to pose a plausible defense to protect its employees from oversexed superiors. Public respondent
therefore is correct in its observation that the Labor Arbiter was in fact
Four. The breach of the companys trust in Villanueva was shown to lenient in his application of the law and jurisprudence for which petitioner
have been committed knowingly and willfully. The conscious design of must be grateful and not gripe against.
issuing another receipt to make it appear that there was a mistake in the
initial transaction with the customers exhibits a culpable act bordering on
dishonesty and deceit. If not for personal gain, why did Villanueva exact 102 DELFIN G. VILLARAMA vs. NLRC AND GOLDEN DONUTS, INC.
from customers amounts in excess of what was required by the
company? What would have Villanueva done had the customers failed to FACTS:
discover the discrepancy between the amount they paid and that VILLARAMA was employed by GOLDEN DONUTS, INC., as its
appearing in the receipts issued to them? Why were there no overages Materials Manager. Villarama was charged with sexual harassment by
reported to his branch supervisor with respect to excess payments which Divina Gonzaga, a clerk-typist assigned in his department. The
were no longer questioned by the customers? These questions arise out humiliating experience compelled her to resign from work. The letter
of the practice which unfortunately corrupted an employee like prompted the President of Golden Donuts, Inc., Prieto, to call Villarama
Villanueva. These doubts sway the Court away from Villanuevas claim to a meeting. Villarama was then required to explain the letter against
that his errors were promptly corrected upon discovery. him. It appears that petitioner agreed to tender his resignation. Private
respondent moved swiftly to separate petitioner. Thus, private
respondent approved petitioner's application for leave of absence with
7. ANALOGOUS CASES
pay from August 5-28, 1989. It also issued an inter-office memorandum
advising "all concerned" that petitioner was no longer connected with the His abusive and distasteful acts unmistakably constituted
company. Mr. Prieto sent a letter to petitioner confirming their sexual harassment because they resulted in an intimidating, hostile, or
agreement that petitioner would be officially separated from the private offensive environment for his female subordinates. Section 8 of Rule 140
respondent. However, Villarama had a change of mind. He sought of the Rules of Court, as amended, considers a violation of the Code of
reconsideration of the management's decision to terminate him. For his Judicial Conduct as a serious offense. A respondent found guilty of a
failure to tender his resignation, petitioner was dismissed by private serious charge may be meted the penalty of:
respondent. Feeling aggrieved, petitioner filed an illegal dismissal case (1) dismissal from the service, forfeiture of all or part of the
against private respondent. benefits as the Court may determine, and disqualification from
reinstatement or appointment to any public office, including government-
ISSUE: owned or controlled corporations, provided, that the forfeiture of
WON the cause of dismissal of complainant was valid. benefits shall in no case include accrued leave credits;
(2) suspension from office without salary and other benefits
HELD: for more than three but notexceeding six months; or
YES. The records show that petitioner was confronted with the (3) a fine of more than P20,000 but not exceeding P40,000.
charge against him. Initially, he voluntarily agreed to be separated from
the company. He took a leave of absence preparatory to this separation.
This agreement was confirmed by the letter to him by Mr. Prieto 104 ATTY. SUSAN M. AQUINO vs. HON. ERNESTO D. ACOSTA
dated August 7, 1989. A few days after, petitioner reneged on the (JUDGE; CASUAL BUSS ON CHEEKS; SH HOW COMMITTED?;
agreement. He refused to be terminated on the ground that the ELEMENTS OF SH)
seriousness of his offense would not warrant his separation from service.
So he alleged in his letter to Mr. Prieto dated August 16, 1989. But even FACTS:
in this letter, petitioner admitted his "error" vis-a-vis Miss Gonzaga. As a Atty. Susan M. Aquino, Chief of the Legal and Technical Staff
manager, Petitioner should know the evidentiary value of his admissions. of the Court of Tax Appeals (CTA), filed a complaint against Judge
Needless to stress, he cannot complain there was no valid cause for his Ernesto Acosta, Presiding Judge of the same court, with sexual
separation. harassment under R.A. 7877 and violation of the Canons of Judicial
Moreover, loss of trust and confidence is a good ground for Ethics and Code of Professional Responsibility. Aquino alleged several
dismissing a managerial employee. It can be proved by substantial instances when respondent judge sexually harassed her. Respondent
evidence which is present in the case at bench. As a managerial judge denied complainants allegation that he sexually harassed her
employee, petitioner is bound by a more exacting work ethics. He failed six times. He claimed that he has always treated her with respect, being
to live up to this higher standard of responsibility when he succumbed to the head of the CTA Legal Staff. In fact, there is no strain in their
his moral perversity. And when such moral perversity is perpetrated professional relationship.
against his subordinate, he provides justifiable ground for his dismissal
for lack of trust and confidence. It is the right, nay, the duty of every HELD:
employer to protect its employees from over sexed superiors. The court found no convincing evidence to sustain
To be sure, employers are given wider latitude of discretion in complainants charges. What we perceive to have been committed by
terminating the employment of managerial employees on the ground of respondent judge are casual gestures of friendship and camaraderie,
lack of trust and confidence. nothing more, nothing less. In kissing complainant, we find no indication
that respondent was motivated by malice or lewd design. Evidently, she
misunderstood his actuations and construed them as work-related sexual
103 ATTY. GRACE M. VELOSO & MA. JOEYLYNN B. QUIONES vs harassment under R.A. 7877. "A mere casual buss on the cheek is not
JUDGE ANACLETO M. CAMINADE a sexual conduct or favor and does not fall within the purview of
(JUDGE; CODE OF JUDICIAL CONDUCT; PENALTY) sexual harassment under R.A. No. 7877. Section 3 (a) thereof provides,
towit:
FACTS: Sec 3. Work, Education or Training - related Sexual
Atty. Veloso, a lawyer of the Public le discussing the case, she Harassment Defined. - Work, education or training-related sexual
was stunned when Judge Caminade suddenly placed his hand on her harassment is committed by an employer, employee, manager,
right thigh and squeezed it. He then took her hand and kissed it. She supervisor, agent of the employer, teacher, instructor, professor, coach,
immediately stood up and headed towards the door leading to the staff trainor, or any other person who, having authority, influence or moral
room. He, however, caught up with her and placed his hand on her ascendancy over another in a work or training or education environment,
shoulder. Before she could open the door, Judge Caminade told her Kiss demands, requests or otherwise requires any sexual favor from the other,
ko bi (Let me kiss you). Atty. Veloso, who was so shocked, retorted Kalo- regardless of whether the demand, request or requirement for
od nimo Judge uy (You are so disgusting, Judge). She then opened the submission is accepted by the object of said Act.
door and went out of his chambers. Justice Morales found Judge a) In a work-related or employment environment, sexual
Caminade guilty of violating Canon 2 and Rule 2.01 of the Code of harassment is committed when:
Judicial Conduct and Canon 3 of the Canons of Judicial Ethics and 1) The sexual favor is made as a condition in the
recommended that respondent be suspended for six months without pay. hiring or in the employment, re employment or continued
employment of said individual, or in granting said individual
ISSUE: favorable compensation, terms, conditions, promotions or
WON the suspension was proper. privileges; or the refusal to grant sexual favor results in limiting,
segregating or classifying the employee which in anyway would
HELD: discriminate, deprive or diminish employment opportunities or
YES. Judge Caminades behavior must be sanctioned. We are otherwise adversely affect said employees;
neither amused by his claims of innocent playfulness nor impressed by 2) The above acts would impair the employee's
his excessive display of congeniality. He acted beyond the bounds of right or privileges under existing labor laws; or
decency, morality and propriety. He failed to meet the standard of 3) The above acts would result in an intimidating,
conduct embodied in the Code of Judicial Conduct. hostile, or offensive environment for the employee.
chair, then extended his hand to her, as if he wanted to shake her hands.
"Clearly, under the foregoing provisions, the elements of She reciprocated by extending her hands and jokingly put his hands on
sexual harassment are as follows: her forehead (agmanmano). She afterwards tried to free her
1) The employer, employee, manager, supervisor, hands off his but he would not let her. Instead, he told her, Wait for a
agent of the employer, teacher, instructor, professor, coach, while, I would just like to tell you something. I love you, is that okay? Tell
trainor, or any other person has authority, influence or me that you love me too. No strings attached. She retorted, As if you are
moralascendancy over another; my father. Spurned, he got mad. This kind of incident happened at
2) The authority, influence or moral ascendancy least ten more times.
exists in a working environment;
3) The employer, employee, manager, supervisor, ISSUE:
agent of the employer, Is he guilty of sexual harassment?

Indeed, from the records on hand, there is no showing that HELD:


respondent judge demanded, requested or required any sexual favor Under the circumstances, we find respondent guilty of sexual
from complainant in exchange for favourable compensation, terms, harassment. His severely outrageous acts, which are an affront to
conditions, promotion or privileges specified under Section 3 of R.A. women, constitute sexual harassment because they necessarily result in
7877. Nor did he, by his actuations, violate the Canons of Judicial Ethics an intimidating, hostile, and offensive working environment for his female
or the Code of Professional Responsibility. subordinates. He abused the power and authority he exercises over
them, which is the gravamen of the offense in sexual harassment.
Sexual harassment in the workplace is not about a man
105 Paiste v Mamenta, Jr. taking advantage of a woman by reason of sexual desire it is about
(SH – POWER EXERCISED BY A SUPERIOR) power being exercised by a superior over his women subordinates.
That power emanates from the fact that he can remove them if they
FACTS: refuse his amorous advances.
In an affidavit-complaint filed with the Office of the Chief There is likewise sufficient evidence in A.M. No. P-03-1 697 to
Justice, dated August 1 0, 2001, complainant Joanne S. Goltiao charged hold respondent administratively liable for his failure to issue official
him with gambling and drinking liquor during office hours, sexual receipt after receiving court fees and for discourtesy. The testimony of
harassment, arrogance and acts unbecoming of a government official. In complainant Paiste is worthy of belief. We find her categorical and
another affidavit complaint filed with the Office of the Court Administrator, consistent declarations credible, especially when viewed in the light of
dated August 28, 2001, the other complainant Jocelyn C. Paiste charged the fact that no ill motive on her part was established. Against her
him with conduct unbecoming of a public officer and with violation of the positive testimony, supported by documentary evidence based on official
AntiGraft and Corrupt Practices Act for his failure to issue official receipt. court records, the mere uncorroborated denial of the respondent
The evidence shows that complainant Goltiao is a deserves scant considerations.
Stenographer I of the MCTC of TayugSan Nicolas since 1 997. She The evidence shows that on April 21 , 2001 , complainant
testified that on August 7, 2000, at about 3:00 p.m., a representative from Paiste gave to the respondent the amount of two thousand four hundred
the Plaridel Insurance Co. came to their office seeking clearance. She pesos (P2,400.00) as bail bond of the accused in Criminal Case No.
immediately prepared the necessary form and, together with the 7461 . When she asked for an official receipt, he told her to get it the
representative, went to see respondent in the courtroom to obtain his following Monday at the court. She sent an office representative that
signature. When she asked him to sign the document, respondent, who Monday as told but respondent did not issue any receipt. Further
was at that time playing tong-its (a card game) at the lawyers table with verification from the records of Criminal Case No. 7461 reveals that no
unnamed individuals, got angry and threw his cards. He shouted at her: copy of the official receipt appears on file indicating that no receipt was
Why did you bring them with you? Did you like them to bring me to the ever issued to complainant Paiste.
Supreme Court? She responded that such was not her intention and We also find the rude and boorish manner respondent treated
reminded him of his requirement that he must first see the applicants complainant Paiste and her client on April 3, 2001 uncalled for. His
before he sign their clearance. He did not sign the clearance, sent then demeanor bordered on arrogance. He made them wait for fifteen minutes
out and shouted Bullshit ka! at her thrice. They all then went out of the before he attended to their concern as they found him playing cards
courtroom and proceeded back to the staff room. She went to her table and gambling. He was very irritable the whole time he was talking to
and buried her face in her hands, crying. Respondent followed her and them. After he examined the record, he slammed it an told them to go to
continued uttering unsavory remarks: Bullshit ka! Vulva of your mother! the court at Rosales. He then got annoyed when they followed him. To
Why did you take the client there and even raised your voice? (Bullshit be sure, this is no way to treat court users. High strung and belligerent
ka! Okinnam nga babai! Apay ta innalam dagita kliyente idiay sanak to behavior has no place in government, especially in the judiciary, where
rinayawan!) She replied that her parents taught them not to answer back the personnel are enjoined to act with self-restraint and civility at all
at older people. He still shouted: Vulva of your mother! I wish you will times, even when confronted with rudeness and insolence. They are
die now! Whom are you bragging of? We will try each other. (Okinnam expected to extend prompt, courteous and adequate service to the
nga babai! Matay ka koma itattan! Apay sinno aya ti paglaslastog mo? people. Such conduct is exacted from them so that they will earn and
Sige, agpipinnadas tayo. ) Thereafter, he asked the utility aide to buy him keep societys high regard for and confidence in the judicial service.
four bottles of beer. Conduct violative of this standard quickly and surely corrodes respect for
Goltiao declared that her working relationship with the the courts. It is the imperative and sacred duty of everyone charged with
respondent is sometimes good and sometimes bad because of his ill the dispensation of justice, from the judge to the lowliest clerk, to
temper. He easily gets mad at her even for small, trivial mistakes. This uphold the courts good name and standing as true temples of justice.
situation started, according to her, when she told him to stop courting and
sending her love notes as she is already a married woman. She related B. ABANDONMENT
an incident which happened early one morning when he asked her to see
him inside the judges chamber. At that time, the designated judge was 106 JENNY M. AGABON and VIRGILIO C. AGABON VS NLRC &
not around. Once inside, she was told to sit in one of the chairs RIVIERA HOME IMPROVEMENTS
in front of the judges table. The respondent, who was sitting at the judges
(FACTORS; SUBCONTRACTING – CLEAR INTENTION TO SEVERE (BURDEN OF PROOF – EMPLOYER; BARRED FROM ENTERING
EE-ER RELATIONSHIP) THE WORKPLACE; NOTICE TO RETURN TO WORK DOES NOT
CONSTITUTE ABANDONEMENT)
FACTS:
Riviera Home Improvements, Inc. is engaged in the business of FACTS:
selling and installing ornamental and construction materials. It employed Respondent Ananias P. Sato was hired by petitioner E.G. & I.
petitioners Virgilio Agabon and Jenny Agabon as gypsum board and Construction Corporation as a grader operator, which is considered as
cornice installers until they were dismissed for abandonment of work. technical labor. He held the position for more than thirteen (13) years. In
Petitioners then filed a complaint for illegal dismissal and 2004, Sato discovered that petitioner corporation had not been remitting
payment of money claims. They assert that they were dismissed because his premium contributions to the Social Security System (SSS). When
the private respondent refused to give them assignments unless they Sato kept on telling petitioners to update his premium contributions, he
agreed to work on a pakyaw basis when they reported for duty. They was removed as a grader operator and made to perform manual labor,
did not agree on this arrangement because it would mean losing benefits such as tilling the land in a private cemetery and/or digging earthworks in
as Social Security System (SSS) members. petitioner corporations construction projects. Thereafter, an inspection
Private respondent, on the other hand, maintained that team from the SSS went to petitioner corporations office to check its
petitioners were not dismissed but had abandoned their work. In fact, compliance with the SSS law. Petitioners told Sato that they could no
private respondent sent two letters to the last known addresses of the longer afford to pay his wages, and he was advised to look for
petitioners advising them to report for work. They even talked to Virgilio employment in other construction companies. Sato, however, found
Agabon to tell him about the new assignment at Pacific Plaza Towers difficulty in finding a job because he had been blacklisted in other
however, petitioners did not report for work because they had construction companies and was prevented from entering the project
subcontracted to perform installation work for another company. sites of petitioners.
Petitioners also demanded for an increase in their wage to P280.00 per Respondent Nilo Berdin was hired by petitioners as a
day. When this was not granted, petitioners stopped reporting for work steelman/laborer; respondent Anecito S. Parantar, Sr. hired as a
and filed the illegal dismissal case. steelman; and respondent Romeo M. Lacida, Jr. was as a laborer. At
the start of their employment, they were required by petitioners to sign
ISSUE: several documents purporting to be employment contracts. They
WON Agabon’s termination was valid. immediately signed the documents without verifying their contents for
fear of forfeiting their employment.
HELD: They were tasked to set up steel bars used in the building
YES. Abandonment is the deliberate and unjustified refusal of foundation, to mix cement, and to perform other tasks required of them
an employee to resume his employment. It is a form of neglect of duty, by petitioners.
hence, a just cause for termination of employment by the employer. For a The project engineer of respondents Berdin, Parantar, and
valid finding of abandonment, these two factors should be present: Lacida instructed them to affix their signatures on various documents.
(1) the failure to report for work or absence without valid or They refused to sign the documents because they were written in
justifiable reason; and English, a language that they did not understand. Irked by their
(2) a clear intention to sever employer-employee relationship, disobedience, the project engineer terminated their employment. On the
with the second as the more determinative factor which is manifested by same date, they were given their weekly wages. However, the wages
overt acts from which it may be deduced that the employees has no more that were paid to them were short of three (3) days worth of wages, as
intention to work. The intent to discontinue the employment must be penalty for their refusal to sign the documents. The following day, they
shown by clear proof that it was deliberate and unjustified. were not allowed to enter the work premises.
In this case, petitioners were frequently absent having Respondents filed their respective complaints with the Regional
subcontracted for an installation work for another company. Arbitration Branch of Cebu City for illegal dismissal, underpayment of
Subcontracting for another company clearly showed the intention to wages (wage differentials), holiday pay, thirteenth (13th) month pay, and
sever the employer-employee relationship with private respondent. This service incentive leave pay.
was not the first time they did this. In January 1996, they did not report The Labor Arbiter held that the respondents were illegally
for work because they were working for another company. Private dismissed from employment. In lieu of reinstatement, due to the strained
respondent at that time warned petitioners that they would be dismissed relations of the parties and as prayed for by respondents, each of them
if this happened again. Petitioners disregarded the warning and exhibited was granted separation pay equivalent to one (1) month pay for every
a clear intention to sever their employer-employee relationship. The year of service. The Labor Arbiter likewise awarded respondents claim
record of an employee is a relevant consideration in determining the for wage differentials, 13th month pay, holiday pay, and service incentive
penalty that should be meted out to him. leave pay.
The law imposes many obligations on the employer such as On appeal, the NLRC, reversed the Labor Arbiters decision.
providing just compensation to workers, observance of the procedural In reversing the decision of the Labor Arbiter, the NLRC
requirements of notice and hearing in the termination of employment. On ratiocinated that, other than respondents bare allegation that they were
the other hand, the law also recognizes the right of the employer to dismissed, they failed to present a written notice of dismissal, and that
expect from its workers not only good performance, adequate work and respondents individual complaints opted for the payment of separation
diligence, but also good conduct and loyalty. The employer may not be pay instead of reinstatement. The NLRC opined that illegal dismissal was
compelled to continue to employ such persons whose continuance in the inconsistent with the prayer for separation pay instead of reinstatement.
service will patently be inimical to his interests. Respondents filed an MR but the same was denied.
Aggrieved, respondents filed a petition for certiorari under Rule
107 E.G & I. CONSTRUCTION CORPORATION and EDSEL GALEOS, 65 of the Rules of Court before the CA.
Petitioners, v. ANANIAS P. SATO, NILO BERDIN, ROMEO M. The CA reinstated the decision of the Labor Arbiter ruling that
LACIDA, JR., and HEIRS OF ANECITO S. PARANTAR, SR., namely: respondents were illegally dismissed. Hence, this petition.
YVONNE, KIMBERLY MAE, MARYKRIS, ANECITO, JR., and JOHN
BRYAN, all surnamed PARANTAR, Respondents. ISSUE:
Whether the CA erred in reinstating the decision of the Labor Labor Arbiter for illegal dismissal. He supported his claim for
Arbiter, declaring that respondents were illegally terminated from commissions with two vouchers evidencing payments for vessel repairs,
employment by petitioner corporation, and that respondents are entitled arguing that he has paid P10,000 for each vessel he repaired.
to their monetary claims. Harpoon averred that on June 15, 2001, Rosit merely met with
Francisco to warn him regarding his habitual absences and tardiness.
HELD: When Francisco continued to be absent, Harpoon sent him memoranda
The decision of the Court of Appeals is sustained. informing him of his absences, which were filed with the DOLE on August
LABOR LAW 15, 2001. Francisco was then terminated on July 30, 2001. With regard
Petitioner corporation failed to prove that respondents were to the commissions claimed, Harpoon averred that Francisco was only a
dismissed for just or authorized cause. In an illegal dismissal case, the regular employee, with a regular salary, and that the supposed
onus probandi rests on the employer to prove that the dismissal of an "commissions" were merely additional money recognizing Franciscos
employee is for a valid cause. efforts.
For abandonment to exist, it is essential (a) that the employee The Labor Arbiter ruled that Francisco was legally dismissed
must have failed to report for work or must have been absent without and that due process was served through the several memoranda sent to
valid or justifiable reason; and (b) that there must have been a clear him. It also ruled that commissions were due Francisco, and gave
intention to sever the employer-employee relationship manifested by credence to the vouchers. The NLRC, however, held that Francisco was
some overt acts. The employer has the burden of proof to show the illegally dismissed, for his timecard for June 2001 only showed three
employee's deliberate and unjustified refusal to resume his absences, which could hardly be called habitual and therefore cannot be
employment without any intention of returning. Mere absence is not a ground for termination. It upheld the Labor Arbiter with regard to the
sufficient. There must be an unequivocal intent on the part of the commissions. The CA affirmed the NLRC, and held that Harpoon
employee to discontinue his employment. Uniwide Sales Warehouse president Rosit should be solidarily liable with the company.
Club v. National Labor Relations Commission, G.R. No. 154503,
February 29, 2008 ISSUES:
The reason why respondents failed to report for work was 1. Whether or not Francisco was illegally dismissed
because petitioner corporation barred them from entering its construction 2. Whether or not he was entitled to his commissions
sites. It is a settled rule that failure to report for work after a notice to 3. Whether Rosit is solidarily liable with Harpoon
return to work has been served does not necessarily constitute
abandonment. The intent to discontinue the employment must be shown HELD:
by clear proof that it was deliberate and unjustified. Petitioner corporation The petition is partly meritorious.
failed to show overt acts committed by respondents from which it may be LABOR LAW: Termination of employment; liability of
deduced that they had no more intention to work. Respondents filing of corporate officers.
the case for illegal dismissal barely four (4) days from their alleged First issue: The SC held that the termination was illegal. As
abandonment is totally inconsistent with our known concept of what stated by the NLRC, Franciscos timecard only showed three consecutive
constitutes abandonment. absences and no record of tardiness, which hardly constitutes gross or
LABOR LAW habitual absence/tardiness. Moreso, the reasons for Franciscos three-
Even as the employee must allege non-payment, the general day absence were not contested by Harpoon before the Labor Arbiter,
rule is that the burden rests on the employer to prove payment, rather and no other evidence was presented before the Labor Arbiter to prove
than on the employee to prove non-payment. The reason for the rule is such "habitual" tardiness/absence. The argument that Francisco
that the pertinent personnel files, payrolls, records, remittances, and abandoned his work and went AWOL also does not hold water, since
other similar documents which will show that overtime, differentials, Harpoon failed to prove that the two elements of work abandonment
service incentive leave, and other claims of the worker have been paid existed: namely, that there is absence of failure to report to work for no
are not in the possession of the worker but in the custody and absolute justifiable reason, and that there is intent to sever the employee-
control of the employer. Agabon v.NLRC, G.R. No. 158693, November employer relationship. Here, Harpoon failed to prove that it was
17, 2004 respondent who voluntarily refused to report back for work by his
In this case, the submission of petitioner corporation of the time defiance and refusal to accept the memoranda and the notices of
records and payrolls of respondents only on their appeal before the absences sent to him. Harpoon failed to present evidence that they sent
NLRC is contrary to elementary precepts of justice and fair play. these notices to respondents last known address for the purpose of
Respondents were not given the opportunity to check the authenticity warning him that his continued failure to report would be construed as
and correctness of the same. Thus, we sustain the ruling of the CA in the abandonment of work. Verily, an absence of three days does not
grant of the monetary claims of respondents. constitute habitual absence justifying a termination from work.
DENIED. Second issue: The SC held that Francisco was not entitled to
the commissions. The check vouchers contained very scant details and
did not state that they were paid for the construction or repair of a vessel.
108 HARPOON MARINE SERVICES, INC., ET AL.., Petitioner, v. They did not state the purpose for which the amounts were paid.
FERNAN H. FRANCISCO, Respondents. Moreover, the list of vessels presented with the vouchers does not
(3 DAYS ABSENT – NOT HABITUAL ABSENCE) validate Franciscos monetary claim for it only contains a list of vessels,
and nothing more. The vouchers patent vagueness makes them
FACTS: unreliable as a basis for Franciscos claim of commissions. Entitlement to
Harpoon Marine Services hired Francisco as a Yard commissions cannot be proved by vouchers which are silent as to the
Supervisor. On June 15, 2001, Francisco averred that Harpoon purpose for which they are issued.
dismissed him from work but promised to pay his separation pay and Third issue: The SC disagrees with the Labor Arbiter and
accrued commissions. He continued reporting for work, but was barred NLRC in according solidary liability on Rosit and Harpoon for the illegal
from entering the premises. He thereafter tried to claim his separation dismissal. As held in the case of MAM Realty Development Corporation
pay and commissions, but Harpoon, through its president Rosit, denied v. National Labor Relations Commission, "obligations incurred by
payment of his commissions. Refusing to sign a quitclaim, Francisco [corporate officers], acting as such corporate agents, are not theirs but
demanded payment of his commissions, then filed a case before the the direct accountabilities of the corporation they represent." As such,
they should not be generally held jointly and solidarily liable with the On the element of the failure of the employee to report for
corporation. The Court, however, cited circumstances when solidary work, we also cannot accept the allegations of petitioners that
liabilities may be imposed, as when the officer acted in bad faith or gross respondents unjustifiably refused to report for duty in their new posts. A
negligence in handling corporate affairs. Here, the CA imposed personal careful review of the records reveals that there is no showing that
liability on Rosit based on bad faith, even though there was no proof that respondents were notified of their new assignments. Granting that the
Rosit acted with bad faith or outside of his authority as company "Duty Detail Orders" were indeed issued, they served no purpose unless
president. At most, his acts merely showed the absence of a just or valid the intended recipients of the orders are informed of such.
cause in terminating the employment of Francisco. The employer cannot simply conclude that an employee is ipso
Petition is PARTLY GRANTED. facto notified of a transfer when there is no evidence to indicate that the
employee had knowledge of the transfer order. Hence, the failure of an
employee to report for work at the new location cannot be taken against
109 Alert Security and Investigation Agency vs. Pasawilan him as an element of abandonment.
(FAILURE TO NOTIFY EMPLOYEES OF NEW ASSIGNMENTS; NO We acknowledge and recognize the right of an employer to
ABANDONMENT) transfer employees in the interest of the service. This exercise is a
management prerogative which is a lawful right of an employer.
Facts: However, like all rights, there are limitations to the right to transfer
Respondents Saidali Pasawilan, Wilfredo Verceles and employees. As ruled in the case of Blue Dairy Corporation v. NLRC:
Melchor Bulusan were all employed by petitioner Alert Security and x x x The managerial prerogative to transfer personnel must be
Investigation Agency, Inc. (Alert Security) as security guards beginning exercised without grave abuse of discretion, bearing in mind the basic
March 31, 1996, January 14, 1997, and January 24, 1997, respectively. elements of justice and fair play. Having the right should not be confused
They were paid 165.00 pesos a day as regular employees, and assigned with the manner in which that right is exercised. Thus, it cannot be used
at the Department of Science and Technology (DOST) pursuant to a as a subterfuge by the employer to rid himself of an undesirable worker.
security service contract between the DOST and Alert Security. In particular, the employer must be able to show that the transfer is not
Respondents aver that because they were underpaid, they filed unreasonable, inconvenient or prejudicial to the employee; nor does it
a complaint for money claims against Alert Security and its president and involve a demotion in rank or a diminution of his salaries, privileges and
general manager, petitioner Manuel D. Dasig, before Labor Arbiter Ariel other benefits. x x x
C. Santos. As a result of their complaint, they were relieved from their In addition to these tests for a valid transfer, there should be
posts in the DOST and were not given new assignments despite the proper and effective notice to the employee concerned. It is the
lapse of six months. On January 26, 1999, they filed a joint complaint for employer’s burden to show that the employee was duly notified of the
illegal dismissal against petitioners. transfer. Verily, an employer cannot reasonably expect an employee to
Petitioners, on the other hand, deny that they dismissed the report for work in a new location without first informing said employee of
respondents. Petitioners presented "Duty Detail Orders" that Alert the transfer. Petitioners’ insistence on the sufficiency of mere issuance of
Security issued to show that respondents were in fact assigned to LRTA. the transfer order is indicative of bad faith on their part.
Respondents, however, failed to report at the LRTA and instead kept
loitering at the DOST and tried to convince other security guards to file 110 POLYFOAM-RGC INTERNATIONAL, CORPORATION and
complaints against Alert Security. Thus, on August 3, 1998, Alert Security PRECILLA A. GRAMAJE, Petitioners, v.EDGARDO CONCEPCION,
filed a "termination report" with the Department of Labor and Employment Respondent.
relative to the termination of the respondents. (LABOR-ONLY; IMMEDIATE ACTION BY EMPLOYEE NEGATES
INTENTION TO SEVERE EE-ER RELATIONSHIP)
Issue:
Whether respondents were illegally dismissed FACTS:
In his February 08, 2000 complaint for illegal dismissal against
Rulings: Polyfoam and Natividad Cheng, Edgardo Concepcion alleged that he
We rule in the affirmative. was hired by Polyfoam as an "all-around" factory worker and served as
As a rule, employment cannot be terminated by an employer such for almost six years. On January 14, 2000, he allegedly discovered
without any just or authorized cause. No less than the 1987 Constitution that his time card was not in the rack and was later informed by the
in Section 3, Article 13 guarantees security of tenure for workers and security guard that he could no longer punch his time card. When he
because of this, an employee may only be terminated for just or protested to his supervisor, the latter allegedly told him that the
authorized causes that must comply with the due process requirements management decided to dismiss him due to an infraction of a company
mandated by law. Hence, employers are barred from arbitrarily removing rule. Cheng, the company manager, also refused to face him.
their workers whenever and however they want. The law sets the valid Respondent counsel later wrote a letter to Polyfoam manager requesting
grounds for termination as well as the proper procedure to take when that respondent be re-admitted to work, but the request remained
terminating the services of an employee. unheeded prompting the latter to file the complaint for illegal dismissal.
Although we recognize the right of employers to shape their On April 28, 2000, Gramaje filed a Motion for Intervention
own work force, this management prerogative must not curtail the basic claiming to be the real employer of respondent. On the other hand,
right of employees to security of tenure. There must be a valid and lawful Polyfoam and Cheng filed a Motion to Dismiss on the grounds that the
reason for terminating the employment of a worker. Otherwise, it is illegal NLRC has no jurisdiction over the case, because of the absence of
and would be dealt with by the courts accordingly. employer-employee relationship between Polyfoam and respondent and
The Labor Code, as amended, enumerates several just and that the money claims had already prescribed.
authorized causes for a valid termination of employment. An employee On May 24, 2000, Labor Arbiter Adolfo Babiano issued an
asserting his right and asking for minimum wage is not among those Order granting Gramaje motion for intervention, it appearing that she is
causes. Dismissing an employee on this ground amounts to retaliation by an indispensable party and denying Polyfoam and Cheng motion to
management for an employee’s legitimate grievance without due dismiss as the lack of employer-employee relationship is only a matter of
process. Such stroke of retribution has no place in Philippine Labor defense.
Laws. In their Position Paper, Polyfoam and Cheng insisted that the
NLRC has no jurisdiction over the case, because respondent was not
their employee. They likewise contended that respondent money claims to the workers in the same manner and extent as if the latter were
had already prescribed. Finally, they fault respondent for including Cheng directly employed by him.
as a party-defendant, considering that she is not even a director of the The test of independent contractorship is "whether one
company. claiming to be an independent contractor has contracted to do the work
In her Position Paper,Gramaje claimed that P.A. Gramaje according to his own methods and without being subject to the control of
Employment Services (PAGES) is a legitimate job contractor who the employer, except only as to the results of the work." In San Miguel
provided some manpower needs of Polyfoam. It was alleged that Corporation v. Semillano, the Court laid down the criteria in determining
respondent was hired as "packer" and assigned to Polyfoam, charged the existence of an independent and permissible contractor relationship,
with packing the latter finished foam products. She argued, however, that to wit:
respondent was not dismissed from employment, rather, he simply x x x [W]hether or not the contractor is carrying on an
stopped reporting for work. independent business; the nature and extent of the work; the skill
On December 14, 2001, Labor Arbiter rendered a Decision required; the term and duration of the relationship; the right to assign the
finding respondent to have been illegally dismissed from employment performance of a specified piece of work; the control and supervision of
and holding Polyfoam and Gramaje/PAGES solidarily liable for the work to another; the employer power with respect to the hiring, firing
respondent money claims. and payment of the contractor workers; the control of the premises; the
On appeal by petitioners, the NLRC modified the LA decision duty to supply the premises, tools, appliances, materials, and labor; and
by exonerating Polyfoam from liability for respondent claim for separation the mode, manner and terms of payment.
pay and deleting the awards of backwages, 13th month pay, damages, Simply put, the totality of the facts and the surrounding
and attorney fees. circumstances of the case are to be considered. Each case must be
Aggrieved, respondent elevated the case to the CA in a special determined by its own facts and all the features of the relationship are to
civil action for certiorari under Rule 65 of the Rules of Court. On be considered.
December 19, 2005, the appellate court granted the petition. The CA Applying the foregoing tests, we agree with the CA conclusion
agreed with the LA conclusion that Gramaje is not a legitimate job that Gramaje is not an independent job contractor, but a "labor-only"
contractor but only a "labor-only" contractor. The appellate court affirmed contractor.
the LA findings of illegal dismissal as respondent was dismissed from the First, Gramaje has no substantial capital or investment. The
service without cause and due process.Consequently, separation pay in presumption is that a contractor is a labor-only contractor unless he
lieu of reinstatement was awarded. The CA quoted with approval the LA overcomes the burden of proving that it has substantial capital,
conclusions on the award of respondent other money claims. investment, tools, and the like. The employee should not be expected to
prove the negative fact that the contractor does not have substantial
ISSUES: capital, investment and tools to engage in job-contracting.
1. Whether or not Gramaje is an independent job contractor? Gramaje claimed that it has substantial capital of its own as
2. Whether or not respondent was illegally dismissed from well as investment in its office, equipment and tools. She pointed out that
employment? she furnished the plastic containers and carton boxes used in carrying
out the function of packing the mattresses of Polyfoam. She added that
HELD: she had placed in Polyfoam workplace ten (10) sealing machines, twenty
The decision of the Court of Appeals is affirmed. (20) hand trucks, and two (2) forklifts to enable respondent and the other
Gramaje is a Labor-Only Contractor - Article 106 of the employees of Gramaje assigned at Polyfoam to perform their job. Finally,
Labor Code explains the relations which may arise between an employer, she explained that she had her own office with her own staff. However,
a contractor, and the contractor employees, thus: aside from her own bare statement, neither Gramaje nor Polyfoam
ART. 106. Contractor or subcontracting. − Whenever an presented evidence showing Gramaje ownership of the equipment and
employer enters into a contract with another person for the performance machineries used in the performance of the alleged contracted job.
of the former work, the employees of the contractor and of the latter Considering that these machineries are found in Polyfoam premises,
subcontractor, if any, shall be paid in accordance with the provisions of there can be no other logical conclusion but that the tools and equipment
this Code. utilized by Gramaje and her "employees" are owned by Polyfoam.
In the event that the contractor or subcontractor fails to pay the Neither did Polyfoam nor Gramaje show that the latter had clients other
wages of his employees in accordance with this Code, the employer shall than the former. Since petitioners failed to adduce evidence that Gramaje
be jointly and severally liable with his contractor or subcontractor to such had any substantial capital, investment or assets to perform the work
employees to the extent of the work performed under the contract, in the contracted for, the presumption that Gramaje is a labor-only contractor
same manner and extent that he is liable to employees directly employed stands.
by him. Second, Gramaje did not carry on an independent business or
The Secretary of Labor and Employment may, by appropriate undertake the performance of its service contract according to its own
regulations, restrict or prohibit the contracting out of labor to protect the manner and method, free from the control and supervision of its principal,
rights of workers established under the Code. In so prohibiting or Polyfoam, its apparent role having been merely to recruit persons to work
restricting, he may make appropriate distinctions between labor-only for Polyfoam.It is undisputed that respondent had performed his task of
contracting and job contracting as well as differentiations within these packing Polyfoam foam products in Polyfoam premises. As to the
types of contracting and determine who among the parties involved shall recruitment of respondent, petitioners were able to establish only that
be considered the employer for purposes of this Code, to prevent any respondent application was referred toGramaje, but that is all. Prior to his
violation or circumvention of any provision of this Code. termination, respondent had been performing the same job in Polyfoam
There is labor-only contracting where the person supplying business for almost six (6) years. He was even furnished a copy of
workers to an employer does not have substantial capital or investment Polyfoam "Mga Alituntunin at Karampatang Parusa,"which embodied
in the form of tools, equipment, machineries, work premises, among Polyfoam rules on attendance, the manner of performing the employee
others, and the workers recruited and placed by such person are duties, ethical standards, cleanliness, health, safety, peace and order.
performing activities which are directly related to the principal business of These rules carried with them the corresponding penalties in case of
such employer. In such cases, the person or intermediary shall be violation.
considered merely as an agent of the employer who shall be responsible While it is true that petitioners submitted the Affidavit of
Polyfoam supervisor Victor Abadia, claiming that the latter did not
exercise supervision over respondent because the latter was not established Insta Printers, a rival printing press, with Edwin Ricardo
Polyfoam but Gramaje employee, said Affidavit is insufficient to prove himself as consultant and owner. Since the establishment of Insta
such claim. Petitioners should have presented the person who they claim Printers, Ricardo became a habitual absentee from his job at GT
to have exercised supervision over respondent and their alleged other Printers. He neglected his duties and responsibilities, and became lax in
employees assigned toPolyfoam. It was never established that Gramaje directing and supervising the work force, resulting in numerous major
took entire charge, control and supervision of the work and service printing errors and failure to meet printing specifications leading to the
agreed upon. And as aptly observed by the CA, "it is likewise highly rejection of several job orders from regular customers. Mrs. Barba
unusual and suspect as to the absence of a written contract specifying noticed that Ricardo not only used GT Printers' bookcloth and other
the performance of a specified service, the nature and extent of the printing materials for his Insta Printers, but he also gave specific
service or work to be done and the term and duration of the relationship." instructions to the production staff to give priority to book and magazine
A finding that a contractor is a "labor-only" contractor, as job orders for Insta Printers. Eventually, the regular customers of GT
opposed to permissible job contracting, is equivalent to declaring that Printers were pirated by Insta Printers. Ricardo also manipulated price
there is an employer-employee relationship between the principal and the quotations during the canvassing of bids to favor his own outfit instead of
employees of the supposed contractor, and the "labor-only" contractor is GT Printers. Because of those irregularities, GT Printers suspended
considered as a mere agent of the principal, the real employer. In this Ricardo as general manager for 30 days. Richard Barba was designated
case, Polyfoam is the principal employer and Gramaje is the labor-only to take his place. Contracts concluded by respondent Ricardo thereafter
contractor. Polyfoam and Gramaje are, therefore, solidarily liable for the were no longer honored. However, he continued to be a sales agent for
rightful claims of respondent. GT Printers, hence, he continued to receive commissions. He stopped
Respondent was Illegally Dismissed From Employment - reporting for work and soon after filed a complaint for illegal dismissal.
Respondent stated that on January 14, 2000, his time card was suddenly
taken off the rack. His supervisor later informed him that Polyfoam ISSUE:
management decided to dismiss him due to infraction of company rule. In WON Ricardo was lawfully dismissed from employment.
short, respondent insisted that he was dismissed from employment
without just or lawful cause and without due process. Polyfoam did not HELD:
offer any explanation of such dismissal. It, instead, explained that YES. The security of tenure accorded to labor under the
respondent real employer is Gramaje. Gramaje, on the other hand, Constitution does not embrace infractions of accepted company rules
denied the claim of illegal dismissal. She shifted the blame on amounting to breach of trust and loss of confidence. The right of an
respondent claiming that the latter in fact abandoned his work. employer to dismiss a managerial employee for breach of trust and loss
The LA gave credence to respondent narration of the of confidence, as in this case, cannot be doubted. As a measure of self-
circumstances of the case. Said conclusion was affirmed by the CA. We preservation against acts inimical to its interests, an employer has the
find no reason to depart from such findings. right to dismiss an employee found committing acts of dishonesty and
Abandonment cannot be inferred from the actuations of disloyalty. The employer may not be compelled to continue to employ
respondent. When he discovered that his time card was off the rack, such a person whose continuance in the service would patently be
he immediately inquired from his supervisor. He later sought the inimical to his employer's interest. The dismissal of a dishonest employee
assistance of his counsel, who wrote a letter addressed to Polyfoam is in the best interest not only of management but also of labor for the law
requesting that he be re-admitted to work. When said request was never intended to impose an unjust situation on either labor or
not acted upon, he filed the instant illegal dismissal case. These management. Reinstatement would be ill-advised and incompatible with
circumstances clearly negate the intention to abandon his work. the labor arbiter's finding that "from those documentary evidences
Petitioners failed to show any valid or authorized cause under presented by respondent, it can be safely conclude[d] that . . . there
the Labor Code which allowed it to terminate the services of respondent. exist visible conflict of interest amounting to willful breach of trust
Neither was it shown that respondent was given ample opportunity to and confidence repose (sic) upon him by his employer, . . as well as
contest the legality of his dismissal. No notice of termination was given to (b) habitual neglect of his duties . . ."
him. Clearly, respondent was not afforded due process. Having failed to
establish compliance with the requirements of termination of employment
under the Labor Code, the dismissal of respondent was tainted with 112 DUNCAN ASSOCIATION OF DETAILMAN-PTGWO and PEDRO
illegality. Consequently, respondent is entitled to reinstatement without A. TECSON vs. GLAXO WELLCOME PHILIPPINES, INC.
loss of seniority rights, and other privileges and to his full backwages (MARRIAGE TO AN EMPLOYEE OF A COMPETITOR’S COMPANY)
inclusive of allowances and to his other benefits or their monetary
equivalent computed from the time his compensation was withheld up to FACTS:
the time of his actual reinstatement. However, if reinstatement is no Pedro A. Tecson (Tecson) was hired by respondent Glaxo
longer feasible as in this case, separation pay equivalent to one-month Wellcome Philippi nes, Inc. (Glaxo) as medical representative. Tecson
salary for every year of service shall be awarded as an alternative. Thus, signed a contract of employment which stipulates, among others, that
the CA is correct in affirming the LA award of separation pay with full he agrees to study and abide by existing company rules; to disclose to
backwages and other monetary benefits. management any existing or future relationship by consanguinity or
DENIED affinity with co-employees or employees of competing drug companies
and should management find that such relationship poses a possible
C. CONFLICT OF INTEREST conflict of interest, to resign from the company. Tecson was initially
assigned to market Glaxos products in the Camarines Sur-Camarines
111 GT PRINTERS and/or TRINIDAD G. BARBA vs. NLRC and Norte sales area. Subsequently, Tecson entered into a romantic
EDWIN RICARDO relationship with Bettsy, an employee of Astra Pharmaceuticals (Astra), a
(COI – HABITUAL NEGLECT AND BREACH OF TRUST) competitor of Glaxo. Bettsy was Astras Branch Coordinator in Albay.
She supervised the district managers and medical representatives of her
FACTS: company and prepared marketing strategies for Astra in that area. Even
Edwin Ricardo, was employed in 1968 as an apprentice of GT before they got married, Tecson received several reminders from his
Printers. He was promote to the position of production manager of GT District Manager regarding the conflict of interest which his relationship
Printers and became general manager. In February, 1985, Ricardo's wife with Bettsy might engender. Still, Tecson married Bettsy. Tecsons
superiors informed him that his marriage to Bettsy gave rise to a conflict Later, the Director of Nursing Services notified Bea that her contract
of interest. would be terminated. Because of this, she wrote a letter requesting for a
reconsideration of the decision to terminate her. Beas contractual
HELD: employment was terminated and she was repatriated to the Philippines
Glaxos policy prohibiting an employee from having a on the recommendation of the Department of Nursing Services after
relationship with an employee of a competitor company is a valid three very poor evaluations.
exercise of management prerogative. Glaxo has a right to guard its trade After her second poor evaluation, she was given intensive
secrets, manufacturing formulas, marketing strategies and other management assistance through a specialized training plan but she did
confidential programs and information from competitors, especially so not improve. She was unable to function as a Senior Head Staff Nurse on
that it and Astra are rival companies in the highly competitive the Neonatal Unit and therefore a recommendation was made that she
pharmaceutical industry. The prohibition against personal or marital should be terminated since she could not fulfill her contractual obligations
relationships with employees of competitor companies upon Glaxos within Nursing Services.
employees is reasonable under the circumstances because relationships Bea thereafter filed a case of illegal dismissal against instant
of that nature might compromise the interests of the company. Glaxo only petitioner before the POEA Adjudication Office.
aims to protect its interests against the possibility that a competitor
company will gain access to its secrets and procedures. Glaxo HELD:
possesses the right to protect its economic interests cannot be denied. Beas termination was illegal for failure of petitioner to
No less than the Constitution recognizes the right of enterprises to adopt prove the existence of a just or authorized cause for terminating
and enforce such a policy to protect its right to reasonable returns on her. We take cognizance of the fact that in any given workplace, not all of
investments and to expansion and growth. Indeed, while our laws the employees perform in accordance with what is expected of them. As
endeavor to give life to the constitutional policy on social justice and the such, it is not uncommon that an employees work performance is found
protection of labor, it does not mean that every labor dispute will be to be unsatisfactory. As a general concept, poor performance is
decided in favor of the workers. The law also recognizes that equivalent to inefficiency and incompetence in the performance of official
management has rights which are also entitled to respect and duties. Under Article 282 of the Labor Code, an unsatisfactory rating
enforcement in the interest of fair play In the case at bar, the record can be a just cause for dismissal only if it amounts to gross and
shows that Glaxo gave Tecson several chances to eliminate the conflict habitual neglect of duties. The fact that an employees’ performance is
of interest brought about by his relationship with Bettsy. When their found to be poor or unsatisfactory does not necessarily mean that the
relationship was still in its initial stage, Tecsons supervisors at Glaxo employee is grossly and habitually negligent of his duties. Gross
constantly reminded him about its effects on his employment with the negligence implies a want or absence of or failure to exercise slight care
company and on the companys interests. After Tecson married Bettsy, or diligence, or the entire absence of care. It evinces a thoughtless
Glaxo gave him time to resolve the conflict by either resigning from the disregard of consequences without exerting any effort to avoid them. In
company or asking his wife to resign from Astra. Glaxo even expressed the present case, petitioner failed to present substantial evidence to
its desire to retain Tecson in its employ because of his satisfactory prove that Beas alleged poor performance in her duties as Senior Head
performance and suggested that he ask Bettsy to resign from her Staff Nurse amounted to gross and habitual neglect. In the first place, the
company instead. Glaxo likewise acceded to his repeated requests for POEA Adjudication Office found that aside from the Memorandum dated
more time to resolve the conflict of interest. When the problem could not June 8, 1993 issued by the Acting Director of Nursing Services of Sultan
be resolved after several years of waiting, Glaxo was constrained to Qaboos University Hospital where Bea was deployed, petitioner failed to
reassign Tecson to a sales area different from that handled by his wife present any other evidence to prove that Beas work performance was
for Astra. Notably, the Court did not terminate Tecson from em ployment indeed poor. Although petitioner contends that three separate
but only reassigned him to another area where his home province, evaluations of Beas work performance were conducted; that after the first
Agusan del Sur, was included. In effecting Tecsons transfer, Glaxo even evaluation, Bea was notified about the poor quality of her work; that
considered the welfare of Tecsons family. Clearly, the foregoing following the second evaluation, she was given an intensive
dispels any suspicion of unfairness and bad faith on the part of Glaxo management assistance through a specialized training program; and,
that only after the third evaluation was made that Bea was advised that
D. ATTITUDE PROBLEM her employment would be terminated, we find no error in the findings of
E. POOR PERFORMANCE OR GROSS INEFFICIENCY the POEA and the NLRC that these claims of petitioner remain to be
allegations since no substantial evidence was presented to prove them.
113 EASTERN OVERSEAS EMPLOYMENT CENTER, INC. vs In termination cases, the burden of proving just and valid cause
CECILIA BEA for dismissing an employee from his employment rests upon the
(SUBSTANTIAL EVIDENCE; POOR PERFORMANCE MUST BE employer, and the latter's failure to discharge that burden would result in
EQUIVALENT TO GROSS AND HABITUAL NEGLECT OF DUTY) a finding that the dismissal is unjustified.

FACTS:
Bea was hired as Senior Head Staff Nurse by Elbualy 114 Realda vs. New Age Graphics Inc., G.R. No. 192190, April 25,
Group/Su ltan Qaboos University Hospital (SQUH), the principal 2012
employer through its placement agency in the Philippines, Eastern (TOTALITY OF INFRACTIONS)
Overseas Employment Center, Inc. (Eastern). Her contractual
employment was for two (2) years. Beas placement with SQUH was Facts:
subject to a three-(3) month probationary period during said contractual Petitioner Realda was dismissed by Respondent New Age
employment. Graphics Inc. for unjustified refusal to render overtime work, unexplained
Beas probationary status ended on May 1992 but she still failure to observe prescribed work standards, habitual tardiness and
continued being in the employ of SQUH. chronic absenteeism despite warning and non-compliance with the
She, like all other employees of the hospital, was also directive for him to explain his numerous unauthorized absences. The
periodically subjected to performance evaluation. After an alleged poor Court of Appeals recognized the existence of just causes for petitioner’s
evaluation of Beas performance as a nurse, she was transferred to the dismissal, however, the appellate court found that the respondent failed
Neo-Natal Unit and her performance was supposedly under observation.
to observe the procedural requirements of due process and, as a Pabayo were subjected to a physical examination where the results
consequence, awarded the petitioner P5,000.00 as Nominal Damages. showed that they were positive of drugs. They were also brought to the
Issues: security office of PAL where they executed written confessions without
WoN the dismissal based on the grounds cited constituted just the benefit of counsel. Roquero and Pabayo then received a notice of
causes; and WoN the amount awarded as Nominal Damages of administrative charge] for violating the PAL Code of Discipline. They
P5,000.00 was valid were required to answer the charges and were placed under
preventive suspension.
Ruling: Eventually, they were dismissed by PAL. Thus, they filed a
First, the petitioner’s arbitrary defiance to Graphics, Inc.’s case for illegal dismissal.
order for him to render overtime work constitutes willful disobedience.
Taking this in conjunction with his inclination to absent himself and to HELD:
report late for work despite being previously penalized, the CA correctly Roquero is guilty of serious misconduct for possessing and
ruled that the petitioner is indeed utterly defiant of the lawful orders and using shabu. He violated Chapter 2, Article VII, section 4 of the PAL
the reasonable work standards prescribed by his employer. Code of Discipline which states: Any employee who, while on company
Second, the petitioner’s failure to observe Graphics, Inc.’s premises or on duty, takes or is under the influence of prohibited or
work standards constitutes inefficiency that is a valid cause for dismissal. controlled drugs, or hallucinogenic substances or narcotics shall be
Failure to observe prescribed standards of work, or to fulfill reasonable dismissed. Serious misconduct is defined as the transgression of some
work assignments due to inefficiency may constitute just cause for established and definite rule of action, a forbidden act, a dereliction of
dismissal. Such inefficiency is understood to mean failure to attain work duty, willful in character, and implies wrongful intent and not mere error in
goals or work quotas, either by failing to complete the same within the judgment. For serious misconduct to warrant the dismissal of an
alloted reasonable period, or by producing unsatisfactory results. As the employee, it (1) must be serious; (2) must relate to the performance of
operator of Graphics, Inc.’s printer, he is mandated to check whether the the employees duty; and (3) must show that the employee has become
colors that would be printed are in accordance with the client’s unfit to continue working for the employer.
specifications and for him to do so, he must consult the General Manager It is of public knowledge that drugs can damage the mental faculties of
and the color guide usedMby Graphics, Inc. before making a full run. the user. Roquero was tasked with the repair and maintenance of PALs
Unfortunately, he failed to observe this simple procedure and proceeded airplanes. He cannot discharge that duty if he is a drug user. His failure
to print without making sure that the colors were at par with the client’s to do his job can mean great loss of lives and properties. Hence, even if
demands. This resulted to delays in the delivery of output, client he was instigated to take drugs he has no right to be reinstated to his
dissatisfaction, and additional costs on Graphics, Inc.’s part. position. He took the drugs fully knowing that he was on duty and more
While a penalty in the form of suspension had already been so that it is prohibited by company rules.
imposed on the petitioner for his habitual tardiness and repeated Instigation is only a defense against criminal liability. It cannot
absenteeism, the principle of “totality of infractions” sanctions the act be used as a shield against dismissal from employment especially when
of Graphics, Inc. of considering such previous infractions in decreeing the position involves the safety of human lives.
dismissal as the proper penalty for his tardiness and unauthorized
absences incurred afterwards, in addition to his refusal to render NOTE: Although both Roquero and Pabayo filed the illegal dismissal
overtime work and conform to the prescribed work standards. case, only Roquero brought this petition for review because Pabayo
This Court cannot likewise agree to the petitioner’s attempt to agreed to monetarily settle with PAL during the pendency of the case.
brush aside his refusal to render overtime work as inconsequential when
Graphics, Inc.’s order for him to do so is justified by Graphics, Inc.’s F. CLOSURE OF ESTABLISHMENTS AND REDUCTION OF
contractual commitments to its clients. Such an order is legal under PERSONNEL; AUTHORIZED CAUSES
Article 89 of the Labor Code and the petitioner’s unexplained refusal to A. INSTALLATION OF LABOR SAVING DEVICES
obey is insubordination that merits dismissal from service.
Nonetheless, while the CA finding that the petitioner is entitled 116 Philippine sheet metal workers' union vs the Court of Industrial
to nominal damages as his right to procedural due process was not Relations, Can Company and Liberal Labor Union
respected despite the presence of just causes for his dismissal is
affirmed, this Court finds the CA to have erred in fixing the amount that FACTS:
the Company is liable to pay. The CA should have taken cognizance of The respondent company filed a motion, in the case pending in
the numerous cases decided by this Court where the amount of nominal the court of industrial relations, asking for authority to lay off at least 15
damages was fixed at P30,000.00 if the dismissal was for a just cause. workers in its can department on the ground that the installation and
operation of nine new labor-saving machines in the said department had
F. DRUG USE rendered the services of the said workers unnecessary. Petitioner
alleged that there was more than sufficient work in the company to keep
115 ALEJANDRO ROQUERO vs. PHILIPPINE AIRLINES, INC. all its workers busy.

FACTS: ISSUE:
Roquero, along with Rene Pabayo, were ground equipment WON the laying off of the 15 employees valid
mechanics of respondent Philippine Airlines, Inc.. They were caught red-
handed possessing and using Methampethamine Hydrochloride or shabu HELD:
in a raid conducted by PAL security officers and NARCOM personnel. Yes. There was justification for reducing the number of workers
They alleged that they did not voluntarily indulge in the said act but were in respondent's factory by the introduction of machinery in the
instigated by a certain Jojie Alipato who was introduced to them by manufacture of its products. There is no question as to the right of the
Joseph Ocul, Manager of the Airport Maintenance Division of PAL. Inside manufacturer to use new labor-saving devices with the view to effecting
the company premises, they locked the door and Alipato lost no time in more economy and efficiency in its method of production. But the right to
preparing the drugs to be used. When they started the procedure of reduce personnel should not be abused. It should not be made a pretext
taking the drugs, armed men entered the room, arrested Roquero and for easing out laborers on account of their union activities. But neither
Pabayo and seized the drugs and the paraphernalia used. Roquero and should it be denied when it is shown that they are not discharging their
duties in a manner consistent with good discipline and efficient operation 118 MAGNOLIA DAIRY PRODUCTS CORPORATION vs. NLRC and
of an industrial enterprise. JENNY A. CALIBO
(ONE MONTH NOTICE; SEPARATION PAY)
NOTE: NO DISCRIMINATION; Their selection was made by a committee
composed of both officers and employees who took no account of the FACTS:
laborers' affiliation to the unions and only considered their proven record. Petitioner entered into a contract of service with Skillpower,
Inc., a duly organized corporation engaged in the business of offering
117 EDGAR AGUSTILO vs. COURT OF APPEALS, SAN MIGUEL and providing manpower services to the public. Skillpower, Inc., assigned
CORPORATION Jenny A. Calibo to petitioners Tetra Paster Division. When petitioners
contract with Skillpower, Inc., expired, Calibo applied with Lippercon
FACTS: Services, Inc., also a corporation engaged in providing manpower
Edgar Agustilo was hired by San Miguel Corporation (SMC) as services.
a temporary employee on its Mandaue Brewery in Mandaue, Cebu. He Lippercon Services, Inc., assigned her to petitioners Tetra
was made permanent and designated as a safety clerk and was later Paster Division as a cleaning aide. Later, Calibo was terminated from
transferred to the Engineering Department of the SMC Mandaue Brewery service due to petitioners installation of automated machines prompting
as an administrative secretary. SMC Mandaue Brewery then adopted a Calibo to institute a complaint for illegal dismissal against petitioner.
policy that managers would no longer be assigned secretaries and that In answer thereto, petitioner averred that it has no employer-employee
only director level positions may be given secretaries. As a result, relationship with private respondent and that the dismissal was prompted
petitioner’s position as administrative secretary was abolished and he by the installation of labor saving devices - an authorized cause for
was transferred to the companys Plant Directors Office-Quality dismissal under the Labor Code, as amended.
Improvement Team (PDO-QIT). Petitioner was informed that 584
employees, including him, would be retrenched due to the modernization ISSUE:
program of the company and that his services would be terminated and WON private respondent was legally dismissed since the
that he would be paid his benefits 30 days after he was cleared of all termination of her employment was due to a cause expressly authorized
accountabilities. SMC notified the DOLE of its modernization program. by the Labor Code
Petitioner was given separation pay representing 175% of his
entitlements under the Labor Code. HELD:
Petitioner then filed a complaint against respondents for unfair YES. Article 283 of the Labor Code provides in part that, the
labor practice, illegal dismissal. employer may also terminate the employment of any employee due to
the installation of labor saving devices, x x x, by serving a written notice
ISSUE: on the workers and the Ministry of Labor and Employment at least one
Whether or not petitioner was illegally dismissed. (1) month before the intended date thereof. In case of termination due to
the installation of labor saving devices or redundancy, the worker
HELD: affected thereby shall be entitled to a separation pay equivalent to at
NO. Complainants termination was justified and that least his one (1) month pay or to at least one (1) month pay for every
respondents adhered to the procedural requirements governing the year of service, whichever is higher. x x x A fraction of at least six (6)
same. We have noted very clearly that petitioners separation from months shall be considered one (1) whole year.
employment was brought about by the installation of labor saving The law authorizes an employer, like the herein petitioner, to
devices and machineries pursuant to the employers reorganizational terminate the employment of any employee due to the installation of
and expansion program. The law in this regard allows such a state of labor saving devices. The installation of these devices is a management
change. Art. 283 of the Labor Code allows the reduction of personnel prerogative, and the courts will not interfere with its exercise in the
with the installation of labor saving devices. While we sympathize with absence of abuse of discretion, arbitrariness, or maliciousness on the
the complainant recognizing the considerable period of his employment part of management, as in this case. Nonetheless, this did not excuse
of more than 11 years, yet equally too, we recognize the respondents petitioner from complying with the required written notice to the employee
judgment in the conduct of its business for which the laws do not and to the Department of Labor and Employment (DOLE) at least one
authorize interference. As a matter of fact, the Labor Code and its month before the intended date of termination. This procedure enables
Implementing Rules do not vest in the Labor Arbiters nor in the different an employee to contest the reality or good faith character of the asserted
divisions of the NLRC managerial authority. The employer is free to ground for the termination of his services before the DOLE.
determine, using his own discretion and business judgment, all elements The failure of petitioner to serve the written notice to private
of employment from hiring to firing (National Federation of Labor Union respondent and to the DOLE, however, does not ipso facto make private
v. NLRC, 202 SCRA 346 (1991)). Moreover, the freedom of management respondents termination from service illegal so as to entitle her to
to conduct its business operations to achieve its purpose cannot be reinstatement and payment of backwages. If at all, her termination from
denied (Yuco Chemical Industries v. Min. of Labor, 185 SCRA 727 service is merely defective because it was not tainted with bad faith or
(1990)). For as we see in the case at bench, complainant was not arbitrariness and was due to a valid cause.
discriminated against. In the respondents program of modernization, The well settled rule is that the employer shall be sanctioned
more than 500 others, to be precise, 583 workers, were likewise affected. for non -compliance with the requirements of, or for failure to observe
And we cannot view this as a manifestation of bad faith and insincerity of due process in terminating from service its employee.
respondents taking into account the installation of machineries and
equipment pursuant to the program as a means of streamlining the B. REDUNDANCY
personnel structure. In a program like this, the eventuality of personnel
being removed cannot be avoided. To contend otherwise would be to 119 WILTSHIRE FILE CO., INC. vs. THE NATIONAL LABOR
intrude into the conduct of an enterprise whose main reason for being is RELATIONS COMMISSION and VICENTE T. ONG, respondents.
the profitability of its operations. (DEFINITION)

FACTS: Private respondent Vicente T. Ong was the Sales Manager of


petitioner Wiltshire File Co., Inc. On 13 June 1985, upon private
respondent’s return from a business and pleasure trip abroad, he was such case, there are no allegations which the employee should refute
informed by the President of petitioner Wiltshire that his services were and defend himself from. Thus, to require petitioner Wiltshire to hold a
being terminated. Private respondent maintains that he tried to get an hearing, at which private respondent would have had the right to be
explanation from management of his dismissal but to no avail. On 18 present, on the business and financial circumstances compelling
June 1985, when private respondent again tried to speak with the retrenchment and resulting in redundancy, would be to impose upon the
President of Wiltshire, the company’s security guard handed him a letter employer an unnecessary and inutile hearing as a condition for legality of
which formally informed him that his services were being terminated termination.
upon the ground of redundancy. This is not to say that the employee may not contest the reality
Private respondent filed, on 21 October 1985, a complaint or good faith character of the retrenchment or redundancy asserted as
before the Labor Arbiter for illegal dismissal alleging that his position grounds for termination of services. The appropriate forum for such
could not possibly be redundant because nobody (save himself) in the controversion would, however, be the Department of Labor and
company was then performing the same duties. Employment and not an investigation or hearing to be held by the
Petitioner company alleged that the termination of respondent’s employer itself. It is precisely for this reason that an employer seeking to
services was a cost cutting measure: that in December 1984, the terminate services of an employee or employees because of “closure of
company had experienced an unusually low volume of orders: and that it establishment and reduction of personnel”, is legally required to give a
was in fact forced to rotate its employees in order to save the written notice not only to the employee but also to the Department of
company. Despite the rotation of employees, petitioner alleged; it Labor and Employment at least one month before effectivity date of the
continued to experience financial losses and private respondent’s termination. In the instant case, private respondent did controvert before
position, Sales Manager of the company, became redundant. the appropriate labor authorities the grounds for termination of services
During pendency, petitioner closed its business. LABOR set out in petitioner’s letter to him dated 17 June 1985.
ARBITER ruled that the dismissal was illegal. NLRC held that the
termination was attended by malice and bad faith on the part of NOTES:
petitioner, considering the manner of private respondent was ordered by Art. 283. Closure of establishment and reduction of personnel. ––
the President to pack up and remove his personal belongings from the The employer may also terminate the employment of any employee due
office. to the installation of labor saving devices, redundancy, retrenchment to
prevent losses or the closing or cessation of operation of the
ISSUE: establishment or undertaking unless the closing is for the purpose of
WON his dismissal was illegal. circumventing the provisions of this Title, by serving a written notice on
the workers and the Ministry of Labor and Employment at least one (1)
HELD: month before the intended date thereof. In case of termination due to the
NO, his dismissal was VALID. installation of labor saving devices or redundancy, the worker affected
In the first place, we note that while the letter informing private thereby shall be entitled to a separation pay equivalent to at least his one
respondent of the termination of his services used the word “redundant“, (1) month pay or to at least one (1) month pay for every year of service,
that letter also referred to the company having “incur[red] financial whichever is higher. In case of retrenchment to prevent losses and in
losses which [in] fact has compelled [it] to resort to retrenchment to cases of closures or cessation of operations of establishment or
prevent further losses”. Thus, what the letter was in effect saying was undertaking not due to serious business losses or financial reverses, the
that because of financial losses, retrenchment was necessary, which separation pay shall be equivalent to one (1) month pay or at least one-
retrenchment in turn resulted in the redundancy of private respondent’s half (1/2) month pay for every of service, whichever is higher. A fraction
position. of at least six (6) months shall be considered one (1) whole year.
In the second place, we do not believe that redundancy in an
employer’s personnel force necessarily or even ordinarily refers to 120 COATS MANILA BAY, INC. vs PURITA M. ORTEGA
duplication of work. That no other person was holding the same position (GROUND; CRITERIA)
that private respondent held prior to the termination of his services, does
not show that his position had not become redundant. Indeed, in any FACTS:
well-organized business enterprise, it would be surprising to find Purita M. Ortega and Marina A. Montero were both employed
duplication of work and two (2) or more people doing the work of one by petitioner as Clerk Analysts in the Industrial Engineering Department.
person. We believe that redundancy, for purposes of our Labor On 27 April 2000, petitioner issued a memorandum announcing that a
Code, exists where the services of an employee are in excess of what is redundancy plan would be implemented. It was stated that the
reasonably demanded by the actual requirements of the enterprise. redundancy program was necessary to prevent further losses. Petitioner
Succinctly put, a position is redundant where it is superfluous, and assured its employees that implementing a redundancy program rather
superfluity of a position or positions may be the outcome of a number of than a retrenchment program would result in better benefits to those
factors, such as overhiring of workers, decreased volume of business, or dismissed.
dropping of a particular product line or service activity previously As a result of this redundancy program, 135 employees were
manufactured or undertaken by the enterprise. terminated, including respondents.
Wiltshire, in view of the contraction of its volume of sales and in Respondents received their respective separation payments
order to cut down its operating expenses, effected some changes in its and thereafter executed release waivers and quitclaims in favor of
organization by abolishing some positions and thereby effecting a petitioner. In the meantime, 11 of the terminated employees were rehired
reduction of its personnel. Thus, the position of Sales Manager was by petitioner to different positions but with lower salaries. On 8 June
abolished and the duties previously discharged by the Sales Manager 2000, respondents filed a complaint for illegal dismissal Respondents
simply added to the duties of the General Manager, to whom the Sales asserted in their position paper that despite their dismissal due to
Manager used to report. redundancy, their functions were assigned to other workers.
In the instant case, the ground for dismissal or termination of Petitioner and Tanco claimed that they had the management
services does not relate to a blameworthy act or omission on the part of prerogative to implement a redundancy program as per Article 283 of the
the employee, there appears to us no need for an investigation and Labor Code. They aver that both respondents were notified that they
hearing to be conducted by the employer who does not, to begin with, would be subject to redundancy and that they never objected thereto
allege any malfeasance or non-feasance on the part of the employee. In as shown by the execution of their respective waivers/quitclaims. It
asserts that the implementation of its redundancy program was not positions of route sales and warehouse personnel were declared
discriminatory, and that it implemented reasonable criteria in selecting redundant. Respondent notified the DOLE Director. SMC thereafter wrote
employees to be retrenched. Moreover, the decision to dismiss a letter to petitioner informing him that, owing to the implementation of
respondents was reached after consultations with the Union the "pre-selling operations" scheme, all positions of route and warehouse
Respondents contend that petitioner cannot invoke redundancy since personnel will be declared redundant and the Sum-ag Sales Office will be
there was no showing that the functions of respondents are duplicitous or closed thus petitioner reported to respondent’s Personnel Department at
superfluous. They also assert that petitioner failed to show that it was the Sta. Fe Brewery, pursuant to a previous directive.
suffering from a serious downturn in business that would warrant Thereafter, the employees of Sum-ag sales force were
redundancy given that such serious business downturn was the cause informed that they can avail of respondent’s early retirement package
given by petitioner in the termination letters sent to respondents. They pursuant to the retrenchment program, while those who will not avail of
also assert that their educational attainment is irrelevant since the early retirement would be redeployed or absorbed at the Brewery or
compelling factor in their acceptance of separation pay was the dire other sales offices. Petitioner opted to remain and manifested his
economic necessity to be caused by their impending loss of jobs. willingness to be assigned to any job, considering that he had three
children in college.
ISSUE: Petitioner was surprised when he was informed by the Acting
WON there’s propriety of the redundancy program Personnel Manager that his name was included in the list of employees
implemented by petitioner; who availed of the early retirement package. Petitioner’s request that he
be given an assignment in the company was ignored by the Acting
HELD: Personnel Manager. Petitioner thus filed a complaint for illegal dismissal
YES. Redundancy exists where the services of an employee
are in excess of what is reasonably demanded by the actual ISSUE:
requirements of the enterprise. Succinctly put, a position is redundant Whether or not the dismissal of petitioner is based on a just
where it is superfluous, and superfluity of a position or positions may be and authorized cause.
the outcome of a number of factors, such as over hiring of workers,
decreased volume of business, or dropping of a particular product line or HELD:
service activity previously manufactured or undertaken by the enterprise. NO. The determination that employee’s services are no longer
That no other person was holding the same position prior to the necessary or sustainable and, therefore, properly terminable is an
termination of ones services, does not show that his position had not exercise of business judgment of the employer. The wisdom or
become redundant. Indeed, in any well-organized business enterprise, it soundness of this judgment is not subject to discretionary review of the
would be surprising to find duplication of work and two (2) or more people Labor Arbiter and the NLRC, provided there is no violation of law and no
doing the work of one person. Just like installation of labor-saving showing that it was prompted by an arbitrary or malicious act. In other
devices, the ground of redundancy does not require the exhibition of words, it is not enough for a company to merely declare that it has
proof of losses or imminent losses. In fact, of all the statutory grounds become over manned. It must produce adequate proof that such is
provided in Article 283 of the Labor Code, it is only retrenchment which the actual situation to justify the dismissal of the affected employees for
requires proof of losses or possible losses as justification for termination redundancy. It bears stressing that whether it be by redundancy or
of employment. retrenchment or any of the other authorized causes, no employee may
It is well settled that the characterization of an employee’s be dismissed without observance of the fundamentals of good faith.
services as no longer necessary or sustainable, and, therefore, properly In selecting employees to be dismissed, a fair and reasonable criteria
terminable, is an exercise of business judgment on the part of the must be used, such as but not limited to (a) less preferred status, e.g.
employer. However, the wisdom or soundness of such characterization temporary employee; (b) efficiency; and (c) seniority. In the case at bar,
or decision is not subject to discretionary review provided, of course, that no criterion whatsoever was adopted by respondent in dismissing
violation of law or arbitrary or malicious action is not shown. In several petitioner. Furthermore, as correctly observed by the NLRC, respondent
instances, the Court has held that it is important for a company to have "has not shown how the cessation of operations of the Sum-ag Sales
fair and reasonable criteria in implementing its redundancy program, Office contributed to the ways and means of improving effectiveness of
such as but not limited to, (a) preferred status, (b) efficiency and (c) the organization with the end in view of efficiency and cutting distribution
seniority. overhead and other related costs. Respondent, thus, clearly resorted to
Records shows that respondents positions were abolished sweeping generalization[s] in dismissing complainant."
because there was duplicity of functions of clerk analysts in the Industrial It is not difficult for employers to abolish positions in the guise
Engineering Section and finishing production clerks in the Operations of a cost-cutting measure and we should not be easily swayed by such
Department. Even the union representatives agreed that respondents schemes which all too often reduce to near nothing what is left of the
positions were redundant. Petitioner found that it was more cost-efficient rubble of rights of our exploited workers. Given the nature of petitioner’s
to maintain only one employee to handle the computation of incentives of job as a Warehouse Checker, it is inconceivable that respondent could
the production employees with the use of computers. not accommodate his services considering that the warehousing
operations at Sum -ag Sales Office has not shut down.
121 BONIFACIO ASUFRIN, JR vs SAN MIGUEL CORPORATION
(ADEQUATE PROOF; FUNDAMENTALS OF GOOD FAITH) RUBEN SERRANO vs. NLRC and ISETANN DEPARTMENT STORE
(EXCEPTION: PROOF OF MALICIOUS OR ARBITRARY MANNER)
FACTS:
Asufrin is a Stock Clerk of Coca Cola Plant, then a department FACTS: Petitioner was hired by Isetann Department Store as a security
of respondent San Miguel Beer Corporation. Sometime in 1984, the sales checker to apprehend shoplifters and prevent pilferage of merchandise. 1
office and operations at the Sum-ag, Bacolod City Sales Office were Initially hired on contractual basis but eventually became a regular
reorganized. Several positions were abolished including petitioner’s employee. He then became head of the Security Checkers Section.
position as Stock Clerk. After reviewing petitioner’s qualifications, he was As a cost-cutting measure, private respondent decided to phase out its
designated warehouse checker at the Sum-ag Sales Office. SMC entire security section and engage the services of an independent
implemented a new marketing system known as the "pre-selling security agency. For this reason, it sent the petitioner a notice that in
scheme" at the Sum-ag Beer Sales Office. As a consequence, all view of company’s retrenchment program, he is notified for his
termination as Security Section Head. The loss of his employment abolition and outsourcing of certain functions and in the identification of
prompted petitioner to file a complaint for illegal dismissal. Petitioner certain redundant positions. The letter also states that petitioner will
contends that abolition of private respondent's Security Checkers Section provide the DOLE a list of affected employees as it implements each
and the employment of an independent security agency do not fall phase of the redundancy program. Petitioner, through a letter, notified
under any of the authorized causes for dismissal under Art. 283 of the private respondent of his termination due to the redundancy of his
Labor Code. position and awarded him a separation package. Due to this, respondent
filed with the Labor Arbiter a complaint for illegal dismissal. Respondent
ISSUE: Whether or not there is a valid ground for the dismissal of the alleged there was no independent proof or evidence presented by
complainant. petitioner to substantiate its claim of redundancy. Petitioner on the other
hand, averred that private respondents dismissal from the service was
HELD: Yes. Closure of establishment and reduction of personnel. — The due to redundancy of his position which was determined after petitioners
employer may also terminate the employment of any employee due to business process re-engineering study and organization review,
the installation of labor-saving devices, redundancy, retrenchment to conducted with private respondents knowledge; that redundancy is an
prevent losses or the closing or cessation of operations of the authorized cause to terminate an employee which is a management
establishment or undertaking unless the closing is for the purpose of prerogative and cannot be interfered with absent any abuse of discretion;
circumventing the provisions of this Title, by serving a written notice on and that there is nothing in the law that requires petitioner to conduct
the, workers and the Department of Labor and Employment at least one impartial investigation or hearing to terminate an employee due to
(1) month before the intended date thereof. In case of termination due to redundancy
the installation of labor-saving devices or redundancy, the worker
affected thereby shall be entitled to a separation pay equivalent to at ISSUE: Whether private respondents termination on the ground of
least one (1) month pay or to at least one (1) month pay for every year of redundancy was valid?
service, whichever is higher. In case of retrenchment to prevent losses
and in cases of closure or cessation of operations of establishment or HELD: NO. There was no substantial evidence presented by petitioner
undertaking not due to serious business losses or financial reverses, the to justify private respondent's dismissal due to redundancy. Petitioners
separation pay shall be equivalent to at least one (1) month pay or at evidence to show redundancy merely consisted of a copy of petitioners
least onehalf (1/2) month pay for every year of service, whichever is letter to the DOLE informing the latter of its intention to implement a
higher. A fraction of at least six (6) months shall be considered as one (1) redundancy program and nothing more. The letter which merely stated
whole year. that petitioner undertook a review, restructuring and streamlining of its
organization which resulted in consolidation, abolition and outsourcing of
The "[management of a company] cannot be denied the faculty of certain functions; and which resulted in identified and redundant positions
promoting efficiency and attaining economy by a study of what units are instead of simplifying its business process restructuring, does not satisfy
essential for its operation. To it belongs the ultimate determination of the requirement of substantial evidence, that is, the amount of evidence
whether services should be performed by its personnel or contracted to which a reasonable mind might accept as adequate to justify a
outside agencies . . . [While there] should be mutual consultation, conclusion.
eventually deference is to be paid to what management decides."
Consequently, absent proof that management acted in a malicious or Petitioner failed to demonstrate the superfluity of private respondents
arbitrary manner, the Court will not interfere with the exercise of position as there was nothing in the records that would establish any
judgment by an employer. concrete and real factors recognized by law and relevant jurisprudence,
such as overhiring of workers, decreased volume of business, or
In the case at bar, we have only the bare assertion of petitioner that, in dropping of a particular product line or service activity previously
abolishing the security section, private respondent's real purpose was to manufactured or undertaken by the enterprise, which were adopted
avoid payment to the security checkers of the wage increases provided in by petitioner in implementing the redundancy program.
the collective bargaining agreement approved in 1990. Such an assertion
is not sufficient basis for concluding that the termination of petitioner's Petitioner also failed to show any fair and reasonable criteria in
employment was not a bona fide decision of management to obtain ascertaining what positions are redundant and how the selection of
reasonable return from its investment, which is a right guaranteed to employees to be dismissed was made. Petitioners failure to show an
employers under the Constitution. Indeed, that the phase-out of the authorized cause for private respondents termination is sufficient
security section constituted a "legitimate business decision" is a factual to declare the dismissal illegal.
finding of an administrative agency which must be accorded respect and
even finality by this Court since nothing can be found in the record which Culili v. Eastern Telecommunications Phils., G.R. No. 165381,
fairly detracts from such finding. Accordingly, the termination of February 9, 2011
petitioner's services was for an authorized cause, i.e., redundancy. (REQUISITES – (1) Good Faith, (2) Fair and Reasonable Criteria)

122 CALTEX (PHILS.), INC. vs NLRC & ROMEO T. STO. TOMAS Facts: Nelson Culili was employed by Eastern Telecommunications a
(SUBSTANTIAL EVIDENCE – reasonable mind might accept as Senior Technician. In 1998, due to business losses, ETPI was compelled
adequate to support a conclusion; (1) Proof of superfluity of to implement a Right-Sizing Program which consisted of two phases: the
position, (2) Fair and reasonable criteria) first phase involved the reduction of ETPI’s workforce to only those
employees that were necessary and which ETPI could sustain; the
FACTS: Romeo T. Sto Tomas was a regular employee of Caltex. He was second phase entailed a company-wide reorganization which would
a Senior Accounting Analyst. Caltex informed the Department of Labor result in the transfer, merger, absorption or abolition of certain
and Employment (DOLE) of its plan to implement a redundancy program departments of ETPI. Among the departments abolished was the Service
in its Marketing Division and some departments in its Batangas Refinery. Quality Department. As a result, Culili’s position was abolished due to
The letter alleged that the redundancy program is a response to the redundancy. Upon filing a complaint, the Labor Arbiter rendered a
market situation which constrained petitioner to rationalize and simplify decision finding ETPI guilty of illegal dismissal and unfair labor practice,
its business processes; that petitioner undertook a review, restructuring which was affirmed by the NLRC. However, the Court of Appeals found
and streamlining of its organization which resulted in consolidation, that Culili’s position was validly abolished due to redundancy. It was
highly unlikely that ETPI would effect a company-wide reorganization They are either officers or members of the Baliwag Mahogany
simply for the purpose of getting rid of Culili. Also, ETPI cannot be held Corporation Union -CFW, the existing collective bargaining agent of the
guilty of unfair labor practice as mere contracting out of services being rank and file employees in the company. In 1988, Baliwag Mahogany
performed by union members does not per se amount to unfair labor Corporation (company) and Baliwag Mahogany Corporation Union-CFW
practice unless it interferes with the employees’ right to self-organization. (union) entered into a collective bargaining agreement containing,
among other things, provisions on conversion into cash of unused
Issue: Whether or not there was an illegal dismissal. vacation and sick leaves; grievance machinery procedure; and the right
of the company to schedule work on Sundays and holidays. The union
Ruling: There was a valid dismissal on the ground of redundancy. There made several requests from the company, one of which was the cash
is redundancy when the service capability of the workforce is greater conversion of unused vacation and sick leave for 1987- 1988 and 1988-
than what is reasonably required to meet the demands of the business 1989. The company ruled to allow payment of unused vacation and sick
enterprise. A position becomes redundant when it is rendered leaves for the period of 1987-1988 but disallowed cash conversion of the
superfluous by any number of factors such as over-hiring of workers, 1988-1989 unused leaves. The company issued suspension orders
decrease in volume of business, or dropping a particular product line. affecting twenty (20) employees for failure to render overtime work on
Among the requisites of a valid redundancy program are: (1) the good December 30, 1989. The suspension was for a period of three (3) days.
faith of the employer in abolishing the redundant position; and (2) fair and On the same day, the union filed a notice of strike on the grounds of
reasonable criteria in ascertaining what positions are to be declared unfair labor practice particularly the violation of the CBA provisions on
redundant such as but not limited to: preferred status, efficiency, and non-payment of unused leaves and illegal dismissal of seven (7)
seniority. employees. The company then issued a notice of termination to three (3)
employees or union members, namely, Cecile de Ocampo, Rene
The records show that ETPI had sufficiently established not only its need Villanueva and Marcelo dela Cruz, of the machinery department,
to reduce its workforce and streamline its organization, but also the allegedly to effect cost reduction and redundancy.
existence of redundancy in the position of a Senior Technician. It was
decided that, in the judgment of ETPI management, the specialized Petitioners contend that the company acted in bad faith when it
functions of a Senior Technician whose sole function was essentially the terminated the services of the three mechanics because the positions
repair and servicing of ETPI’s telecommunications equipment was no held by them were not at all abolished but merely given to Gemac
longer needed since the Business and Consumer [Accounts] Department Machineries. On the contrary, the company stresses that when it
had to remain economical and focused yet versatile enough to meet all contracted the services of Gemac Machineries for the maintenance and
the multifarious needs of its small and medium sized clients. It is repair of its industrial machinery, it only adopted a cost saving and cost-
inconceivable that ETPI would effect a company-wide reorganization of consciousness program in order to improve production efficiency.
this scale for the mere purpose of singling out Culili and terminating him.
What ETPI did was to abolish the position itself for being too specialized ISSUE: Whether or not the dismissals of petitioners Cecile de Ocampo,
and limited. Rene Villanueva, and Marcelo dela Cruz from their positions by the
company on the ground of redundancy was done in good faith.
SC finds Culili’s dismissal was for a lawful cause and not an act of unfair
labor practice, ETPI, however, was remiss in its duty to observe HELD: YES. Petitioners' dismissal was justified by redundancy due to
procedural due process in effecting the termination of Culili. In Mayon superfluity and hence legal. We believe that redundancy, for purposes of
Hotel & Restaurant v.Adana, SC observed that the requirement of law our Labor Code, exists where the services of an employee are in excess
mandating the giving of notices was intended: not only to enable the of what is reasonably demanded by the actual requirement of the
employees to look for another employment and therefore ease the impact enterprise. Succinctly put, a position is redundant where it is superfluous,
of the loss of their jobs and the corresponding income, but more and superfluity of a position or positions may be the outcome of a
importantly, to give the Department of Labor and Employment (DOLE) number of factors, such as over hiring of workers, decreased volume of
the opportunity to ascertain the verity of the alleged authorized business, or dropping of a particular product line or service activity
cause of termination. previously manufactured or undertaken by the enterprise. The employer
had no legal obligation to keep in its payroll more employees, than are
With regard to the impleaded corporate officers, they cannot be held necessary for the operation of its business. The reduction of the number
liable for acts done in his official capacity because a corporation, by legal of workers in a company made necessary by the introduction of the
fiction, has a personality separate and distinct from its officers, services of Gemac Machineries in the maintenance and repair of its
stockholders, and members. To pierce this fictional veil, it must be shown industrial machinery is justified. There can be no question as to the right
that the corporate personality was used to perpetuate fraud or an illegal of the company to contract the services of Gemac Machineries to replace
act, or to evade an existing obligation, or to confuse a legitimate issue. In the services rendered by the terminated mechanics with a view to
illegal dismissal cases, corporate officers may be held solidarily liable effecting more economic and efficient methods of production. In the
with the corporation if the termination was done with malice or bad faith. same case, We ruled that "(t)he characterization of (petitioners') services
as no longer necessary or sustainable, and therefore properly terminable,
Culili has failed to prove that his dismissal was orchestrated by the was an exercise of business judgment on the part of (private respondent)
individual respondents herein for the mere purpose of getting rid of him. company. The wisdom or soundness of such characterization or decision
Hence, the dismissal is declared valid but Eastern Telecommunications was not subject to discretionary review on the part of the Labor Arbiter
Philippines, Inc. is ordered to pay petitioner Nelson A. Culili the amount nor of the NLRC so long, of course, as violation of law or merely arbitrary
of P50,000.00 as nominal damages for non-compliance with statutory and malicious action is not shown" (ibid, p. 673).
due process, in addition to the mandatory separation pay required under
Article 283 of the Labor Code. In contracting the services of Gemac Machineries, as part of the
company's cost-saving program, the services rendered by the mechanics
123 CECILE DE OCAMPO, et.al. vs NLRC and BALIWAG became redundant and superfluous, and therefore properly terminable.
MAHOGANY CORPORATION The company merely exercised its business judgment or management
prerogative. And in the absence of any proof that the management
FACTS: Petitioners are employees of Baliwag Mahogany Corporation.
abused its discretion or acted in a malicious or arbitrary manner, the the decision to close the entire establishment or to close or abolish a
court will not interfere with the exercise of such prerogative. department or section thereof for economic reasons, such as to minimize
expenses and reduce capitalization. In the present case, when petitioner
C. RETRENCHMENT decided to cease operating its F & B Department and open the same to a
concessionaire, it did not reduce the number of personnel assigned
124 ALABANG COUNTRY CLUB INC. vs. NLRC, ALABANG thereat. It terminated the employment of ALL personnel assigned at the
COUNTRY CLUB INDEPENDENT EMPLOYEES UNION department. As in the case of retrenchment, however, for the closure
(CONDITIONS – (1) SUBSTANTIAL, (2) REASONABLY IMMINENT, (3) of a business or a department due to serious business losses to be
REASONABLY NECESSARY, (4) SUFFICIENT & CONVINCING regarded as an authorized cause for terminating employees, it must be
EVIDENCE; RETRENCHMENT VS. CLOSURE) proven that the losses incurred are substantial and actual or
reasonably imminent; that the same increased through a period of
FACTS: time; and that the condition of the company is not likely to improve
Francisco Ferrer, then President of ACCI, requested its Internal in the near future. Petitioners failure to prove that the closure of its F &
Auditor, Irene CamposUgalde, to conduct a study on the profitability of B Department was due to substantial losses notwithstanding, this Court
ACCIs Food and Beverage Departmen (F & B Department). In her report, finds that individual respondents were dismissed on the ground of
it showed that F & B Department had been incurring substantial losses. closure or cessation of an undertaking not due to serious business losses
Realizing that it was no longer profitable for ACCI to maintain its own F & or financial reverses, which is allowed under Article 283 of the Labor
B Department, the management decided to cease from operating the Code.
department and to open the same to a contractor, such as a
concessionaire, which would be willing to operate its own food and 125 LOPEZ SUGAR CORPORATION vs. FEDERATION OF FREE
beverage business within the club. ACCI subsequently entered into an WORKERS
agreement with La Tasca Restaurant Inc. (La Tasca), for it to operate the (PREVENT LOSSES; PROOF OF ACTUAL DECLINE OF GROSS AND
F & B Department. Subsequently, ACCI sent its F & B Department NET REVENUES)
employees individual letters informing them that their services were
being terminated one month from the date and that they would be paid FACTS:
separation pay equivalent to one hundred twenty five (125%) percent of Petitioner, allegedly to prevent losses due to major economic
their monthly salary for every year of service. ACCI also informed them problems, and exercising its privilege under their Collective Bargaining
that La Tasca agreed to absorb all affected employees immediately with Agreement ("CBA") entered into between petitioner and Philippine Labor
the status of regular employees without need of undergoing a Union Association ("PLUA-NACUSIP"), caused the retrenchment and
probationary period, and that all affected employees would receive the retirement of a number of its employees. Petitioner filed with DOLE a
same salary they were receiving from ACCI at the time of their combined report on retirement and application for clearance to retrench
termination. The Union, with the authority of individual respondents, affecting eighty six (86) of its employees in order to prevent losses.
filed before the NLRC a complaint for illegal dismissal. The Union and Federation of Free Workers ("FFW"), as the certified bargaining agent
individual respondents alleged that the F & B Division had been reporting of the rank-and-file employees of petitioner, filed a complaint for unfair
gaining profits as shown by the Statement of Income and Deficit labor practices and recovery of union.
prepared by SGV&Co. They thus argued that compliance with the FFW claimed that the terminations undertaken by petitioner
standards for losses to justify their retrenchment was not met by ACCI. were violative of the security of tenure of its members and were intended
ACCI averred, however, that it may exercise management prerogatives to "bust" the union and hence constituted an unfair labor practice. FFW
to adopt a cost-saving and cost-consciousness program to improve claimed that after the termination of the services of its members,
efficiency in its operations, prevent losses, and concentrate on core petitioner advised 110 casuals to report to its personnel office. FFW
businesses, and to lay-off workers and contract out their jobs. further argued that to justify retrenchment, serious business reverses
must be "actual, real and amply supported by sufficient and convincing
ISSUE: evidence.
WON the respondents were dismissed due to retrenchment. Petitioner denied having hired casuals to replace those it had
retired or retrenched. It explained that the announcement calling for 110
HELD: workers to report to its personnel office was only for the purpose of
NO. Retrenchment on the ground of serious business losses is organizing a pool of extra workers which could be tapped whenever
allowed subject to the conditions that (1) the losses expected should be there were temporary vacancies by reason of leaves of absence of
substantial and not merely de minimis in extent; (2) the substantial regular workers.
losses apprehended must be reasonably imminent as such imminence
can be perceived objectively in good faith by the employer; (3) ISSUE:
retrenchment must be reasonably necessary and likely to effectively WON the retrenchment was valid.
prevent the expected losses; and (4) the alleged losses, if already
realized and the expected imminent losses sought to be forestalled, must HELD:
be proven by sufficient and convincing evidence. However, the case NO. Article 283 of the Labor Code provides:
at bar is one involving closure of a business undertaking. Article 283. Closure of establishment and reduction of
Retrenchment is the reduction of personnel for the purpose of personnel. — The employer may also terminate then employment of any
cutting down on costs of operations in terms of salaries and wages employee due to the installation of labor saving devices, redundancy,
resorted to by an employer because of losses in operation of a business retrenchment to prevent losses or the closing or cessation of operation of
occasioned by lack of work and considerable reduction in the volume of the establishment or undertaking unless the closing is for the purpose of
business. circumventing the provisions of this Title, by serving a written notice on
Closure of a business or undertaking due to business the workers and the Ministry of Labor and Employer at least one (1)
losses is the reversal of fortune of the employer whereby there is a month before the intended date thereof. In case of termination due
complete cessation of business operations to prevent further financial to the installation of labor saving devices or redundancy, the worker
drain upon an employer who cannot pay anymore his employees since affected thereby shall be entitled to a se pay equivalent to at least his
business has already stopped. One of the prerogatives of management is one (1) month pay or to at least one (1) month pay for every year of
service, whichever is higher. In case of retrenchment to prevent losses Antipuesto, et al., consulted with the Regional Director of the Department
and in cases, of closures or cessation of operations of establishment or of Labor and Employment ("DOLE") who opined that it would be best for
undertaking not due to serious business losses or financial reverses, the them to receive the separation pay being offered by the corporation. His
separation pay shall be equivalent to one (1) month pay or at least one advice was heeded.
half (1/2) month pay for every year of service, whichever is higher. A The subsequent receipt of their separation pay benefits,
fraction of at least six (6) months shall be considered one (1) whole year. nevertheless, did not deter Antipuesto, et al., from later going through
(Emphasis supplied) The phrase "to Prevent losses" means with their complaint for illegal dismissal against the corporation. The
that retrenchment or termination of the services of some employees is charge averred that the retrenchment program was a mere subterfuge
authorized to be undertaken by the employer sometime before the losses used by Edge Apparel to give a semblance of regularity and validity to
anticipated are actually sustained or realized. It is not, in other words, the the dismissal of the complainants.
intention of the lawmaker to compel the employer to stay his hand and Edge Apparel countered that its financial obligations,
keep all his employees until sometime after losses shall have in fact amounting to about P8 Million, had begun to eat up most of its capital
materialized. outlay and resulted in unabated losses of P681,280.00 in 1989,
We consider it may be useful to sketch the general standards P262,741.00 in 1990, P162,170.00 in 1991 and P749,294.00 in 1992,
in terms of which the acts of petitioner employer must be appraised. constraining the company to adopt and implement a retrenchment
Firstly, the losses expected should be substantial and not merely de program.
minimis in extent. If the loss purportedly sought to be forestalled by
retrenchment is clearly shown to be insubstantial and inconsequential in ISSUE:
character, the bona fide nature of the retrenchment would appear to be WON there was a valid retrenchment.
seriously in question. Secondly, the substantial loss apprehended must
be reasonably imminent, as such imminence can be perceived HELD:
objectively and in good faith by the employer. There should, in other YES. Retrenchment is an economic ground to reduce the
words, be a certain degree of urgency for the retrenchment, which is after number of employees. In order to be justified, the termination of
all a drastic recourse with serious consequences for the livelihood of the employment by reason of retrenchment must be due to business losses
employees retired or otherwise laid-off. Because of the consequential or reverses which are serious, actual and real. Not every loss incurred
nature of retrenchment, it must, thirdly, be reasonably necessary and or expected to be incurred by the employer will justify retrenchment,
likely to effectively prevent the expected losses. The employer should since, in the nature of things, the possibility of incurring losses is
have taken other measures prior or parallel to retrenchment to forestall constantly present, in greater or lesser degree, in carrying on the
losses, i.e., cut other costs than labor costs. An employer who, for business operations.
instance, lays off substantial numbers of workers while continuing Retrenchment is normally resorted to by management during
to dispense fat executive bonuses and perquisites or so-called "golden periods of business reverses and economic difficulties occasioned by
parachutes", can scarcely claim to be retrenching in good faith to avoid such events as recession, industrial depression, or seasonal fluctuations.
losses. To impart operational meaning to the constitutional policy of It is an act of the employer of reducing the work force because of losses
providing "full protection" to labor, the employer's prerogative to bring in the operation of the enterprise, lack of work, or considerable reduction
down labor costs by retrenching must be exercised essentially as a on the volume of business. Retrenchment is, in many ways, a measure of
measure of last resort, after less drastic means — e.g., reduction of both last resort when other less drastic means have been tried and found to
management and rank-and-file bonuses and salaries, going on reduced be inadequate. A lull caused by lack of orders or shortage of materials
time, improving manufacturing efficiencies, trimming of marketing must be of such nature as would severely affect the continued business
and advertising costs, etc. — have been tried and found wanting. operations of the employer to the detriment of all and sundry if not
Lastly, but certainly not the least important, alleged if already realized, properly addressed. The institution of "new methods or more efficient
and the expected imminent losses sought to be forestalled, must be machinery, or of automation" is technically a ground for termination of
proved by sufficient and convincing evidence. The reason for requiring employment by reason of installation of labor-saving devices but
this quantum of proof is readily apparent: any less exacting standard of where the introduction of these methods is resorted to not merely to
proof would render too easy the abuse of this ground for termination of effect greater efficiency in the operations of the business but principally
services of employees. because of serious business reverses and to avert further losses, the
In this case, there was no proof of actual declining gross and device could then verily be considered one of retrenchment. In this case,
net revenues submitted. No audited financial statements showing the the Labor Arbiter and the NLRC both concluded that there had been a
financial condition of petitioner corporation during the above mentioned valid ground for the retrenchment of private respondents. The documents
crop years were submitted. Since financial statements audited by presented in evidence were found to "conclusively show that (petitioner)
independent external auditors constitute the normal method of proof of suffered serious financial losses." The general standards or elements
the profit and loss performance of a company, it is not easy to needed for the retrenchment to be valid — i.e., that the losses expected
understand why petitioner should have failed to submit such financial are substantial and not merely de minimis in extent; that the expected
statements. Petitioner made passing reference to cost reduction losses are reasonably imminent such as can be perceived objectively
measures it had allegedly undertaken, it was, once more, a fairly and in good faith by the employer; that the retrenchment is reasonably
conspicuous failure to specify the cost-reduction measures actually necessary and likely to effectively prevent the expected losses; and that
undertaken in good faith before resorting to retrenchment. the imminent losses sought to be forestalled are substantiated — were
adequately shown in the present case.
126 EDGE APPAREL, INC. vs NLRC
(JUSTIFIED – Serious, actual and real business loss; LAST 127 FE S. SEBUGUERO vs NLRC and G.T.I. SPORTSWEAR
RESORT) CORPORATION
(BASIC REQUISITES; MANDATORY NOTICE; MERELEY
FACTS: DEFECTIVE)
Pursuing its retrenchment program, petitioner Edge Apparel,
Inc., dismissed private respondents Josephine Antipuesto, Norina Ando, FACTS:
Juliet Baguio, Apolinaria Velonta, Corazon Pino and Josephine Cañete Petitioners were among the thirty-eight (38) regular employees
from employment effective 03 September 1992. Feeling aggrieved, of private respondent GTI Sportswear Corporation, who were given
"temporary lay-off" notices by the latter due to alleged lack of work and notice because by this time, their lay-off is to become permanent and
heavy losses caused by the cancellation of orders from abroad and by they were definitely losing their employment.
the garments embargo of 1990. The lack of written notice to the petitioners and to the DOLE
Believing that their "temporary lay-off" was a ploy to dismiss does not, however, make the petitioners' retrenchment illegal such that
them, resorted to because of their union activities and was in violation of they are entitled to the payment of back wages and separation pay in lieu
their right to security of tenure since there was no valid ground therefor, of reinstatement as they contend. Their retrenchment, for not having
the 38 laid-off employees filed with the Labor Arbiter's office in the been effected with the required notices, is merely defective. In those
National Capital Region complaints for illegal dismissal, unfair labor cases where we found the retrenchment to be illegal and ordered the
practice, underpayment of wages under Wage Orders Nos. 01 and 02, employees' reinstatement and the payment of back wages, the validity of
and non -payment of overtime pay and 13th month pay. the cause for retrenchment, that is the existence of imminent or actual
GTI denied the claim of illegal dismissal and asserted that it serious or substantial losses, was not proven. But here, such a cause is
was its prerogative to lay-off its employees temporarily for a period not present as found by both the Labor Arbiter and the NLRC. There is only a
exceeding six months to prevent losses due to lack of work or job orders violation by GTI of the procedure prescribed in Article 283 of the Labor
from abroad, and that the lay-off affected both union and non-union Code in effecting the retrenchment of the petitioners. It is now settled that
members. It justified its failure to recall the 38 laid-off employees after the where the dismissal of an employee is in fact for a just and valid cause
lapse of six months because of the subsequent cancellations of job and is so proven to be but he is not accorded his right to due process,
orders made by its foreign principals, a fact which was communicated to i.e., he was not furnished the twin requirements of notice and the
the petitioners and the other complainants who were all offered opportunity to be heard, the dismissal shall be upheld but the employer
severance pay must be sanctioned for non-compliance with the requirements of
or for failure to observe due process.
ISSUE:
WON there was a valid retrenchment. 128 Plastimer Industrial Corp. v. Gopo, G.R. No. 183390, February
16, 2011
HELD: (DEFECTIVE NOTICE AND TEMPORARY NET INCOME DOES NOT
YES but it is defective due to lack of procedural notice. RENDER RETRENCHMENT ILLEGAL)
Retrenchment, on the other hand, is used interchangeably with the term
"lay-off." It is the termination of employment initiated by the employer Facts:
through no fault of the employee's and without prejudice to the latter, The Personnel and Administration Manager of Plastimer issued
resorted to by management during periods of business recession, a Memorandum informing all its employees of the decision of the Board
industrial depression, or seasonal fluctuations, or during lulls occasioned of Directors to downsize and reorganize its business operations due to
by lack of orders, shortage of materials, conversion of the plant for a new withdrawal of investments and shares of stocks which resulted in the
production program or the introduction of new methods or more efficient change of its corporate structure. On 14 May 2004, the employees of
machinery, or of automation. Plastimer, including respondent Gopo and other employees were served
Simply put, it is an act of the employer of dismissing employees written notices of their termination effective 13 June 2004. Plastimer and
because of losses in the operation of a business, lack of work, and Plastimer Industrial Corporation Christian Brotherhood (PICCB), the
considerable reduction on the volume of his business, a right consistently incumbent sole and exclusive collective bargaining representative of all
recognized and affirmed by this Court. rank and file employees, entered into a Memorandum of Agreement
To determine, therefore, whether the petitioners were validly (MOA) relative to the terms and conditions that would govern the
retrenched or were illegally dismissed, we must determine whether there retrenchment of the affected employees. On 26 May 2004, Plastimer
was compliance with the law regarding a valid retrenchment at anytime submitted to the DOLE an Establishment Termination Report containing
within the six month-period that they were temporarily laid-off. Under the the list of the employees affected by the reorganization and downsizing.
aforequoted Article 283 of the Labor Code, there are three basic The affected employees, including respondents, signed individual
requisites for a valid retrenchment: “Release Waiver and Quitclaim.”
(1) the retrenchment is necessary to prevent losses and such Thereafter, respondents filed a complaint against Plastimer and
losses are proven; its President Teo Kee Bin (petitioners) before the Labor Arbiter for illegal
(2) written notice to the employees and to the Department of dismissal with prayer for reinstatement and full backwages,
Labor and Employment at underpayment of separation pay, moral and exemplary damages and
least one month prior to the intended date of retrenchment; and attorney’s fees.
(3) payment of separation pay equivalent to one month pay or Respondents alleged that they did not voluntarily relinquish
at least 1/2 month pay for every year of service, whichever is higher. their jobs and that they were required to sign the waivers and quitclaims
The requirement of notice to both the employees concerned without giving them an opportunity to read them and without explaining
and the Department of Labor and Employment (DOLE) is mandatory their contents; and that Plastimer failed to establish the causes/valid
and must be written and given at least one month before the reasons for the retrenchment and to comply with the one-month notice to
intended date of retrenchment. In this case, it is undisputed that the the DOLE as well as the standard prescribed under the Collective
petitioners were given notice of the temporary layoff. There is, however, Bargaining Agreement between Plastimer and the employees. Petitioners
no evidence that any written notice to permanently retrench them was countered that the retrenchment was a management prerogative and that
given at least one month prior to the date of the intended retrenchment. respondents got their retrenchment or separation pay even before the
The NLRC found that GTI conveyed to the petitioners the impossibility of effective date of their separation from service.
recalling them due to the continued unavailability of work. But what the The Labor Arbiter ruled in favor of petitioners. It held that
law requires is a written notice to the employees concerned and petitioners were able to prove that there was a substantial withdrawal of
that requirement is mandatory. The notice must also be given at least stocks that led to the downsizing of the workforce; that notice to the
one month in advance of the intended date of retrenchment to enable the affected employees were given on 14 May 2004, 30 days before its
employees to look for other means of employment and therefore to ease effective date on 14 June 2004, and it was only the notice to the DOLE
the impact of the loss of their jobs and the corresponding income. That that was filed short of the 30-day period; that respondents claimed their
they were already on temporary lay-off at the time notice should have separation pay in accordance with the MOA; and that respondents could
been given to them is not an excuse to forego the one-month written not claim ignorance of the contents of the waivers and quitclaims
because they were assisted by the union President and their counsel in The Court has ruled that a waiver or quitclaim is a valid and
signing them. binding agreement between the parties, provided that it constitutes a
On appeal, the NLRC affirmed the Labor Arbiter’s decision. credible and reasonable settlement, and that the one accomplishing it
The Court of Appeals reversed the NLRC decision and found has done so voluntarily and with a full understanding of its import. We
that petitioners have been illegally dismissed. agree with the Labor Arbiter and the NLRC that respondents were
The Court of Appeals ruled that there was no valid cause for sufficiently apprised of their rights under the waivers and quitclaims that
retrenchment; that while Plastimer claimed financial losses from 2001 to they signed. Each document contained the signatures of Marcaida,
2004, records showed an improvement of its finances in 2003; that PICCB President, and Atty. Diwa, the counsel for the union, which
Plastimer failed to use a reasonable and fair standard or criteria in proved that respondents were duly assisted when they signed the
ascertaining who would be dismissed and who would be retained among waivers and quitclaims. Further, Marcaida’s letter to Teo Kee Bin, dated
its employees; that the MOA between Plastimer and PICCB only 28 May 2004, proved that proper assistance was extended upon
recognized the need for partial retrenchment and the computation of respondents. Hence, we rule that the waivers and quitclaims that
retrenchment pay without disclosing the criteria in the selection of the respondents signed were valid.
employees to be retrenched; and that the union President and the WHEREFORE, we SET ASIDE the Decision and Resolution of
PICCB’s counsel were not present when the retrenched employees were the Court of Appeals, and hereby REINSTATE the Decision of the Labor
made to sign the waivers and quitclaims. Arbiter and the Resolution of the NLRC upholding the validity of
Hence, the petition before this Court. respondents’ retrenchment with MODIFICATION that petitioners pay
each of the
Issue: respondents the amount of P30,000 as nominal damages for non-
WON respondents were illegally retrenched by petitioners. compliance with statutory due process.

Ruling: 129-130 Internation management Services vs. Logarta, G.R. No.


The petition has merit. 163657, April 18, 2012
This Court is not precluded from reviewing the factual issues (5 STRICT REQUIREMENTS)
when there are conflicting findings by the Labor Arbiter, the NLRC and
the Court of Appeals. In this case, we find that the findings of the Labor Facts:
Arbiter and the NLRC are more in accord with the evidence on record. Recruitment agency, International Management Services
One-Month Notice of Termination of Employment Article 283 of (IMS), owned and operated by Marilyn C. Pascual, deployed respondent
the Labor Code provides: Roel P. Logarta to work for Petrocon Arabia Limited (Petrocon) in
ART. 283. Closure of establishment and reduction of Alkhobar, Kingdom of Saudi Arabia, in connection with general
personnel. - The employer may also terminate the employment of any engineering services of Petrocon for the Saudi Arabian Oil Company
employee due to the installation of labor-saving devices, redundancy, (Saudi Aramco). Respondent was employed for a period of two (2) years,
retrenchment to prevent losses or the closing or cessation of operation of commencing on October 2, 1997, with a monthly salary of eight hundred
the establishment or undertaking unless the closing is for the purpose of US Dollars (US$800.00).
circumventing the provisions of this Title, by serving a written notice on On April 29, 1998, Saudi Aramco notified Petrocon that due to
the workers and the Department of Labor and Employment at least one changes in the general engineering services work forecast for 1998, the
(1) month before the intended date thereof. xxx man hours that were formerly allotted to Petrocon is going to be reduced
In this case, Plastimer submitted the notice of termination of employment by 40% which constrained Petrocon to reduce its personnel.
to the DOLE on 26 May 2004. However, notice to the affected employees Thus, on June 1, 1998, Petrocon gave respondent a written notice
were given to them on 14 May 2004 or 30 days before the effectivity of informing the latter that due to the lack of project works related to his
their termination from employment on 13 June 2004. While notice to the expertise, he is given a 30-day notice of termination, and that his last
DOLE was short of the one-month notice requirement, the affected day of work with Petrocon will be on July 1, 1998. Petrocon also informed
employees were sufficiently informed of their retrenchment 30 days respondent that all due benefits in accordance with the terms and
before its effectivity. Petitioners’ failure to comply with the one-month conditions of his employment contract will be paid to respondent,
notice to the DOLE is only a procedural infirmity and does not render including his ticket back to the Philippines.
the retrenchment illegal. In Agabon v. NLRC, we ruled that when the Before his departure from Saudi Arabia, respondent received his final
dismissal is for a just cause, the absence of proper notice should not paycheck from Petrocon amounting SR7,488.57.
nullify the dismissal or render it illegal or ineffectual. Instead, the Upon his return, respondent filed a complaint with the Regional
employer should indemnify the employee for the violation of his statutory Arbitration Branch VII, National Labor Relations Commission (NLRC),
rights. Here, the failure to fully comply with the one-month notice of Cebu City, against petitioner as the recruitment agency which employed
termination of employment did not render the retrenchment illegal but it him for employment abroad. In filing the complaint, respondent sought to
entitles respondents to nominal damages. recover his unearned salaries covering the unexpired portion of his
Validity of Retrenchment employment contract with Petrocon on the ground that he was illegally
The fact that there was a net income in 2003 does not justify dismissed.
the Court of Appeals’ ruling that there was no valid reason for the The Labor Arbiter rendered judgment in favor of the respondent
retrenchment. and ordered petitioner to pay the peso equivalent of US$5,600.00 based
Records showed that the net income of P6,185,707.05 for 2003 on the rate at the time of actual payment, as payment of his wages for
was not even enough for petitioners to recover from the P52,904,297.88 the unexpired portion of his contract of employment. The NLRC on
loss in 2002. Article 283 of the Labor Code recognizes retrenchment to appeal affirmed the Labor Arbiter’s decision but reduced the award to
prevent losses as a right of the management to meet clear and only US$4,800.00 or its peso equivalent at the time of payment. The CA
continuing economic threats or during periods of economic recession to likewise dismissed the petition and affirmed the NLRC decision.
prevent losses. There is no need for the employer to wait for substantial
losses to materialize before exercising ultimate and drastic option to Issue: Whether or not respondents dismissal through retrenchment
prevent such losses. illegal.
Validity of Waivers and Quitclaims
Ruling: No. Retrenchment is the reduction of work personnel usually due
to poor financial returns, aimed to cut down costs for operation 8042. A plain reading of the said provision clearly reveals that it applies
particularly on salaries and wages. It is one of the economic grounds to only to an illegally dismissed overseas contract worker or a worker
dismiss employees and is resorted by an employer primarily to avoid or dismissed from overseas employment without just, valid or authorized
minimize business losses. cause.
Retrenchment programs are purely business decisions within In the case at bar, notwithstanding the fact that respondent's
the purview of a valid and reasonable exercise of management termination from his employment was procedurally infirm, having not
prerogative. It is one way of downsizing an employer's workforce and is complied with the notice requirement, nevertheless the same remains to
often resorted to by the employer during periods of business recession, be for a just, valid and authorized cause, i.e., retrenchment as a valid
industrial depression, or seasonal fluctuations, and during lulls in exercise of management prerogative. To stress, despite the employer's
production occasioned by lack of orders, shortage of materials, failure to comply with the one-month notice to the DOLE prior to
conversion of the plant for a new production program, or introduction of respondent's termination, it is only a procedural infirmity which does not
new methods or more efficient machinery or automation. It is a valid render the retrenchment illegal. In Agabon v. NLRC, this Court ruled that
management prerogative, provided it is done in good faith and the when the dismissal is for a just cause, the absence of proper notice
employer faithfully complies with the substantive and procedural should not nullify the dismissal or render it illegal or ineffectual. Instead,
requirements laid down by law and jurisprudence. the employer should indemnify the employee for violation of his statutory
Philippine Law recognizes retrenchment as a valid cause rights. Consequently, it is Article 283 of the Labor Code and not Section
for the dismissal of a migrant or overseas Filipino worker under 10 of R.A. No. 8042 that is controlling. Thus, respondent is entitled to
Article 283 of the Labor Code. payment of separation pay equivalent to one (1) month pay, or at least
Thus, retrenchment is a valid exercise of management one-half (1/2) month pay for every year of service, whichever is higher.
prerogative subject to the strict requirements set by jurisprudence, to wit: Considering that respondent was employed by Petrocon for a period of
(1)That the retrenchment is reasonably necessary and likely eight (8) months, he is entitled to receive one (1) month pay as
to prevent business losses which, if already incurred, are not merely de separation pay. In addition, pursuant to current jurisprudence, for failure
minimis, but substantial, serious, actual and real, or if only expected, are to fully comply with the statutory due process of sufficient notice,
reasonably imminent as perceived objectively and in good faith by the respondent is entitled to nominal damages in the amount P50,000.00.
employer;
(2)That the employer served written notice both to the 131 WATERFRONT CEBU CITY HOTEL, Petitioner, v. MA. MELANIE
employees and to the Department of Labor and Employment at least P. JIMENEZ, JACQUELINE C. BAGUIO, LOVELLA V. CARILLO, and
one month prior to the intended date of retrenchment; MAILA G. ROBLE, Respondents.
(3)That the employer pays the retrenched employees (UNIQUE DEFENSE – ONLY THE DEPARTMENT IS INCURRING THE
separation pay equivalent to one month pay or at least 1/2 month pay LOSS, NOT THE OVERALL HOTEL PERFORMANCE)
for every year of service, whichever is higher;
(4)That the employer exercises its prerogative to retrench FACTS:
employees in good faith for the advancement of its interest and not to Herein respondents were hired for Club Waterfront, a division
defeat or circumvent the employees' right to security of tenure; and under petitioner Waterfront Cebu City Hotel which catered to foreign high
(5)That the employer used fair and reasonable criteria in stakes gamblers for different positions.
ascertaining who would be dismissed and who would be retained among On 12 May 2003, respondents, along with 41 other employees,
the employees, such as status,…efficiency, seniority, physical fitness, received identical letters of termination from petitioner Director of Human
age, and financial hardship for certain workers. 28 Applying the above- Resources informing them of the temporary suspension of business of
stated requisites for a valid retrenchment in the case at bar, it is apparent the Club.
that the first, fourth and fifth requirements were complied with by The following day, petitioner served the notice of suspension of
respondent's employer. However, the second and third requisites were business with the DOLE). The dismissed employees were offered
absent when Petrocon terminated the services of respondent. separation pay equivalent to half-month pay for every year of service.
As aptly found by the NLRC and justly sustained by the CA, The Club closure took effect on 15 June 2003.
Petrocon exercised its prerogative to retrench its employees in good faith On 26 June 2003, respondents filed a complaint before the
and the considerable reduction of work allotments of Petrocon by Saudi Labor Arbiter for illegal dismissal, illegal suspension, and non-payment of
Aramco was sufficient basis for Petrocon to reduce the number of its salaries and other monetary benefits. They likewise prayed for damages
personnel. and attorney fees.
As for the notice requirement, however, contrary to petitioner's Respondents maintained that they are employees of petitioner
contention, proper notice to the DOLE within 30 days prior to the assigned to the Club, hence they should have been allowed to work in
intended date of retrenchment is necessary and must be complied with other departments of the hotel.
despite the fact that respondent is an overseas Filipino worker. In the Oppositely, petitioner averred that since April 2002, the Club
present case, although respondent was duly notified of his termination by has been incurring losses that it had to temporarily cease its operations
Petrocon 30 days before its effectivity, no allegation or proof was effective 15 June 2003. To support the allegations of losses, petitioner
advanced by petitioner to establish that Petrocon ever sent a notice to presented financial statements of Waterfront Promotion, Ltd. Petitioner
the DOLE 30 days before the respondent was terminated. Thus, this argued that pursuant to Article 286 of the Labor Code, the temporary
requirement of the law was not complied with. suspension of business operations does not terminate employment.
In the case at bar, despite the fact that respondent was Thus, respondents have no cause of action against them.
employed by Petrocon as an OFW in Saudi Arabia, still both he and his On 12 December 2003, the labor arbiter ruled in favor of
employer are subject to the provisions of the Labor Code when petitioner and upheld the closure of the Club business operations as a
applicable. The basic policy in this jurisdiction is that all Filipino workers, management prerogative. The petitioner was, however, directed to
whether employed locally or overseas, enjoy the protective mantle of comply with Article 283 of the Labor Code and to pay complainants their
Philippine labor and social legislations. separation pay equivalent to one-half month pay for every year of
Also, respondent is entitled to the payment of his separation service, a fraction of at least 6 months being considered as one year.
pay. However, this Court disagrees with the conclusion of the Labor Respondents appealed to the NLRC which issued a Decision
Arbiter, the NLRC and the CA, that respondent should be paid his affirming the ruling of the Labor Arbiter. After the denial of respondents
separation pay in accordance with the provision of Section 10 of R.A. No.
motion for reconsideration, they elevated the case to the Court of Verily, retrenchment and not closure was effected to warrant
Appeals. the valid dismissal of respondents. Petitioner has not totally ceased its
Respondents argued that the NLRC should have considered operations. It merely closed down a department.
the financial statements of the petitioner Hotel and not merely of the Retrenchment is the termination of employment initiated by the
Club, which is only a division of the Hotel. According to respondents, the employer through no fault of and without prejudice to the employees. It is
permanent closure of the Club resulted in retrenchment but petitioner resorted to during periods of business recession, industrial depression, or
failed to prove that it complied with the standards for retrenchment. On 5 seasonal fluctuations or during lulls occasioned by lack of orders,
July 2006, the Court of Appeals rendered a Decision reversing the shortage of materials, conversion of the plant for a new production
findings and conclusions of the NLRC. program or the introduction of new methods or more efficient machinery
The appellate court found that petitioner Hotel is the actual or of automation. It is an act of the employer of dismissing employees
employer of respondents, thus the evidence of losses and closure of the because of losses in the operation of a business, lack of work, and
Club is immaterial and irrelevant. considerable reduction on the volume of his business.
Petitioner filed a motion for reconsideration but it was denied in In case of retrenchment, proof of financial losses becomes the
a Resolution dated 15 August 2006. Hence, this petition for review on determining factor in proving its legitimacy. In establishing a unilateral
certiorari. claim of actual or potential losses, financial statements audited by
independent external auditors constitute the normal method of proof of
ISSUE: profit and loss performance of a company. The condition of business
Whether or not the evidence of losses and closure of Club losses justifying retrenchment is normally shown by audited financial
Waterfront is immaterial and irrelevant to the termination of petitioners? documents like yearly balance sheets and profit and loss statements as
well as annual income tax returns.
HELD: Retrenchment is subject to faithful compliance with the
The ruling of the Court of Appeals is reversed and set substantative and procedural requirements laid down by law and
aside. jurisprudence. For a valid retrenchment, the following elements must be
At the outset, it should be stated that the respondents cannot present:
be accommodated in other departments of the Hotel. The duties and (1) That retrenchment is reasonably necessary and likely to
functions they perform are peculiar to the positions they hold in the Club. prevent business losses which, if already incurred, are not merely de
It is likewise undisputed that the Club remained closed and there is no minimis, but substantial, serious, actual and real, or if only expected, are
other department in the Hotel similar to the Club and which catered to reasonably imminent as perceived objectively and in good faith by the
foreign high stakes gamblers. Verily, reinstatement cannot be and could employer;
not have been an option for petitioner Hotel. (2) That the employer served written notice both to the
For the purpose of proving financial losses, petitioner employees and to the Department of Labor and Employment at least one
presented the financial statements of Waterfront Promotion, Ltd. which month prior to the intended date of retrenchment;
petitioner describes as the company which promotes, markets and (3) That the employer pays the retrenched employees
finances the Club. separation pay equivalent to one
A review of the corporate structure of the Club as contained in (1) month pay or at least ½ month pay for every year of service,
the financial statements submitted by petitioner reveals that it is actually whichever is higher;
a wholly-owned subsidiary of Waterfront Promotion, Ltd. Strictly (4) That the employer exercises its prerogative to retrench
speaking, the Club is not related to petitioner except to say that they are employees in good faith for the advancement of its interest and not to
two different subsidiaries of one parent corporation, i.e., Waterfront defeat or circumvent the employees right to security of tenure; and
Philippines. Petitioner, then, could have right at the beginning avoided (5) That the employer used fair and reasonable criteria in
the conflict with respondents by setting itself apart from them. Petitioner ascertaining who would be dismissed and who would be retained among
could have invoked the separateness from the Hotel of the Club which the employees, such as status, efficiency, seniority, physical fitness, age,
employed respondents. Petitioner did not do so. Instead, and at the and financial hardship for certain workers.
outset, it formally presented itself as the respondents employer when, All these elements were successfully proven by petitioner.
through its Director of Human Resources, it informed respondents about First, the huge losses suffered by the Club for the past two years had
the temporary suspension of the business of the Club and forthwith forced petitioner to close it down to avert further losses which would
served the notices of suspension of business on DOLE. eventually affect the operations of petitioner. Second, all 45 employees
The consolidated financial statements that were prepared in working under the Club were served with notice of termination. The
the name of Waterfront Promotion refer to the casino operations of the corresponding notice was likewise served to the DOLE one month prior
Club. A consolidated financial statement is usually prepared for a parent to retrenchment. Third, the employees were offered separation pay,
company and its subsidiaries, the purpose of which is to provide an most of whom have accepted and opted not to join in this complaint.
overview of the financial condition of the group of companies as a single Fourth, cessation of or withdrawal from business operations was bona
entity. The Club, being a wholly-owned subsidiary of Waterfront fide in character and not impelled by a motive to defeat or circumvent the
Promotion, Ltd. operates under the management, supervision and control tenurial rights of employees. Neither is there a showing that petitioner
of Waterfront Promotion, Ltd. The relationship between these two carried out the closure of the business in bad faith. No labor dispute
companies is so intertwined that the Club is practically considered a existed between management and the employees when the latter were
department or division of Waterfront Promotion, Ltd. terminated.
A review of the consolidated financial statement proves
petitioner assertion that the losses there reflected refer to the losses of 132 Legend Hotel (Manila) vs Realuyo AKA Roa
the Club. The consolidated financial statement and the corporate (SUFFICIENT AND CONVINCING EVIDENCE)
relationships it indicates, cannot, however, be relied upon by petitioner to
avoid this particular labor dispute because, as already stated, petitioner Facts:
itself has been claiming from the very beginning that the Club is only a Respondent averred that he had worked as a pianist at the
division/department of the hotel. Legend Hotel’s Tanglaw Restaurant from September 1992 with an initial
rate of P400.00/night that was given to him after each night’s
performance; that his rate had increased to P750.00/night; and that
during his employment, he could not choose the time of performance, reserves the right to control both the end achieved and the manner and
which had been fixed from 7:00 pm to 10:00 pm for three to six means used to achieve that end.
times/week. He added that the Legend Hotel’s restaurant manager had A review of the records shows, however, that respondent
required him to conform with the venue’s motif; that he had been performed his work as a pianist under petitioner’s supervision and
subjected to the rules on employees’ representation checks and chits, a control. Specifically, petitioner’s control of both the end achieved and the
privilege granted to other employees; that on July 9, 1999, the manner and means used to achieve that end was demonstrated by the
management had notified him that as a cost-cutting measure his services following, to wit: a. He could not choose the time of his performance,
as a pianist would no longer be required effective July 30, 1999; that he which petitioners had fixed from 7:00 pm to 10:00 pm, three to six times a
disputed the excuse, insisting that Legend Hotel had been lucratively week; b. He could not choose the place of his performance; c. The
operating as of the filing of his complaint; and that the loss of his restaurant’s manager required him at certain times to perform only
employment made him bring his complaint. Tagalog songs or music, or to wear barong Tagalog to conform to the
Filipiniana motif; and d. He was subjected to the rules on employees’
Issues: representation check and chits, a privilege granted to other employees.
1. Whether or not petition for certiorari to the CA is proper. Relevantly, it is worth remembering that the employer need not actually
2. Whether or not there is ER-EE relationship. supervise the performance of duties by the employee, for it sufficed that
3. Whether or not retrenchment as a ground for respondent’s the employer has the right to wield that power.
dismissal is valid. NO. Retrenchment is one of the authorized causes for the
dismissal of employees recognized by the Labor Code. It is a
Held: management prerogative resorted to by employers to avoid or to
YES. There is no longer any doubt that a petition for certiorari minimize business losses. On this matter, Article 283 of the Labor Code.
brought to assail the decision of the NLRC may raise factual issues, and The Court has laid down the following standards that an
the CA may then review the decision of the NLRC and pass upon such employer should meet to justify retrenchment and to foil abuse, namely:
factual issues in the process.8 The power of the CA to review factual (a) The expected losses should be substantial and not merely de minimis
issues in the exercise of its original jurisdiction to issue writs of certiorari in extent; (b) The substantial losses apprehended must be reasonably
is based on Section 9 of Batas Pambansa Blg. 129, which pertinently imminent; (c) The retrenchment must be reasonably necessary and likely
provides that the CA “shall have the power to try cases and conduct to effectively prevent the expected losses; and (d) The alleged losses, if
hearings, receive evidence and perform any and all acts necessary to already incurred, and the expected imminent losses sought to be
resolve factual issues raised in cases falling within its original and forestalled must be proved by sufficient and convincing evidence.
appellate jurisdiction, including the power to grant and conduct new trials Anent the last standard of sufficient and convincing evidence, it
or further proceedings.” ought to be pointed out that a less exacting standard of proof would
YES. Petitioner actually wielded the power of selection at the render too easy the abuse of retrenchment as a ground for termination of
time it entered into the service contract dated September 1, 1992 with services of employees.
respondent. This is true, notwithstanding petitioner’s insistence that In termination cases, the burden of proving that the dismissal
respondent had only offered his services to provide live music at was for a valid or authorized cause rests upon the employer. Here,
petitioner’s Tanglaw Restaurant, and despite petitioner’s position that petitioner did not submit evidence of the losses to its business operations
what had really transpired was a negotiation of his rate and time of and the economic havoc it would thereby imminently sustain. It only
availability. The power of selection was firmly evidenced by, among claimed that respondent’s termination was due to its “present
others, the express written recommendation dated January 12, 1998 by business/financial condition.” This bare statement fell short of the norm to
Christine Velazco, petitioner’s restaurant manager, for the increase of his show a valid retrenchment. Hence, we hold that there was no valid cause
remuneration. for the retrenchment of respondent.
Respondent’s remuneration, albeit denominated as talent fees,
was still considered as included in the term wage in the sense and
context of the Labor Code, regardless of how petitioner chose to 133 DIGITAL TELECOMMUNIC ATIONS PHIL. , INC. VS. DIGITEL E
designate the remuneration. Anent this, Article 97(f) of the Labor Code MPLOYEES UNION
clearly states: (NO GOOD FAITH)
xxx wage paid to any employee shall mean the remuneration or
earnings, however designated, capable of being expressed in terms of FACTS:
money, whether fixed or ascertained on a time, task, piece, or By virtue of a certification election, Digitel Employees Union
commission basis, or other method of calculating the same, which is (Union) became the exclusive bargaining agent of all rank and file
payable by an employer to an employee under a written or unwritten employees of Digitel in 1994. The Union and Digitel then commenced
contract of employment for work done or to be done, or for services collective bargaining negotiations which resulted in a bargaining
rendered or to be rendered, and includes the fair and reasonable value, deadlock. The Union threatened to go on strike, but then the Labor
as determined by the Secretary of Labor, of board, lodging, or other Secretary assumed jurisdiction over the dispute and eventually directed
facilities customarily furnished by the employer to the employee. the parties to execute a CBA. 2 However, no CBA was forged between
That respondent worked for less than eight hours/day was of Digitel and the Union. Some Union members abandoned their
no consequence and did not detract from the CA’s finding on the employment with Digitel. The Union later became dormant.
existence of the employer-employee relationship. In providing that the Ten (10) years thereafter or on 28 September 2004, Digitel
“normal hours of work of any employee shall not exceed eight (8) hours a received from Esplana, who was President of the Union, a letter
day,” Article 83 of the Labor Code only set a maximum of number of containing the list of officers, CBA proposals and ground rules. 3 Digitel
hours as “normal hours of work” but did not prohibit work of less than was reluctant to negotiate with the Union and demanded that the latter
eight hours. Union show compliance with the provisions of the Union’s Constitution
The power of the employer to control the work of the employee and By-laws on union membership and election of officers.
is considered the most significant determinant of the existence of an On 4 November 2004, Esplana and his group filed a case for
employer-employee relationship. This is the so-called control test, and is Preventive Mediation before the National Conciliation and Mediation
premised on whether the person for whom the services are performed Board based on Digitel’s violation of the duty to bargain. On 25
November 2004, Esplana filed a notice of strike. On 10 March 2005, the
then Labor Secretary issued an Order4 assuming jurisdiction over the Digitel. It is undisputed that as early as March 1994, the affected
labor dispute. employees, except for two, were already performing their job as Traffic
During the pendency of the controversy, Digitel Service, Inc. (Digiserv), a Operator which was later renamed as Customer Service Representative
non-profit enterprise engaged in call center servicing, filed with the DOLE (CSR). It is equally undisputed that all throughout their employment, their
an Establishment Termination Report stating that it will cease its function as CSR remains the same until they were terminated effective
business operation. The closure affected at least 100 employees, 42 of May 30, 2005. Their long period of employment as such is an indication
whom are members of the herein respondent Union. Alleging that the that their job is directly related to the main business of DIGITEL which is
affected employees are its members and in reaction to Digiserv’s action, telecommunications.
Esplana and his group filed another Notice of Strike for union busting, Furthermore, Digiserv does not exercise control over the
illegal lock-out, and violation of the assumption order. On 23 May 2005, affected employees. Digiserv shared the same Human Resources,
the Labor Secretary ordered the second notice of strike subsumed by the Accounting, Audit and Legal Departments with Digitel which manifested
previous Assumption Order.5 Meanwhile, on 14 March 2005, Digitel filed that it was Digitel who exercised control over the performance of the
a petition with the Bureau of Labor Relations (BLR) seeking cancellation affected employees. The NLRC also relied on the letters of
of the Union’s registration. In a Decision dated 11 May 2005, the commendation, plaques of appreciation and certification issued by Digitel
Regional Director of the DOLE dismissed the petition for to the Customer Service Representatives as evidence of control.
cancellation of union registration for lack of merit. The appeal filed by Considering that Digiserv has been found to be engaged in
Digitel with the BLR was eventually dismissed for lack of merit in a labor-only contracting, the dismissed employees are deemed employees
Resolution dated 9 March 2007. of Digitel.
In an Order dated 13 July 2005, the Secretary of Labor directed The affected employees were illegally dismissed. In
Digitel to commence the CBA negotiation with the Union and certified for addition to finding that Digiserv is a labor-only contractor, records teem
compulsory arbitration before the NLRC the issue of unfair labor practice. with proof that its dismissed employees are in fact employees of Digitel.
In accordance with the 13 July 2005 Order of the Secretary of Labor, the The NLRC enumerated these pieces of evidence, thus:
unfair labor practice issue was certified for compulsory arbitration before  The remaining affected employees, except for two (2), were
the NLRC. On 31 January 2006, NLRC rendered a Decision dismissing already hired by DIGITEL even before the existence of DIGISERV.
the unfair labor practice charge against Digitel but declaring the dismissal Likewise, the remaining affected employees continuously held the
of the 13 employees of Digiserv as illegal and ordering their position of Customer Service Representative, which was earlier known
reinstatement. 10 as Traffic Operator, from the time they were appointed on March 1, 1994
The Union manifested that out of 42 employees, only 13 until they were terminated on May 30, 2005.
remained, as most had already accepted separation pay.  Further, the Certificates issued to Customer Service
In view of this unfavorable decision, Digitel filed a petition on 9 Representative likewise show that they are employees of DIGITEL, Take
June 2006 before the Court of Appeals, challenging the above NLRC for example the "Service Award" issued to Ma. Loretta C. Esen, one of
Decision and Resolution and arguing mainly that Digiserv employees are the remaining affected employees. The "Service Award" was signed by
not employees of Digitel. the officers of DIGITEL – the VP-Customer Services Division, the
On 18 June 2008, CA partially granted the case for ULP, thus VPHuman Resources Division and the Group Head-Human Resources
modifying the assailed NLRC dispositions. The CA likewise sustained the Division. It cannot be gainsaid that it is only the employer that issues
finding that Digiserv is engaged in labor-only contracting and that its service award to its employees. 22 As an alternative argument, Digitel
employees are actually employees of Digitel. maintains that the affected employees were validly dismissed on the
Digitel filed a motion for reconsideration but was denied in a grounds of closure of Digiserv, a department within Digitel.
Resolution dated 9 October 2008. Hence, this petition for review on In the recent case of Waterfront Cebu City Hotel v. Jimenez, 23
certiorari. we referred to the closure of a department or division of a company as
retrenchment. For a valid retrenchment, the following elements must be
ISSUES: present:
1) Whether Digiserv is a legitimate contractor; and (1) That retrenchment is reasonably necessary and likely to
2) Whether there was a valid dismissal. prevent business losses which, if already incurred, are not merely de
minimis, but substantial, serious, actual and real, or if only expected, are
RULING: reasonably imminent as perceived objectively and in good faith by the
employer;
Digiserv is a labor-only contractor. Labor-only contracting is (2) That the employer served written notice both to the
expressly prohibited by our labor laws. After an exhaustive review of the employees and to the Department of Labor and Employment at least one
records, there is no showing that first, Digiserv has substantial month prior to the intended date of retrenchment;
investment in the form of capital, equipment or tools. The NLRC, as (3) That the employer pays the retrenched employees
echoed by the CA, did not find substantial Digiserv’s authorized capital separation pay equivalent to one (1) month pay or at least ½ month pay
stock of P 1,000,000.00. It pointed out that only P 250,000.00 of the for every year of service, whichever is higher;
authorized capital stock had been subscribed and only P 62,500.00 had (4) That the employer exercises its prerogative to retrench
been paid up. There was no increase in capitalization for the last 10 employees in good faith for the advancement of its interest and not to
years. 19 defeat or circumvent the employees’ right to security of tenure; and
Moreover, in the Amended Articles of Incorporation, as well as in the (5) That the employer used fair and reasonable criteria in
General Information Sheets for the years 1994, 2001 and 2005, the ascertaining who would be dismissed and who would be retained among
primary purpose of Digiserv is to provide manpower services. In PCI the employees, such as status, efficiency, seniority, physical fitness, age,
Automation Center, Inc. v. National Labor Relations Commission,20 the and financial hardship for certain workers.24 Only the first 3 elements of
Court made the following distinction: "the legitimate job contractor a valid retrenchment had been here satisfied. Indeed, it is management
provides services while the labor-only contractor provides only prerogative to close a department of the company. Digitel’s decision to
manpower. The legitimate job contractor undertakes to perform a specific outsource the call center operation of the company is a valid reason to
job for the principal employer while the labor-only contractor merely close down the operations of a department under which the affected
provides the personnel to work for the principal employer." The services employees were employed. The fifth element regarding the criteria to be
provided by employees of Digiserv are directly related to the business of
observed by Digitel clearly does not apply because all employees under service, whichever is higher. A fraction of at least six (6) months shall be
Digiserv were dismissed. considered one (1) whole year.
The instant case is all about the fourth element, that is, Article 283 of the Labor Code shows that closure or cessation
whether or not the affected employees were dismissed in good of business operation as a valid and authorized ground of terminating
faith. We find that there was no good faith in the retrenchment. Prior to employment is not limited to those resulting from business losses or
the cessation of Digiserv’s operations, the Secretary of Labor had issued reverses. Said provision in fact provides for the payment of separation
the first and second assumption order. The effects of the assumption pay to employees terminated because of closure of business not due to
order issued by the Secretary of Labor are two-fold. It enjoins an losses, thus implying that termination of employees other than closure of
impending strike on the part of the employees and orders the employer to business due to losses may be valid. In the present case, while
maintain the status quo. petitioners did not sufficiently establish substantial losses to justify
There is no doubt that Digitel defied the assumption order by closure of the business, its income statement shows declining sales
abruptly closing down Digiserv. The closure of a department is not illegal prompting the petitioners to suspend its business operations, eventually
per se. What makes it unlawful is when the closure is undertaken in bad leading to its permanent closure. On this score, we agree that undue
faith. In St. John Colleges, Inc. v. St. John Academy Faculty and interference with an employers judgment in the conduct of his business is
Employees Union,26 bad faith was evidenced by the timing of and uncalled for. Even as the law is solicitous of the welfare of employees, it
reasons for the closure and the timing of and reasons for the must also protect the right of an employer to exercise what is clearly a
subsequent opening. management prerogatives. As long as the companys exercise of the
same is in good faith to advance its interest and not for the purpose of
D. CLOSURE OF BUSINESS OPERATIONS defeating or circumventing the rights of employees under the law or a
valid agreement such exercise will be upheld.
134 J.A.T. GENERAL SERVICES & JESUSA ADLAWAN TOROBU vs. In the event, under Article 283 of the Labor Code, three
NLRC & JOSE F. MASCARINAS requirements are necessary for a valid cessation of business
(DEFINITION; REQUIREMENTS – (1) WRITTEN NOTICE, (2) BONA operations, namely: (a) service of a written notice to the employees and
FIDE IN CHARACTER, (3) TERMINATION PAY) to the DOLE at least one (1) month before the intended date thereof; (b)
the cessation of business must be bona fide in character; and (c)
FACTS: payment to the employees of termination pay amounting to at least one-
Jesusa Adlawan Trading & General Services (JAT) hired Jose half (1/2) month pay for every year of service, or one (1) month pay,
F. Mascarinas as helper tasked to coordinate with the cleaning and whichever is higher.
delivery of the heavy equipment sold to customers. Initially, private The closure of business operation by petitioners, in our view, is
respondent was hired as a probationary employee and was paid P165 not tainted with bad faith or other circumstance that arouses undue
per day that was increased to P180 in July 1997 and P185 in January suspicion of malicious intent. The decision to permanently close business
1998. In October 1997, the sales of heavy equipment declined because operations was arrived at after a suspension of operation for several
of the Asian currency crisis. Consequently, JAT temporarily suspended months precipitated by a slowdown in sales without any prospects of
its operations. It advised its employees, including private respondent, not improving. There were no indications that an impending strike or any
to report for work starting on the first week of March 1998 JAT indefinitely labor related union activities precipitated the sudden closure of business.
closed shop effective May 1998. A few days after, private respondent
filed a case for illegal dismissal and underpayment of wages against 135 LA UNION CEMENT WORKERS UNION & ARNULFO ALMOITE
petitioners before the NLRC. On December 14, 1998, JAT filed an vs NLRC & BACNOTAN CEMENT CORPORATION (now HOLCIM
Establishment Termination Report with the Department of Labor and PHILIPPINES, INC.)
Employment (DOLE), notifying the latter of its decision to close its (CLOSURE LEADING TO REDUNDANCY; SUPPORT SERVICES)
business operations due to business losses and financial reverses.
FACTS:
ISSUE: Bacnotan Cement Corporation had been utilizing the wet
Whether or not private respondent was illegally dismissed from process technology in its operations. It then later introduced the dry
employment due to closure of petitioners business process technology as part of its modernization program. When the new
dry process technology became fully operational, the company
HELD: discovered that the dry process technology or the dry line proved to be
NO. Closure of business, is the reversal of fortune of the more efficient than the wet process technology. Thus, after studying the
employer whereby there is a complete cessation of business operations situation, the company concluded that it would be uncompetitive and
and/or an actual locking-up of the doors of establishment, usually due to impractical to operate the wet line and decided to close it down. To
financial losses. Closure of business as an authorized cause for implement the closure of the wet line, the company and La Union
termination of employment aims to prevent further financial drain upon an Cement Workers Union entered into a Memorandum of Agreement
employer who cannot pay anymore his employees since business has whereby the company committed to grant separation pay equivalent to
already stopped. 150% of the monthly basic pay for every year of service plus the
ART. 283. Closure of establishment and reduction of additional fixed amount of P27,000.00 to employees who would be
personnel. The employer may also terminate the employment of any terminated as a result of the closure of the wet line. In a letter, Senior
employee due to the installation of labor-saving devices, redundancy, Executive Vice President, notified the employees of the its decision to
retrenchment to prevent losses or the closing or cessation of operation of mothball the wet line and the termination of those whose employment
the establishment or undertaking unless the closing is for the purpose of would become unnecessary as a result of the closure. The company sent
circumventing the provisions of this Title, by serving a written notice on a letter to the Department of Labor and Employment (DOLE), informing
the workers and the Department of Labor and Employment at least one him about the companys decision to shut down the wet line and
(1) month before the intended date thereof. In case of retrenchment to furnishing him the list of affected employees. The company then sent
prevent losses and in cases of closures or cessation of operations of notices of termination to more or less 200 employees including petitioner
establishment or undertaking not due to serious business losses or Almoite. Upon the receipt of the separation pay, a number of the affected
financial reverses, the separation pay shall be equivalent to one (1) employees signed individual Release Waiver and Quitclaim. However,
month pay or to at least one-half (1/2) month pay for every year of some 80 of its members including Almoite filed complaints for unfair labor
practice, illegal lay-off and illegal dismissal. They alleged that while the companies availed of the special redundancy program. Later, the two
closure affected only the wet line, among the employees terminated were companies sent letters to sixty-six (66) employees informing them that
operating the dry line or performing support services for both wet and dry their respective positions had been declared redundant. The notices
lines. They further alleged that after the closure of the wet line, the likewise stated that their services would be terminated effective thirty (30)
company contracted out the services performed by the employees who days from receipt thereof.
were terminated. Separation benefits, including the conversion of all earned
leave credits and other benefits due under existing CBAs were thereafter
ISSUE: Whether the termination was valid. paid to those affected.
Petitioners accused private respondents, among others, of
HELD: unfair labor practice, violation of CBA and discrimination. They averred
YES. Petitioner Almoites work as an oiler for both the wet line that in the dismissal of sixty-six (66) union officers and members on the
and dry line has become redundant or superfluous following the closure ground of redundancy, private respondents circumvented the provisions
of the wet line. By and large, the determination of whether to maintain or in their CBA, which states that “In all cases of lay-off or retrenchment
phase out an entire department or section or to reduce personnel lies resulting in termination of employment in the line of work, the Last-In-
with the management. Thus, his termination on the ground of First-Out (LIFO) Rule must always be strictly observed.” They also
redundancy is an authorized cause for termination under Article 283 of alleged that the companies' claim that they were in economic crisis
the Labor Code. was fabricated because in 1990, a net income of over 83 million pesos
As concluded by the NLRC: was realized by Liberty Flour Mills Group of Companies.
xxx Private respondents contend that their decision to implement a
There is no dispute as to the fact that there was a partial special redundancy program was an exercise of management
closure or cessation of operations with the mothballing of the old wet- prerogative which could not be interfered with unless it is shown to be
process production line of the company a situation which falls among the tainted with bad faith and ill motive. Private respondents explained that
authorized causes for termination allowed under Article 283 of the Labor they had no choice but to reduce their work force, otherwise, they would
Code. suffer more losses.
xxx
Neither is there any dispute that the logical and consequence ISSUE: WON their termination was valid.
[sic] of such partial cessation of operations was to render certain
employees redundant. Obviously enough, since there was a curtailment HELD:
in operations, certain activities were rendered either excess or no longer YES. The court sustained the companies' prerogative to adopt
necessary, hence, redundant. the alleged redundancy/retrenchment program to minimize if not, to avert
xxxx losses in the conduct of its operations. However, the companies' decision
The only ostensible argument presented by appellant is the on this matter is not absolute. The basis for such an action must be far
bare allegation that most of them were not exclusively assigned to the from being whimsical and the same must be proved by substantial
wet process line but were performing support services for both the wet evidence. In addition, the implementation of such a decision or policy
line and the dry line. Therefore, they argue that they could not be must be in accordance with existing laws, rules and procedure and
declared redundant by virtue of the closure of the wet line alone. This line provisions of the CBA between the parties, if there be any. Short of any
of argument is non sequitur, fallacious and totally untenable. It proceeds of these conditions, management policy to pursue and terminate its
from the erroneous premise that only those exclusively assigned to the employees allegedly to avert losses, must fail.
wet line can be declared redundant. The mere fact that an employee was In subject case, the 66 complaining employees were separated
performing support services for both the wet and the dry line does not in from service as a result of the decision of management to limit its
any way exclude him from being declared as redundant. On the contrary, operations and streamline positions and personnel requirements.
with the closure of the wet line and the consequent scaling down of A close examination of the positions retained by management
activities requiring support services, it stands to reason that there was show that said positions such as egg sorter, debonner were but the
already an excess of employees performing support services. minimal positions required to sustain the limited functions/operations of
Respondent had therefore all the reason to include such employees the meat processing department. In the absence of any evidence to
among those whom it considered redundant. prove bad faith on the part of management in arriving at such decision,
which records on hand failed to show in instant case, the rationality of the
136 MAYA FARMS EMPLOYEES ORGANIZATION,,et.al. vs. NLRC, act of management in this regard must be sustained. While it may be true
MAYA REALTY & LIVESTOCK, INC., MAYA FARMS, INC., and that the Liberty Flour Mills Group of Companies as a whole posted a net
LIBERTY FLOUR MILLS, INC. income of P83.3 Million, it is admitted that with respect to operations of
the meat processing and livestock which were undertaken by herein
FACTS: companies sustained losses in the sum of P2,257,649.88 (Exh. "3"). This
Maya Farms, Inc. and Maya Realty and Livestock Corporation is the reason, as advanced by management, for its decision to streamline
belong to the Liberty Mills group of companies whose undertakings positions resulting in the reduction of manpower compliment.
include the operation of a meat processing plant which produces ham,
bacon, cold cuts, sausages and other meat and poultry products. 137 MAC ADAMS METAL ENGINEERING WORKERS UNION-
Petitioners, on the other hand, are its exclusive bargaining agents of the INDEPENDENT vs vs. MAC ADAMS
employees. Private respondents announced the adoption of an early METAL ENGINEERING and/or LYDIA SISON, et.al.
retirement program as a cost-cutting measure considering that their
business operations suffered major setbacks over the years. The FACTS:
program was voluntary and could be availed of originally only by MAMEWU charged private respondents MAME and GBS with
employees with at least eight (8) years of service but was reduced to five unfair labor practices (ULP) committed through union busting and illegal
(5) years. But the response to the program was nil. Thus, the early closure, and illegal dismissal. They insisting that the closure of MAME
retirement program was converted into a special redundancy program and GBS was illegal as it was calculated to bust their union, claiming
intended to reduce the work force to an optimum number so as to make that MAME and GBS continued doing business under new business
operations more viable. A total of sixty-nine (69) employees from the two names, i.e., MBS Machine and Industrial Supply (MBS) and MVS Heavy
Equipment Rental and Builders (MVS). Private respondent spouses ongoing reorganization. Petitioner requested that her transfer be made
Geronimo and Lydia V. Sison, proprietors of GBS and MAME effective in October or November 2003 and that she be given time to
respectively, denied the allegations explaining the closure of MAME and discuss it with her husband and daughter.
GBS, because Lydia V. Sison decided to retire from business when she A week later, however, or on August 11, 2003, petitioner was
became sickly, when her health did not improve despite proper medical informed that her transfer would be effective August 18, 2003. On even
attention she announced her plan to close the shop. The announcement date, she was placed under investigation for the delayed released of
in advance was intended to give the workers ample time to look for BCRs (cash budget for customer representation in sealed envelopes
alternative employment. Accordingly, she declined to accept new projects which are given to loyal clients) which she received for distribution earlier
and proceeded with the winding up of her business. Geronimo B. Sison in July 2003.
admitted being a part-owner of MBS which, he maintained, was an Finding that the delay in releasing the BCRs amounted to loss
entirely separate and distinct business enterprise from MAME and GBS. of trust and confidence, petitioner claims that in a meeting with the
MBS was engaged in manufacturing carton boxes and other allied respondents, she was given four options: resignation, termination,
products. On the other hand, MAME and GBS were both engaged in the availment of an early retirement package worth P40,000, or transfer to
businesses of machine shop operations, fabrication and construction. Pasig City. Without availing of any option, petitioner took a leave of
absence on August 28, 29 and September 1, 2003.
ISSUE: WoN the cessation and closure of business was lawful On September 3, 2003, respondent company sent petitioner a
memorandum-directive for her to immediately report to the head office in
HELD: Pasig City and to return the company vehicle assigned to her to the Cebu
YES. Closure or cessation of business operations is Office within 24 hours. Petitioner did not heed the directive, however.
allowed even if the business is not undergoing economic losses. She instead filed an application for sick leave until September 11, 2003,
The owner, for any bona fide reason, can lawfully close shop at anytime. and another until September 27, 2003.
Just as no law forces anyone to go into business, no law can compel On October 6, 2003, petitioner requested that she be given her
anybody to continue in it. It would indeed be stretching the intent and daily work assignment in Cebu, which request was later to be denied by
spirit of the law if we were to unjustly interfere with the managements Olga by letter dated October 8, 2003. On October 7, 2003, petitioner was
prerogative to close or cease its business operations just because said given a show cause notice for her to explain in writing why she should
business operation or undertaking is not suffering from any loss or simply not be sanctioned for insubordination for failure to comply with the
to provide the workers continued employment. transfer order.
The employer need only comply with the following On November 4, 2002, respondent company sent petitioner a notice of
requirements for a valid cessation of business operations. (a) service of a termination effective November 7, 2003 for insubordination, prompting
written notice to the employees and to the DOLE at least one month petitioner to file a complaint for illegal dismissal, non-payment of overtime
before the intended date thereof; (b) the cessation of or withdrawal from pay, 13th month pay, service incentive leave pay, separation pay,
business operations must be bona fide in character and (c) payment of damages and attorney's fees before the Labor Arbiter.
termination pay equivalent to at least one-half month pay for each year of The Court of Appeals ruled that petitioner was not entitled to
service, or one month pay, whichever is higher. The records reveal that separation pay because, contrary to the NLRC's finding, she "lacked
private respondents complied with the aforecited requirements. MAMEs good faith." It noted that petitioner, from the start, knew and accepted the
employees were adequately informed of the intended business closure company policy on transfers whenever so required, and could not thus
and a written notice to the Regional Director of the Department of Labor refuse "another valid reassignment by treating it as an imposition and
and Employment (DOLE) was filed by private respondents, informing the burden."
DOLE that except for winding-up operations, MAME will be closed
effective March 8, 1993. Similar notices were served by Lydia V. Sison to ISSUE:
the Social Security System (SSS), Bureau of Internal Revenue (BIR), Whether petitioner is entitled separation pay by way of financial
Department of Trade and Industry (DTI) and the Municipal Licensing assistance.
Division of Antipolo, Rizal. Thus, the licenses and registration of
respondent MAME with the SSS, the Municipality of Antipolo, Rizal and RULING:
the DTI were subsequently cancelled and/or withdrawn. NO. Reno Foods, Inc. v. Nagkakaisang Lakas ng Manggagawa
Further, private respondents closure of business was bona fide (NLM)-Katipunan 16 explains the propriety of granting separation pay in
and that private respondents did not engage in the operation of run-away termination cases in this wise:
shops. Finally, since private respondents cessation and closure of The law is clear. Separation pay is only warranted when the
business was lawful, there was no illegal dismissal to speak of. cause for termination is not attributable to the employee's fault, such as
those provided in Articles 283 and 284 of the Labor Code, as well as in
H. EFFECTS OF DISMISSAL cases of illegal dismissal in which reinstatement is no longer feasible. It is
A. WITH JUST OR AUTHORIZED CAUSE; COMPLIANCE OF DUE not allowed when an employee is dismissed for just cause, such as
PROCESS serious misconduct.
xxx xxx xxx
138 Apacible vs. Multimed Industries Inc., G.R. No. 178903, May 30, It is true that there have been instances when the Court
2011 awarded financial assistance to employees who were terminated for just
(NON-ENTITLEMENT OF SEPARATION PAY) causes, on grounds of equity and social justice. The same, however, has
been curbed and rationalized in Philippine Long Distance Telephone
Facts: Company v. National Labor Relations Commission. In that case, we
Petitioner Juliet Apacible was hired sometime in 1994 by recognized the harsh realities faced by employees that forced them,
respondent. She rose from the ranks to become Assistant Area Sales despite their good intentions, to violate company policies, for which the
Manager for Cebu Operations, the position she held at the time she was employer can rightly terminate their employment. For these instances,
separated from the service in 2003. the award of financial assistance was allowed. But, in clear and
On August 4, 2003, petitioner was informed by respondent unmistakable language, we also held that the award of financial
Marlene Orozco (Marlene), her immediate superior, that she would be assistance shall not be given to validly terminated employees, whose
transferred to the company's main office in Pasig City on account of the offenses are iniquitous or reflective of some depravity in their moral
character. When the employee commits an act of dishonesty, depravity, Labor Arbiter Antonio R. Macam rendered his Decision,
or iniquity, the grant of financial assistance is misplaced compassion. It is declaring the petitioner illegally dismissed from the service and hence,
tantamount not only to condoning a patently illegal or dishonest act, but entitled to reinstatement plus backwages and attorney's fees. NLRC
an endorsement thereof. It will be an insult to all the laborers who despite rendered its Resolution dismissing the herein respondents' appeal for
their economic difficulties, strive to maintain good values and moral lack of merit.
conduct. CA rendered the now assailed Decision reversing the rulings of
In fact, in the recent case of Toyota Motors Philippines, Corp. the NLRC. In finding the petitioner's dismissal lawful, the appellate court
Workers Association (TMPCWA) v. National Labor Relations attributed unto Yabut authorship of the meter tampering and illegal use of
Commission, we ruled that separation pay shall not be granted to all electricity acts which it regarded as serious misconduct.
employees who are dismissed on any of the four grounds provided in
Article 282 of the Labor Code. Such ruling was reiterated and further ISSUE:
explained in Central Philippines Bandag Retreaders, Inc. v. Diasnes: Whether or not petitioners dismissal is illegal?
To reiterate our ruling in Toyota, labor adjudicatory officials and
the CA must demur the award of separation pay based on social justice HELD:
when an employee's dismissal is based on serious misconduct or wilful Court of Appeals decision is sustained.
disobedience; gross and habitual neglect of duty; fraud or wilful breach of LABOR LAW
trust; or commission of a crime against the person of the employer or his Article 279 of the Labor Code of the Philippines provides that
immediate family — grounds under Art. 282 of the Labor Code that (i)n cases of regular employment, the employer shall not terminate the
sanction dismissals of employees. They must be most judicious and services of an employee except for a just cause or when authorized by
circumspect in awarding separation pay or financial assistance as the this Title. x x x The just causes are enumerated in Article 282, which
constitutional policy to provide full protection to labor is not meant to be provides:
an instrument to oppress the employers. The commitment of the Court to Article 282.Termination by employer. - An employer may
the cause of labor should not embarrass us from sustaining the terminate an employment for any of the following causes:
employers when they are right, as assistance to the undeserving and (a) Serious misconduct or willful disobedience by the employee
those who are unworthy of the liberality of the law. (italics in the original, of the lawful orders of his employer or representative in connection with
emphasis and underscoring supplied) ASTIED Petitioner was, it bears his work;
reiteration, dismissed for wilfully disobeying the lawful order of her (b) Gross and habitual neglect by the employee of his duties;
employer to transfer from Cebu to Pasig City. As (c) Fraud or willful breach by the employee of the trust reposed
correctly noted by the appellate court, petitioner knew and accepted in him by his employer or duly authorized representative;
respondent company's policy on transfers when she was hired and was (d) Commission of a crime or offense by the employee against
in fact even transferred many times from one area of operations to the person of his employer or any immediate member of his family or his
another — Bacolod City, Iloilo City and Cebu. duly authorized representative; and
Clearly, petitioner's adamant refusal to transfer, coupled with (e) Other causes analogous to the foregoing.
her failure to heed the order for her return the company vehicle assigned Significantly, tampering with electric meters or metering
to her and, more importantly, allowing her counsel to write letters installations of the Company or the installation of any device, with the
couched in harsh language to her superiors unquestionably show that purpose of defrauding the Company is classified as an act of dishonesty
she was guilty of insubordination, hence, not entitled to the award of from Meralco employees, expressly prohibited under company rules. It is
separation pay reasonable that its commission is classified as a severe act of
dishonesty, punishable by dismissal even on its first commission, given
139 NORMAN YABUT, Petitioner, v. MANILA ELECTRIC COMPANY the nature and gravity of the offense and the fact that it is a grave wrong
AND MANUEL M. LOPEZ, Respondents. directed against their employer.
Article 282 (a) provides that an employer may terminate an
FACTS: employment because of an employee's serious misconduct, a cause
This case stems from a complaint for illegal dismissal and that was present in this case in view of the petitioner's violation of his
monetary claims filed by herein petitioner Norman Yabut (Yabut) against employer's code of conduct. Misconduct is defined as the transgression
respondents Manila Electric Company (Meralco) and Meralco officer of some established and definite rule of action, a forbidden act, a
Manuel M. Lopez (Lopez). dereliction of duty, willful in character, and implies wrongful intent and not
The petitioner had worked with Meralco from February 1989 mere error in judgment. For serious misconduct to justify dismissal, the
until his dismissal from employment on February 5, 2004. Meralco's following requisites must be present:
Inspection Office issued a memorandum informing it of an illegal service (a) it must be serious;
connection at the petitioner's residence. Given this report, Meralco's (b) it must relate to the performance of the employee's duties;
Head of Investigation-Litigation Office issued to the petitioner a notice of and
investigation. (c) it must show that the employee has become unfit to
Meralcos Litigation Investigation Office summarized the results continue working for the employer.
of Meralco's findings in a memorandum which indicated that Yabuts The dismissal is also justified as the act imputed upon the
electric service was disconnected for account delinquency. petitioner qualifies as fraud or willful breach by the employee of the
Notwithstanding the disconnection and the fact that Meralcos service had trust reposed in him by his employer or duly authorized representative
not been reconnected, Yabut's meter registered electric consumption. In under Article 282 (c) of the Labor Code. While the petitioner contests this
view of these findings, respondent Meralco, issued a notice of dismissal ground by denying that his position is one of trust and confidence, it is
addressed to the petitioner. The notice cites violation of Section 7, undisputed that at the time of his dismissal, he was holding a supervisory
paragraph 3 of Meralco's Company Code on Employee Discipline and position after he rose from the ranks since commencement of his
Article 282 (a), (c), (d) and (e) of the Labor Code of the Philippines as employment with Meralco. As a supervisor with duty and power that
bases for the dismissal. included testing of service meters and investigation of violations of
Aggrieved by the decision of the management, Yabut filed with contract of customers, his position can be treated as one of trust and
the National Labor Relations Commission (NLRC) a complaint for illegal confidence, requiring a high degree of honesty as compared with
dismissal and money claims against Meralco and Lopez. ordinary rank-and-file employees.
We emphasize that dismissal of a dishonest employee is to the of due process. The LA observed that the alleged board resolution
best interest not only of the management but also of labor. As a measure dismissing Galang was unsubstantiated and self-serving, and carries no
of self-protection against acts inimical to its interest, a company has the probative value. The LA also noted that there was no proof that Galang
right to dismiss its erring employees. An employer cannot be compelled was notified of the charges against him before he was dismissed.
to continue employing an employee guilty of acts inimical to the On appeal, the National Labor Relations Commission (NLRC)
employer's interest, justifying loss of confidence in him. affirmed the labor arbiter findings.
After reaching the CA, the appellate court annulled the NLRC
The requirements of procedural due process were satisfied. decision and declared that Galang had been dismissed for a just cause.
The CA took exception to the conclusion of both the labor arbiter and the
On the matter of procedural due process, it is well-settled that notice NLRC that the respondents failed to discharge the burden of proving that
and hearing constitute the essential elements of due process in the Galang had been dismissed for cause. It pointed out that the records are
dismissal of employees. The employer must furnish the employee with replete with proof that Galang committed acts justifying the termination of
two written notices before termination of employment can be legally his employment.
effected. The first apprises the employee of the particular acts or The CA stressed that prior to the incidents leading to Galang
omissions for which dismissal is sought. The second informs the dismissal, he had already committed serious negligence in his work. It
employee of the employer's decision to dismiss him. With regard to the referred to the flooding of the 32nd floor of the condominium where he
requirement of a hearing, the essence of due process lies simply in an was assigned, due to his failure to secure tightly the valve filter room.
opportunity to be heard, and not that an actual hearing should always The flooding severely damaged the building elevator, resulting in repair
and indispensably be held.27 work amounting to P23,952.65. The CA stressed that despite this act of
gross negligence, he still remained in employment and it was only "on
These requirements were satisfied in this case. The first required notice account of subsequent events x x x that [the respondents] were
was dated November 3, 2003, sufficiently notifying the petitioner of the compelled to dismiss him."
particular acts being imputed against him, as well as the applicable law While the CA had no doubt that Galang dismissal was for
and the company rules considered to have been violated. cause, it nonetheless believed that he was not afforded procedural due
process for lack of notice. Consequently, it awarded Galang nominal
On November 17, 2003, Meralco conducted a hearing on the charges damages of P30,000.00, pursuant to the Agabon doctrine.
against the petitioner. During said time, the petitioner was accorded the
right to air his side and present his defenses on the charges against him. ISSUE:
Whether or not there was a just cause for Galang dismissal
Finally, Meralco served a notice of dismissal dated February 4, 2004 based on evidence not presented before the labor arbiter and the NLRC.
upon the petitioner. Such notice notified the latter of the company's
decision to dismiss him from employment on the grounds clearly HELD:
discussed therein. Galang petition is unmeritorious.
There was just cause for the dismissal - The CA committed no
140 ROMEO A. GALANG, Petitioner, v. CITYLAND SHAW TOWER, reversible error and neither did it commit grave abuse of discretion in
INC. and VIRGILIO BALDEMOR, Respondents. declaring that Galang had been dismissed for cause. Contrary to Galang
submission, there is substantial evidence such relevant evidence that a
FACTS: reasonable mind might accept as adequate to support a conclusion
Romeo Galang was employed by Gayren Maintenance supporting the CA decision.
Services as a janitor. When his contract with the agency expired, he was The affidavits executed in 2005, simply amplified the evidence
absorbed by Cityland Shaw Tower. Cityland submitted in 2002, including documents, which cited Galang
Galang alleged that he was absorbed as a janitor by Cityland serious negligence in causing the flooding of his assigned condominium
with a promise of regular employment after the completion of his six- floor, which resulted in a costly repair of the building selevator.
month probation. He claimed that even after the lapse of the period, he Additionally, there was Tupas memo to Cityland President which
continued working for Cityland although he had no idea about his "pertains to the case of Romeo Galang xxx for harassment to co-janitors,
employment status. He did not know his status for certain until he was insubordination to Supervisor and conduct unbecoming an employee."
shown a document on May 21, 2002 informing him that his employment Tupas made a report of an incident where Galang took pictures
would be terminated effective May 20, 2002. Thus, he filed a complaint of his co-janitors whom he considered as suspects in the alleged loss of
for illegal dismissal against Cityland and its Building Manager, Virgilio money (P4,000.00) kept in his locker. Tupas called a meeting to
Baldemor. investigate the matter. She asked Galang to surrender the pictures, but
Cityland countered that they absorbed Galang as a casual he refused and harassed the janitors and insulted Tupas in front of
employee after the expiration of his contract with Gayren Maintenance everybody. Tupas also reported that on several occasions, Galang
Services. They alleged that during his employment with them, they found disobeyed her orders, often finding fault with his co-employees, and was
him to be remiss in the performance of his job and he failed, too, to very hard to deal with. She believed that Galang had been grossly
conduct himself as a good employee. insubordinate and had committed acts of harassment against his co-
The respondents further alleged that in the face of Galang employees. Thus, he was already a liability to the organization.
negative work attitude and job performance, Cityland charged him with In light of the circumstances, the Supreme Court found that
gross insubordination, harassment of his co-employees and conduct Galang had become unfit to continue in employment. The evidence
unbecoming an employee. supports the view that he continued to exhibit undesirable traits as an
The respondents stressed that Cityland Board of Directors employee and as a person, in relation to both his co-workers and his
terminated Galang services, for gross insubordination, effective May 20, superiors, particularly Tupas, her immediate supervisor.
2002, after a "comprehensive examination of the accusation against On the due process - The finding of a just cause for Galang
complainant." dismissal notwithstanding, the Court concurs with the CA conclusion that
In a decision dated September 22, 2003, the Labor Arbiter Cityland did not afford Galang the required notice before he was
found that Galang had been illegally dismissed because Cityland failed to dismissed. As the CA noted, the investigation conference Tupas called to
present evidence to support Galang dismissal for cause after observance look into the janitors complaints against Galang, did not constitute the
written notice required by law as he had no clear idea what the charges Case was elevated to the Supreme Court under Rule 45 Review on
were. Thus, the CA committed no error in sustaining his dismissal and Certiorari Comment was made that WWWEC hired petitioner on a
awarding him nominal damages as indemnity. probationary basis and fired him before he became a regular employee.
As a final point, Galang posits that vis--vis the matter of
ISSUE:
dismissal for just cause without due process, the CA "was incorrect when
it retroactively applied the later ruling of the High Court in Agabon v. Whether or not Aliling was illegally dismissed.
NLRC, considering that when this case was filed, the applicable doctrine
was Serrano." RULING:
The Supreme Court disagrees with this position. As the
respondents correctly pointed out, the decision of the NLRC did not attain Petitioner was illegally dismissed.
finality as it was brought to the CA on a petition for certiorari and was
To justify fully the dismissal of an employee, the employer must, as a
overturned. Galang simply did not have the benefit of any final arbiter or rule, prove that the dismissal was for a just cause and that the employee
NLRC decision to which the Serrano ruling could be applied. When the was afforded due process prior to dismissal. As a complementary
CA ruled on the case, this Court had abandoned the Serrano doctrine in principle, the employer has the onus of proving with clear, accurate,
favor of Agabon. Thus, the CA committed no error in applying Agabon to consistent, and convincing evidence the validity of the dismissal.
the case.
DENIED Aliling was dismissed for an unjust cause

First off, the attendant circumstances in the instant case aptly show that
141-142 Armando Ailing vs. Jose B. Feliciano the issue of petitioner’s alleged failure to achieve his quota, as a ground
for terminating employment, strikes the Court as a mere afterthought on
FACTS: the part of WWWEC.

Respondent Wide Wide World Express Corporation (WWWEC) offered to What WWWEC considered as the evidence purportedly showing it gave
employ petitioner Armando Aliling (Aliling) on June 2, 2004 as “Account Aliling the chance to explain his inability to reach his quota was a
Executive (Seafreight Sales),”. The offer came with a six (6)-month purported September 20, 2004 memo of San Mateo addressed to the
probation period condition with this express caveat: “Performance during latter. However, Aliling denies having received such letter and WWWEC
probationary period shall be made as basis for confirmation to Regular or has failed to refute his contention of non-receipt. In net effect, WWWEC
Permanent Status.” was at a loss to explain the exact just reason for dismissing Aliling.

On June 11, 2004, Aliling and WWWEC inked an Employment Contract At any event, assuming for argument that the petitioner indeed failed to
under the terms of conversion to regular status shall be determined on achieve his sales quota, his termination from employment on that ground
the basis of work performance; and employment services may, at any would still be unjustified. Article 282 of the Labor Code considers any of
time, be terminated for just cause or in accordance with the standards the following acts or omission on the part of the employee as just cause
defined at the time of engagement. or ground for terminating employment:

However, instead of a Sea freight Sale assignment, WWWEC asked (a) Serious misconduct or willful disobedience by the employee
Aliling to handle Ground Express (GX), a new company product launched of the lawful orders of his employer or representative in
on June 18, 2004 involving domestic cargo forwarding service for Luzon. connection with his work;
Marketing this product and finding daily contracts for it formed the core of
Aliling’s new assignment. (b) Gross and habitual neglect by the employee of his
duties;
A month after, Manuel F. San Mateo III (San Mateo), WWWEC Sales
and Marketing Director, emailed Aliling to express dissatisfaction with the (c) Fraud or willful breach by the employee of the trust reposed
latter’s performance. in him by his employer or duly authorized representative;

On October 15, 2004, Aliling tendered his resignation to San Mateo. (d) Commission of a crime or offense by the employee against
While WWWEC took no action on his tender, Aliling nonetheless the person of his employer or any immediate member of his
demanded reinstatement and a written apology, claiming in a subsequent family or his duly authorized representatives; and
letter dated October 1, 2004 to management that San Mateo had forced
him to resign. (e) Other causes analogous to the foregoing. (Emphasis
supplied)
On October 6, 2004, Lariosa again wrote, this time to advise Aliling of the
termination of his services effective as of that date owing to his “non- An employee’s failure to meet sales or work quotas falls under the
satisfactory performance” during his probationary period. Records show concept of gross inefficiency, which in turn is analogous to gross neglect
that Aliling, for the period indicated, was paid his outstanding salary. of duty that is a just cause for dismissal under Article 282 of the Code.
However, in order for the quota imposed to be considered a valid
However, or on October 4, 2004, Aliling filed a Complaint for illegal productivity standard and thereby validate a dismissal, managements
dismissal due to forced resignation, nonpayment of salaries as well as prerogative of fixing the quota must be exercised in good faith for the
damages with the NLRC against WWWEC. advancement of its interest. The duty to prove good faith, however, rests
with WWWEC as part of its burden to show that the dismissal was for a
On April 25, 2006, the Labor Arbiter issued a decision declaring that the just cause. WWWEC must show that such quota was imposed in good
grounds upon which complainant’s dismissal was based did not conform faith. This WWWEC failed to do, perceptibly because it could not. The
not only the standard but also the compliance required under Article 281 fact of the matter is that the alleged imposition of the quota was a
of the Labor Code, Necessarily, complainant’s termination is not justified desperate attempt to lend a semblance of validity to Alilings illegal
for failure to comply with the mandate the law requires. Respondents dismissal.
should be ordered to pay salaries corresponding to the unexpired portion
of the contract of employment and all other benefits amounting to a total Being an experimental activity and having been launched for the first
of P35,811.00 covering the period from October 6 to December 7, 2004. time, the sales of GX services could not be reasonably quantified.
WWWEC failed to demonstrate the reasonableness and the bona
Both parties appealed the decision to the NLRC, which affirmed the fides on the quota imposition. Respondent WWWEC miserably failed to
decision of the Labor Arbiter and was later on sustained by the Court of prove the termination of petitioner was for a just cause nor was there
Appeals. substantial evidence to demonstrate the standards were made known to
the latter at the time of his engagement. Hence, petitioners right to receipt of the notices with a warning that failure to do so would mean
security of tenure was breached. waiver of their answer. They were also placed under preventive
suspension in the meantime.
Alilings right to procedural due process was violated
Petitioners failed to submit their written explanation within the
As earlier stated, to effect a legal dismissal, the employer must show not stated period. Subsequently, Kingspoint Express issued to them separate
only a valid ground therefor, but also that procedural due process has yet uniformly worded notices on January 20, 2006, informing them of
properly been observed. When the Labor Code speaks of procedural due their dismissal for the abovementioned charges based on the following
process, the reference is usually to the two (2)-written notice rule acts: fabrication of baseless money claims against the company,
envisaged in Section 2 (III), Rule XXIII, Book V of the Omnibus Rules misleading fellow co-workers to sign the malicious complaint for money
Implementing the Labor Code, which provides: claims against the company, refusal to undergo the company's general
drug test, and extorting money from co-workers to fund activities that
Section 2. Standard of due process: requirements of notice. In
all cases of termination of employment, the following standards they were never fully informed of. Also, petitioner Dacara was dismissed
of due process shall be substantially observed. for consummating his sexual relations with Co’s helper inside her
residence and thus impregnating the help.
I. For termination of employment based on just causes as defined in A complaint for illegal dismissal was subsequently filed,
Article 282 of the Code: alleging that the charges against them were fabricated and that their
dismissal was prompted by Kingspoint Express' aversion to their union
(a) A written notice served on the employee specifying the ground
activities. The Labor Arbiter ruled in favor of the petitioners as the
or grounds for termination, and giving to said employee reasonable
opportunity within which to explain his side; charges are purportedly mere unsubstantiated allegations. This was
affirmed by the NLRC on appeal but the latter reversed itself on a
(b) A hearing or conference during which the employee subsequent MR filed by Kingspoint. The CA initially reversed the NLRC’s
concerned, with the assistance of counsel if the employee so ruling but on an MR, they too reversed their earlier ruling and favored
desires, is given opportunity to respond to the charge, present his Kingspoint. Thus, this petition for certiorari before the SC.
evidence or rebut the evidence presented against him; and

(c) A written notice [of] termination served on the employee Issue:


indicating that upon due consideration of all the circumstance, WON the dismissal was valid.
grounds have been established to justify his termination.
Ruling:
Here, the first and second notice requirements have not been properly Yes, the dismissal was valid. It is fundamental that in order to
observed, thus tainting petitioners dismissal with illegality. validly dismiss an employee, the employer is required to observe both
substantive and procedural due process — the termination of
The adverted memo dated September 20, 2004 of WWWEC supposedly
informing Aliling of the likelihood of his termination and directing him to employment must be based on a just or authorized cause and the
account for his failure to meet the expected job performance would have dismissal must be effected after due notice and hearing.
had constituted the charge sheet, sufficient to answer for the first notice As to the substantive requirements of due process, the
requirement, but for the fact that there is no proof such letter had been employees' refusal to submit themselves to drug test is a just cause for
sent to and received by him. In fact, in his December 13, 2004 their dismissal.
Complainants Reply Affidavit, Aliling goes on to tag such An employer may terminate an employment on the ground of
letter/memorandum as fabrication. WWWEC did not adduce proof to
serious misconduct or willful disobedience by the employee of the lawful
show that a copy of the letter was duly served upon Aliling. Clearly
enough, WWWEC did not comply with the first notice requirement. orders of his employer or representative in connection with his work.
Willful disobedience requires the concurrence of two elements: (1) the
Neither was there compliance with the imperatives of a hearing or employee's assailed conduct must have been willful, that is,
conference. The Court need not dwell at length on this particular breach characterized by a wrongful and perverse attitude; and, (2) the order
of the due procedural requirement. Suffice it to point out that the record is violated must have been reasonable, lawful, made known to the
devoid of any showing of a hearing or conference having been
employee, and must pertain to the duties which he had been engaged to
conducted. On the contrary, in its October 1, 2004 letter to Aliling, or
barely five (5) days after it served the notice of termination, WWWEC discharge. Both elements are present in this case.
acknowledged that it was still evaluating his case. And the written notice As to the first element, the dismissed employees did not deny
of termination itself did not indicate all the circumstances involving the their refusal to undergo drug testing nor did they explain their refusal.
charge to justify severance of employment. The utter lack of reason or justification for their insubordination indicates
that it was prompted by mere obstinacy, hence, willful and warranting of
143 Kakampi and Its Members Panuelos vs. Kingspoint Express & dismissal. As to the second element, the subject order is relevant in the
Logistics performance of their functions as drivers of Kingspoint Express. As the
(REASONABLE OPPORTUNITY – AT LEAST 5 CALENDAR DAYS) NLRC correctly pointed out, drivers are indispensable to Kingspoint
Express' primary business of rendering door-to-door delivery services. It
Facts: is common knowledge that the use of dangerous drugs has adverse
Petitioners were former drivers of the respondent Kingspoint effects on driving abilities that may render the dismissed employees
Express, a sole proprietorship under the name of Co which is engaged in incapable of performing their duties to Kingspoint Express and acting
the business of transporting goods. against its interests, in addition to the threat they pose to the public.
They were dismissed from service on January 20, 2006 on the The existence of a single just cause is enough to order their
grounds of serious misconduct, dishonesty, loss of trust and confidence dismissal and it is now inconsequential if the other charges against them
and commission of acts inimical to the interest of Kingspoint Express. do not merit their dismissal from service. Nonetheless, while Kingspoint
Kingspoint Express issued separate notices to explain to the Express had reason to sever their employment relations, this Court finds
individual petitioners on January 16, 2006 the charges of dishonesty, its supposed observance of the requirements of procedural due process
serious misconduct and loss of confidence by filing with the NLRC false, pretentious. While Kingspoint Express required the dismissed employees
malicious and fabricated cases against the company, and their allegedly to explain their refusal to submit to a drug test, the two (2) days afforded
unwarranted refusal to undergo drug testing. They were required to to them to do so cannot qualify as "reasonable opportunity", which the
submit their answer to the charges within forty-eight (48) hours from
Court construed in King of Kings Transport, Inc. v. Mamacas a period of Article 282. Termination by Employer. - An employer may
at least five (5) calendar days from receipt of the notice. terminate an employment for any of the following causes:
Thus, even if Kingspoint Express' defective attempt to comply (a)Serious misconductor willful disobendience by the employee
with procedural due process does not negate the existence of a just of the lawful orders of his employer or his representativesin connection
cause for their dismissal, Kingspoint Express is still liable to indemnify with his work;
the dismissed employees, with the exception of Panuelos, Dizon and xxxxxxxxx
Dimabayao, who did not appeal the dismissal of their complaints, with (e) Other causes analogous to the foregoing.
nominal damages in the amount of P30,000.00. LABOR LAW: misconduct
Misconduct involves the transgression of some established and
definite rule of action, forbidden act, a dereliction of duty, willful in
144 COSMOS BOTTLING CORP., Petitioner, v. WILSON FERMIN, character, and implies wrongful intent and not mere error in judgment.
Respondent; WILSON B. FERMIN,Petitioner, v. COSMOS BOTTLING For misconduct to be serious and therefore a valid ground for dismissal,
CORPORATION and CECILIA BAUTISTA, Respondents. it must be:
1. of grave and aggravated character and not merely trivial or
FACTS: unimportant and
Wilson B. Fermin (Fermin) was a forklift operator at Cosmos 2. connected with the work of the employee.
Bottling Corporation (COSMOS), where he started his employment on 27 In this case, petitioner dismissed respondent based on the
August 1976.On 16 December 2002, he was accused of stealing the NBI's finding that the latter stole and used Yusecos credit cards. But
cellphone of his fellow employee, Luis Braga (Braga). Fermin was then since the theft was not committed against petitioner itself but against one
given a Show Cause Memorandum, requiring him to explain why the of its employees, respondent's misconduct was not work-related and
cellphone was found inside his locker. In compliance therewith, he therefore, she could not be dismissed for serious misconduct.
submitted an affidavit the following day, explaining that he only hid the Nonetheless, Article 282(e) of the Labor Code talks of other
phone as a practical joke and had every intention of returning it to analogous causes or those which are susceptible of comparison to
Braga. another in general or in specific detail .For an employee to be validly
After conducting an investigation, COSMOS found Fermin dismissed for a cause analogous to those enumerated in Article 282, the
guilty of stealing Bragas phone in violation of company rules and cause must involve a voluntary and/or willful act or omission of the
regulations. Consequently, on 2 October 2003,the company terminated employee.
Fermin from employment after 27 years of service, effective on 6 October A cause analogous to serious misconduct is a voluntary and/or
2003. willful act or omission attesting to an employees moral depravity. Theft
Following the dismissal of Fermin from employment, Braga committed by an employee against a person other than his employer, if
executed an affidavit, which stated the belief that the former had merely proven by substantial evidence, is a cause analogous to serious
pulled a prank without any intention of stealing the cellphone, and misconduct.
withdrew from COSMOS his complaint against Fermin. In this case, the LA has already made a factual finding, which
Meanwhile, Fermin filed a Complaint for Illegal Dismissal, was affirmed by both the NLRC and the CA, that Fermin had committed
which the Labor Arbiter (LA) dismissed for lack of merit on the ground theft when he took Bragas cellphone. Thus, this act is deemed analogous
that the act of taking a fellow employees cellphone amounted to gross to serious misconduct, rendering Fermins dismissal from service just and
misconduct. Further, the LA likewise took into consideration Fermins valid.
other infractions, namely: (a) committing acts of disrespect to a superior Further, the CA was correct in ruling that previous infractions
officer, and (b) sleeping on duty and abandonment of duty. may be cited as justification for dismissing an employee only if they are
Fermin filed an appeal with the National Labor Relations related to the subsequent offense. However, it must be noted that such a
Commission (NLRC), which affirmed the ruling of the LA[and denied discussion was unnecessary since the theft, taken in isolation from
Fermins subsequent Motion for Reconsideration. Fermins other violations, was in itself a valid cause for the termination of
Thereafter, Fermin filed a Petition for Certiorari with the Court his employment.
of Appeals (CA),which reversed the rulings of the LA and the NLRC and Finally, it must be emphasized that the award of financial
awarded him his full retirement benefits. Although the CA accorded with compensation or assistance to an employee validly dismissed from
finality the factual findings of the lower tribunals as regards Fermins service has no basis in law. Therefore, considering that Fermins act of
commission of theft, it nevertheless held that the penalty of dismissal taking the cellphone of his co-employee is a case analogous to serious
from service was improper on the ground that the said violation did not misconduct, this Court is constrained to reverse the CAs ruling as
amount to serious misconduct or wilful disobedience. regards the payment of his full retirement benefits. In the same breath,
COSMOS and Fermin moved for reconsideration, but the CA neither can this Court grant his prayer for backwages.
likewise denied their motions. Thus, both parties filed the present G.R. No. 194303 is DENIED.
Petitions for Review. G.R. No. 193676 is GRANTED.

ISSUE: 145 ALEX Q. NARANJO, DONNALYN DE GUZMAN, RONALD V.


Whether the imposition of the penalty of dismissal was CRUZ, ROSEMARIE P. PIMENTEL, and ROWENA B.
appropriate BARDAJE,Petitioners, v. BIOMEDICA HEALTH CARE, INC. and
CARINA "KAREN" J. MOTOL, Respondents.
HELD: (ABSENT FOR VARIOUS PERSONAL REASON – NOT STRIKE)
We rule in the affirmative.
LABOR LAW: theft against a co-employee is analogous to FACTS:
serious misconduct Petitioners Alex Naranjo (Naranjo), Ronald Allan Cruz, Rowena
Theft committed against a co-employee is considered as a Bardaje, Donnalyn De Guzman and Rosemarie Pimentel were all
case analogous to serious misconduct, for which the penalty of dismissal employees of Biomedica Health Care, Inc. (Biomedica).
from service may be meted out to the erring employee,viz: On November 7, 2006, Naranjo, et al. were all absent for
Article 282 of the Labor Code provides: various personal reasons. The next day, Naranjo, et al. came in for
work but were not allowed to enter the premises. Carina Motol (Motol),
Biomedicas president, informed them using foul language, to just find Petitioners did not stage a mass leave. The term "Mass Leave"
other employment. has been left undefined by the Labor Code. Plainly, the legislature
Subsequently, Biomedica issued notices to Naranjo, et al. intended that the terms ordinary sense be used. "Mass" is defined as
accusing them of having conducted an illegal strike and were accordingly "participated in, attended by, or affecting a large number of individuals;
directed to explain within twenty-four (24) hours to explain why they having a large-scale character." While the term "Leave" is defined as "an
should not be held guilty of and dismissed for violating the company authorized absence or vacation from duty or employment usually with
policy against illegal strikes under Article XI, Category Four, Sections 6, pay." Thus, the phrase "mass leave" may refer to a simultaneous
8, 12, 18 and 25 of the Company Policy.Biomedica, however, failed to availment of authorized leave benefits by a large number of employees in
furnish them with the copy of the said company policy. a company. It is undeniable that going on leave or absenting ones self
Naranjo, et al. failed to submit their written explanation. Thus, from work for personal reasons when they have leave benefits available
Biomedica served Notices of Termination stating that Naranjo, et al. is an employees right. Here, the five (5) petitioners were absent on
engaged in illegal strike. Consequently, Naranjo et al. filed a complaint November 7, 2006. The records are bereft of any evidence to establish
for illegal dismissal. The LA dismissed the complaint. The NLRC how many workers are employed in Biomedica. There is no evidence on
reversed the LA. On appeal to the CA, the CA reinstated the decision of record that 5 employees constitute a substantial number of employees of
the LA. Biomedica.
Petitioners did not go on strike. Art. 212(o) of the Labor Code
ISSUE: defines a strike as "any temporary stoppage of work by the concerted
Whether or not Naranjo, et al. were illegally dismissed? action of employees as a result of any industrial or labor dispute."
"Concerted" is defined as "mutually contrived or planned" or "performed
HELD: in unison." In the case at bar, the 5 petitioners went on leave for various
The petition is meritorious. reasons. Petitioners were in different places on November 7, 2006 to
LABOR LAW: illegal dismissal; mass leave; strike attend to their personal needs or affairs. They did not go to the company
Petitioners were not afforded procedural due process. premises to petition Biomedica for their grievance. This shows that there
Thus, the Court elaborated in King of Kings Transport, Inc. v. Mamac that was NO intent to go on strike.
a mere general description of the charges against an employee by the Dismissal is not the proper penalty. But setting aside from the
employer is insufficient to comply with the above provisions of the law. nonce the facts established above, the most pivotal argument against the
Clearly, petitioners were charged with conducting an illegal strike, not a dismissal of petitioners is that the penalty of dismissal from employment
mass leave, without specifying the exact acts that the company considers cannot be imposed even if we assume that petitioners went on an illegal
as constituting an illegal strike or violative of company policies. strike. It has not been shown that petitioners are officers of the Union. On
Further, while Biomedica cites the provisions of the company this issue, the NLRC correctly cited Gold City Integrated Port Service,
policy which petitioners purportedly violated, it failed to quote said Inc. v. NLRC, wherein We ruled that: "An ordinary striking worker cannot
provisions in the notice so petitioners can be adequately informed of the be terminated for mere participation in an illegal strike. There must be
nature of the charges against them and intelligently file their explanation proof that he committed illegal acts during a strike."
and defenses to said accusations. The CA is REVERSED and SET ASIDE. The NLRC is
Moreover, the period of 24 hours allotted to petitioners to REINSTATED with MODIFICATION.
answer the notice was severely insufficient and in violation of the
implementing rules of the Labor Code. Under the implementing rule of B. NO JUST AND AUTHORIZED CAUSE; NO DUE PROCESS
Art. 277, an employee should be given "reasonable opportunity" to file a I. REINSTATEMENT
response to the notice. King of Kings Transport, Inc. elucidates in this
wise: " Reasonable opportunity under the Omnibus Rules means every 146. 3RD ALERT SECURITY AND DETECTIVE SERVICES, INC.,
kind of assistance that management must accord to the employees to Petitioner, v. ROMUALDO NAVIA, Respondent.
enable them to prepare adequately for their defense. This should be (FAILURE TO EXECUTE FINALITY OF CASE; ORDER IS
construed as a period of at least five (5) calendar days from receipt of the IMMEDIATELY EXECUTORY; NO EFFORT TO REINSTATE)
notice to give the employees an opportunity to study the accusation
against them, consult a union official or lawyer, gather data and FACTS:
evidence, and decide on the defenses they will raise against the Romualdo Navia filed an illegal dismissal case against 3rd
complaint." Alert. In its November 30, 2005 decision, the Labor Arbiter issued a
In addition, Biomedica did not set the charges against decision in favor or 3rd Alert. The NLRC, after appeal, affirmed the ruling
petitioners for hearing or conference in accordance with Sec. 2, Book V, of the Labor Arbiter and on October 19, 2008, it also denied 3rd Alert
Rule XIII of the Implementing Rules and Regulations of the Labor Code motion for reconsideration.
and in line with ruling in King of Kings Transport, Inc., where the Court From this ruling, 3rd Alert filed an appeal with the CA (CA-G.R.
explained: "After serving the first notice, the employers should schedule SP No. 106963) with a prayer for the issuance of a temporary restraining
and conduct a hearing or conference wherein the employees will be order. The CA denied the appeal; 3rd Alert moved for a motion for
given the opportunity to: (1) explain and clarify their defenses to the reconsideration but the motion was also denied.
charge against them; (2) present evidence in support of their defenses; In the meantime, on January 29, 2009, the NLRC issued an
and (3) rebut the evidence presented against them by the management." Entry of Judgment certifying that the NLRC resolution dated October 19,
Petitioners were denied substantive due process. Clearly, 2008 has become final and executory. Thus, Navia filed with the labor
to justify the dismissal of an employee on the ground of serious arbiter an ex-parte motion for recomputation of back wages and an ex-
misconduct, the employer must first establish that the employee is guilty parte motion for execution based on the recomputed back wages.
of improper conduct, that the employee violated an existing and valid On November 10, 2009, the labor arbiter issued a writ of
company rule or regulation, or that the employee is guilty of a execution to enforce the recomputed monetary awards. 3rd Alert
wrongdoing. In the instant case, Biomedica failed to even establish that appealed the recomputed amount stated in the writ of execution to the
petitioners indeed violated company rules, failing to even present a copy NLRC. 3rd Alert also alleged that the writ was issued with grave abuse of
of the rules and to prove that petitioners were made aware of such discretion since there was already a notice of reinstatement sent to
regulations. Navia.
The NLRC dismissed the appeal, ruling that 3rd Alert is guilty was not acted upon. Instead, he was terminated from service effective
of bad faith since there was no earnest effort to reinstate Navia. The May 1, 2007.
NLRC also ruled that there was no notice or reinstatement sent to Navia Thus, Dakila filed a complaint for constructive illegal dismissal.
counsel. A motion for reconsideration was filed, but it was likewise He averred that the consultancy contract was a scheme to deprive him of
denied. the benefits of regularization, claiming to have assumed tasks necessary
3rd Alert filed a petition for certiorari with the CA which found and desirable in the trade or business of Skylanders and under their
the petition without merit because Navia had not been reinstated either direct control and supervision. On the contrary, Skylanders argued that
physically or in the payroll. The CA also denied the motion for Dakila was not their regular employee as he was not required to observe
reconsideration filed by 3rd Alert; hence, this petition. regular working hours and was free to adopt means and methods to
accomplish his task except as to the results of the work required of him.
ISSUE: Hence, no employer- employee relationship existed between them.
Did the Court of Appeals err in ruling that the NLRC did Both the Labor Arbiter and the NLRC ruled that Dakila was
not commit any grave abuse of discretion? illegally dismissed. The Court of Appeals affirmed the findings of the
Labor Arbiter and the NLRC.
HELD:
After a close examination of the petition and the attached ISSUE:
records where 3rd Alert insists that a copy of the manifestation on Whether or not Dakila was illegally dismissed?
reinstatement had been sent to Navia counsel and was received by a
certain "Biznar", the Supreme Court did not see any grave abuse of HELD:
discretion. The petition is partly granted.
Since it was ruled that there had been no notice of The issue of illegal dismissal is premised on the existence of
reinstatement sent to Navia or his counsel, as also affirmed by the CA, an employer-employee relationship between the parties herein. Records
the Court cannot rule otherwise in the absence of any compelling reveal that both the LA and the NLRC, as affirmed by the CA, have found
evidence. Time and again, it has been held that this Court is not a trier of substantial evidence to show that respondent Dakila was a regular
facts. In the absence of any attendant grave abuse of discretion, these employee who was dismissed without cause.
findings are entitled not only to respect, but to our final recognition in this Following Article 279 of the Labor Code, an employee who is
appellate review. unjustly dismissed from work is entitled to reinstatement without loss of
Article 223 of the Labor Code provides that in case there is an seniority rights and other privileges and to his full backwages computed
order of reinstatement, the employer must admit the dismissed employee from the time he was illegally dismissed. However, considering that
under the same terms and conditions, or merely reinstate the employee respondent Dakila was terminated on May 1, 2007, or one (1) day prior to
in the payroll. The order shall be immediately executory. Thus, 3rd Alert his compulsory retirement on May 2, 2007, his reinstatement is no longer
cannot escape liability by simply invoking that Navia did not report for feasible. His backwages should be computed only for days prior to his
work. The law states that the employer must still reinstate the employee compulsory retirement which in this case is only a day.
in the payroll. Where reinstatement is no longer viable as an option, Petition is PARTLY GRANTED.
separation pay equivalent to one (1) month salary for every year of
service could be awarded as an alternative.
Since the proceedings below indicate that 3rd Alert failed to 148 NORKIS TRADING CORPO RATION VS. JOAQUIN B
adduce additional evidence to show that it tried to reinstate Navia, either UENAVISTA, ET AL.
physically or in the payroll, it s safe to conclude that there was no earnest
effort to reinstate Navia. FACTS:
It is also noteworthy that 3rd Alert resorted to legal tactics to The respondents were hired by Norkis Trading, a domestic
frustrate the execution of the labor arbiter order; for about four (4) years, corporation engaged in the business of manufacturing and marketing of
it evaded the obligation to reinstate Navia. By so doing, 3rd Alert has Yamaha motorcycles and multi-purpose vehicles, on separate dates and
made a mockery of justice. It is thus proper to impose treble costs for various positions as welders and operators.
against 3rd Alert for its utter disregard to comply with the writ of Although they worked for Norkis Trading as skilled workers
execution. To reiterate, no indication exists showing that 3rd Alert assigned in the operation of industrial and welding machines owned and
exerted any efforts to reinstate Navia; worse, 3rd Alert lame excuse of used by Norkis Trading for its business, they were not treated as regular
having sent a notice of reinstatement to a certain "Biznar" only employees by Norkis Trading.
compounded the intent to mislead the courts. Instead, they were regarded by Norkis Trading as members of
DENIED Panaghiusa sa Kauswagan Multi-Purpose Cooperative (PASAKA) and
deemed an independent contractor that merely deployed the
respondents to render services for Norkis Trading.4 The respondents
147 THE NEW PHILIPPINE SKYLANDERS, INC. and/or JENNIFER M. nonetheless believed that they were regular employees of Norkis Trading
ENANO-BOTE, Petitioners, v. FRANCISCO N. DAKILA, Respondent. citing various circumstances in their position paper. Hence, the
(REINSTATEMENT NO LONGER POSSIBLE – BACKWAGES) respondents filed with the DOLE a complaint against Norkis Trading and
PASAKA for labor-only contracting and non-payment of minimum wage
FACTS: and overtime pay.
Respondent Francisco Dakila (Dakila) was employed by The The filing of the complaint for labor-only contracting allegedly
New Philippine Skylanders, Inc. (Skylanders) as early as 1987 and led to the suspension of the respondents’ membership with PASAKA.
terminated for cause in April 1997 when the latter was sold. In May 1997, The suspension prompted the respondents to file with the NLRC the
he was rehired as consultant by Skylanders under a Contract for complaint for illegal suspension against Norkis Trading and PASAKA. On
Consultancy Services. October 13, 1999, the respondents were to report back to work but
Thereafter, in a letter dated April 19, 2007, Dakila informed during the hearing in their NLRC case, they were informed by PASAKA
Skylanders of his compulsory retirement effective May 2, 2007 and that they would be transferred to Norkis Tradings’ sister company, Porta
sought for the payment of his retirement benefits. His request, however, Coeli Industrial Corporation (Porta Coeli), as washers of Multicab
vehicles. The respondents opposed the transfer as it would allegedly
result in a change of employers and that the transfer would result in a NLRC, in its assailed decision or resolution, has made a factual finding
demotion since from being skilled workers in Norkis Trading; they would that is not supported by substantial evidence. It is within the jurisdiction of
be reduced to being utility workers. the CA, whose jurisdiction over labor cases has been expanded to review
Labor Arbiter (LA) Gutierrez dismissed the complaint for lack of the findings of the NLRC. 47 This case falls within the exception to the
merit. LA directed complainants to report back to PASAKA for work general rule that findings of fact of labor officials are to be accorded
assignment. Likewise, respondent PASAKA is directed to accept the respect and finality on appeal.
complainants back for work. In the meantime (in a separate case) DOLE Norkis Trading is the principal employer of the
Regional Director Balanag issued on August 22, 2000 his Order20 in respondents, considering that PASAKA is a mere labor-only
LSED Case No. RO700-9906-CI-CS-168. Regional Director Balanag contractor. Labor-only contracting, a prohibited act, is an arrangement
ruled that PASAKA was engaged in labor-only contracting.21 It was where the contractor or subcontractor merely recruits, supplies, or places
found that: (1) PASAKA had failed to prove that it had substantial capital; workers to perform a job, work, or service for a principal. In labor-only
22 (2) the machineries, equipment and supplies used by the respondents contracting, the following elements are present: (a) the contractor or
in the performance of their duties were all owned by Norkis Trading and subcontractor does not have substantial capital or investment to actually
not by PASAKA; 23 (3) the respondents’ membership with PASAKA as a perform the job, work, or service under its own account and
cooperative was inconsequential to their employment with Norkis responsibility; and (b) the employees recruited, supplied or placed by
Trading; 24 (4) Norkis Trading and PASAKA failed to prove that their such contractor or subcontractor perform activities which are directly
sub-contracting arrangements were covered by any of the conditions set related to the main business of the principal. These differentiate it from
forth in Section 6 of Department Order No. 10, Series of 1997; 25 (5) permissible or legitimate job contracting or subcontracting, which refers
Norkis Trading and PASAKA failed to dispute the respondents’ claim that to an arrangement whereby a principal agrees to put out or farm out with
their work was supervised by leadmen and production supervisors of the contractor or subcontractor the performance or completion of a
Norkis Trading; 26 and (6) Norkis Trading and PASAKA failed to dispute specific job, work, or service within a definite or predetermined period,
the respondents’ allegation that their salaries were paid by employees of regardless of whether such job, work, or service is to be performed or
Norkis Trading.27 Norkis Trading and PASAKA were then declared completed within or outside the premises of the principal. A person is
solidarily liable for the monetary claims of therein complainants. considered engaged in legitimate job contracting or subcontracting if the
Regional Director Balanag’s Order was later affirmed by then DOLE following conditions concur: (a) the contractor carries on a distinct and
Secretary Sto. Tomas. When the rulings of the DOLE Secretary were independent business and partakes the contract work
appealed before the CA via the petitions for certiorari, the CA affirmed on his account under his own responsibility according to his own manner
the Orders of the DOLE Secretary.31 A motion for reconsideration of the and method, free from the control and direction of his employer or
CA decision was denied in a Resolution32. principal in all matters connected with the performance of his work except
The two petitions docketed as G.R. Nos. 180078-79, which as to the results thereof; (b) the contractor has substantial capital or
were brought before the SC to question the CA’s rulings, were later investment; and (c) the agreement between the principal and the
denied with finality by the SC. The respondents informed the NLRC of contractor or subcontractor assures the contractual employees’
Regional Director Balanag’s Order by filing a Manifestation. However, the entitlement to all labor and occupational safety and health standards, free
NLRC rendered its Decision35 affirming with modification the decision of exercise of the right to self-organization, security of tenure, and social
LA Gutierrez. It held that the respondents were not illegally suspended welfare benefits. 49 SC emphasized that the petitioner’s arguments
from work. The NLRC declared that the LA had no jurisdiction over the against the respondents’ claim that PASAKA is a labor-only contractor,
dispute because the respondents which is thus to be regarded as a mere agent of Norkis Trading for which
were not employees, but members of PASAKA. the respondents rendered service, are already
The respondents’ motion for reconsideration was denied by the mooted by the finality of this SC’s Resolutions in G.R. Nos. 180078-79,
NLRC. Undaunted, the respondents questioned the NLRC’s rulings which stems from the CA’s and the DOLE Secretary’s review of the
before the CA via a petition for certiorari. Finding merit in the petition for DOLE Regional Director’s Order dated August 22, 2000 in LSED Case
certiorari, the CA rendered its decision reversing and setting aside the No. RO700-9906-CI-CS-168. Applying the doctrine of res judicata, all
decision and resolution of the NLRC. The CA considered Regional matters that have been fully resolved with finality by this Court’s
Director Balanag’s finding in LSED Case No. RO700-9906-CI-CS-168 dismissal of the appeal that stemmed from Regional Director Balanag’s
that PASAKA was engaged in labor-only contracting. In ruling that the Order in LSED Case No. RO700-9906-CI-CS-168 are already
respondents were illegally dismissed, the CA held that Norkis Trading’s conclusive between the parties. The rule on conclusiveness of judgment
refusal to accept the respondents back to their former positions, offering then now precludes this Court from re-opening the issues that were
them instead to accept a new already settled with finality in G.R. Nos. 180078-79, which effectively
assignment as washers of vehicles in its sister company, was a demotion affirmed the CA’s findings that PASAKA was engaged in labor-only
that amounted to a constructive dismissal. Norkis Trading’s motion for contracting, and that Norkis Trading shall be treated as the employer of
reconsideration was denied by the CA. Hence, this petition. the respondents.
Termination of an employment for no just or authorized
ISSUES: cause amounts to an illegal dismissal. Where an entity is declared to
(1) Whether the CA erred in reversing of LA Gutierrez’s and the be a labor-only contractor, the employees supplied by said contractor to
NLRC’s rulings. the principal employer become regular employees of the latter. Having
(2) Whether PASAKA is a labor-only contractor. gained regular status, the employees are entitled to security of tenure
(3) Whether the respondents were illegally dismissed by Norkis and can only be dismissed for just or authorized causes and after they
Trading had been afforded due process. 66 Termination of employment without
just or authorized cause and without observing procedural due process is
RULING: illegal.
Factual findings of labor officials may be examined by the In claiming that they were illegally dismissed from their
courts when there is a showing that they were arrived at arbitrarily employment, the respondents alleged having been informed by PASAKA
or in disregard of evidence on record. Nonetheless, these findings are that they would be transferred, upon the behest of Norkis Trading, as
not infallible. When there is a showing that they were arrived at arbitrarily Multicab washers or utility workers to Porta Coeli, a sister company of
or in disregard of the evidence on record, they may be examined by the Norkis Trading. Norkis. Where labor-only contracting exists, the Labor
courts. The CA can then grant a petition for certiorari if it finds that the Code itself establishes an employer-employee relationship between the
employer and the employees of the labor-only contractor. The statute dismissal. [37] It is not private compensation or damages but is awarded
establishes this relationship for a comprehensive purpose: to prevent a in furtherance and effectuation of the public objective of the Labor Code.
circumvention of labor laws. The contractor is considered merely an Nor is it a redress of a private right but rather in the nature of a command
agent of the principal employer and the latter is responsible to the to the employer to make public reparation for dismissing an employee
employees of the labor-only contractor as if such employees had been either due to the formers unlawful act or bad faith. Article 279 mandates
directly employed by the principal employer. that an employees full backwages shall be inclusive of allowances
and other benefits or their monetary equivalent. Contrary to the ruling of
II. ENTITLEMENT TO FULL BACKWAGES the Court of Appeals, we do not see that a salary increase can be
interpreted as either an allowance or a benefit. Salary increases are not
149 EQUITABLE BANKING CORPORATION (EQUITABLE PCI BANK) akin to allowances or benefits, and cannot be confused with either. The
vs RICARDO SADAC term allowances is sometimes used synonymously with emoluments, as
(SALARY INCREASE NOT INCLUDED AS ALLOWANCES AND indirect or contingent remuneration, which may or may not be earned, but
OTHER BENEFITS; ALLOWANCE VS. SALARY INCREASE) which is sometimes in the nature of compensation, and sometimes in the
nature of reimbursement. [47]Allowances and benefits are granted to the
FACTS: employee apart or separate from, and in addition to the wage or salary.
Sadac was appointed Vice President of the Legal Department In contrast, salary increases are amounts which are added to the
of PCI Bank and subsequently General Counsel thereof. Nine lawyers of employees salary as an increment thereto for varied reasons deemed
PCI Banks Legal Department, accused Sadac of abusive conduct, inter appropriate by the employer. Salary increases are not separate grants by
alia, and ultimately, petitioned for a change in leadership of the themselves but once granted, they are deemed part of the employees
department. On the ground of lack of confidence in Sadac, under the salary. To extend the coverage of an allowance or a benefit to include
rules of client and lawyer relationship, PCI Bank instructed Sadac to salary increases would be to strain both the imagination of the Court and
deliver all materials in his custody in all cases in which the latter was the language of law. As aptly observed by the NLRC, to otherwise give
appearing as its counsel of record. In reaction thereto, Sadac requested the meaning other than what the law speaks for by itself, will open the
for a full hearing and formal investigation but the same remained floodgates to various interpretations.[48] Indeed, if the intent were to
unheeded. Sadac filed a complaint for illegal dismissal with damages include salary increases as basis in the computation of backwages, the
against PCI Bank and individual members of the Board of Directors same should have been explicitly stated in the same manner that the law
thereof. After learning of the filing of the complaint, PCI Bank terminated used clear and unambiguous terms in expressly providing for the
the services of Sadac. Finally, Sadac was removed from his office and inclusion of allowances and other benefits.
ordered disentitled to any compensation and other benefits. In this case,
the SC already decided that Sadacs dismissal was not grounded on 150 Timoteo H. Sarona v. NLRC
any of the causes stated in Article 282 of the Labor Code and that (COMPUTED FROM TIME OF DISMISSAL UPTO FINALITY OF THE
PCI Bank disregarded the procedural requirements in terminating CASE IF SEPARATION PAY WAS NOT GIVEN; ACTUAL RECEIPT)
respondent Sadacs employment, thus Sadac was entitled to
backwages from termination of employment until turning sixty (60) FACTS:
years of age (in 1995 ) xxx. However, in Sadacs computation, the total The petitioner, who was hired by Sceptre as a security guard,
amount of the monetary award is P6,030,456.59, representing his was asked by Karen Therese Tan, Sceptre's Operations Manager, to
backwages and other benefits, including the general increases which he submit a resignation letter as the same was supposedly required for
should have earned during the period of his illegal termination. According applying for a position at Royale.
to Sadac, the catena of cases uniformly holds that it is the obligation of Martin informed him that he would no longer be given any
the employer to pay an illegally dismissed employee the whole amount of assignment per the instructions of Aida Sabalones-Tan, general manager
the salaries or wages, plus all other benefits and bonuses and general of Sceptre. This prompted him to file a complaint for illegal dismissal.
increases to which he would have been normally entitled had he not While complainant is entitled to backwages, we are aware that his stint
been dismissed; and therefore, salary increases should be deemed a with respondent Royale lasted only for one (1) month and three (3) days
component in the computation of backwages. Moreover, Sadac such that it is our considered view that his backwages should be limited
contended that his checkup benefit, clothing allowance, and cash to only three (3) months. The petitioner does not deny that he has
conversion of vacation leaves must be included in the computation of his received the full amount of his backwages and separation pay as
backwages. PCI Bank disputed respondent Sadacs computation. Per its provided under the NLRC's November 2005 Decision. However,
computation, the amount of monetary award due respondent Sadac is he claims that this does not preclude this Court from modifying a decision
P2,981,442.98 only, to the exclusion of the latters general salary that is tainted with grave abuse of discretion or issued without
increases and other claimed benefits which, it maintained, were jurisdiction.
unsubstantiated, that an unqualified award of backwages means that the
employee is paid at the wage rate at the time of his dismissal. ISSUE:
Furthermore, petitioner Bank argued before the Labor Arbiter that the Whether the petitioner's backwages should be limited to his
award of salary differentials is not allowed, the established rule being that salary for three (3) months
upon reinstatement, illegally dismissed employees are to be paid their
backwages without deduction and qualification as to any wage increases RULING:
or other benefits that may have been received by their co-workers who No. In case separation pay is awarded and reinstatement is no
were not dismissed or did not go on strike. longer feasible, backwages shall be computed from the time of illegal
dismissal up to the finality of the decision should separation pay not be
ISSUE: paid in the meantime. It is the employee's actual receipt of the full
Whether general salary increases should be included in the amount of his separation pay that will effectively terminate the
base figure to be used in the computation of backwages. employment of an illegaly dismissed employee. Otherwise, the employer-
employee relationship subsists and the illegally dismissed employee is
HELD: entitled to backwages, taking into account the increases and
NO. Backwages in general are granted on grounds of equity other benefits, including the 13th month pay, that were received by his co-
for earnings which a worker or employee has lost due to his illegal employees who are not dismissed. It is the obligation of the employer to
pay an illegally dismissed employee or worker the whole amount of the A probationary employee, like a regular employee, enjoys
salaries or wages, plus all other benefits and bonuses and general security of tenure. However, in cases of probationary employment, aside
increases, to which he would have been normally entitled had he not from just or authorized causes of termination, an additional ground is
been dismissed and had not stopped working. provided under Article 281 of the Labor Code,i.e., the probationary
employee may also be terminated for failure to qualify as a regular
NOTE: employee in accordance with reasonable standards made known by the
a) full backwages and other benefits computed from October 1, 2003 (the employer to the employee at the time of the engagement. Thus, the
date Royale illegally dismissed the petitioner) until the finality of this services of an employee who has been engaged on probationary basis
decision; may be terminated for any of the following:
(1) a just or
b) separation pay computed from April 1976 until the finality of this (2) an authorized cause; and
decision at the rate of one month pay per year of service; (3) when he fails to qualify as a regular employee in
accordance with reasonable standards prescribed by the employer.
c) ten percent (10%) attorney’s fees based on the total amount of the Article 277(b) of the Labor Code mandates that the employer
awards under (a) and (b) above; shall furnish the worker, whose employment is sought to be terminated, a
written notice containing a statement of the causes of termination, and
d) moral damages of Twenty-Five Thousand Pesos (₱25,000.00); and shall afford the latter ample opportunity to be heard and to defend himself
with the assistance of a representative if he so desires, in accordance
e) exemplary damages of Twenty-Five Thousand Pesos (₱25,000.00). with company rules and regulations pursuant to the guidelines set by the
Department of Labor and Employment.
III. WHEN REINSTATEMENT NOT POSSIBLE In the instant case, based on the facts on record,
petitioners failed to accord respondent substantive and procedural
151 ROBINSONS GALLERIA/ROBINSONS SUPERMARKET due process. haphazard manner in the investigation of the missing
CORPORATION and/or JESS MANUEL, petitioners, vs. IRENE R. cash, which was left to the determination of the police authorities and the
RANCHEZ, respondents. Prosecutor's Office, left respondent with no choice but to cry foul.
Administrative investigation was not conducted by petitioner
FACTS: Supermarket. On the same day that the missing money was reported by
Respondent Ranchez was a probationary employee for 5 respondent to her immediate superior, the company already pre-judged
months. She was hired as a cashier by Robinsons sometime within that her guilt without proper investigation, and instantly reported her to the
period. Two weeks after she was hired, she reported the loss of cash police as the suspected thief, which resulted in her languishing in jail for
which she had placed in the company locker. She offered to pay for the two weeks.
lost amount but the Operations Manager of Robinsons had her strip- The due process requirements under the Labor Code are
searched then reported her to the police even though they found nothing mandatory and may not be replaced with police investigation or court
on her person. An information for Qualified Theft was filed with the proceedings. An illegally or constructively dismissed employee,
Quezon City Regional Trial Court. She was detained for 2 weeks for respondent is entitled to: (1) either reinstatement, if viable, or separation
failure to immediately post bail. Weeks later, respondent Ranchez filed a pay, if reinstatement is no longer viable; and (2) backwages. These two
complaint for illegal dismissal and damages. A year later, Robinsons sent reliefs are separate and distinct from each other and are awarded
to respondent by mail a notice of termination and/or notice of expiration conjunctively.
of probationary employment. In this case, since respondent was a probationary employee at
The Labor Arbiter dismissed the complaint for illegal dismissal, the time she was constructively dismissed by petitioners, she is entitled
alleging that at the time of filing respondent Ranchez had not yet been to separation pay and backwages. Reinstatement of respondent is no
terminated. She was merely investigated. However, the NLRC reversed longer viable considering the circumstances.
this ruling, stating that Ranchez was illegally dismissed and that
Robinson's should reinstate her. It held that Ranchez was deprived of 152 BERNADETH LONDONIO AND JOAN CORCORO v. BIO
due process when she was strip-searched and sent to jail for two weeks RESEARCH, INC. AND WILSON Y. ANG
because such amounted to constructive dismissal, making it impossible (QUITCLAIM DOES NOT PREVENT FILLING OF ILLEGAL
for the respondent to continue under the employment. Even though she DISMISSAL)
was merely a probationary employee, the lapse of the probationary
contract did not amount to a valid dismissal because there was already FACTS:
an unwarranted constructive dismissal beforehand. Petitioners were employees of respondent Bio Research. They
The NLRC denied Robinson's motion for reconsideration. The were subsequently dismissed from employment. Bio Research claims
CA affirmed the decision of the NLRC. that due to financial difficulties, it was forced to retrench petitioners in a
move towards financial efficiency. Petitioners were made to execute a
ISSUE: quitclaim as they were given their separation pay. Petitioners still
Whether respondent was illegally terminated from employment pursued a complaint with the Labor Arbiter for illegal dismissal. They also
by petitioners. claim that the illegal dismissal was done as retaliation for a sexual
harassment complaint they filed against one of the company managers.
HELD: The Labor Arbiter found the dismissal to be illegal as the financial
The petition is unmeritorious. difficulties of Bio Research were unproven, and that no criteria was
LABOR LAW: Probationary employees; termination of established in selecting the employees to be let go. The LA ordered
employment for payment of backwages and reinstatement, as well as the subsidiary
There is probationary employment when the employee upon liability of respondent Ang, for having acted in bad faith. The NLRC, on
his engagement is made to undergo a trial period during which the appeal, affirmed the findings of the LA. Upon appeal to the Court of
employer determines his fitness to qualify for regular employment based Appeals, the appellate court affirmed the finding of illegal dismissal, but
on reasonable standards made known to him at the time of engagement. upheld the quitclaim executed by the petitioners. In addition, the CA
found that since there was no proof of bad faith on the part of
Respondent Ang, he should not be held subsidiarily liable. Petitioners In the present case, Bides has consistently maintained, from
question these findings in the Supreme Court. the proceedings in the LA up to the CA, his refusal to be reinstated due
to his fear of reprisal which he could experience as a consequence of his
ISSUES: return. By doing so, Bides unequivocally foreclosed reinstatement as a
Is the quitclaim is valid? relief. DENIED.
Is Respondent Ang subsidiarily liable to petitioners?
C. NO JUST OR AUTHORIZED CAUSE; WITH DUE PROCESS
HELD: D. WITH JUST OR AUTHORIZED CAUSE; NO DUE PROCESS
A quitclaim does not prevent an illegally dismissed employee
from instituting an illegal dismissal case. And since the CA affirmed the 154 WENPHIL CORPORATION, vs. NLRC AND ROBERTO MALLARE
findings of illegal dismissal, Bio Research is indeed liable. Petitioners are
ordered reinstated, with payment of backwages. If not possible, then a FACTS:
payment of separation pay of one-half month salary for every year Mallare was hired by petitioner as a crew member at its Cubao
served. Branch. He thereafter became the assistant head of the Backroom
The acts of corporate officers, done in behalf of the department of the same branch. Private respondent had an altercation
corporation, are separate from their personal acts. Absent a showing of (caused trouble during office hours and even defied his superiors
bad faith, the ruling of the CA is proper. Respondent Ang should not have as they tried to pacify him) with a co-employee, Job Barrameda, as a
been held liable for the corporate acts. result of which he and Barrameda were suspended on the following
morning and in the afternoon of the same day a memorandum was
issued by the Operations Manager advising private respondent of his
153 APO CHEMICAL MANUFACTURING CORPORATION and dismissal from the service in accordance with their Personnel Manual,
MICHAEL CHENG, Petitioners, v. RONALDO A. BIDES, Respondent. prompting private respondent to file a complaint against petitioner for
(DOCTRINE OF STRAINED RELATIONSHIP; REFUSAL TO BE unfair labor practice, illegal suspension and illegal dismissal.
REINSTATED AND DEMANDS SEPARATION PAY = WAIVER OF
RELIEF OF REINSTATEMENT) ISSUE:
WON Mallare was afforded with due process.
FACTS:
Ronaldo Bides (Bides) was an employee of Apo Chemical HELD:
Manufacturing Corporation (ACMC) for eleven (11) years. Matthew NO. Under Section 1, Rule XIV of the Implementing
Cheng (Matthew), the plant manager of ACMC, sent a written Regulations of the Labor Code, it is provided that "No worker shall be
memorandum requiring Bides to explain his refusal to sign the dismissed except for just or authorized cause provided by law and after
disciplinary form in connection with his alleged infractions of loitering in due process." Sections 2, 5, 6, and 7 of the same rules require that
the comfort room for about five (5) to eight (8) minutes, two (2) to three before an employer may dismiss an employee the latter must be given a
(3) times a day, on March 5, 6, 7, 8, 9 and 10, 2003 under pain of written notice stating the particular act or omission constituting the
revocation of his housing privileges. Bides explained that urinating, as he grounds thereof; that the employee may answer the allegations within a
was “nababalisawsaw” at the time, was not an infraction. reasonable period; that the employer shall afford him ample opportunity
Allegedly, Matthew confronted Bides and prohibited him from to be heard and to defend himself with the assistance of his
working the following day as he would be terminated from the ACMC. representative, if he so desires; and that it is only then that the employer
Thus, Bides filed a complaint for illegal dismissal against ACMC. may dismiss the employee by notifying him of the decision in writing
The LA ruled that Bides was illegally dismissed. The NLRC stating clearly the reasons therefor. Such dismissal is without prejudice to
reversed the LA’s decision. The CA affirmed with modification the the right of the employee to contest its validity in the Regional Branch of
NLRC’s decision. The CA ruled that there was no illegal dismissal but the NLRC.
ordered ACMC to pay Bides separation pay in lieu of reinstatement, The failure of petitioner to give private respondent the benefit of
taking into account the strained relations between the parties. a hearing before he was dismissed constitutes an infringement of his
constitutional right to due process of law and equal protection of the laws.
ISSUE: The standards of due process in judicial as well as administrative
Whether strained relations exist between ACMC and Bides to proceedings have long been established. In its bare minimum due
bar the latter’s reinstatement and justify the award of separation pay? process of law simply means giving notice and opportunity to be heard
before judgment is rendered. Hence, since Petitioner committed an
HELD: infraction of the second requirement. Thus, it must be imposed a
The Court finds no merit in the petition. sanction for its failure to give a formal notice and conduct an investigation
LABOR LAW: doctrine of strained relations as required by law before dismissing petitioner from employment.
The Court is well aware that reinstatement is the rule and,
for the exception of “strained relations” to apply, it should be proved 155 RUBEN SERRANO vs.NLRC and ISETANN DEPARTMENT
that it is likely that, if reinstated, an atmosphere of antipathy and STORE
antagonism would be generated as to adversely affect the efficiency and
productivity of the employee concerned. FACTS:
Under the doctrine of strained relations, the payment of Serrano was hired by Isetann Department Store as a security
separation pay is considered an acceptable alternative to reinstatement checker to apprehend shoplifters and prevent pilferage of merchandise.
when the latter option is no longer desirable or viable. On one hand, such Initially hired on contractual basis and eventually became a regular
payment liberates the employee from what could be a highly oppressive employee He later became head of the Security Checkers Section
work environment. On the other hand, it releases the employer from the of private respondent. As a cost-cutting measure, private respondent
grossly unpalatable obligation of maintaining in its employ a worker it decided to phase out its entire security section and engage the services
could no longer trust. Moreover, the doctrine of strained relations has of an independent security agency. For this reason, the company sent
been made applicable to cases where the employee decides not to Serrano of a verbal notice of its termination In view of the retrenchment
be reinstated and demands for separation pay. program of the company. The loss of his employment prompted petitioner
to file complaint for illegal dismissal, illegal layoff, unfair labor practice, employment, and the question is the appropriate sanction for the violation
underpayment of wages, and non-payment of salary and overtime pay. of petitioner's right.
The Labor Arbiter rendered a decision finding petitioner to have The employer's failure to comply with the notice requirement
been illegally dismissed. He ruled that private respondent failed to does not constitute a denial of due process but a mere failure to observe
establish that it had retrenched its security section to prevent or minimize a procedure for the termination of employment which makes the
losses to its business; that private respondent failed to accord due termination of employment merely ineffectual. It is similar to the
process to petitioner; that private respondent failed to use reasonable failure to observe the provisions of Art. 1592, in relation to Art. 1191, of
standards in selecting employees whose employment would be the Civil Code in rescinding a contract for the sale of immovable property.
terminated; that private respondent had not shown that petitioner and Under these provisions, while the power of a party to rescind a contract is
other employees in the security section were so inefficient so as to justify implied in reciprocal obligations, nonetheless, in cases involving the sale
their replacement by a security agency, or that "cost-saving devices of immovable property, the vendor cannot exercise this power even
[such as] secret video cameras (to monitor and prevent shoplifting) and though the vendee defaults in the payment of the price, except by
secret code tags on the merchandise" could not have been employed; bringing an action in court or giving notice of rescission by means of a
instead, the day after petitioner's dismissal, private respondent employed notarial demand. Consequently, a notice of rescission given in the letter
a safety and security supervisor with duties and functions similar to those of an attorney has no legal effect, and the vendee can make payment
of petitioner. Private respondent appealed to the NLRC which, in its even after the due date since no valid notice of rescission has been
resolution of March 30, 1994; reversed the decision of the Labor Arbiter given.
and ordered petitioner to be given separation pay equivalent to one Indeed, under the Labor Code, only the absence of a just
month pay for every year of service, unpaid salary, and proportionate 13th cause for the termination of employment can make the dismissal of an
month pay. Petitioner contends that abolition of private respondent's employee illegal, that if in proceedings for reinstatement under Art. 283, it
Security Checkers Section and the employment of an independent is shown that the termination of employment was due to an authorized
security agency do not fall under any of the authorized causes for cause, then the employee concerned should not be ordered reinstated
dismissal under Art. 283 of the Labor Code. even though there is failure to comply with the 30-day notice
requirement. Instead, he must be granted separation pay in accordance
ISSUE: with Art. 283.
WON the dismissal was for an authorized cause. If the employee's separation is without cause, instead of being
given separation pay, he should be reinstated. In either case, whether he
HELD: is reinstated or only granted separation pay, he should be paid full
YES. The "[management of a company] cannot be denied the backwages if he has been laid off without written notice at least 30 days
faculty of promoting efficiency and attaining economy by a study of what in advance.
units are essential for its operation. To it belongs the ultimate On the other hand, with respect to dismissals for cause under
determination of whether services should be performed by its personnel Art. 282, if it is shown that the employee was dismissed for any of the just
or contracted to outside agencies . . . [While there] should be mutual causes mentioned in said Art. 282, then, in accordance with that article,
consultation, eventually deference is to be paid to what management he should not be reinstated. However, he must be paid backwages
decides." Consequently, absent proof that management acted in a from the time his employment was terminated until it is determined
malicious or arbitrary manner, the Court will not interfere with the that the termination of employment is for a just cause because the
exercise of judgment by an employer. failure to hear him before he is dismissed renders the termination of
In the case at bar, we have only the bare assertion of petitioner his employment without legal effect.
that, in abolishing the security section, private respondent's real purpose
was to avoid payment to the security checkers of the wage increases 156 JENNY M. AGABON and VIRGILIO C. AGABON,vs NLRC &
provided in the collective bargaining agreement approved in 1990. Such RIVIERA HOME IMPROVEMENTS, INC.
an assertion is not sufficient basis for concluding that the termination of
petitioner's employment was not a bona fide decision of management to FACTS:
obtain reasonable return from its investment, which is a right guaranteed Riviera Home Improvements, Inc. employed petitioners Virgilio
to employers under the Constitution. Indeed, that the phase-out of the Agabon and Jenny Agabon as gypsum board and cornice installers until
security section constituted a "legitimate business decision" is a factual they were dismissed for abandonment of work. Petitioners assert that
finding of an administrative agency which must be accorded respect and they were dismissed because the private respondent refused to give
even finality by this Court since nothing can be found in the record which them assignments unless they agreed to work on a pakyaw basis. They
fairly detracts from such finding. Accordingly, the termination of did not agree on this arrangement because it would mean losing benefits
petitioner's services was for an authorized cause, i.e., redundancy. as Social Security System (SSS) members. Petitioners also claim that
Hence, pursuant to Art. 283 of the Labor Code, petitioner private respondent did not comply with the twin requirements of notice
should be given separation pay at the rate of one month pay for every and hearing. Private respondent, on the other hand, maintained that
year of service. petitioners were not dismissed but had abandoned their work. In fact,
private respondent sent two letters to the last known addresses of the
ISSUE: petitioners advising them to report for work. Private respondents
WON the employer's failure to comply with the notice manager even talked to petitioner Virgilio Agabon by telephone sometime
requirement constitute a denial of due process. in June 1999 to tell him about the new assignment at Pacific Plaza
Towers involving 40,000 square meters of cornice installation work.
HELD: However, petitioners did not report for work because they had
NO. Art. 283 also provides that to terminate the employment of subcontracted to perform installation work for another company.
an employee for any of the authorized causes the employer must serve
"a written notice on the workers and the Department of Labor and ISSUE:
Employment at least one (1) month before the intended date thereof." In WON the procedure for dismissal were observed
the case at bar, petitioner was given a notice of termination on October
11, 1991. On the same day, his services were terminated. He was thus HELD:
denied his right to be given written notice before the termination of his
The procedure for terminating an employee is found in Book VI, On another breath, a dismissal for an authorized cause under
Rule I, Section 2(d) of the Omnibus Rules Implementing the Labor Code: Article 283 does not necessarily imply delinquency or culpability on the
Standards of due process: requirements of notice. In all part of the employee. Instead, the dismissal process is initiated by the
cases of termination of employment, the following standards of due employers exercise of his management prerogative, i.e. when the
process shall be substantially observed: employer opts to install labor saving devices, when he decides to cease
I. For termination of employment based on just causes as defined in business operations or when, as in this case, he undertakes to
Article 282 of the Code: implement a retrenchment program. In dismissal for just cause under
(a) A written notice served on the employee specifying the Article 282, the payment of separation pay, as a rule, is not required and
ground or grounds for termination, and giving to said employee a dismissal for authorized cause under Article 283, the law requires
reasonable opportunity within which to explain his side; payment of separation pay.
(b) A hearing or conference during which the employee Accordingly, it is wise to hold that: (1) if the dismissal is based
concerned, with the assistance of counsel if the employee so desires, is on a just cause under Article 282 but the employer failed to comply with
given opportunity to respond to the charge, present his evidence or rebut the notice requirement, the sanction to be imposed upon him should be
the evidence presented against him; and tempered because the dismissal process was, in effect, initiated by an
(c) A written notice of termination served on the employee act imputable to the employee; and (2) if the dismissal is based on an
indicating that upon due consideration of all the circumstances, grounds authorized cause under Article 283 but the employer failed to comply
have been established to justify his termination. In case of termination, with the notice requirement, the sanction should be stiffer because the
the foregoing notices shall be served on the employees last dismissal process was initiated by the employers exercise of his
known address. management prerogative.
In this case, the dismissal should be upheld because it was In this case, JAKA was suffering from serious business losses
established that the petitioners abandoned their jobs to work for another at the time it terminated respondents employment. It is, therefore,
company, a just cause under Art. 282. Private respondent, however, did established that there was ground for respondents dismissal, i.e.,
not follow the notice requirements and instead argued that sending retrenchment, which is one of the authorized causes enumerated under
notices to the last known addresses would have been useless because Article 283 of the Labor Code. Likewise, it is established that JAKA failed
they did not reside there anymore. Unfortunately for the private to comply with the notice requirement under the same Article.
respondent, this is not a valid excuse because the law mandates the twin Considering the factual circumstances in the instant case and the above
notice requirements to the employees last known address. Thus, it ratiocination, we, therefore, deem it proper to fix the indemnity at
should be held liable for non-compliance with the procedural P50,000.00. In all cases of business closure or cessation of operation or
requirements of due process. undertaking of the employer, the affected employee is entitled to
The violation of petitioners right to statutory due process separation pay. This is consistent with the state policy of treating labor as
by the private respondent warrants the payment of indemnity in the a primary social economic force, affording full protection to its rights as
form of nominal damages. The amount of such damages is well as its welfare. The exception is when the closure of business or
addressed to the sound discretion of the court, taking into account cessation of operations is due to serious business losses or
the relevant circumstances. financial reverses; duly proved, in which case, the right of affected
employees to separation pay is lost for obvious reasons.
157 JAKA FOOD PROCESSING CORPORATION vs. DARWIN
PACOT, et.al 158 NELSON A. CULILI, Petitioner, v. EASTERN
TELECOMMUNICATIONS PHILIPPINES, INC., SALVADOR HIZON
FACTS: (President and Chief Executive Officer), EMILIANO JURADO
Respondents were earlier hired by JAKA Foods Processing Corporation (Chairman of the Board), VIRGILIO GARCIA (Vice President) and
until the latter terminated their employment because the corporation was STELLA GARCIA (Assistant Vice President), Respondents.
in dire financial straits. It is not disputed, however, that the termination
was effected without JAKA complying with the requirement under Article FACTS:
283 of the Labor Code regarding the service of a written notice upon Respondent Eastern Telecommunications Philippines, Inc.
the employees and the Department of Labor and Employment at least (ETPI) is a telecommunications company engaged mainly in the business
one (1) month before the intended date of termination. Respondents of establishing commercial telecommunications systems and leasing of
separately filed complaints for illegal dismissal wherein after due international datalines or circuits that pass through the international
proceedings, the Labor Arbiter rendered a decision declaring the gateway facility (IGF). The other respondents are ETPIs officers.
termination illegal and ordering JAKA and its HRD Manager to reinstate Petitioner Nelson A. Culili was employed by ETPI as a
respondents with full backwages, and separation pay if reinstatement is Technician in its Field Operations Department in 1981. In 1996, Culili
not possible. was promoted to Senior Technician in the Customer Premises
Equipment Management Unit of the Service Quality Department.
ISSUE: As a telecommunications company and an authorized IGF
What are the legal implications of a situation where an operator, ETPI was required, under RA No. 7925 and EO No. 109, to
employee is dismissed for cause but such dismissal was effected without establish landlines in Metro Manila and certain provinces. However, due
the employers compliance with the notice requirement under the Labor to interconnection problems with the PLDT, poor subscription and
Code. cancellation of subscriptions, and other business difficulties, ETPI was
forced to halt its roll out of 129,000 landlines already allocated to a
HELD: number of its employees.
A dismissal for just cause under Article 282 implies that the In 1998, due to business troubles and losses, ETPI was
employee concerned has committed, or is guilty of, some violation compelled to implement a Right-Sizing Program which consisted of two
against the employer, i.e. the employee has committed some serious phases: the first phase involved the reduction of ETPIs workforce to only
misconduct, is guilty of some fraud against the employer, or, as in those employees that were necessary and which ETPI could sustain; the
Agabon, he has neglected his duties. Thus, it can be said that the second phase entailed a company-wide reorganization which would
employee himself initiated the dismissal process. result in the transfer, merger, absorption or abolition of certain
departments of ETPI.
As part of the first phase, ETPI offered to its employees who undertaken by the enterprise. Soriano, Jr. v. NLRC, G.R. No. 165594,
had rendered at least fifteen years of service, the Special Retirement April 23, 2007
Program, which consisted of the option to voluntarily retire at an earlier This Court also held that the following evidence may be
age and a retirement package equivalent to two and a half (2) months proffered to substantiate redundancy: the new staffing pattern, feasibility
salary for every year of service. This offer was initially rejected by the studies/ proposal on the viability of the newly created positions, job
Eastern Telecommunications Employees Union (ETEU), ETPIs duly description and the approval by the management of the restructuring.
recognized bargaining agent, which threatened to stage a strike. ETPI In the case at bar, ETPI was upfront with its employees about
explained to ETEU the exact details of the Right-Sizing Program and the its plan to implement a Right-Sizing Program. Even in the face of initial
Special Retirement Program and after consultations with ETEUs opposition from and rejection of the said program by ETEU, ETPI
members, ETEU agreed to the implementation of both programs. Thus, patiently negotiated with ETEUs officers to make them understand ETPIs
ETPI re-offered the Special Retirement Program and the corresponding business dilemma and its need to reduce its workforce and streamline its
retirement package to the one hundred two (102) employees who organization. This evidently rules out bad faith on the part of ETPI.
qualified for the program. Of all the employees who qualified to avail of The records show that ETPI had sufficiently established not
the program, only Culili rejected the offer. only its need to reduce its workforce and streamline its organization, but
Among the departments abolished was the Service Quality also the existence of redundancy in the position of a Senior Technician.
Department. The functions of the Customer Premises Equipment ETPI explained how it failed to meet its business targets and the factors
Management Unit, Culilis unit, were absorbed by the Business and that caused this, and how this necessitated it to reduce its workforce and
Consumer Accounts Department. As a result, Culilis position was streamline its organization. ETPI also submitted its old and new tables of
abolished due to redundancy and his functions were absorbed by the organization and sufficiently described how limited the functions of the
Business and Consumer Accounts Department. abolished position of a Senior Technician were and how it decided on
ETPI, through its Assistant Vice President Stella Garcia, whom to absorb these functions.
informed Culili of his termination from employment effective April 8, 1999. LABOR LAW
Culili alleged that neither he nor the DOLE were formally Although the Court finds Culilis dismissal was for a lawful
notified of his termination. Culili believed that ETPI had already decided cause and not an act of unfair labor practice, ETPI, however, was remiss
to dismiss him even prior to the March 8, 1999 letter. Moreover, Culili in its duty to observe procedural due process in effecting the termination
asserted that ETPI had contracted out the services he used to perform to of Culili.
a labor-only contractor which not only proved that his functions had not For termination of employment as defined in Article 283 of the
become unnecessary, but which also violated their Collective Bargaining Labor Code, the requirement of due process shall be deemed complied
Agreement (CBA) and the Labor Code. Aside from these, Culili also with upon service of a written notice to the employee and the appropriate
alleged that he was discriminated against when ETPI offered some of his Regional Office of the Department of Labor and Employment at least
co-employees an additional benefit in the form of motorcycles to induce thirty days before effectivity of the termination, specifying the ground or
them to avail of the Special Retirement Program, while he was not. grounds for termination.
ETPI denied singling Culili out for termination. ETPI claimed ETPI does not deny its failure to provide DOLE with a written
that because there was no more work for Culili, it was constrained to notice regarding Culilis termination. It, however, insists that it has
serve a final notice of termination to Culili, which Culili ignored. Thus, on complied with the requirement to serve a written notice to Culili as
March 26, 1999, ETPI tendered to Culili his final pay check of evidenced by his admission of having received it and forwarding it to his
P859,033.99 consisting of his basic salary, leaves, 13th month pay and union president.
separation pay. ETPI claimed that Culili refused to accept his termination The Court of Appeals, in finding that Culili was not afforded
and continued to report for work. procedural due process, held that Culilis dismissal was ineffectual, and
Culili filed a complaint against ETPI and its officers for illegal required ETPI to pay Culili full backwages in accordance with our
dismissal, unfair labor practice, and money claims before the Labor decision in Serrano v. NLRC, 387 Phil. 345 (2000).
Arbiter. Hence, since it has been established that Culilis termination
The Labor Arbiter found ETPI guilty of illegal dismissal and was due to an authorized cause and cannot be considered unfair labor
unfair labor practice. practice on the part of ETPI, his dismissal is valid. However, in view of
On appeal, the NLRC affirmed the Labor Arbiters decision but ETPIs failure to comply with the notice requirements under the Labor
modified the amount of moral and exemplary damages awarded. Code, Culili is entitled to nominal damages in addition to his separation
The Court of Appeals found that Culilis position was validly pay.
abolished due to redundancy. It further held that ETPI cannot be held
guilty of unfair labor practice as mere contracting out of services being 159 PHILIPPINE CHARITY SWEEPSTAKES OFFICE BOARD OF
performed by union members does not per se amount to unfair labor DIRECTORS and REYNALDO P. MARTIN v. MARIE JEAN C. LAPID
practice unless it interferes with the employees right to self-organization.
Hence, this petition. FACTS:
An administrative complaint was filed against the Respondent
ISSUE: for allegedly confronting, badmouthing and shouting invectives at Mr.
Whether or not Culili is illegally dismissed. Guemo, in the presence of other employees and seeking assistance from
the PSCO. The PCSO Board of Directors found her guilty of discourtesy
HELD: in the course of official duties and grave misconduct and imposed on her
The decision of the Court of Appeals is sustained. the penalty of dismissal from service.
LABOR LAW On appeal with the CSC, the Commission dismissed the
There is redundancy when the service capability of the respondent’s appeal for being moot and academic. Moreover, they ruled
workforce is greater than what is reasonably required to meet the that the respondent is a casual employee which means that she is not
demands of the business enterprise. A position becomes redundant entitled to security of tenure. However, the CA reversed the decision of
when it is rendered superfluous by any number of factors such as over- the Commission by reinstating the respondent in the service until the
hiring of workers, decrease in volume of business, or dropping a expiration of her casual employment.
particular product line or service activity previously manufactured or
ISSUE:
Did the CA gravely err in granting the respondent’s reiterating that on the time the alleged crime took place she, together
petition, in effect, reversing the CSC’s resolutions. with her two salesgirls, had first counted the cash before placing it in a
plastic bag that she deposited inside the drawer of the cabinet with the
HELD: knowledge of the other salesgirls. One of the salesgirls however averred
A new ruling recognizes that casual employees are covered by that she had left the petitioner alone because the latter had still to change
the security of tenure and cannot be terminated within the period of his her clothes; and that that was the first time that the petitioner had ever
employment except for cause. Despite this new ruling, it is not the asked to be left behind, for they had previously left the kiosk together.
intention of the Court to make the status of a casual employee at par with Respondent Vina declared that the petitioner did not call the office of
that of a regular employee, who enjoys permanence of employment. The Minex for the pick-up of the P39,194.50 cash sales on that faithful day in
rule is still that casual employment will cease automatically at the end of violation of the standard operating procedure (SOP) requiring cash
the period unless renewed as stated in the Plantilla of Casual proceeds exceeding P10,000.00 to be reported for pick-up if the amount
Employment. Casual employees may also be terminated anytime though could not be
subject to certain conditions or qualifications. Thus, they may be laid-off deposited in the bank. After the preliminary investigation, the fiscal
anytime before the expiration of the employment period provided any of rendered a resolution finding probable cause for qualified theft and
the following occurs:(1) when their services are no longer needed; (2) recommending the filing of an information against the petitioner. Thus,
funds are no longer available; (3) the project has already been she was charged with qualified theft before the Regional Trial Court.
completed/finished; or (4) their performance are below par. The petitioner argued that there was no evidence at all upon
Equally important, they are entitled to due process especially if which Minex could validly dismiss her considering that she had not yet
they are to be removed for more serious causes or for causes other than been found guilty beyond reasonable doubt of the crime of qualified theft.
the reasons mentioned in CSC Form No. 001. The reason for this is that
their termination from the service could carry a penalty affecting their Issues:
rights and future employment in the government. Whether or not there was valid ground to terminate the
In the case at bench, the CSC itself found that Lapid was petitioner.
denied due process as she was never formally charged with the
administrative offenses of Discourtesy in the Course of Official Duties Ruling:
and Grave Misconduct, for which she was dismissed from the service. To The petitioner’s argument is not novel. It has been raised and
somehow remedy the situation, the petitioners mentioned in their rejected many times before on the basis that neither conviction beyond
Memorandum before the CA that there was no reason anymore to reasonable doubt for a crime against the employer nor acquittal after
pursue the administrative charge against Lapid and to investigate further criminal prosecution was indispensable. Nor was a formal charge in court
as this was superseded by Memorandum dated September 14, 2005 for the acts prejudicial to the interest of the employer a pre-requisite for a
recommending the termination of respondent Lapid’s casual valid dismissal. The criminal charges initiated by the company against
employment. They pointed out that this was precisely the reason why no private respondents and the finding after preliminary investigation of their
Formal Charge was issued. Clearly, the action of petitioners clearly prima facie guilt of the offense charged constitute substantial evidence
violated Lapid’s basic rights as a casual employee. termination based on loss of trust and confidence.
Therefore, the petition is denied and the respondent is allowed The Labor Tribunal need not have gone further as to require
to continue rendering services as teller of PCSO and is also entitled to private respondent’s conviction of the crime charged, or inferred
payment of backwages. innocence on their part from their release from detention, which was
mainly due to their posting of bail. While there is a valid ground to
160 Concepcion vs. Minex Import Corp., G.R. No. 153569, January terminate petitioner, respondent however failed to comply with the
24, 2012 requirements of due process prior to the termination under the
implementing rules and regulations of the Labor Code.
Facts: In all cases of termination of employment, the following
Respondent is engaged in the retail of semi-precious stones, standards of due process shall be substantially observed. For termination
selling them in kiosks or stalls installed in various shopping centers. It of employment based on just causes as defined in Article 282 of the
employed the petitioner initially as a salesgirl then later on as supervisor. Labor Code:
Working under her supervision were salesgirls Cristina Calung and Lida (i) A written notice served on the employee specifying the
Baquilar. ground or grounds for termination, and giving said employee reasonable
One day the petitioner and her salesgirls had sales of crystal opportunity within which to explain his side.
items totaling P39,194.50. At the close of business that day, they (ii) A hearing or conference during which the employee
conducted a cashcount of their sales proceeds, including those from the concerned, with the assistance of counsel if he so desires is given
previous two days and determined their total for the three days to be opportunity to respond to the charge, present his evidence, or rebut the
P50,912.00. The petitioner wrapped the amount in a plastic bag and evidence presented against him.
deposited it in the drawer of the locked wooden cabinet of the kiosk. (iii) A written notice of termination served on the employee,
The following day petitioner phoned respondent Vina Mariano indicating that upon due consideration of all the circumstances, grounds
to report that the P50,912.00 was missing, explaining how she and her have been established to justify his termination
salesgirls had placed the wrapped amount at the bottom of the cabinet In this case the respondents immediately had her arrested and
the night before, and how she had found upon reporting to work that investigated by the police authorities for qualified theft which constitutes
morning that the contents of the cabinet were in disarray and the money. a denial of her right to due process of law, consisting in the opportunity to
Later, while the petitioner was giving a detailed statement on be heard and to defend herself. In fact, their decision to dismiss her was
the theft to the security investigator of Harrison Plaza, Vina and Sylvia already final even before the police authority commenced an
Mariano, her superiors, arrived with a policeman who immediately placed investigation of the theft, the finality being confirmed by no less than
the petitioner under arrest and brought her to a police station where she Sylvia Mariano herself telling the petitioner during their phone
was investigated her and detained for a day. conversation following the latter’s release from police custody that she
Subsequently petitioner filed a case for illegal dismissal against (Sylvia) “no longer wanted to see” her.
respondent and two days later respondent filed a criminal case for The fact that the petitioner was the only person suspected of
qualified theft against petitioner. The petitioner insisted on her innocence, being responsible for the theft aggravated the denial of due process.
terminated their contracts. This telex was given credibility and weight by
161 Skippers United Pacific vs. Doza, G.R. No. 175558, February 8, the Labor Arbiter and NLRC in deciding that there was pre-termination of
2012 the employment contract "akin to resignation" and no illegal dismissal.
(DEMAND FOR REPATRIATION, NOT VOLUNTARY RESIGNATION) However, as correctly ruled by the CA, the telex message is "a biased
and self-serving document that does not satisfy the requirement of
Facts: substantial evidence." If, indeed, De Gracia, et al., voluntarily pre-
Petitioner deployed De Gracia, Lata and Aprosta to work on terminated their contracts, then De Gracia, et al., should have submitted
board the vessel MV Wisdom Star. On December 3 1998, Skippers their written resignations. Article 285 of the Labor Code recognizes
alleges that De Garcia smelling strongly of alcohol, went to the cabin of termination by the employee of the employment contract by "serving
Gabriel Oleszek, MV Wisdom Stars’ Master. Skippers claims that he was written notice on the employer at least one (1) month in advance." Given
rude and shouted noisily to the master. De Gracia left the master’s cabin that provision, the law contemplates the requirement of a written notice of
after a few minutes and was heard shouting very loudly somewhere resignation. In the absence of a written resignation, it is safe to presume
down the corridors. The incident was evidenced by the Captain’s Report that the employer terminated the seafarers. In addition, the telex
sent on said date. message relied upon by the Labor Arbiter and NLRC bore conflicting
Furthermore, Skippers also claim that on January 22, 1999, dates of 22 January 1998 and 22 January 1999, giving doubt to the
Aprosta, De Gracia, Lata and Daza arrived in the master’s cabin and veracity and authenticity of the document.
demanded immediate repatriation because they were not satisfied with In 22 January 1998, De Gracia, et al., were not even employed yet by the
the ship. De Gracia, et al. threatened that they may become crazy any foreign principal.
moment and demanded for all outstanding payments due to them. The
incident is evidenced by a telex of Cosmoship MV Wisdom to skippers 162 SEBASTIAN F. OASAY, JR. Petitioner, v. PALACIO DEL
but had conflicting dates. GOBERNADOR CONDOMINIUM CORPORATION and/or OMAR T.
De Gracia claims that Skippers failed to remit their respective CRUZ, Respondents.
allotments, compelling them to vent their grievances with the Romanian (BUILDING ADMIN OF PDGCC, a GOCC; BREACH OF TRUST)
Seafarers Union. On January 28, 1999, the Filipino seafarers were
unceremoniously discharged and immediately repatriated. Upon arrival in FACTS:
the Philippines, they filed a complaint for illegal dismissal with the LA. Respondent Palacio Del Gobernador Condominium
The LA dismissed the seafarers’ complaint as the seafarers’ Corporation (PDGCC) is a government-owned and controlled corporation
demand for immediate repatriation due to the dissatisfaction with the ship organized for the purpose of owning and arranging the common areas of
is considered a voluntary pre-termination of employment. Such act was Palacio Del Gobernador Condominium. The said condominium, all the
deemed akin to resignation recognized under Article 285 of the LC. The units therein having been acquired by the government, houses various
LA gave credence to the telex of the master’s report that the seafarers government agencies such as the Commission on Elections
indeed demanded immediate repatriation. (COMELEC), Bureau of Treasury and the Intramuros.
The NLRC agreed with the LA’s decision. The CA however The Board of Directors of PDGCC, through its Board
reversed the LA’s and the NLRC’s decision. The Court deemed the telex Resolution No. 013 dated October 27, 1994, appointed the petitioner as
message as a self-serving document that does not its permanent Building Administrator effective September 1, 1994.
satisfy the requirement of substantial evidence, or that amount of PDGCC President Omar T. Cruz (Cruz) required the petitioner
relevant evidence which a reasonable mind might accept as adequate to to submit a written report on the allowances and other compensation, in
justify the conclusion that petitioners indeed voluntarily demanded their connection with his duties as Building Administrator, that he received
immediate repatriation. Aggrieved, Skippers appeals the case with the from the government offices housed in the condominium. Apparently, the
Supreme Court. petitioner had been earning additional income for services that he
rendered for the COMELEC.
Issue: Petitioner submitted his written report wherein he admitted that
Whether or not the seafarer’s demand for immediate he had received additional compensation from the COMELEC for
repatriation can be considered an act of voluntary resignation. services which he rendered after his regular working hours and on
Saturdays, Sundays and holidays. He explained that the COMELEC had
Ruling: caused the rehabilitation of the 8th floor of the condominium and that he
For a worker's dismissal to be considered valid, it must comply was tasked by the former, for a stated compensation, to supervise and
with both procedural and substantive due process. The legality of the monitor the rehabilitation.
manner of dismissal constitutes procedural due process, while the After investigating the allegations against the petitioner, Atty.
legality of the act of dismissal constitutes substantive due process. Bernardo recommended to Cruz and the PDGCC Board of Directors the
Procedural due process in dismissal cases consists of the twin filing of appropriate charges against the petitioner for violation of
requirements of notice and hearing. The employer must furnish the Republic Act No. 3019 (Anti-Graft and Corrupt Practices Act) and
employee with two written notices before the termination of employment Republic Act No. 6713 (Code of Conduct and Ethical Standards for
can be effected: (1) the first notice apprises the employee of the Public Officials and Employees).
particular acts or omissions for which his dismissal is sought; and (2) the Cruz then directed the petitioner to turn over all of his
second notice informs the employee of the employer's decision to accountabilities to PDGCC. The foregoing was acknowledged by the
dismiss him. Before the issuance of the second notice, the requirement petitioner in his letter to the PDGCC Board of Directors dated November
of a hearing must be complied with by giving the worker an opportunity to 17, 2006.
be heard. It is not necessary that an actual hearing be conducted. Nevertheless, on January 23, 2007, the petitioner filed a
Substantive due process, on the other hand, requires that dismissal by Complaint for constructive dismissal with the arbitration branch of the
the employer be made under a just or authorized cause under Articles National Labor Relations Commission (NLRC) in Quezon City against
282 to 284 of the Labor Code. PDGCC and Cruz.
In this case, there was no written notice furnished to De Gracia, The Labor Arbiter (LA) rendered a Decision dismissing the
et al., regarding the cause of their dismissal. Cosmoship furnished a petitioners complaint, finding that there was substantial evidence to
written notice (telex) to Skippers, the local manning agency, claiming that conclude that the petitioner had breached the trust and confidence of
De Gracia, et al., were repatriated because the latter voluntarily pre- PDGCC. The NLRC affirmed the LA decision. Petitioner filed a petition
for certiorari before the CA but the same was denied. Undaunted, the
petitioner instituted the instant petition for review on certiorari before this 163 Lynvil Fishing Enterprises vs. Ariola, G.R. No. 181974, February
Court. 1, 2012

ISSUE: Whether or not the dismissal by reason of breach of trust was Facts:
valid Petitioner Lynvil Fishing Enterprises, Inc. (Lynvil) is engaged in
deep-sea fishing. Respondents’ services were engaged in various
HELD: capacities:
Yes. Andres G. Ariola, captain; Jessie D. Alcovendas, chief mate;
Labor Law Jimmy B. Calinao, chief engineer; Ismael G. Nubla, cook; Elorde Bañez,
The first requisite for dismissal on the ground of loss of trust oiler; and Leopoldo G. Sebullen, bodegero.
and confidence is that the employee concerned must be holding a On Aug. 1, 1998, Lynvil received a report from Ramonito
position of trust and confidence. Clarido, one of its employees, that on July 31, 1998, he witnessed that
Here, it is indubitable that the petitioner holds a position of trust while on board the company vessel Analyn VIII, respondents conspired
and confidence. The position of Building Administrator, being managerial with one another and stole eight tubs of “pampano” and “tangigue” fish
in nature, necessarily enjoys the trust and confidence of the employer. and delivered them to another vessel.
The second requisite is that there must be an act that would Petitioner filed a criminal complaint against respondents before
justify the loss of trust and confidence. Loss of trust and confidence, to the office of the City Prosecutor of Malabon City which found probable
be a valid cause for dismissal, must be based on a willful breach of trust cause for indictment of respondents for the crime of qualified theft.
and founded on clearly established facts. The basis for the dismissal Relying on the finding and Nasipit Lumber Company v. NLRC, 257 Phil.
must be clearly and convincingly established but proof beyond 937 (1989), Lynvil asserted there was sufficient basis for valid
reasonable doubt is not necessary. termination of employment of respondents based on serious misconduct
PDGCC had established, by clear and convincing evidence, and/or loss of trust and confidence.
the petitioners acts which justified its loss of trust and confidence on the
former. On this score, the LA keenly observed that: Issues:
Complainants breach of the trust reposed in him as Building Whether a finding of the city prosecutor of probable cause to
Administrator is sufficiently supported by the evidence on record. indict employees of qualified theft is sufficient basis for valid termination
Complainants admission that he received remuneration from for serious misconduct and/or loss of trust or confidence?
Commission on Elections (COMELEC) whose office is housed at Whether the employees were validly terminated?
respondent Palacio Del Gobernador Condominium justified his
termination of employment. Complainant cannot assert that he rendered Ruling:
services to COMELEC only after office hours as his functions as Building On the first issue, the Supreme Court ruled in the negative. We
Coordinator would definitely have favored COMELEC in the performance ruled that proof beyond reasonable doubt of an employee’s misconduct is
of his functions during regular office hours. not required when loss of confidence is the ground for dismissal. It is
Likewise, as Building Administrator, his active vigilance in sufficient if the employer has “some basis” to lose confidence or that the
reporting and informing the respondents as to the expired license to employer has reasonable ground to believe or to entertain the moral
operate of the EGB Security Agency and its revoked SEC Certificate of conviction that the employee concerned is responsible for the misconduct
Registration was his duty and look-out. In the instant case, complainant and that the nature of his participation therein rendered him absolutely
instead of informing the respondents, kept this information from the unworthy of the trust and confidence demanded by his position.
knowledge of the respondents and allowed the security agency to render Lynvil cannot argue that since the Office of the Prosecutor
security services to the premises of respondents despite its expired found probable cause for theft the Labor Arbiter must follow the finding
license and revoked SEC Certificate of Registration. as a valid reason for the termination of respondents’ employment. The
What escapes the foregoing argument of the petitioner is that proof required for purposes that differ from one and the other are likewise
he is an employee of PDGCC and not of the COMELEC. It is undisputed different.
that PDGCC did not authorize nor was it informed of the services On the second question, the Court stated that nonetheless,
rendered by the petitioner in favor of the COMELEC. To make matters even without reliance on the prosecutor’s finding, we find that there was
worse, the said services rendered by the petitioner are, essentially, valid cause for respondents’ dismissal.
related to the performance of his duties as a Building Administrator of the Just cause is required for a valid dismissal. The Labor Code
condominium. provides that an employer may terminate an employment based on fraud
or willful breach of the trust reposed on the employee. Such breach is
On the procedural aspect, we find that PDGCC had observed due considered willful if it is done intentionally, knowingly, and purposely,
process in effecting the dismissal of the petitioner. without justifiable excuse, as distinguished from an act done carelessly,
thoughtlessly, heedlessly or inadvertently. It must also be based on
DGCC complied with the first notice requirement, i.e. notice informing the substantial evidence and not on the employer’s whims or caprices or
petitioner of his infractions, as shown by the following: (1) the suspicions otherwise, the employee would eternally remain at the mercy
Memorandum dated September 27, 2005 sent by Cruz to the petitioner of the employer. Loss of confidence must not be indiscriminately used as
requiring the latter to explain and to submit his report on the additional a shield by the employer against a claim that the dismissal of an
compensation he received from COMELEC; and (2) the letter dated employee was arbitrary. And, in order to constitute a just cause for
December 9, 2005 sent by Cruz to the petitioner requiring him to explain dismissal, the act complained of must be work-related and shows that the
why he allowed the EGB Security Investigation and General Services, employee concerned is unfit to continue working for the employer. In
Inc. to render services to the condominium. addition, loss of confidence as a just cause for termination of
employment is premised on the fact that the employee concerned holds a
The second notice requirement was likewise complied with by PDGCC position of responsibility, trust and confidence or that the employee
when it sent to the petitioner the Memorandum dated October 28, 2006 concerned is entrusted with confidence with respect to delicate matters,
which, in essence, informed the latter that a new Building Administrator such as the handling or care and protection of the property and assets of
had been appointed.
the employer. The betrayal of this trust is the essence of the offense for company time for his personal affairs, but only for a few hours and not
which an employee is penalized. Breach of trust is present in this case. the whole day.
However, Lynvil contends that it cannot be guilty of illegal While respondent was still suspended, Norkis also found that
dismissal because the private respondents were employed under a fixed- Respondent committed some inappropriate and irregular acts such as
term contract which expired at the end of the voyage. Contrarily, the unexplained low performance of his branch, missing funds, unauthorized
private respondents (employees) contend that they became regular disbursement of funds, irregular transactions.
employees by reason of their continuous hiring and performance of tasks Petitioners terminated respondent’s services for loss of trust
necessary and desirable in the usual trade and business of Lynvil. and confidence and gross inefficiency. Respondent filed a complaint for
Jurisprudence, laid two conditions for the validity of a fixed- illegal suspension and illegal dismissal. LA favored respondent.
contract agreement between the employer and employee: first, the Petitioners appealed to NLRC. NLRC reversed the LA’s decision and
fixed period of employment was knowingly and voluntarily agreed upon found respondent to have been validly dismissed. The NLRC, however,
by the parties without any force, duress, or improper pressure being upheld the LA’s finding that petitioners are liable to respondent for unpaid
brought to bear upon the employee and absent any other circumstances wages. Respondent filed MR. It was denied so he filed with the CA a
vitiating his consent; or second, it satisfactorily appears that the petition for certiorari. CA reinstated with modification the decision of the
employer and the employee dealt with each other on more or less equal LA. Respondent filed a motion for clarification as to the awards of
terms with no moral dominance exercised by the former or the latter. separation pay and back wages while petitioners filed MR. CA issued a
In the context of the facts that: (1) the respondents were doing tasks Resolution stating that as regards respondent’s motion for clarification,
necessarily to Lynvil’s fishing business with positions ranging from the separation pay and back wages shall be reckoned from the time
captain of the vessel to bodegero; (2) after the end of a trip, they will respondent was illegally suspended until finality of its earlier Decision.
again be hired for another trip with new contracts; and (3) this The CA likewise denied petitioners’ MR. Hence, petitioners filed the
arrangement continued for more than ten years, the clear intention is to present petition.
go around the security of tenure of the respondents as regular
employees. And respondents are so by the express provisions of the ISSUE:
second paragraph of Article 280, thus: xxx Provided, That any employee Was respondent lawfully dismissed for grounds of loss of trust
who has rendered at least one year of service, whether such service is and confidence?
continuous or broken, shall be considered a regular employee with
respect to the activity in which he is employed and his employment shall RULING:
continue while such activity exists. NO. Loss of trust and confidence as a ground for termination of
Having found that respondents are regular employees who an employee under Article 282 of the Labor Code requires that the
may be, however, dismissed for cause as we have so found in this case, breach of trust be willful, meaning it must be done intentionally,
there is a need to look into the procedural requirement of due process in knowingly, and purposely, without justifiable excuse. The basic premise
Section 2, Rule XXIII, Book V of the Rules Implementing the Labor Code. for dismissal on the ground of loss of confidence is that the employee
It is required that the employer furnish the employee with two written concerned holds a position of trust and confidence. It is the breach of this
notices: (1) a written notice served on the employee specifying the trust that results in the employer’s loss of confidence in the employee.
ground or grounds for termination, and giving to said employee Here, there is no question that as petitioners’ Branch Manager
reasonable opportunity within which to explain his side; and (2) a written in Iligan City, respondent was holding a position of trust and confidence.
notice of termination served on the employee indicating that upon due He was responsible for the administration of the branch, and exercised
consideration of all the circumstances, grounds have been established to supervision and control over all the employees. He was also incharge of
justify his termination. In this case, it is clear that the employees were not sales and collection.
given the final written notices of dismissal. In termination cases, the burden of proof rests upon the
The Court ruled that since employees were dismissed for just employer to show that the dismissal is for a just and valid cause and
cause, they were not entitle to separation pay and backwages. However, failure to do so would necessarily mean that the dismissal was illegal.
they were to be granted nominal damages for failure of the employer to The quantum of proof required in determining the legality of an
comply with statutory due process. employee’s dismissal is only substantial evidence. CA correctly held that
petitioners failed to discharge this burden.
164 Norkis vs. Buat - GR No. 185255 Petitioners having failed to establish by substantial evidence
any valid ground for terminating respondent’s services, we uphold the
FACTS: finding of the Labor Arbiter and the CA that respondent was illegally
Respondent Delfin S. Descallar was assigned at the Iligan City dismissed.
Branch of petitioner Norkis Distributors, Inc., a distributor of Yamaha An illegally dismissed employee is entitled to two reliefs:
motorcycles. He became a regular employee and was promoted as back wages and reinstatement. The two reliefs provided are separate
Branch Manager. He acted as branch administrator and had supervision and distinct. In instances where reinstatement is no longer feasible
and control of all the employees. Respondent was also responsible for because of strained relations between the employee and the employer,
sales and collection separation pay is granted. The normal consequences of respondent’s
In a memorandum, petitioners required respondent to explain illegal dismissal, then, are reinstatement without loss of seniority rights,
in writing within 48 hrs why he should not be penalized or terminated for and payment of back wages computed from the time compensation was
being absent without official leave (AWOL) or rendering under-time withheld from him up to the date of actual reinstatement. Where
service on certain dates. Respondent explained that he reported to the reinstatement is no longer viable as an option, separation pay equivalent
office on those dates, but he either went to the bank or followed-up on to one month salary for every year of service should be awarded as an
prospects. As he was still within city limits, he did not file any official alternative. The payment of separation pay is in addition to payment of
leave or travel record. back wages.
Norkis conducted an investigation. Finding that respondent was The CA merely clarified the period of payment of back wages
not able to prove that he was really in the branch or on official travel, and separation pay up to the finality of its decision modifying the LA’s
petitioners suspended him for 15 days without pay. According to decision. In view of the modification of monetary awards in the Labor
petitioners, respondent admitted during the investigation that he used Arbiter’s decision, the time frame for the payment of back wages and
separation pay is accordingly modified to the finality of the CA decision.
WHEREFORE, the petition for review on certiorari is DENIED. principle, the employer has the onus of proving with clear, accurate,
consistent, and convincing evidence the validity of the dismissal.

Aliling was dismissed for an unjust cause


165 Armando Ailing, Petitioner vs. Jose B. Feliciano, Manuel F. San
Mateo III, et al., Respondents First off, the attendant circumstances in the instant case aptly show that
the issue of petitioner’s alleged failure to achieve his quota, as a ground
FACTS: for terminating employment, strikes the Court as a mere afterthought on
the part of WWWEC.
Respondent Wide Wide World Express Corporation (WWWEC) offered to
employ petitioner Armando Aliling (Aliling) on June 2, 2004 as “Account What WWWEC considered as the evidence purportedly showing it gave
Executive (Seafreight Sales),”. The offer came with a six (6)-month Aliling the chance to explain his inability to reach his quota was a
probation period condition with this express caveat: “Performance during purported September 20, 2004 memo of San Mateo addressed to the
probationary period shall be made as basis for confirmation to Regular or latter. However, Aliling denies having received such letter and WWWEC
Permanent Status.” has failed to refute his contention of non-receipt. In net effect, WWWEC
was at a loss to explain the exact just reason for dismissing Aliling.
On June 11, 2004, Aliling and WWWEC inked an Employment Contract
under the terms of conversion to regular status shall be determined on At any event, assuming for argument that the petitioner indeed failed to
the basis of work performance; and employment services may, at any achieve his sales quota, his termination from employment on that ground
time, be terminated for just cause or in accordance with the standards would still be unjustified. Article 282 of the Labor Code considers any of
defined at the time of engagement. the following acts or omission on the part of the employee as just cause
or ground for terminating employment:
However, instead of a Sea freight Sale assignment, WWWEC asked
Aliling to handle Ground Express (GX), a new company product launched (a) Serious misconduct or willful disobedience by the employee
on June 18, 2004 involving domestic cargo forwarding service for Luzon. of the lawful orders of his employer or representative in
Marketing this product and finding daily contracts for it formed the core of connection with his work;
Aliling’s new assignment.
(b) Gross and habitual neglect by the employee of his
A month after, Manuel F. San Mateo III (San Mateo), WWWEC Sales duties;
and Marketing Director, emailed Aliling to express dissatisfaction with the
latter’s performance. (c) Fraud or willful breach by the employee of the trust reposed
in him by his employer or duly authorized representative;
On October 15, 2004, Aliling tendered his resignation to San Mateo.
While WWWEC took no action on his tender, Aliling nonetheless (d) Commission of a crime or offense by the employee against
demanded reinstatement and a written apology, claiming in a subsequent the person of his employer or any immediate member of his
letter dated October 1, 2004 to management that San Mateo had forced family or his duly authorized representatives; and
him to resign.
(e) Other causes analogous to the foregoing. (Emphasis
On October 6, 2004, Lariosa again wrote, this time to advise Aliling of the supplied)
termination of his services effective as of that date owing to his “non-
satisfactory performance” during his probationary period. Records show An employee’s failure to meet sales or work quotas falls under the
that Aliling, for the period indicated, was paid his outstanding salary. concept of gross inefficiency, which in turn is analogous to gross neglect
of duty that is a just cause for dismissal under Article 282 of the Code.
However, or on October 4, 2004, Aliling filed a Complaint for illegal However, in order for the quota imposed to be considered a valid
dismissal due to forced resignation, nonpayment of salaries as well as productivity standard and thereby validate a dismissal, managements
damages with the NLRC against WWWEC. prerogative of fixing the quota must be exercised in good faith for the
advancement of its interest. The duty to prove good faith, however, rests
On April 25, 2006, the Labor Arbiter issued a decision declaring that the with WWWEC as part of its burden to show that the dismissal was for a
grounds upon which complainant’s dismissal was based did not conform just cause. WWWEC must show that such quota was imposed in good
not only the standard but also the compliance required under Article 281 faith. This WWWEC failed to do, perceptibly because it could not. The
of the Labor Code, Necessarily, complainant’s termination is not justified fact of the matter is that the alleged imposition of the quota was a
for failure to comply with the mandate the law requires. Respondents desperate attempt to lend a semblance of validity to Alilings illegal
should be ordered to pay salaries corresponding to the unexpired portion dismissal.
of the contract of employment and all other benefits amounting to a total
of P35,811.00 covering the period from October 6 to December 7, 2004. Being an experimental activity and having been launched for the first
time, the sales of GX services could not be reasonably quantified.
Both parties appealed the decision to the NLRC, which affirmed the WWWEC failed to demonstrate the reasonableness and the bona
decision of the Labor Arbiter and was later on sustained by the Court of fides on the quota imposition. Respondent WWWEC miserably failed to
Appeals. prove the termination of petitioner was for a just cause nor was there
substantial evidence to demonstrate the standards were made known to
Case was elevated to the Supreme Court under Rule 45 Review on the latter at the time of his engagement. Hence, petitioners right to
Certiorari Comment was made that WWWEC hired petitioner on a security of tenure was breached.
probationary basis and fired him before he became a regular employee.
Alilings right to procedural due process was violated
ISSUE:
As earlier stated, to effect a legal dismissal, the employer must show not
Whether or not Aliling was illegally dismissed. only a valid ground therefor, but also that procedural due process has
properly been observed. When the Labor Code speaks of procedural due
RULING: process, the reference is usually to the two (2)-written notice rule
envisaged in Section 2 (III), Rule XXIII, Book V of the Omnibus Rules
Petitioner was illegally dismissed. Implementing the Labor Code, which provides:

To justify fully the dismissal of an employee, the employer must, as a Section 2. Standard of due process: requirements of notice. In
rule, prove that the dismissal was for a just cause and that the employee all cases of termination of employment, the following standards
was afforded due process prior to dismissal. As a complementary of due process shall be substantially observed.
I. For termination of employment based on just causes as defined in Secretary were asked to sign each page of the printout. Vallota, however,
Article 282 of the Code: was not given a copy of the printed file.
On November 14, 2005, Vallota received a memorandum
(a) A written notice served on the employee specifying the ground
directing him to explain within 72 hours why highly confidential files were
or grounds for termination, and giving to said employee reasonable
opportunity within which to explain his side; stored in his computer, which also informed him that he was being placed
under preventive suspension for 30 days effective upon receipt of the
(b) A hearing or conference during which the employee said notice. A second memorandum, also dated November 14, 2005,
concerned, with the assistance of counsel if the employee so notified Vallota of the extension of his preventive suspension for another
desires, is given opportunity to respond to the charge, present his 30 days, in view of the fact that the management needed more time to
evidence or rebut the evidence presented against him; and evaluate the administrative case against him.
On November 24, 2005, PGAI sent him another memorandum
(c) A written notice [of] termination served on the employee
indicating that upon due consideration of all the circumstance, requesting further details on some of the matters he raised in his
grounds have been established to justify his termination. response. In a letter dated December 6, 2005, Vallota requested a
conference, to be attended by a Union representative and counsel. PGAI
Here, the first and second notice requirements have not been properly sent Vallota another memorandum dated December 7, 2005, which,
observed, thus tainting petitioners dismissal with illegality. among others, set a new deadline for Vallota to submit his reply and
evidence in his defense. In compliance with the deadline set, Vallota
The adverted memo dated September 20, 2004 of WWWEC supposedly
submitted his reply-memorandumdated December 12, 2005, outlining his
informing Aliling of the likelihood of his termination and directing him to
account for his failure to meet the expected job performance would have response to the charges.
had constituted the charge sheet, sufficient to answer for the first notice Meanwhile, the Union sent a letter to PGAI President Philip K.
requirement, but for the fact that there is no proof such letter had been Rico requesting that a grievance committee be convened and that the
sent to and received by him. In fact, in his December 13, 2004 contents of the computers of other IT personnel be similarly produced.
Complainants Reply Affidavit, Aliling goes on to tag such The request for the convening of a grievance committee was ignored. On
letter/memorandum as fabrication. WWWEC did not adduce proof to
December 21, 2005, Vallota was given a notice of termination of his
show that a copy of the letter was duly served upon Aliling. Clearly
enough, WWWEC did not comply with the first notice requirement. employment effective January 10, 2006 on the ground of loss of trust and
confidence.
Neither was there compliance with the imperatives of a hearing or Thus, the petitioners filed a complaint for illegal dismissal with
conference. The Court need not dwell at length on this particular breach claims for full backwages, moral and exemplary damages, and attorney
of the due procedural requirement. Suffice it to point out that the record is fees.
devoid of any showing of a hearing or conference having been On March 31, 2006, Labor Arbiter Aliman D. Mangandog
conducted. On the contrary, in its October 1, 2004 letter to Aliling, or
rendered a decision in favor of the petitioners. The respondents filed their
barely five (5) days after it served the notice of termination, WWWEC
acknowledged that it was still evaluating his case. And the written notice Memorandum of Appeal dated May 19, 2006. On June 30, 2006, the
of termination itself did not indicate all the circumstances involving the National Labor Relations Commission dismissed the appeal on the
charge to justify severance of employment. ground that the respondents failed to submit a certificate of non-forum
shopping in accordance with the Rules of Procedure of the NLRC.
166 PRUDENTIAL GUARANTEE AND ASSURANCE EMPLOYEE The respondents filed their Motion for Reconsideration dated
LABOR UNION and SANDY T. VALLOTA, Petitioners, vs. NATIONAL July 17, 2006, which the Union opposed. On October 31, 2007, the
LABOR RELATIONS COMMISSION, PRUDENTIAL GUARANTEE NLRC granted the respondents motion for reconsideration and reversed
AND ASSURANCE INC., and/or JOCELYN RETIZOS, Respondents. and set aside the decision of the LA.
(HEARING IS MANDATORY; FAILURE TO OBSERVE = NOMINAL
DAMAGES) ISSUE:
Whether or not Vallota was validly dismissed on the ground of
FACTS: loss of trust and confidence?
Vallota was employed by Prudential Guarantee as a Junior
Programmer on May 16, 1995. He reported directly to Gerald Dy Victory, HELD:
then head of the EDP, until his replacement by respondent Jocelyn The Court discussion in Mabeza v. National Labor
Retizos sometime in 1997. In August of 2005, Vallota was elected to the Relations Commission is instructive:
Board of Directors of the Union. Loss of confidence as a just cause for dismissal was never
On November 11, 2005, HR Manager, Atty. Rillo informed intended to provide employers with a blank check for terminating their
Union President, Mike Apostol that PGAI was going to conduct an on- employees. Such a vague, all-encompassing pretext as loss of
the-spot security check in the Information and Technology Department. confidence, if unqualifiedly given the seal of approval by this Court, could
The inspection team proceeded to the IT Department, and the readily reduce to barren form the words of the constitutional guarantee of
EDP head, through PGAI network administrator Angelo Gutierrez security of tenure. Having this in mind, loss of confidence should ideally
initiated the spot check of IT Department computers, beginning with the apply only to cases involving employees occupying positions of trust and
one assigned to Vallota. After exploring the contents of all the folders and confidence or to those situations where the employee is routinely
subfolders in the "My Documents" folder, a folder named AAwas found, charged with the care and custody of the employer's money or property.
which Vallota claimed to be about a mutual life fund. Retizos, on the To the first class belong managerial employees, i.e., those vested with
other hand, asked Vallota if was working for MAA Mutual Life and the powers or prerogatives to lay down management policies and/or to
sending them confidential documents of PGAI. hire, transfer, suspend, lay-off, recall, discharge, assign or discipline
Sensing that Vallota was being singled out, Apostol insisted employees or effectively recommend such managerial actions; and to the
that all the computers in the IT Department, including that of Retizos, be second class belong cashiers, auditors, property custodians, etc., or
also subjected to a spot security check. Later, at Retizos office, and in those who, in the normal and routine exercise of their functions, regularly
the presence of Atty. Rillo, Vallota was informed that Retizos and Atty. handle significant amounts of money or property.
Rillo would print the files found in his computer under the folder "MAA." In Bristol Myers Squibb (Phils.), Inc. v. Baban,the Court
Vallota did not object. After the files were printed, Vallota and the Union discussed the requisites for a valid dismissal on the ground of loss of
trust and confidence:
It is clear that Article 282(c) of the Labor Code allows an files reveals some degree of carelessness or neglect in his failure to
employer to terminate the services of an employee for loss of trust and delete them, but it is an extremely farfetched conclusion bordering on
confidence. The right of employers to dismiss employees by reason of paranoia to state that it is part of a larger conspiracy involving corporate
loss of trust and confidence is well established in jurisprudence. espionage.
The first requisite for dismissal on the ground of loss of trust Moreover, contrary to the respondents allegations, the MAA
and confidence is that the employee concerned must be one holding a files found in Vallota computer, the prospectus and corporate profile, are
position of trust and confidence. There are two (2) classes of positions of not sensitive corporate documents. These are documents routinely made
trust. The first class consists of managerial employees. They are defined available to the public, and serve as means to inform the public about the
as those vested with the powers or prerogatives to lay down company and to disseminate information about the products it sells or the
management policies and to hire, transfer suspend, lay-off, recall, services it provides, in order that potential clients may make a sound and
discharge, assign or discipline employees or effectively recommend such informed decision whether or not to purchase or avail of such goods and
managerial actions. The second class consists of cashiers, auditors, services.
property custodians, etc. They are defined as those who in the normal If anything, the presence of the files would merely merit the
and routine exercise of their functions, regularly handle significant development of some suspicion on the part of the employer, but should
amounts of money or property. Xxx not amount to a loss of trust and confidence such as to justify the
The second requisite is that there must be an act that would termination of his employment. Such act is not of the same class, degree
justify the loss of trust and confidence. Loss of trust and confidence to be or gravity as the acts that have been held to be of such character. While
a valid cause for dismissal must be based on a willful breach of trust and Vallota act or omission may have been done carelessly, it falls short of
founded on clearly established facts. The basis for the dismissal must be the standard required for termination of employment. It does not manifest
clearly and convincingly established but proof beyond reasonable doubt either that the employee concerned is unfit to continue working for his
is not necessary. employer.
Thus, the first question to be addressed is whether Vallota held Procedural due process requirements for termination - In
a position of trust and confidence. Vallota was employed by PGAI as a this case, the two-notice requirement was complied with. By the
Junior Programmer assigned to the EDP Department. Based on the petitioners own admission, PGAI issued to Vallota a written Notice of
standards set by previous jurisprudence, Vallota position as Junior Charges & Preventive Suspension dated November 14, 2005. After an
Programmer is analogous to the second class of positions of trust and exchange of memoranda, PGAI then informed Vallota of his dismissal in
confidence. Though he did not physically handle money or property, he its decision dated December 21, 2005.
became privy to confidential data or information by the nature of his Given, however, that the petitioners expressly requested a
functions. At a time when the most sensitive of information is found not conference or a convening of a grievance committee, following the
printed on paper but stored on hard drives and servers, an employee Court ruling in the Perez case, which was later cited in the recent
who handles or has access to data in electronic form naturally becomes case of Lopez v. Alturas Group of Companies, such formal hearing
the unwilling recipient of confidential information. became mandatory. After PGAI failed to affirmatively respond to
Having addressed the nature of his position, the next question such request, it follows that the hearing requirement was not
is whether the act complained of justified the loss of trust and confidence complied with and, therefore, Vallota was denied his right to
of Vallota employer so as to constitute a valid cause for dismissal. It procedural due process.
must, thus, be determined whether the alleged basis for dismissal was Reinstatement and backwages - In light of the above
based on clearly established facts. discussion, Vallota is entitled to reinstatement and backwages, reckoned
The act alleged to have caused the loss of trust and confidence from the date he was illegally dismissed until the finality of this decision in
of PGAI in Vallota was the presence in his computer hard drive of a accordance with jurisprudence.
folder named "MAA" allegedly containing files with information on MAA In view of the strained relations between Vallota and PGAI,
Mutual Life Philippines, a domestic corporation selling life insurance however, it is not in the best interest of the parties, nor is it advisable or
policies to the buying public, and files relating to PGAI internal affairs. practical to order reinstatement. Where reinstatement is no longer viable
While the law and this Court recognize the right of an employer as an option, separation pay equivalent to one (1) month salary for every
to dismiss an employee based on loss of trust and confidence, the year of service should be awarded as an alternative. It must be stressed,
evidence of the employer must clearly and convincingly establish the however, that an illegally dismissed employee is entitled to two reliefs:
facts upon which the loss of trust and confidence in the employee is backwages and reinstatement, which are separate and distinct. In
based. Golden Ace Builders v. Tagle, it was written:
To be a valid ground for dismissal, loss of trust and confidence Thus, an illegally dismissed employee is entitled to two reliefs:
must be based on a willful breach of trust and founded on clearly backwages and reinstatement. The two reliefs provided are separate and
established facts. A breach is willful if it is done intentionally, knowingly distinct. In instances where reinstatement is no longer feasible because
and purposely, without justifiable excuse, as distinguished from an act of strained relations between the employee and the employer, separation
done carelessly, thoughtlessly, heedlessly or inadvertently. It must rest pay is granted. In effect, an illegally dismissed employee is entitled to
on substantial grounds and not on the employer arbitrariness, whims, either reinstatement, if viable, or separation pay if reinstatement is no
caprices or suspicion; otherwise, the employee would remain eternally at longer viable, and backwages.
the mercy of the employer. Further, in order to constitute a just cause for The normal consequences of respondentsillegal dismissal,
dismissal, the act complained of must be work-related and show that the then, are reinstatement without loss of seniority rights, and payment of
employee concerned is unfit to continue working for the employer. Such backwages computed from the time compensation was withheld up to the
ground for dismissal has never been intended to afford an occasion for date of actual reinstatement. Where reinstatement is no longer viable as
abuse because of its subjective nature. an option, separation pay equivalent to one (1) month salary for every
In this case, there was no other evidence presented to prove year of service should be awarded as an alternative. The payment of
fraud in the manner of securing or obtaining the files found in Vallota separation pay is in addition to payment of backwages.
computer. In fact, aside from the presence of these files in Vallota hard Velasco v. National Labor Relations Commission,
drive, there was no other evidence to prove any gross misconduct on his emphasizes:
part. There was no proof either that the presence of such files was part of The accepted doctrine is that separation pay may avail in lieu
an attempt to defraud his employer or to use the files for a purpose other of reinstatement if reinstatement is no longer practical or in the best
than that for which they were intended. If anything, the presence of the
interest of the parties. Separation pay in lieu of reinstatement may
likewise be awarded if the employee decides not to be reinstated. THE CASE:
Under the doctrine of strained relations, the payment of Reyes hired respondents as chief bakers in his three franchise
separation pay is considered an acceptable alternative to reinstatement branches of Julie’s Bakeshop in Sibalom and San Jose, Antique.
when the latter option is no longer desirable or viable. On one hand, such Respondents filed separate complaints against petitioners for
payment liberates the employee from what could be a highly oppressive underpayment of wages, payment of premium pay for holiday and rest
work environment. On the other hand, it releases the employer from the day, service incentive leave pay, 13th month pay, cost of living allowance
grossly unpalatable obligation of maintaining in its employ a worker it (COLA) and attorney’s fees. These complaints were later on
could no longer trust. consolidated.
GRANTED Subsequently, in a memorandum dated February 16, 2000,
Reyes reassigned respondents as utility/security personnel tasked
I. CONSTRUCTIVE DISMISSAL to clean the outside vicinity of his bakeshops and to maintain peace
and order in the area. Upon service of the memo, respondents,
167 DANILO LEONARDO vs. NATIONAL LABOR RELATIONS however, refused to sign the same and likewise refused to perform
COMMISSION their new assignments by not reporting for work.

FACTS: LABOR ARBITER:


AURELIO FUERTE was employed by REYNALDOS expressed dismay over respondents’ lack of good faith in
MARKETING CORPORATION his compensation was P122.00 a day, negotiating a settlement. The Labor Arbiter denounced the way
augmented by a weekly supervisors allowance of P600.00. FUERTE respondents dealt with Atty. Delicana during their discussions for a
alleges that he was instructed to report at private respondents main office possible settlement since respondents themselves later on informed the
where he was informed by the companys personnel manager that he said tribunal that at the time of the said discussions, they no longer
would be transferred to its Sucat plant due to his failure to meet his sales considered Atty. Delicana as their counsel. Despite this, the Labor Arbiter
quota, and for that reason, his supervisors allowance would be still required the parties to submit their respective position papers. And as
withdrawn. For a short time, FUERTE reported for work at the Sucat respondents’ position paper was filed late and no evidence was attached
plant; however, he protested his transfer, subsequently filing a complaint to prove the allegations therein, the Labor Arbiter resolved to dismiss the
for illegal termination. Private respondent contends that it never complaints.
terminated petitioners services. Private respondent claims that Fuerte NLRC overruled the Decision of the Labor Arbiter and held that
was demoted pursuant to a company policy intended to foster the burden of proof lies on herein petitioners as Reyes admitted being
competition among its employees. the employer of Tolores. Hence, petitioners not Tolores, had the duty to
Under this scheme, private respondents employees are advance proof. With respect to Arnaiz and Napal, the NLRC noted that
required to comply with a monthly sales quota. Should a supervisor such since their alleged employer was not impleaded, said respondents’ cases
as FUERTE fail to meet his quota for a certain number of consecutive should be remanded to the Labor Arbiter, and tried as new and separate
months, he will be demoted, whereupon his supervisors allowance will cases.
be withdrawn and be given to the individual who takes his place. When NLRC (MR) found merit in respondents’ Motion for
the employee concerned succeeds in meeting the quota again, he is re- Reconsideration. The NLRC ruled that respondents’ demotion in rank
appointed supervisor and his allowance is restored. from chief bakers to utility/security personnel is tantamount to
constructive dismissal which entitles them to the reliefs available to
ISSUE: illegally dismissed employees. NLRC ratiocinated that the employer
WON Fuerte was constructively dismissed. bears the burden of proving that the employees received their wages and
benefits. In this case, however, no proof of such payment was presented
HELD: by the petitioners.
NO. An employer acts well within its rights in transferring an NLRC (MR NANAMAN), in its Resolution dated December 18,
employee as it sees fit provided that there is no demotion in rank or 2003, again reconsidered its own ruling and held that respondents were
diminution in pay. ] The two circumstances are deemed badges of bad not dismissed, either actually or constructively, but instead willfully
faith, and thus constitutive of constructive dismissal. Constructive disobeyed the return to work order of their employer. The NLRC upheld
dismissal is defined in the following manner: an involuntary resignation petitioners’ prerogative to transfer respondents if only to serve the
resorted to when continued employment becomes impossible, greater interest, safety and well-being of the buying public by forestalling
unreasonable, or unlikely; when there is a demotion in rank or diminution irregular acts of said employees. The NLRC then put the blame on
in pay; or when a clear discrimination, insensibility or disdain by an respondents for disobeying the lawful orders of their employer, noting
employer becomes unbearable to the employee. To constitute such a that it was the same attitude displayed by them in their dealings with their
ground for dismissal, there must be (1) failure to report for work or counsel, Atty. Delicana, in the proceedings before the Labor Arbiter.
absence without valid or justifiable reason; and (2) a clear intention, as CA ruled that respondents were constructively dismissed
manifested by some overt acts, to sever the employer-employee since their designation from chief bakers to utility/security
relationship. personnel is undoubtedly a demotion in rank which involved “a
drastic change in the nature of work resulting to a demeaning and
168 Julie’s Bakeshop and/or Edgar Reyes, Petitioners, vs. HENRY humiliating work condition.” Further, respondents could not be held
ARNAIZ, EDGAR NAPAL,⃰ and Jonathan Tolores, Respondents. guilty of abandonment of work as this was negated by their immediate
filing of complaints to specifically ask for reinstatement.
FACTS:
Julie’s Bakeshop and/or Edgar Reyes (Reyes) assail the ISSUE:
decision of the CA which reversed the Resolutions of the NLRC and WAS THE TRANSFER/REASSIGNMENT OF RESPONDENTS
ordered petitioners to reinstate respondents Henry Arnaiz (Arnaiz), Edgar TO ANOTHER POSITION WITHOUT DIMINUTION IN PAY AND
Napal (Napal) and Jonathan Tolores (Tolores) and to pay them their OTHER PRIVILEGES TANTAMOUNT TO CONSTRUCTIVE
backwages for having been constructively dismissed, as well as their DISMISSAL?
other monetary benefits.
HELD: Respondents cannot be faulted for refusing to report for work
The Court of Appeals is correct in reviewing the findings of the as they were compelled to quit their job due to a demotion without any
National Labor Relations Commission. ( reinstatement without loss of just cause. Moreover, we have consistently held that a charge of
seniority rights, full backwages, inclusive of allowances, and other abandonment is inconsistent with the filing of a complaint for constructive
benefits or their monetary equivalent, computed from the time their dismissal. Respondents’ demand to maintain their positions as chief
compensation was withheld up to the time of their actual reinstatement, bakers by filing a case and asking for the relief of reinstatement belies
should be granted) abandonment.
The transfer/reassignment of respondents constitutes constructive 169 MA. MELISSA A. GALANG, Petitioner, v. JULIA MALASUGUI,
dismissal. Respondent.
We have held that management is free to regulate, according
to its own discretion and judgment, all aspects of employment, including FACTS:
hiring, work assignments, working methods, time, place and manner of Malasugui filed a complaint for illegal dismissal before the
work, processes to be followed, supervision of workers, working National Labor Relations Commission against Galang. Malasugui alleged
regulations, transfer of employees, work supervision, lay off of workers that she was hired by Galang to take care, oversee and man the
and discipline, dismissal and recall of workers. The exercise of premises of the Davao Royal Garden Compound (Pangi Property) the
management prerogative, however, is not absolute as it must be main compound of Galang where the orchids and other ornamental
exercised in good faith and with due regard to the rights of labor. plants used for the business were nursed and propagated. In November
In constructive dismissal cases, the employer has the 1998, she became sick with severe cough and asked for financial
burden of proving that the transfer of an employee is for just or assistance from Galang for medical check-up. The coughing became
valid ground, such as genuine business necessity. The employer incessant which prompted Galang to bring her to a doctor and made to
must demonstrate that the transfer is not unreasonable, undergo a series of examinations including chest radiographic
inconvenient, or prejudicial to the employee and that the transfer examination. Thereafter, she was terminated from work and barred from
does not involve a demotion in rank or a diminution in salary and entering the Pangi property.
other benefits. “If the employer fails to overcome this burden of Galang, on the other hand, denied that Malasugui was her
proof, the employee’s transfer is tantamount to unlawful employee. When the family driver left the other bunkhouse, Malasugui
constructive dismissal.” occupied it and brought along her family as well. The Galang family
In this case, petitioners insist that the transfer of respondents tolerated this arrangement for around six years as an act of kindness.
was a measure of self-preservation and was prompted by a desire to During these times, Malasugui did not look for any job as initially
protect the health of the buying public, claiming that respondents should intended. They did not require Malasugui to pay for rentals, electricity,
be transferred to a position where they could not sabotage the business water and other utilities. In return, Malasugui helped in weeding,
pending resolution of their cases. According to petitioners, the possibility watering, spraying chemicals on the orchids in gratitude for the hospitality
that respondents might introduce harmful substances to the bread while of the Galang family. Admittedly, Galang occasionally gave money to
in the performance of their duties as chief bakers is not imaginary but Malasugui out of charity. She even answered for the medical expenses of
real as borne out by what Tolores did in one of the bakeshops in Culasi, Malasugui when the latter became sick of excessive coughing early in
Antique where he was assigned as baker. 1999. She even made an arrangement with a radiologist for her
This postulation is not well-taken. On the contrary, petitioners diagnostic examination but Malasugui did not show up at the appointed
failed to satisfy the burden of proving that the transfer was based on just time. When confronted by Galang about this, Malasugui packed her
or valid ground. Petitioners’ bare assertions of imminent threat from the belongings and left the Pangi property. She was not asked nor forced to
respondents are mere accusations which are not substantiated by any leave the premises by any member of the Galang family.
proof. This Court is proscribed from making conclusions based on mere The Labor Arbiter ruled that there was an employer-employee
presumptions or suppositions. An employee’s fate cannot be justly relationship between Galang and Malasugui, however, it ruled that there
hinged upon conjectures and surmises. was no substantial evidence that Malasugui was illegally dismissed and
The act attributed against Tolores does not even convince us barred from entering the property after she, without any notice to her
as he was merely a suspected culprit in the alleged sabotage for which employer, packed her belongings and left the Pangi property. The NLRC
no investigation took place to establish his guilt or culpability. Besides, affirmed the decision of the Labor Arbiter. The CA however ruled that
Reyes still retained Tolores as an employee and chief baker when he respondent was illegally dismissed by Galang.
could have dismissed him for cause if the allegations were indeed found
true. In view of these, this Court finds no compelling reason to justify ISSUES:
the transfer of respondents from chief bakers to utility/security 1. Whether or not Malasugui is an employee of Galang.
personnel. What appears to this Court is that respondents’ transfer 2. Whether or not Malasugui was constructively dismissed.
was an act of retaliation on the part of petitioners due to the
former’s filing of complaints against them, and thus, was clearly HELD:
made in bad faith. In fact, petitioner Reyes even admitted that he caused Court of Appeals decision is affirmed.
the reassignments due to the pending complaints filed against him. LABOR LAW
“[D]emotion involves a situation in which an employee is All three, Labor Arbiter, the NLRC and the CA ruled that there
relegated to a subordinate or less important position constituting a was an employer-employee relationship between Galang and Malasugui,
reduction to a lower grade or rank, with a corresponding decrease in therefore, there is no need to routinely undertake the re-examination of
duties and responsibilities, and usually accompanied by a decrease in the evidence presented by the contending parties for the factual findings
salary.“ of the labor officials who have acquired expertise in their own fields are
Although there was no diminution in pay, there was accorded respect and even finality if affirmed on appeal to the Court of
undoubtedly a demotion in titular rank. One cannot deny the disparity Appeals.
between the duties and functions of a chief baker to that of a Such principle cannot, however, apply to the finding of illegal
utility/security personnel tasked to clean and manage the orderliness of dismissal against Galang. The Labor Arbiter and the NLRC both ruled
the outside premises of the bakeshop. Respondents were even that there was no illegal dismissal, but the Court of Appeals reversed
prohibited from entering the bakeshop. The change in the nature of their such findings. When supported by substantial evidence, the findings of
work undeniably resulted to a demeaning and humiliating work condition. fact of the CA are conclusive and binding on the parties and are not
reviewable by this Court, unless the case falls under any of the following - In a letter dated May 15, 2002, cagalawan assailed his
recognized exceptions: transfer claiming he was effctively demoted to his position as head of the
(1) When the conclusion is a finding grounded entirely on disconnection crew to a mere member. He also averted that such
speculation, surmises and conjectures; transfer was inconvenient and prejudicial to him.
(2) When the inference made is manifestly mistaken, absurd or - In a memorandum dated May 16, 2002 the Ke-e explained
impossible; that said transfer was not a demotion since he was holding the position
(3) Where there is a grave abuse of discretion; only by mere designation and not appointment. Meanwhile and in view of
(4) When the judgment is based on a misapprehension of facts; Cagalawan’s transfer, Ke-e issued an order recalling the former’s
(5) When the findings of fact are conflicting; previous designation as Acting Head of the disconnection crew of the
(6) When the Court of Appeals, in making its findings, went Balingasag sub-office.
beyond the issues of the case and the same is contrary to the - Cagalawan eventually stopped reporting for work. On July 1,
admissions of both appellant and appellee; 2002, he filed a complaint for constructive dismissal before the Arbitration
(7) When the findings are contrary to those of the trial court [in branch of the NLRC against MORESCO II and its officers, Ke-e and
this case the administrative bodies of Labor Arbiter and NLRC]; Danilo Subrado, in their capacities as General Manager and Board
(8) When the findings of fact are conclusions without citation of Chairman, respectively.
specific evidence on which they are based; - In reply, Cagalawan claimed that was transferred because he
(9) When the facts set forth in the petition as well as in the executed an Affidavit in support of his co-employee Jessie Rances, who
petitioners' main and reply briefs are not disputed by the respondents; fied an illegal dismissal case against MORESCO II.
and
(10) When the findings of fact of the Court of Appeals are Issues
premised on the supposed absence of evidence and contradicted by the MORESCO II thus fied this petition raising the following issues:
evidence on record. (1)Was the respondent constructively dismissed by the
Jurisprudence provides that the burden of proof to show that petitioner?
the dismissal was for a just cause is on the employer. (2) Did the Court of Appeals err in reversing the NLRC?
Respondent has been in the employ of petitioner for six years
when the alleged abandonment happened. Being scolded, if it were true, Ruling
is hardly a reason for a gardener of six years to just pack up and leave The petition has no merit MORESCO II’s belated submission of
the work premises where she was even allowed to reside, at a time when evidence cannot be permitted.
she was ill and needed medical attention. Indeed, the alleged scolding is Labor tribunals, such as the NLRC, are not precluded from
itself incredible. The given reason was that respondent failed to show up receiving evidence submitted on appeal as technical rules are not binding
at her arranged appointment with the radiologist. It is hard to believe that in cases submitted before them. However, any delay in the submission of
a sick gardener, certainly of minimal means, would refuse the offer of evidence should be adequately explained and should adequately prove
medical services. In fact, the basic allegation in respondents complaint the allegations sought to be proven. In the present case, MORESCO II
for illegal dismissal was that petitioners treatment to her became sour did not cite any reason why it had failed to file its position paper or
especially when she requested that she be examined by a doctor for her present its cause before the Labor Arbiter despite sufficient notice and
cough. And, completely belying the petitioners assertion that respondent time given to do so. Only after an adverse decision was rendered did it
failed to show up at the appointed time with the radiologist are two present its defense and rebut the evidence of Cagalawan by alleging that
certificates issued by Radiologist Susan R. Gaspar stating that on 30 his transfer was made in response to the letter-request of the area
January 1999 and on 1 February 1999 respondent had her chest x-ray manager of the Gingoog sub-officer asking for additional personnel to
taken at the Radiology Section of the Polyclinic Davao. meet its collection quota. To our mind, however, the belated submission
The overt act relied upon by petitioner is not only a doubtful of the said letter- request without any valid explanation casts doubt on
occurrence but is, if it did transpire, even consistent with the dismissal its credibility, especially so when the same is not a newly discovered
from employment posited by the respondent. The factual appraisal of the evidence. For one, the letter-request was dated May 8, 2002 or a day
Court of Appeals is correct. Petitioner was displeased after incurring before the memorandum for Cagalawan’s transfer was issued.
expenses for respondents medical check-up and, it is credible that, MORESCO II could have easily presented the letter in the
thereafter, respondent was prevented entry into the work premises. This proceedings before the Labor Arbiter for serious examination. Why it was
is tantamount to constructive dismissal. not presented at the earliest opportunity is a serious question which
Constructive dismissal exists where there is cessation of work lends credence to Cagalawan’s theory that it may have just
because continued employment is rendered impossible, unreasonable or been fabricated for the purpose of appeal.
unlikely, as an offer involving a demotion in rank and a diminution in pay. It should also be recalled that after Cagalawan received the
Constructive dismissal is a dismissal in disguise or an act amounting to memorandum for his transfer to the Gingoog sub-officer, he immediately
dismissal but made to appear as if it were not. questioned the basis thereof through a letter addressed to Ke-e. If at that
DENIED time there was already a letter- request from the Gingoog area manager,
Ke-e could have easily referred to o specifid this in his subsequent
170 MISAMIS ORIENTAL II ELECTRIC SERVICE COOPERATIVE memorandum of May 16, 2002 which served as his response to
(MORESCO II) VS VIRGILIO CAGALAWAN Cagalawan’s queries about the transfer. However, the said memorandum
was silent in this respect.
Facts Nevertheless, Cagalawan, for his part, faithfully complied with
- MORESCO II, a rural electric cooperative, hired Cagalawan the transfer order but with the reservation to contest its validity precisely
as a Disconnection Lineman on a probationary basis. On March 1, 1994 because he was not adequately informed of its real basis.
Cagalawan was appointed to the same post this time on a permanent The rule is that it is within the ambit of the employer’s
basis. prerogative to transfer an employee for valid reasons and according to
- July 17, 2001, he was designated as Acting Head of the the requirement of its business, provided that the transfer does not result
disconnection crew in one of the sub-offi in Misamis Oriental. In a in demotion in rank or diminution of salary, benefits and other
Memorandum, MORESCO II General Manager Ke-e transferred privileges.45 This Court has always considered the management’s
Cagalawan to another area as a member of the disconnection crew. prerogative to transfer its employees in pursuit of its legitimate interests.
But this prerogative should be exercised without grave abuse of Respondent Javiers absence from August 9, 1995 cannot be deemed as
discretion and with due regard to the basic elements of justice and fair an abandonment of his work.
play, such that if there is a showing that the transfer was unnecessary or Abandonment is a matter of intention and cannot lightly be
inconvenient and prejudicial to the employee, it cannot be upheld.46 inferred or legally presumed from certain equivocal acts. To constitute as
Here, while we find that the transfer of Cagalawan neither entails any such, two requisites must concur: first, the employee must have failed to
demotion in rank since he did not have tenurial security over report for work or must have been absent without valid or justifiable
the position of head of the disconnection crew, nor result to diminution in reason; and second, there must have been a clear intention on the part
pay as this was not sufficiently proven by him, MORESCO II’s evidence of the employee to sever the employer-employee relationship as
is nevertheless not enough to show that said transfer was required by the manifested by some overt acts, with the second element being the more
exigency of the electric cooperative’s business interest. Simply stated, determinative factor. Abandonment as a just ground for dismissal
the evidence sought to be admitted by MORESCO II is not substantial to requires clear, willful, deliberate, and unjustified refusa l of the employee
prove that there was a genuine business urgency that necessitated the to resume his employment.
transfer. Mere absence or failure to report for work, even after notice to
xxx xxx Clearly, not only was the delay in the submission of return, is not tantamount to abandonment.
MORESCO II’s evidence not explained, there was also failure on its part The petitioner acted with precipitate haste in terminating
to sufficiently support its allegation that the transfer of Cagalawan was for respondent Javiers employment on the ground that he had raped the
a legitimate purpose. This being the case, MORESCO II’s plea that its complainant therein. Respondent Javier had yet to be tried for the said
evidence be admitted in the interest of justice does not deserve any charge. In fine, the petitioner prejudged him, and pre-empted the
merit. xxx xxx ruling of the RTC. The petitioner had, in effect, adjudged respondent
WHEREFORE, the petition is DENIED. The Decision dated Javier guilty without due process of law. While it may be true that after
July 26, 2005 or the Court of Appeals in CA-G.R. SP No. 84991 and its the preliminary investigation of the complaint, probable cause for rape
Resolution dated September 6, 2006, are AFFIRMED. was found and respondent Javier had to be detained, these cannot be
SO ORDERED. made as legal bases for the immediate termination of his employment.
Moreover, the petitioner did not accord respondent Javier an
J. DISMISSAL BASED ON FALSE OR NON-EXISTENT CAUSE opportunity to explain his absences. It bears stressing that for a dismissal
to be validly effected, the twin requirements of due process notice and
171 STANDARD ELECTRIC MANUFACTURING CORPORATION vs hearing must be observed. In dismissing an employee, an employer has
STANDARD ELECTRIC the burden of proving that the former worker has been served two
EMPLOYEES UNION-NAFLU-KMU and ROGELIO JAVIER, notices: (1) one to apprise him of the particular acts or omissions for
which his dismissal is sought; and (2) the other to inform him of his
FACTS: employers decision to dismiss him. As to the requirement of a hearing,
Rogelio Javier was employed by the Standard Electric the essence of due process lies in an opportunity to be heard, and not
Manufacturing Corporation (SEMC) as radial spot machine operator in its always and indispensably in an actual hearing.
Production Department. Javier was a member of the Standard Electric But Javier is not entitled to any salary during the period of his
Employees Union-NAFLU (Union). Javier failed to report for work and detention. His entitlement to full backwages commenced from the
failed to notify the SEMC of the reason for his absences. He was time the petitioner refused his reinstatement. In the instant case,
arrested and detained for the charge of rape. Later, the SEMC when respondent Javier was freed by virtue of the judgment of acquittal,
received a letter from Javier, through counsel, informing the SEMC that heimmediately proceeded to the petitioner but was not accepted back to
Javier was detained for the charge of rape and for that reason failed to work; hence, the reckoning point for the grant of backwages started
report for work. He requested the SEMC to defer the implementation of therefrom.
its intention to dismiss him. The SEMC denied Javiers request and
issued a Memorandum terminating his employment for (a) having been K. PREVENTIVE SUSPENSION
absent without leave (AWOL) for more than fifteen days; and (b) for
committing rape. Javier was release from jail and shortly thereafter 172 RUFINA SORIANO vs NLRC & KINGLY COMMODITIES
reported for work, but the SEMC refused to accept him back. TRADERS AND MULTI-RESOURCES, INC.
Javier filed a Complaint for illegal dismissal and averred that
since the reason for his detention for rape was non-existent, the FACTS:
termination of his employment was illegal. The SEMC averred that Rufina Soriano worked as Investment Counselor in 1977 and
Javiers prolonged absences caused irreparable damages to its orderly eventually became the Vice-President, Marketing of Kingly Commodities
operation; he had to be replaced so that the continuity and flow of Traders and Multi-Resources, Inc. On 1984, she was charged with
production would not be jeopardized. It could not afford to wait for Javiers allowing or failing to supervise and monitor certain activities of
indefinite return from detention, if at all. The SEMC insisted that investment counsellors in her department (which includes the signing of a
conformably with its Rules and Regulations, it was justified in dismissing contract opening an account for client, transfer of funds and so on,
Javier for being absent without leave for fifteen days or so. without the knowledge and authority of the client of the said Corporation)
which results in loss of confidence in her. Soriano was preventively
ISSUE: suspended and was required to explain her acts of failure to act. She
WON Javier dismissal was valid. submitted her detailed answer in her explanation. Thereafter, unsatisfied
with the explanation of Soriano, the Executive VP and General Manager
HELD: of Kingly notified Soriano that the Corporation has lost confidence in her
NO. Respondent Javier was dismissed by the petitioner ability to discharge the functions of her office, thus terminated her
effective February 5, 1996 for (a) being AWOL from July 31, 1995 up to services.
January 30, 1996; and (b) committing rape. However, on demurrer to Soriano filed a complaint for illegal suspension and dismissal
evidence, respondent Javier was acquitted of the charge. With against Kingly. That, Kingly acted in bad faith in suspending and
respondent Javiers acquittal, the cause of his dismissal from his termination her services and that kingly violated her right to due process
employment turned out to be non-existent. by suspending her immediately without the benefit of hearing and argues
that the notice of preventive suspension was the living proof that the
corporation had already concluded she was guilty of the charges levelled issue of whether he could validly refuse to obey the transfer orders was
against her even before she could submit her written explanation. brought before this Court, we ruled thus: The refusal to obey a valid
transfer order constitutes willful disobedience of a lawful order of an
ISSUE: employer. Employees may object to, negotiate and seek redress against
Whether or not Rufina Soriano was illegally suspended. employers for rules or orders that they regard as unjust or illegal.
However, until and unless these rules or orders are declared illegal or
HELD: improper by competent authority, the employees ignore or disobey them
No. Preventive suspension does not in itself prove that the at their peril. For Galanida’s continued refusal to obey Allied Bank's
company had prejudged that Soriano was guilty of the charges she was transfer orders, we hold that the bank dismissed Galanida for just cause
asked to answer and explain. in accordance with Article 282(a) of the Labor Code.
Preventive suspension may be necessary for the protection of Galanida is thus not entitled to reinstatement or to separation
the company, its operation and assets, pending investigation of the pay. (Emphasis supplied, citations omitted).
alleged malfeasance or misfeasance on the part of officers or employees It is important to note what the PVA said on Delada’s defiance
of the company and pending a decision on the part of the company. of the transfer order:
Hence, considering the very senior and sensitive character of In fact, Delada cannot hide under the legal cloak of the grievance
petitioner’s position as head of a Department, a line position and machinery of the CBA or the voluntary arbitration proceedings to disobey
considering the unauthorized transactions then just discovered by the a valid order of transfer from the management of the hotel. While it is true
Kingly, the Court belie that the preventive suspension was an arbitrary that Delada’s transfer to Seasons is the subject of the grievance
and capricious act amounting to bad faith on the part of Kingly. machinery in accordance with the provisions of their CBA, Delada is
expected to comply first with the said lawful directive while awaiting th
173 Manila Pavilion Hotel vs. Delada, G.R. No. 189947, January 25, results of the decision in the grievance proceedings. This issue falls
2012 squarely in the case of Allied Banking Corporation vs. Court of Appeals x
x x.
Facts: Pursuant to Allied Banking, unless the order of MPH is
Delada was the Union President of the Manila Pavilion rendered invalid, there is a presumption of the validity of that order.
Supervisors Association at MPH. He was originally assigned as Head Since the PVA eventually ruled that the transfer order was a valid
Waiter of Rotisserie, a fine-dining restaurant operated by petitioner. exercise of management prerogative, we hereby reverse the Decision
Pursuant to a supervisory personnel reorganization program, MPH and the Resolution of the CA affirming the Decision of the PVA in this
reassigned him as Head Waiter of Seasons Coffee Shop, another respect. MPH had the authority to continue with the administrative
restaurant operated by petitioner at the same hotel. Respondent declined proceedings for insubordination and willful disobedience against Delada
the inter-outlet transfer and instead asked for a grievance meeting on the and to impose on him the penalty of suspension. As a consequence,
matter, pursuant to their Collective Bargaining Agreement (CBA). He also petitioner is not liable to pay back wages and other benefits for the period
requested his retention as Head Waiter of Rotisserie while the grievance corresponding to the penalty of 90-day suspension.
procedure was ongoing. MPH replied and told respondent to report to his
new assignment for the time being, without prejudice to the resolution of PREVENTIVE SUSPENSION:
the grievance involving the transfer. He adamantly refused to assume his
new post at the Seasons Coffee Shop and instead continued to report to Preventive suspension is a disciplinary measure resorted to by the
his previous assignment at Rotisserie. Thus, MPH sent him several employer pending investigation of an alleged malfeasance or
memoranda on various dates, requiring him to explain in writing why he misfeasance committed by an employee. The employer temporarily bars
should not be penalized for the following offenses: serious misconduct; the employee from working if his continued employment poses a serious
willful disobedience of the lawful orders of the employer; gross and imminent threat to the life or property of the employer or of his co-
insubordination; gross and habitual neglect of duties; and willful breach of workers.
trust. Despite the notices from MPH, Delada persistently rebuffed orders
for him to report to his new assignment. According to him, since the On the other hand, the penalty of suspension refers to the disciplinary
grievance machinery under their CBA had already been initiated, his action imposed on the employee after an official investigation or
transfer must be held in abeyance. Thus, on 9 May 2007, MPH initiated administrative hearing is conducted.9 The employer exercises its right to
administrative proceedings against him. discipline erring employees pursuant to company rules and regulations.

Issue:
Whether MPH retained the authority to continue with the
administrative case against Delada for insubordination and willful
disobedience of the transfer order.

Rulings:
Accordingly, we rule in this case that MPH did not lose its
authority to discipline respondent for his continued refusal to report to his
new assignment. In relation to this point, we recall our Decision in Allied
Banking Corporation v. Court of Appeals. In Allied Banking Corporation,
employer Allied Bank reassigned respondent Galanida from its Cebu City
branch to its Bacolod and Tagbilaran branches. He refused to follow the
transfer order and instead filed a Complaint before the Labor Arbiter for
constructive dismissal. While the case was pending, Allied Bank insisted
that he report to his new assignment. When he continued to refuse, it
directed him to explain in writing why no disciplinary action should be
meted out to him. Due to his continued refusal to report to his new
assignment, Allied Bank eventually terminated his services. When the

Potrebbero piacerti anche